ΠšΠΎΡΠΈΠ½ΡƒΡ ΠΊΡ€ΡƒΠ³: ВригономСтричСский ΠΊΡ€ΡƒΠ³

Π‘ΠΎΠ΄Π΅Ρ€ΠΆΠ°Π½ΠΈΠ΅

ВригономСтричСский ΠΊΡ€ΡƒΠ³

ВригономСтричСский ΠΊΡ€ΡƒΠ³ — это ΠΎΠΊΡ€ΡƒΠΆΠ½ΠΎΡΡ‚ΡŒ с Π΅Π΄ΠΈΠ½ΠΈΡ‡Π½Ρ‹ΠΌ радиусом ΠΈ Ρ†Π΅Π½Ρ‚Ρ€ΠΎΠΌ Π² Π½Π°Ρ‡Π°Π»Π΅ осСй ΠΊΠΎΠΎΡ€Π΄ΠΈΠ½Π°Ρ‚, каТдая Ρ‚ΠΎΡ‡ΠΊΠ° ΠΊΠΎΡ‚ΠΎΡ€ΠΎΠΉ ΠΎΠ±Ρ€Π°Π·ΡƒΠ΅Ρ‚ Ρ‚Ρ€Π΅ΡƒΠ³ΠΎΠ»ΡŒΠ½ΠΈΠΊ с Ρ‚ΠΎΡ‡ΠΊΠ°ΠΌΠΈ:
  • Ρ‚ΠΎΡ‡ΠΊΠ° Π² Π½Π°Ρ‡Π°Π»Π΅ осСй ΠΊΠΎΠΎΡ€Π΄ΠΈΠ½Π°Ρ‚ (Ρ‚ΠΎΡ‡ΠΊΠ° 0;0)
  • Ρ‚ΠΎΡ‡ΠΊΠ° Π½Π° окруТности (Π²Ρ‹Π±Ρ€Π°Π½Π° Π½Π°ΠΌΠΈ)
  • Ρ‚ΠΎΡ‡ΠΊΠ° Π½Π° оси X, которая являСтся ΠΏΡ€ΠΎΠ΅ΠΊΡ†ΠΈΠ΅ΠΉ Π²Ρ‹Π±Ρ€Π°Π½Π½ΠΎΠΉ Π½Π°ΠΌΠΈ Ρ‚ΠΎΡ‡ΠΊΠΈ Π½Π° эту ось (пСрпСндикуляр ΠΊ оси X)

Как Π²ΠΈΠ΄Π½ΠΎ, Ρ‚Π°ΠΊΠΎΠΉ Ρ‚Ρ€Π΅ΡƒΠ³ΠΎΠ»ΡŒΠ½ΠΈΠΊ являСтся ΠΏΡ€ΡΠΌΠΎΡƒΠ³ΠΎΠ»ΡŒΠ½Ρ‹ΠΌ, Ρ‚Π°ΠΊ ΠΊΠ°ΠΊ ΠΈΠ· Π²Ρ‹Π±Ρ€Π°Π½Π½ΠΎΠΉ Π½Π°ΠΌΠΈ Ρ‚ΠΎΡ‡ΠΊΠΈ Π½Π° ось абсцисс всСгда опускаСтся пСрпСндикуляр. Π’ΠΎ Π΅ΡΡ‚ΡŒ сторона, ΡΠΎΠ΅Π΄ΠΈΠ½ΡΡŽΡ‰Π°Ρ Π½Π°Ρ‡Π°Π»ΠΎ ΠΊΠΎΠΎΡ€Π΄ΠΈΠ½Π°Ρ‚ ΠΈ Π²Ρ‹Π±Ρ€Π°Π½Π½ΡƒΡŽ Π½Π°ΠΌΠΈ Ρ‚ΠΎΡ‡ΠΊΡƒ Π½Π° тригономСтричСском ΠΊΡ€ΡƒΠ³Π΅ ( Π½Π° ΠΏΡ€ΠΈΠ²Π΅Π΄Π΅Π½Π½ΠΎΠΌ рисункС ΠΎΠ±ΠΎΠ·Π½Π°Ρ‡Π΅Π½Π½ΡƒΡŽ ΠΊΠ°ΠΊ B, B1. B2, B3) всСгда являСтся Π³ΠΈΠΏΠΎΡ‚Π΅Π½ΡƒΠ·ΠΎΠΉ ΠΏΡ€ΡΠΌΠΎΡƒΠ³ΠΎΠ»ΡŒΠ½ΠΎΠ³ΠΎ Ρ‚Ρ€Π΅ΡƒΠ³ΠΎΠ»ΡŒΠ½ΠΈΠΊΠ°, проСкция Π²Ρ‹Π±Ρ€Π°Π½Π½ΠΎΠΉ Ρ‚ΠΎΡ‡ΠΊΠΈ — это ΠΊΠ°Ρ‚Π΅Ρ‚, Π° сторона ΠΎΡ‚ Ρ‚ΠΎΡ‡ΠΊΠΈ пСрСсСчСния с осью X ΠΎΠ±Ρ€Π°Π·ΡƒΠ΅Ρ‚ Π²Ρ‚ΠΎΡ€ΠΎΠΉ ΠΊΠ°Ρ‚Π΅Ρ‚.

Π£Π³ΠΎΠ», ΠΊΠΎΡ‚ΠΎΡ€Ρ‹ΠΉ образуСтся ΠΌΠ΅ΠΆΠ΄Ρƒ осью абсцисс (осью X) ΠΈ Π³ΠΈΠΏΠΎΡ‚Π΅Π½ΡƒΠ·ΠΎΠΉ Ρ‚Ρ€Π΅ΡƒΠ³ΠΎΠ»ΡŒΠ½ΠΈΠΊΠ° — являСтся ΡƒΠ³Π»ΠΎΠΌ, для ΠΊΠΎΡ‚ΠΎΡ€ΠΎΠ³ΠΎ ΠΈ Π²Ρ‹Ρ‡ΠΈΡΠ»ΡΡŽΡ‚ΡΡ значСния тригономСтричСских Ρ„ΡƒΠ½ΠΊΡ†ΠΈΠΉ.

Π­Ρ‚ΠΎΡ‚ ΡƒΠ³ΠΎΠ» отсчитываСтся ΠΎΡ‚ ΠΏΠΎΠ»ΠΎΠΆΠΈΡ‚Π΅Π»ΡŒΠ½ΠΎΠ³ΠΎ направлСния оси абсцисс (оси X) ΠΊΠ°ΠΊ ноль, Π΄Π°Π»Π΅Π΅ ΠΏΡ€ΠΎΡ‚ΠΈΠ² часовой стрСлки. Π’Π°ΠΊΠΈΠΌ ΠΎΠ±Ρ€Π°Π·ΠΎΠΌ, ΠΏΠΎΠ»Π½Ρ‹ΠΉ ΠΊΡ€ΡƒΠ³ составляСт 360 градусов ΠΈΠ»ΠΈ 2Ο€ Ρ€Π°Π΄ΠΈΠ°Π½.

Π§Ρ‚ΠΎΠ±Ρ‹ Π²Ρ‹Ρ‡ΠΈΡΠ»ΠΈΡ‚ΡŒ Π·Π½Π°Ρ‡Π΅Π½ΠΈΠ΅ тригономСтричСской Ρ„ΡƒΠ½ΠΊΡ†ΠΈΠΈ для Π²Ρ‹Π±Ρ€Π°Π½Π½ΠΎΠ³ΠΎ ΡƒΠ³Π»Π° тригономСтричСского ΠΊΡ€ΡƒΠ³Π° достаточно Π²ΠΎΡΠΏΠΎΠ»ΡŒΠ·ΠΎΠ²Π°Ρ‚ΡŒΡΡ ΠΊΠΎΠΎΡ€Π΄ΠΈΠ½Π°Ρ‚Π°ΠΌΠΈ Ρ‚ΠΎΡ‡ΠΊΠΈ, ΠΏΡ€ΠΈΠ½Π°Π΄Π»Π΅ΠΆΠ°Ρ‰Π΅ΠΉ окруТности тригономСтричСского ΠΊΡ€ΡƒΠ³Π°. На ΠΏΡ€ΠΈΠ²Π΅Π΄Π΅Π½Π½ΠΎΠΌ Π²Ρ‹ΡˆΠ΅ рисункС, ΠΏΠΎΠΊΠ°Π·Π°Π½ΠΎ вычислСниС значСния синуса для всСх ΡƒΠ³Π»ΠΎΠ².

НапримСр, sinΒ Ξ± для Ρ‚Ρ€Π΅ΡƒΠ³ΠΎΠ»ΡŒΠ½ΠΈΠΊΠ° OBC (Π³Π΄Π΅ ΠΊΠΎΠΎΡ€Π΄ΠΈΠ½Π°Ρ‚Ρ‹ Ρ‚ΠΎΡ‡ΠΊΠΈ B Ρ€Π°Π²Π½Ρ‹ (x,y) ) ,Π±ΡƒΠ΄Π΅Ρ‚ Ρ€Π°Π²Π΅Π½: y / √ ( x2 + y2)Β 

Бвойства тригономСтричСского ΠΊΡ€ΡƒΠ³Π°

Если ΠΏΠΎΡΠ»Π΅Π΄ΠΎΠ²Π°Ρ‚Π΅Π»ΡŒΠ½ΠΎ Π²Ρ‹Ρ‡ΠΈΡΠ»ΡΡ‚ΡŒ значСния тригономСтричСских Ρ„ΡƒΠ½ΠΊΡ†ΠΈΠΉ для тригономСтричСского ΠΊΡ€ΡƒΠ³Π°, Ρ‚ΠΎ становится Π²ΠΈΠ΄Π½ΠΎ, Ρ‡Ρ‚ΠΎ Ρ€Π΅Π·ΡƒΠ»ΡŒΡ‚Π°Ρ‚ Ρ‚Π°ΠΊΠΈΡ… вычислСний мСняСт свой Π·Π½Π°ΠΊ Π² зависимости ΠΎΡ‚ Ρ‚ΠΎΠ³ΠΎ, Π² ΠΊΠ°ΠΊΠΎΠΉ Ρ‡Π΅Ρ‚Π²Π΅Ρ€Ρ‚ΠΈ тригономСтричСского ΠΊΡ€ΡƒΠ³Π° Π²Ρ‹Π±Ρ€Π°Π½Π° Ρ‚ΠΎΡ‡ΠΊΠ°. ΠŸΡ€ΠΈ этом Π·Π½Π°ΠΊ тригономСтричСской Ρ„ΡƒΠ½ΠΊΡ†ΠΈΠΈ Π² ΠΏΡ€Π΅Π΄Π΅Π»Π°Ρ… ΠΎΠ΄Π½ΠΎΠΉ ΠΈ Ρ‚ΠΎΠΉ ΠΆΠ΅ Ρ‡Π΅Ρ‚Π²Π΅Ρ€Ρ‚ΠΈ сохраняСтся.
Β 

Π—Π½Π°ΠΊΠΈ тригономСтричСских Ρ„ΡƒΠ½ΠΊΡ†ΠΈΠΉ Π² ΠΊΠΎΠΎΡ€Π΄ΠΈΠ½Π°Ρ‚Π½Ρ‹Ρ… чСтвСртях Π² тригономСтричСском ΠΊΡ€ΡƒΠ³Π΅




ΠŸΡ€Π΅ΠΎΠ±Ρ€Π°Π·ΠΎΠ²Π°Π½ΠΈΠ΅ ΡƒΠ³Π»ΠΎΠ² большС 360 градусов ΠΈΠ»ΠΈ 2Ο€ Ρ€Π°Π΄ΠΈΠ°Π½

Как Π²ΠΈΠ΄Π½ΠΎ ΠΈΠ· ΠΊΠ°Ρ€Ρ‚ΠΈΠ½ΠΎΠΊ, послС Ρ‚ΠΎΠ³ΠΎ, ΠΊΠ°ΠΊ Π·Π½Π°Ρ‡Π΅Π½ΠΈΠ΅ ΡƒΠ³Π»Π° прСвысит 360 градусов (ΠΈΠ»ΠΈ 2Ο€ Ρ€Π°Π΄ΠΈΠ°Π½), Ρ‚ΠΎ Ρ€Π΅Π·ΡƒΠ»ΡŒΡ‚Π°Ρ‚ вычислСния значСния Π±ΡƒΠ΄Π΅Ρ‚ Ρ‚Π΅ΠΌ ΠΆΠ΅ самым. Π’ΠΎ Π΅ΡΡ‚ΡŒ, для Ρ‚ΠΎΠ³ΠΎ, Ρ‡Ρ‚ΠΎΠ±Ρ‹ привСсти Π·Π½Π°Ρ‡Π΅Π½ΠΈΠ΅ ΠΊ «Π½ΠΎΡ€ΠΌΠ°Π»ΡŒΠ½ΠΎΠΌΡƒ» — Π½ΡƒΠΆΠ½ΠΎ Π²Ρ‹Ρ‡Π΅ΡΡ‚ΡŒ ΠΈΠ· ΠΈΠΌΠ΅ΡŽΡ‰Π΅Π³ΠΎΡΡ значСния 360 градусов ΠΈΠ»ΠΈ 2Ο€ Ρ€Π°Π΄ΠΈΠ°Π½ ΠΈ ΠΏΠΎΠ²Ρ‚ΠΎΡ€ΡΡ‚ΡŒ ΠΎΠΏΠ΅Ρ€Π°Ρ†ΠΈΡŽ ΡΡ‚ΠΎΠ»ΡŒΠΊΠΎ Ρ€Π°Π·, ΠΏΠΎΠΊΠ° Ρ€Π΅Π·ΡƒΠ»ΡŒΡ‚Π°Ρ‚ Π½Π΅ станСт мСньшС 360 ΠΈΠ»ΠΈ 2Ο€.


 ВригономСтричСскиС ΡΠΎΠΎΡ‚Π½ΠΎΡˆΠ΅Π½ΠΈΡ Π² ΠΏΡ€ΡΠΌΠΎΡƒΠ³ΠΎΠ»ΡŒΠ½ΠΎΠΌ Ρ‚Ρ€Π΅ΡƒΠ³ΠΎΠ»ΡŒΠ½ΠΈΠΊΠ΅ | ОписаниС курса | Π Π°Π΄ΠΈΠ°Π½Ρ‹ ΠΈ градусы. Π Π°Π΄iΠ°Π½ΠΈ i градуси 

Β Β Β 

ВригономСтричСский ΠΊΡ€ΡƒΠ³. ΠžΡΠ½ΠΎΠ²Π½Ρ‹Π΅ значСния тригономСтричСских Ρ„ΡƒΠ½ΠΊΡ†ΠΈΠΉ

Если Π²Ρ‹ ΡƒΠΆΠ΅ Π·Π½Π°ΠΊΠΎΠΌΡ‹ с тригономСтричСским ΠΊΡ€ΡƒΠ³ΠΎΠΌ, ΠΈ Ρ…ΠΎΡ‚ΠΈΡ‚Π΅ лишь ΠΎΡΠ²Π΅ΠΆΠΈΡ‚ΡŒ Π² памяти ΠΎΡ‚Π΄Π΅Π»ΡŒΠ½Ρ‹Π΅ элСмСнты, Β ΠΈΠ»ΠΈ Π²Ρ‹ совсСм Π½Π΅Ρ‚Π΅Ρ€ΠΏΠ΅Π»ΠΈΠ²Ρ‹,  – Ρ‚ΠΎ Π²ΠΎΡ‚ ΠΎΠ½, тригономСтричСский ΠΊΡ€ΡƒΠ³:

ΠœΡ‹ ΠΆΠ΅ здСсь Π±ΡƒΠ΄Π΅ΠΌ всС ΠΏΠΎΠ΄Ρ€ΠΎΠ±Π½ΠΎ Ρ€Π°Π·Π±ΠΈΡ€Π°Ρ‚ΡŒ шаг Π·Π° шагом + ΠΏΠΎΠΊΠ°Π·Π°Ρ‚ΡŒ

ВригономСтричСский ΠΊΡ€ΡƒΠ³ – Π½Π΅ Ρ€ΠΎΡΠΊΠΎΡˆΡŒ, Π° Π½Π΅ΠΎΠ±Ρ…ΠΎΠ΄ΠΈΠΌΠΎΡΡ‚ΡŒ

ВригономСтрия Ρƒ ΠΌΠ½ΠΎΠ³ΠΈΡ… ассоциируСтся с Π½Π΅ΠΏΡ€ΠΎΡ…ΠΎΠ΄ΠΈΠΌΠΎΠΉ Ρ‡Π°Ρ‰Π΅ΠΉ. Π’Π΄Ρ€ΡƒΠ³ наваливаСтся ΡΡ‚ΠΎΠ»ΡŒΠΊΠΎ Π·Π½Π°Ρ‡Π΅Π½ΠΈΠΉ тригономСтричСских Ρ„ΡƒΠ½ΠΊΡ†ΠΈΠΉ,Β  ΡΡ‚ΠΎΠ»ΡŒΠΊΠΎ формул…  А ΠΎΠ½ΠΎ вСдь, ΠΊΠ°ΠΊ, – нСзаладилось Π²Π½Π°Ρ‡Π°Π»Π΅, и… пошло-поСхало… сплошноС нСпониманиС…

ΠžΡ‡Π΅Π½ΡŒ Π²Π°ΠΆΠ½ΠΎ Π½Π΅ ΠΌΠ°Ρ…Π°Ρ‚ΡŒ Ρ€ΡƒΠΊΠΎΠΉ Π½Π° значСния тригономСтричСских Ρ„ΡƒΠ½ΠΊΡ†ΠΈΠΉ, – ΠΌΠΎΠ», всСгда ΠΌΠΎΠΆΠ½ΠΎ ΠΏΠΎΡΠΌΠΎΡ‚Ρ€Π΅Ρ‚ΡŒ Π² ΡˆΠΏΠΎΡ€Ρƒ с Ρ‚Π°Π±Π»ΠΈΡ†Π΅ΠΉ Π·Π½Π°Ρ‡Π΅Π½ΠΈΠΉ.

Если Π²Ρ‹ постоянно смотритС Π² Ρ‚Π°Π±Π»ΠΈΡ†Ρƒ со значСниями тригономСтричСских Ρ„ΠΎΡ€ΠΌΡƒΠ», Β Π΄Π°Π²Π°ΠΉΡ‚Π΅ ΠΈΠ·Π±Π°Π²Π»ΡΡ‚ΡŒΡΡ ΠΎΡ‚ этой ΠΏΡ€ΠΈΠ²Ρ‹Ρ‡ΠΊΠΈ!

Нас Π²Ρ‹Ρ€ΡƒΡ‡ΠΈΡ‚ тригономСтричСский ΠΊΡ€ΡƒΠ³! Π’Ρ‹ нСсколько Ρ€Π°Π· ΠΏΠΎΡ€Π°Π±ΠΎΡ‚Π°Π΅Ρ‚Π΅ с Π½ΠΈΠΌ, ΠΈ Π΄Π°Π»Π΅Π΅ ΠΎΠ½ Ρƒ вас сам Π±ΡƒΠ΄Π΅Ρ‚ Π²ΡΠΏΠ»Ρ‹Π²Π°Ρ‚ΡŒ Π² Π³ΠΎΠ»ΠΎΠ²Π΅. Π§Π΅ΠΌ ΠΎΠ½ Π»ΡƒΡ‡ΡˆΠ΅ Ρ‚Π°Π±Π»ΠΈΡ†Ρ‹? Π”Π° Π² Ρ‚Π°Π±Π»ΠΈΡ†Π΅-Ρ‚ΠΎ Π²Ρ‹ Π½Π°ΠΉΠ΄Π΅Ρ‚Π΅ ΠΎΠ³Ρ€Π°Π½ΠΈΡ‡Π΅Π½Π½ΠΎΠ΅ число Π·Π½Π°Ρ‡Π΅Π½ΠΈΠΉ, Π° Π½Π° ΠΊΡ€ΡƒΠ³Π΅ – Π’Π‘Π•!

К ΠΏΡ€ΠΈΠΌΠ΅Ρ€Ρƒ, скаТитС, глядя Π² ΡΡ‚Π°Π½Π΄Π°Ρ€Ρ‚Π½ΡƒΡŽ Ρ‚Π°Π±Π»ΠΈΡ†Ρƒ Π·Π½Π°Ρ‡Π΅Π½ΠΈΠΉ тригономСтричСских Ρ„ΠΎΡ€ΠΌΡƒΠ», Ρ‡Π΅ΠΌΡƒ Ρ€Π°Π²Π΅Π½ синус, скаТСм, градусов, ΠΈΠ»ΠΈ .

Никак?.. ΠΌΠΎΠΆΠ½ΠΎ, ΠΊΠΎΠ½Π΅Ρ‡Π½ΠΎ, ΠΏΠΎΠ΄ΠΊΠ»ΡŽΡ‡ΠΈΡ‚ΡŒ Ρ„ΠΎΡ€ΠΌΡƒΠ»Ρ‹ привСдСния…  А глядя Π½Π° тригономСтричСский ΠΊΡ€ΡƒΠ³, Π»Π΅Π³ΠΊΠΎ ΠΌΠΎΠΆΠ½ΠΎ ΠΎΡ‚Π²Π΅Ρ‚ΠΈΡ‚ΡŒ Π½Π° Ρ‚Π°ΠΊΠΈΠ΅ вопросы. И Π²Ρ‹ скоро Π±ΡƒΠ΄Π΅Ρ‚Π΅ Π·Π½Π°Ρ‚ΡŒ ΠΊΠ°ΠΊ!

А ΠΏΡ€ΠΈ Ρ€Π΅ΡˆΠ΅Π½ΠΈΠΈ тригономСтричСских ΡƒΡ€Π°Π²Π½Π΅Π½ΠΈΠΉ ΠΈ нСравСнств Β  Π±Π΅Π· тригономСтричСского ΠΊΡ€ΡƒΠ³Π°  – Π²ΠΎΠΎΠ±Ρ‰Π΅ Π½ΠΈΠΊΡƒΠ΄Π°.

Знакомство с тригономСтричСским ΠΊΡ€ΡƒΠ³ΠΎΠΌ

Π”Π°Π²Π°ΠΉΡ‚Π΅ ΠΏΠΎ порядку.

Π‘Π½Π°Ρ‡Π°Π»Π° Π²Ρ‹ΠΏΠΈΡˆΠ΅ΠΌ Π²ΠΎΡ‚ Ρ‚Π°ΠΊΠΎΠΉ ряд чисСл:

А Ρ‚Π΅ΠΏΠ΅Ρ€ΡŒ Ρ‚Π°ΠΊΠΎΠΉ:

И, Π½Π°ΠΊΠΎΠ½Π΅Ρ†, Ρ‚Π°ΠΊΠΎΠΉ:

ΠšΠΎΠ½Π΅Ρ‡Π½ΠΎ, понятно, Ρ‡Ρ‚ΠΎ, Π½Π° самом-Ρ‚ΠΎ Π΄Π΅Π»Π΅, Π½Π° ΠΏΠ΅Ρ€Π²ΠΎΠΌ мСстС стоит , Π½Π° Π²Ρ‚ΠΎΡ€ΠΎΠΌ мСстС стоит , Π° Π½Π° послСднСм – . Π’ΠΎ Π΅ΡΡ‚ΡŒ нас Π±ΡƒΠ΄Π΅Ρ‚ большС ΠΈΠ½Ρ‚Π΅Ρ€Π΅ΡΠΎΠ²Π°Ρ‚ΡŒ Ρ†Π΅ΠΏΠΎΡ‡ΠΊΠ° .

Но ΠΊΠ°ΠΊ красиво ΠΎΠ½Π° ΠΏΠΎΠ»ΡƒΡ‡ΠΈΠ»Π°ΡΡŒ! Π’ случаС Ρ‡Π΅Π³ΠΎ – восстановим эту «лСсСнку-чудСсСнку».

И Π·Π°Ρ‡Π΅ΠΌ ΠΎΠ½ΠΎ Π½Π°ΠΌ?

Π­Ρ‚Π° Ρ†Π΅ΠΏΠΎΡ‡ΠΊΠ° – ΠΈ Π΅ΡΡ‚ΡŒ основныС значСния синуса ΠΈ косинуса Π² ΠΏΠ΅Ρ€Π²ΠΎΠΉ Ρ‡Π΅Ρ‚Π²Π΅Ρ€Ρ‚ΠΈ.

НачСртим Π² ΠΏΡ€ΡΠΌΠΎΡƒΠ³ΠΎΠ»ΡŒΠ½ΠΎΠΉ систСмС ΠΊΠΎΠΎΡ€Π΄ΠΈΠ½Π°Ρ‚ ΠΊΡ€ΡƒΠ³ Π΅Π΄ΠΈΠ½ΠΈΡ‡Π½ΠΎΠ³ΠΎ радиуса (Ρ‚ΠΎ Π΅ΡΡ‚ΡŒ радиус-Ρ‚ΠΎ ΠΏΠΎ Π΄Π»ΠΈΠ½Π΅ Π±Π΅Ρ€Π΅ΠΌ любой, Π° Π΅Π³ΠΎ Π΄Π»ΠΈΠ½Ρƒ объявляСм Π΅Π΄ΠΈΠ½ΠΈΡ‡Π½ΠΎΠΉ).

ΠžΡ‚ Π»ΡƒΡ‡Π° Β Β«0-Π‘Ρ‚Π°Ρ€Ρ‚Β» ΠΎΡ‚ΠΊΠ»Π°Π΄Ρ‹Π²Π°Π΅ΠΌ Π² Π½Π°ΠΏΡ€Π°Π²Π»Π΅Π½ΠΈΠΈ стрСлки (см. рис.) ΡƒΠ³Π»Ρ‹ .

ΠŸΠΎΠ»ΡƒΡ‡Π°Π΅ΠΌ ΡΠΎΠΎΡ‚Π²Π΅Ρ‚ΡΡ‚Π²ΡƒΡŽΡ‰ΠΈΠ΅ Ρ‚ΠΎΡ‡ΠΊΠΈ Π½Π° ΠΊΡ€ΡƒΠ³Π΅. Π’Π°ΠΊ Π²ΠΎΡ‚ Ссли ΡΠΏΡ€ΠΎΠ΅Ρ†ΠΈΡ€ΠΎΠ²Π°Ρ‚ΡŒ Ρ‚ΠΎΡ‡ΠΊΠΈ Π½Π° ΠΊΠ°ΠΆΠ΄ΡƒΡŽ ΠΈΠ· осСй, Ρ‚ΠΎ ΠΌΡ‹ Π²Ρ‹ΠΉΠ΄Π΅ΠΌ ΠΊΠ°ΠΊ Ρ€Π°Π· Π½Π° значСния ΠΈΠ· ΡƒΠΊΠ°Π·Π°Π½Π½ΠΎΠΉ Π²Ρ‹ΡˆΠ΅ Ρ†Π΅ΠΏΠΎΡ‡ΠΊΠΈ.

Π­Ρ‚ΠΎ ΠΏΠΎΡ‡Π΅ΠΌΡƒ ΠΆΠ΅, спроситС Π²Ρ‹?

НС Π±ΡƒΠ΄Π΅ΠΌ Ρ€Π°Π·Π±ΠΈΡ€Π°Ρ‚ΡŒ всС. Рассмотрим ΠΏΡ€ΠΈΠ½Ρ†ΠΈΠΏ, ΠΊΠΎΡ‚ΠΎΡ€Ρ‹ΠΉ ΠΏΠΎΠ·Π²ΠΎΠ»ΠΈΡ‚ ΡΠΏΡ€Π°Π²ΠΈΡ‚ΡŒΡΡ ΠΈ с Π΄Ρ€ΡƒΠ³ΠΈΠΌΠΈ, Π°Π½Π°Π»ΠΎΠ³ΠΈΡ‡Π½Ρ‹ΠΌΠΈ ситуациями.

Π’Ρ€Π΅ΡƒΠ³ΠΎΠ»ΡŒΠ½ΠΈΠΊ ΠΠžΠ’ – ΠΏΡ€ΡΠΌΠΎΡƒΠ³ΠΎΠ»ΡŒΠ½Ρ‹ΠΉ, Π² Π½Π΅ΠΌ Β . А ΠΌΡ‹ Π·Π½Π°Π΅ΠΌ, Ρ‡Ρ‚ΠΎ ΠΏΡ€ΠΎΡ‚ΠΈΠ² ΡƒΠ³Π»Π° Π² Π»Π΅ΠΆΠΈΡ‚ ΠΊΠ°Ρ‚Π΅Ρ‚ Π²Π΄Π²ΠΎΠ΅ мСньший Π³ΠΈΠΏΠΎΡ‚Π΅Π½ΡƒΠ·Ρ‹ (Π³ΠΈΠΏΠΎΡ‚Π΅Π½ΡƒΠ·Π° Ρƒ нас = радиусу ΠΊΡ€ΡƒΠ³Π°, Ρ‚ΠΎ Π΅ΡΡ‚ΡŒ ).

Π—Π½Π°Ρ‡ΠΈΡ‚, АВ= (Π° ΡΠ»Π΅Π΄ΠΎΠ²Π°Ρ‚Π΅Π»ΡŒΠ½ΠΎ, ΠΈ ОМ=). А ΠΏΠΎ Ρ‚Π΅ΠΎΡ€Π΅ΠΌΠ΅ ΠŸΠΈΡ„Π°Π³ΠΎΡ€Π°

НадСюсь, ΡƒΠΆΠ΅ Ρ‡Ρ‚ΠΎ-Ρ‚ΠΎ становится понятно?

НаконСц, Ρ‡Ρ‚ΠΎ Ρ‚Π°ΠΊΠΎΠ΅ синус, косинус Π² ΠΏΡ€ΡΠΌΠΎΡƒΠ³ΠΎΠ»ΡŒΠ½ΠΎΠΌ Ρ‚Ρ€Π΅ΡƒΠ³ΠΎΠ»ΡŒΠ½ΠΈΠΊΠ΅?

Β 

Π’Π°ΠΊ Π²ΠΎΡ‚ Ρ‚ΠΎΡ‡ΠΊΠ° Β Π’ ΠΈ Π±ΡƒΠ΄Π΅Ρ‚ ΡΠΎΠΎΡ‚Π²Π΅Ρ‚ΡΡ‚Π²ΠΎΠ²Π°Ρ‚ΡŒ Β Π·Π½Π°Ρ‡Π΅Π½ΠΈΡŽ , Π° Ρ‚ΠΎΡ‡ΠΊΠ° М – Π·Π½Π°Ρ‡Π΅Π½ΠΈΡŽ

Аналогично с ΠΎΡΡ‚Π°Π»ΡŒΠ½Ρ‹ΠΌΠΈ значСниями ΠΏΠ΅Ρ€Π²ΠΎΠΉ Ρ‡Π΅Ρ‚Π²Π΅Ρ€Ρ‚ΠΈ.

Как Π²Ρ‹ ΠΏΠΎΠ½ΠΈΠΌΠ°Π΅Ρ‚Π΅, привычная Π½Π°ΠΌ ось (ox) Π±ΡƒΠ΄Π΅Ρ‚ осью косинусов, Π° ось Β (oy) – осью синусов. Β  ΠŸΡ€ΠΎ тангСнс ΠΈ котангСнс ΠΏΠΎΠ·ΠΆΠ΅.

Π‘Π»Π΅Π²Π° ΠΎΡ‚ нуля ΠΏΠΎ оси косинусов (Π½ΠΈΠΆΠ΅ нуля ΠΏΠΎ оси синусов) Π±ΡƒΠ΄ΡƒΡ‚, ΠΊΠΎΠ½Π΅Ρ‡Π½ΠΎ, ΠΎΡ‚Ρ€ΠΈΡ†Π°Ρ‚Π΅Π»ΡŒΠ½Ρ‹Π΅ значСния.

Π˜Ρ‚Π°ΠΊ, Π²ΠΎΡ‚ ΠΎΠ½, Π’Π‘Π•ΠœΠžΠ“Π£Π©Π˜Π™ тригономСтричСский ΠΊΡ€ΡƒΠ³, Π±Π΅Π· ΠΊΠΎΡ‚ΠΎΡ€ΠΎΠ³ΠΎ Π½ΠΈΠΊΡƒΠ΄Π° Π² Ρ‚Ρ€ΠΈΠ³ΠΎΠ½ΠΎΠΌΠ΅Ρ‚Ρ€ΠΈΠΈ.

А Π²ΠΎΡ‚ ΠΊΠ°ΠΊ ΠΏΠΎΠ»ΡŒΠ·ΠΎΠ²Π°Ρ‚ΡŒΡΡ тригономСтричСским ΠΊΡ€ΡƒΠ³ΠΎΠΌ, ΠΌΡ‹ ΠΏΠΎΠ³ΠΎΠ²ΠΎΡ€ΠΈΠΌ Π² ΡΠ»Π΅Π΄ΡƒΡŽΡ‰Π΅ΠΉ ΡΡ‚Π°Ρ‚ΡŒΠ΅.

ВригономСтричСский ΠΊΡ€ΡƒΠ³ синус ΠΈ косинус

ВригономСтричСский ΠΊΡ€ΡƒΠ³ прСдставляСт значСния тригономСтричСских Ρ„ΡƒΠ½ΠΊΡ†ΠΈΠΉ синус (sin) ΠΈ косинус (cos) Π² Π²ΠΈΠ΄Π΅ ΠΊΠΎΠΎΡ€Π΄ΠΈΠ½Π°Ρ‚ Ρ‚ΠΎΡ‡Π΅ΠΊ Π΅Π΄ΠΈΠ½ΠΈΡ‡Π½ΠΎΠΉ окруТности ΠΏΡ€ΠΈ Ρ€Π°Π·Π»ΠΈΡ‡Π½Ρ‹Ρ… значСниях ΡƒΠ³Π»Π° Π°Π»ΡŒΡ„Π° Π² градусах ΠΈ Ρ€Π°Π΄ΠΈΠ°Π½Π°Ρ….

ΠŸΠΎΡΠΊΠΎΠ»ΡŒΠΊΡƒ я сам Π²Π΅Ρ‡Π½ΠΎ ΠΏΡƒΡ‚Π°ΡŽΡΡŒ ΠΏΡ€ΠΈ ΠΏΠ΅Ρ€Π΅Π²ΠΎΠ΄Π΅ ΠΊΠΎΠΎΡ€Π΄ΠΈΠ½Π°Ρ‚ Ρ‚ΠΎΡ‡Π΅ΠΊ окруТности Π² синусы ΠΈ косинусы, для простоты всС значСния косинусов (cos) для ΡƒΠ³Π»ΠΎΠ² ΠΎΡ‚ 0 Π΄ΠΎ 360 градусов (ΠΎΡ‚ 0 ΠΏΠΈ Π΄ΠΎ 2 ΠΏΠΈ) ΠΏΠΎΠ΄Ρ‡Π΅Ρ€ΠΊΠ½ΡƒΡ‚Ρ‹ Π·Π΅Π»Π΅Π½ΠΎΠΉ Ρ‡Π΅Ρ€Ρ‚ΠΎΡ‡ΠΊΠΎΠΉ. Π”Π°ΠΆΠ΅ ΠΏΡ€ΠΈ распСчаткС этого рисунка тригономСтричСского ΠΊΡ€ΡƒΠ³Π° Π½Π° Ρ‡Π΅Ρ€Π½ΠΎ-Π±Π΅Π»ΠΎΠΌ ΠΏΡ€ΠΈΠ½Ρ‚Π΅Ρ€Π΅ всС значСния косинуса Π±ΡƒΠ΄ΡƒΡ‚ ΠΏΠΎΠ΄Ρ‡Π΅Ρ€ΠΊΠ½ΡƒΡ‚Ρ‹, Π° значСния синуса Π±ΡƒΠ΄ΡƒΡ‚ Π±Π΅Π· подчСркивания. Если Π²Π°ΠΌ интСрСсно, Ρ‚ΠΎ ΠΌΠΎΠΆΠ΅Ρ‚Π΅ ΠΏΠΎΡΠΌΠΎΡ‚Ρ€Π΅Ρ‚ΡŒ ΠΎΡ‚Π΄Π΅Π»ΡŒΠ½Ρ‹Π΅ тригономСтричСскиС ΠΊΡ€ΡƒΠ³ΠΈ для синуса ΠΈ косинуса.

Напротив ΡƒΠΊΠ°Π·Π°Π½Π½Ρ‹Ρ… ΡƒΠ³Π»ΠΎΠ² Π½Π° окруТности располоТСны Ρ‚ΠΎΡ‡ΠΊΠΈ, Π° Π² ΠΊΡ€ΡƒΠ³Π»Ρ‹Ρ… скобках ΡƒΠΊΠ°Π·Π°Π½Ρ‹ ΠΊΠΎΠΎΡ€Π΄ΠΈΠ½Π°Ρ‚Ρ‹ этих Ρ‚ΠΎΡ‡Π΅ΠΊ. ΠŸΠ΅Ρ€Π²ΠΎΠΉ записана ΠΊΠΎΠΎΡ€Π΄ΠΈΠ½Π°Ρ‚Π°

Π₯ (косинус)

Π”Π°Π²Π°ΠΉΡ‚Π΅ ΠΏΡ€ΠΎΠ²Π΅Π΄Π΅ΠΌ ΠΎΠ±Π·ΠΎΡ€Π½ΡƒΡŽ ΡΠΊΡΠΊΡƒΡ€ΡΠΈΡŽ ΠΏΠΎ этому ΡƒΠ³ΠΎΠ»ΠΊΡƒ матСматичСского Π·ΠΎΠΎΠΏΠ°Ρ€ΠΊΠ°. ΠŸΡ€Π΅ΠΆΠ΄Π΅ всСго, Π½ΡƒΠΆΠ½ΠΎ ΠΎΡ‚ΠΌΠ΅Ρ‚ΠΈΡ‚ΡŒ, Ρ‡Ρ‚ΠΎ здСсь присутствуСт Π΄Π΅ΠΊΠ°Ρ€Ρ‚ΠΎΠ²Π° систСма ΠΊΠΎΠΎΡ€Π΄ΠΈΠ½Π°Ρ‚ — ΠΎΠ΄Π½Π° чСрная Π³ΠΎΡ€ΠΈΠ·ΠΎΠ½Ρ‚Π°Π»ΡŒΠ½Π°Ρ линия с Π±ΡƒΠΊΠΎΠ²ΠΊΠΎΠΉ Π₯ Π²ΠΎΠ·Π»Π΅ стрСлочки, вторая — Π²Π΅Ρ€Ρ‚ΠΈΠΊΠ°Π»ΡŒΠ½Π°Ρ линия с Π±ΡƒΠΊΠΎΠ²ΠΊΠΎΠΉ Π£. На оси Π₯, ΠΊΠΎΡ‚ΠΎΡ€ΡƒΡŽ Π΅Ρ‰Π΅ Π½Π°Π·Ρ‹Π²Π°ΡŽΡ‚ ось абсцисс (это ΡƒΠΌΠ½ΠΎΠ΅ слово ΠΌΠ°Ρ‚Π΅ΠΌΠ°Ρ‚ΠΈΠΊΠΈ ΠΏΡ€ΠΈΠ΄ΡƒΠΌΠ°Π»ΠΈ ΡΠΏΠ΅Ρ†ΠΈΠ°Π»ΡŒΠ½ΠΎ, Ρ‡Ρ‚ΠΎ Π±Ρ‹ Π·Π°ΠΏΡƒΡ‚Π°Ρ‚ΡŒ Π±Π»ΠΎΠ½Π΄ΠΈΠ½ΠΎΠΊ) ΠΆΠΈΠ²ΡƒΡ‚ косинусы — cos. На оси Π£, ΠΊΠΎΡ‚ΠΎΡ€ΡƒΡŽ Π½Π°Π·Ρ‹Π²Π°ΡŽΡ‚ ось ΠΎΡ€Π΄ΠΈΠ½Π°Ρ‚ (Π΅Ρ‰Π΅ ΠΎΠ΄Π½ΠΎ ΡƒΠΌΠ½ΠΎΠ΅ слово, ΠΊΠΎΡ‚ΠΎΡ€ΠΎΠ΅ Π² устах Π±Π»ΠΎΠ½Π΄ΠΈΠ½ΠΊΠΈ ΠΌΠΎΠΆΠ΅Ρ‚ ΡΡ‚Π°Ρ‚ΡŒ убийствСнным ΠΎΡ€ΡƒΠΆΠΈΠ΅ΠΌ), ΠΆΠΈΠ²ΡƒΡ‚ синусы — sin. Если ΠΏΠΎΡΠΌΠΎΡ‚Ρ€Π΅Ρ‚ΡŒ Π½Π° ΡΠ΅ΠΌΠ΅ΠΉΠ½ΡƒΡŽ Тизнь этих тригономСтричСских Ρ„ΡƒΠ½ΠΊΡ†ΠΈΠΉ, Ρ‚ΠΎ Π½Π΅ Ρ‚Ρ€ΡƒΠ΄Π½ΠΎ Π·Π°ΠΌΠ΅Ρ‚ΠΈΡ‚ΡŒ, Ρ‡Ρ‚ΠΎ синусы всСгда Π½Π° ΠΊΡƒΡ…Π½Π΅ Ρƒ ΠΏΠ»ΠΈΡ‚Ρ‹ ΠΏΠΎ Π²Π΅Ρ€Ρ‚ΠΈΠΊΠ°Π»ΠΈ, Π° косинусы — Π½Π° Π΄ΠΈΠ²Π°Π½Π΅ ΠΏΠ΅Ρ€Π΅Π΄ Ρ‚Π΅Π»Π΅Π²ΠΈΠ·ΠΎΡ€ΠΎΠΌ ΠΏΠΎ Π³ΠΎΡ€ΠΈΠ·ΠΎΠ½Ρ‚Π°Π»ΠΈ.

Π’ этой систСмС ΠΊΠΎΠΎΡ€Π΄ΠΈΠ½Π°Ρ‚ нарисована ΠΎΠΊΡ€ΡƒΠΆΠ½ΠΎΡΡ‚ΡŒ радиусом, Ρ€Π°Π²Π½Ρ‹ΠΌ Π΅Π΄ΠΈΠ½ΠΈΡ†Π΅. Π¦Π΅Π½Ρ‚Ρ€ окруТности находится Π² Π½Π°Ρ‡Π°Π»Π΅ систСмы ΠΊΠΎΠΎΡ€Π΄ΠΈΠ½Π°Ρ‚ — Ρ‚Π°ΠΌ, Π³Π΄Π΅ Π² Ρ†Π΅Π½Ρ‚Π΅ рисунка ΠΏΠ΅Ρ€Π΅ΡΠ΅ΠΊΠ°ΡŽΡ‚ΡΡ

оси абсцисс (ось Π₯) ΠΈ ΠΎΡ€Π΄ΠΈΠ½Π°Ρ‚ (ось Π£).

Из Ρ†Π΅Π½Ρ‚Ρ€Π° окруТности ΠΏΡ€ΠΎΠ²Π΅Π΄Π΅Π½Ρ‹ Ρ‚ΠΎΠ½Π΅Π½ΡŒΠΊΠΈΠ΅ Ρ‡Π΅Ρ€Ρ‚ΠΎΡ‡ΠΊΠΈ, ΠΊΠΎΡ‚ΠΎΡ€Ρ‹Π΅ ΠΏΠΎΠΊΠ°Π·Ρ‹Π²Π°ΡŽΡ‚ ΡƒΠ³Π»Ρ‹ 30, 45, 60, 120, 135, 150, 210, 225, 240, 300, 315, 330 градусов. Π’ Ρ€Π°Π΄ΠΈΠ°Π½Π½ΠΎΠΉ ΠΌΠ΅Ρ€Π΅ ΡƒΠ³Π»ΠΎΠ² это ΠΏΠΈ Π΄Π΅Π»Π΅Π½Π½ΠΎΠ΅ Π½Π° 6, ΠΏΠΈ Π½Π° 4, ΠΏΠΈ Π½Π° 3, 2 ΠΏΠΈ Π½Π° 3, 3 ΠΏΠΈ Π½Π° 4, 5 ΠΏΠΈ Π½Π° 6, 7 ΠΏΠΈ Π½Π° 6, 5 ΠΏΠΈ Π½Π° 4, 4 ΠΏΠΈ Π½Π° 3, 3 ΠΏΠΈ Π½Π° 2, 5 ΠΏΠΈ Π½Π° 3, 7 ΠΏΠΈ Π½Π° 4, 11 ΠΏΠΈ Π΄Π΅Π»Π΅Π½Π½ΠΎΠ΅ Π½Π° 6. Π‘ осями ΠΊΠΎΠΎΡ€Π΄ΠΈΠ½Π°Ρ‚ ΡΠΎΠ²ΠΏΠ°Π΄Π°ΡŽΡ‚ Ρ‚Π°ΠΊΠΈΠ΅ значСния ΡƒΠ³Π»ΠΎΠ²: 0, 90, 180, 270 градусов ΠΈΠ»ΠΈ 0 ΠΏΠΈ, ΠΏΠΈ Π΄Π΅Π»Π΅Π½Π½ΠΎΠ΅ Π½Π° 2, ΠΏΠΈ, 3 ΠΏΠΈ Π΄Π΅Π»Π΅Π½Π½ΠΎΠ΅ Π½Π° 2. ΠŸΠΎΠ»ΡŒΠ·ΡƒΡΡΡŒ ΠΊΠ°Ρ€Ρ‚ΠΈΠ½ΠΊΠΎΠΉ, ΠΎΡ‡Π΅Π½ΡŒ просто ΠΏΠ΅Ρ€Π΅Π²ΠΎΠ΄ΠΈΡ‚ΡŒ ΡƒΠ³Π»Ρ‹ ΠΈΠ· градусов Π² Ρ€Π°Π΄ΠΈΠ°Π½Ρ‹ ΠΈ ΠΈΠ· Ρ€Π°Π΄ΠΈΠ°Π½ Π² градусы. ΠžΠ΄ΠΈΠ½Π°ΠΊΠΎΠ²Ρ‹Π΅ значСния Π² Ρ€Π°Π·Π½Ρ‹Ρ… систСмах измСрСния ΡƒΠ³Π»ΠΎΠ² написаны Π½Π° ΠΎΠ΄Π½ΠΎΠΉ Π»ΠΈΠ½ΠΈΠΈ, ΠΈΠ·ΠΎΠ±Ρ€Π°ΠΆΠ°ΡŽΡ‰Π΅ΠΉ этот ΡƒΠ³ΠΎΠ».

Π›ΠΈΠ½ΠΈΠΈ ΡƒΠ³Π»ΠΎΠ² Π·Π°ΠΊΠ°Π½Ρ‡ΠΈΠ²Π°ΡŽΡ‚ΡΡ Ρ‚ΠΎΡ‡ΠΊΠ°ΠΌΠΈ Π½Π° Π΅Π΄ΠΈΠ½ΠΈΡ‡Π½ΠΎΠΉ окруТности. Π’ΠΎΠ·Π»Π΅ ΠΊΠ°ΠΆΠ΄ΠΎΠΉ Ρ‚ΠΎΡ‡ΠΊΠΈ, Π² ΠΊΡ€ΡƒΠ³Π»Ρ‹Ρ… скобках, записаны ΠΊΠΎΠΎΡ€Π΄ΠΈΠ½Π°Ρ‚Ρ‹ этой Ρ‚ΠΎΡ‡ΠΊΠΈ. ΠŸΠ΅Ρ€Π²ΠΎΠΉ записана ΠΊΠΎΠΎΡ€Π΄ΠΈΠ½Π°Ρ‚Π°

Π₯, которая соотвСтствуСт косинусу ΡƒΠ³Π»Π°, ΠΎΠ±Ρ€Π°Π·ΠΎΠ²Π°Π²ΡˆΠ΅Π³ΠΎ эту Ρ‚ΠΎΡ‡ΠΊΡƒ. Π’Ρ‚ΠΎΡ€ΠΎΠΉ записана ΠΊΠΎΠΎΡ€Π΄ΠΈΠ½Π°Ρ‚Π° Π£ этой Ρ‚ΠΎΡ‡ΠΊΠΈ, Ρ‡Ρ‚ΠΎ соотвСтствуСт Π·Π½Π°Ρ‡Π΅Π½ΠΈΡŽ синуса ΡƒΠ³Π»Π°. По ΠΊΠ°Ρ€Ρ‚ΠΈΠ½ΠΊΠ΅ довольно Π»Π΅Π³ΠΊΠΎ Π½Π°Ρ…ΠΎΠ΄ΠΈΡ‚ΡŒ синус ΠΈ косинус Π·Π°Π΄Π°Π½Π½ΠΎΠ³ΠΎ ΡƒΠ³Π»Π° ΠΈ Π½Π°ΠΎΠ±ΠΎΡ€ΠΎΡ‚, ΠΏΠΎ Π·Π°Π΄Π°Π½Π½ΠΎΠΌΡƒ Π·Π½Π°Ρ‡Π΅Π½ΠΈΡŽ синуса ΠΈΠ»ΠΈ косинуса, ΠΌΠΎΠΆΠ½ΠΎ Π»Π΅Π³ΠΊΠΎ Π½Π°ΠΉΡ‚ΠΈ Π·Π½Π°Ρ‡Π΅Π½ΠΈΠ΅ ΡƒΠ³Π»Π°. Π“Π»Π°Π²Π½ΠΎΠ΅, Π½Π΅ ΠΏΠ΅Ρ€Π΅ΠΏΡƒΡ‚Π°Ρ‚ΡŒ синус с косинусом.

ΠžΠ±Ρ€Π°Ρ‰Π°ΡŽ особоС Π²Π½ΠΈΠΌΠ°Π½ΠΈΠ΅ Π½Π° Ρ‚ΠΎΡ‚ Ρ„Π°ΠΊΡ‚, Ρ‡Ρ‚ΠΎ Ссли Π²Ρ‹ ΠΏΠΎ Π·Π½Π°Ρ‡Π΅Π½ΠΈΡŽ синуса ΠΈΠ»ΠΈ косинуса ΠΈΡ‰ΠΈΡ‚Π΅ ΡƒΠ³ΠΎΠ», ΠΎΠ±ΡΠ·Π°Ρ‚Π΅Π»ΡŒΠ½ΠΎ Π½ΡƒΠΆΠ½ΠΎ Π΄ΠΎΠΏΠΈΡΡ‹Π²Π°Ρ‚ΡŒ ΠΏΠ΅Ρ€ΠΈΠΎΠ΄ ΡƒΠ³Π»Π°. ΠœΠ°Ρ‚Π΅ΠΌΠ°Ρ‚ΠΈΠΊΠΈ ΠΎΡ‡Π΅Π½ΡŒ Ρ‚Ρ€Π΅ΠΏΠ΅Ρ‚Π½ΠΎ относятся ΠΊ этому Π°ΠΏΠΏΠ΅Π½Π΄ΠΈΡ†ΠΈΡ‚Ρƒ тригономСтричСских Ρ„ΡƒΠ½ΠΊΡ†ΠΈΠΉ ΠΈ ΠΏΡ€ΠΈ Π΅Π³ΠΎ отсутствии ΠΌΠΎΠ³ΡƒΡ‚ Π²Π»Π΅ΠΏΠΈΡ‚ΡŒ Π΄Π²ΠΎΠΉΠΊΡƒ Π·Π°, казалось Π±Ρ‹, ΠΏΡ€Π°Π²ΠΈΠ»ΡŒΠ½Ρ‹ΠΉ ΠΎΡ‚Π²Π΅Ρ‚. Π§Ρ‚ΠΎ Ρ‚Π°ΠΊΠΎΠ΅ ΠΏΠ΅Ρ€ΠΈΠΎΠ΄ ΠΏΡ€ΠΈ Π½Π°Ρ…ΠΎΠΆΠ΄Π΅Π½ΠΈΠΈ ΡƒΠ³Π»Π° ΠΏΠΎ Π·Π½Π°Ρ‡Π΅Π½ΠΈΡŽ тригономСтричСской Ρ„ΡƒΠ½ΠΊΡ†ΠΈΠΈ? Π­Ρ‚ΠΎ такая ΡˆΡ‚ΡƒΡ‡ΠΊΠ°, которая ΠΏΡ€ΠΈΠ΄ΡƒΠΌΠ°Π½Π° ΠΌΠ°Ρ‚Π΅ΠΌΠ°Ρ‚ΠΈΠΊΠ°ΠΌΠΈ ΡΠΏΠ΅Ρ†ΠΈΠ°Π»ΡŒΠ½ΠΎ для Ρ‚ΠΎΠ³ΠΎ, Ρ‡Ρ‚ΠΎΠ±Ρ‹ Π·Π°ΠΏΡƒΡ‚Ρ‹Π²Π°Ρ‚ΡŒΡΡ самим ΠΈ Π·Π°ΠΏΡƒΡ‚Ρ‹Π²Π°Ρ‚ΡŒ Π΄Ρ€ΡƒΠ³ΠΈΡ…. ОсобСнно Π±Π»ΠΎΠ½Π΄ΠΈΠ½ΠΎΠΊ. Но ΠΎΠ± этом ΠΌΡ‹ ΠΏΠΎΠ³ΠΎΠ²ΠΎΡ€ΠΈΠΌ ΠΊΠ°ΠΊ-Π½ΠΈΠ±ΡƒΠ΄ΡŒ Π² Π΄Ρ€ΡƒΠ³ΠΎΠΉ Ρ€Π°Π·.

Всё, Ρ‡Ρ‚ΠΎ собрано Π² ΠΊΡƒΡ‡ΠΊΡƒ Π½Π° рисункС тригономСтричСского ΠΊΡ€ΡƒΠ³Π° синуса ΠΈ косинуса, ΠΌΠΎΠΆΠ½ΠΎ Π²Π½ΠΈΠΌΠ°Ρ‚Π΅Π»ΡŒΠ½ΠΎ Ρ€Π°ΡΡΠΌΠΎΡ‚Ρ€Π΅Ρ‚ΡŒ Π½Π° ΠΎΡ‚Π΄Π΅Π»ΡŒΠ½Ρ‹Ρ… ΠΊΠ°Ρ€Ρ‚ΠΈΠ½ΠΊΠ°Ρ… с ΠΏΠΎΡ€Ρ‚Ρ€Π΅Ρ‚Π°ΠΌΠΈ синуса 0, 30, 45 градусов (ссылки Π½Π° ΠΎΡ‚Π΄Π΅Π»ΡŒΠ½Ρ‹Π΅ странички я Π±ΡƒΠ΄Ρƒ Π΄ΠΎΠ±Π°Π²Π»ΡΡ‚ΡŒ ΠΏΠΎ ΠΌΠ΅Ρ€Π΅ увСличСния Ρ„ΠΎΡ‚ΠΎΠ³Π°Π»Π΅Ρ€Π΅ΠΈ синусов ΠΈ косинусов).

Найти Ρ€Π΅ΡˆΠ΅Π½ΠΈΠ΅:

Бинусы ΠΈ косинусы ΠΊΡ€ΡƒΠ³ — здСсь ΠΊΠ°Ρ€Ρ‚ΠΈΠ½ΠΊΠ° Π²ΠΎ всСй своСй тригономСтричСской красС.

Π£Π³ΠΎΠ» 120 градусов Π² Ρ€Π°Π΄ΠΈΠ°Π½Π°Ρ… — Ρ€Π°Π²Π΅Π½ 2/3 ΠΏΠΈ ΠΈΠ»ΠΈ 2 ΠΏΠΈ Π΄Π΅Π»Π΅Π½Π½ΠΎΠ΅ Π½Π° 3, Π½Π° ΠΊΠ°Ρ€Ρ‚ΠΈΠ½ΠΊΠ΅ ΠΎΡ‡Π΅Π½ΡŒ красиво нарисовано.

ЗначСния синусов косинусов ΡƒΠ³Π»ΠΎΠ² Π² Ρ€Π°Π΄ΠΈΠ°Π½Π°Ρ… — Π½Π° ΠΊΠ°Ρ€Ρ‚ΠΈΠ½ΠΊΠ΅ Π΅ΡΡ‚ΡŒ Ρ‚Π°ΠΊΠΈΠ΅, надСюсь, ΠΈΠΌΠ΅Π½Π½ΠΎ Ρ‚Π΅ ΡƒΠ³Π»Ρ‹, ΠΊΠΎΡ‚ΠΎΡ€Ρ‹Π΅ Π²Ρ‹ ΠΈΡ‰Π΅Ρ‚Π΅.

Π—Π½Π°Ρ‡Π΅Π½ΠΈΠ΅ косинуса ΡƒΠ³Π»Π° Π² 45 градусов — Ρ€Π°Π²Π½ΠΎ ΠΊΠΎΡ€Π½ΡŽ ΠΈΠ· Π΄Π²ΡƒΡ… Π΄Π΅Π»Π΅Π½Π½ΠΎΠΌΡƒ Π½Π° Π΄Π²Π°, ΠΌΠΎΠΆΠ΅Ρ‚Π΅ ΠΏΡ€ΠΎΠ²Π΅Ρ€ΠΈΡ‚ΡŒ ΠΏΠΎ рисунку.

ВригономСтричСская ΠΎΠΊΡ€ΡƒΠΆΠ½ΠΎΡΡ‚ΡŒ — я Π½Π΅ совсСм ΡƒΠ²Π΅Ρ€Π΅Π½, Ρ‡Ρ‚ΠΎ прСдставлСнная Π½Π° ΠΊΠ°Ρ€Ρ‚ΠΈΠ½ΠΊΠ΅ ΠΎΠΊΡ€ΡƒΠΆΠ½ΠΎΡΡ‚ΡŒ являСтся тригономСтричСской, Π½ΠΎ Ρ‡Ρ‚ΠΎ-Ρ‚ΠΎ ΠΎΡ‚ Ρ‚Ρ€ΠΈΠ³ΠΎΠ½ΠΎΠΌΠ΅Ρ‚Ρ€ΠΈΠΈ Π² этой окруТности ΠΎΠΏΡ€Π΅Π΄Π΅Π»Π΅Π½Π½ΠΎ Π΅ΡΡ‚ΡŒ, Π½Π°ΠΏΡ€ΠΈΠΌΠ΅Ρ€, синусы ΠΈ косинусы Π½Π° окруТности — вылитая тригономСтрия.

ВригономСтричСский ΠΊΡ€ΡƒΠ³ рисунок — Π΅ΡΡ‚ΡŒ здСсь Ρ‚Π°ΠΊΠΎΠΉ. ΠŸΡ€Π°Π²Π΄Π°, Π½Π΅ самый красивый рисунок, ΠΌΠΎΠΆΠ½ΠΎ Π½Π°Ρ€ΠΈΡΠΎΠ²Π°Ρ‚ΡŒ Π³ΠΎΡ€Π°Π·Π΄ΠΎ красивСС ΠΈ понятнСС. МнС минус Π² Ρ€Π΅ΠΏΡƒΡ‚Π°Ρ†ΠΈΡŽ — ΠΏΠΎΡ‡Π΅ΠΌΡƒ я Π΄ΠΎ сих ΠΏΠΎΡ€ Π½Π΅ нарисовал Π΅Π³ΠΎ для Π±Π»ΠΎΠ½Π΄ΠΈΠ½ΠΎΠΊ? ΠŸΡ€Π΅Π΄ΡΡ‚Π°Π²Π»ΡΠ΅Ρ‚Π΅ ΡΠΈΡ‚ΡƒΠ°Ρ†ΠΈΡŽ Π² ΠΊΠ°Ρ€Ρ‚ΠΈΠ½Π½ΠΎΠΉ Π³Π°Π»Π΅Ρ€Π΅Π΅ Π±ΡƒΠ΄ΡƒΡ‰Π΅Π³ΠΎ: экскурсовод ΠΎΠ±ΡŠΡΡΠ½ΡΠ΅Ρ‚ Π³Ρ€ΡƒΠΏΠΏΠ΅ школьников «ΠŸΠ΅Ρ€Π΅Π΄ Π²Π°ΠΌΠΈ всСмирно извСстноС ΠΏΠΎΠ»ΠΎΡ‚Π½ΠΎ «Π’ригономСтричСская ΠΌΠ°Π΄ΠΎΠ½Π½Π° с Π΅Π΄ΠΈΠ½ΠΈΡ‡Π½Ρ‹ΠΌ ΠΎΡ‚Ρ€Π΅Π·ΠΊΠΎΠΌ Π½Π° Ρ€ΡƒΠΊΠ°Ρ…» — ΠΊΠ°Ρ€Ρ‚ΠΈΠ½Π° гСниального Ρ…ΡƒΠ΄ΠΎΠΆΠ½ΠΈΠΊΠ° эпохи Π Π°Π½Π½Π΅Π³ΠΎ ΠœΠ°Ρ‚Π΅ΠΌΠ°Ρ‚ΠΈΡ‡Π΅ΡΠΊΠΎΠ³ΠΎ ВозроТдСния . ..» Π”Π°Π»ΡŒΡˆΠ΅ ΠΎΠ½Π° Π½Π°Π·Ρ‹Π²Π°Π΅Ρ‚ имя этого самого Ρ…ΡƒΠ΄ΠΎΠΆΠ½ΠΈΠΊΠ° (ΠΈΠ»ΠΈ Ρ…ΡƒΠ΄ΠΎΠΆΠ½ΠΈΡ†Ρ‹). Π­Ρ‚ΠΎ имя ΠΌΠΎΠΆΠ΅Ρ‚ Π±Ρ‹Ρ‚ΡŒ вашим!

ΠšΡ€ΡƒΠ³ синусов ΠΈ косинусов — ΠΈΠΌΠ΅Π½Π½ΠΎ Ρ‚Π°ΠΊΠΎΠΉ ΠΊΡ€ΡƒΠ³ ΡΠΎΠ²Π΅Ρ€ΡˆΠ΅Π½Π½ΠΎ случайно оказался здСсь Π½Π° ΠΊΠ°Ρ€Ρ‚ΠΈΠ½ΠΊΠ΅.

Π£Π³ΠΎΠ» 9 градусов сколько это Π² ΠΏΠΈ — Π² ΠΏΠΈ это 1/20 ΠΈΠ»ΠΈ ΠΏΠΈ/20.
РСшСниС: для ΠΏΠ΅Ρ€Π΅Π²ΠΎΠ΄Π° градусов Π² ΠΏΠΈ Ρ€Π°Π΄ΠΈΠ°Π½, Π½ΡƒΠΆΠ½ΠΎ ΠΈΠΌΠ΅ΡŽΡ‰ΠΈΠ΅ΡΡ Ρƒ нас градусы Ρ€Π°Π·Π΄Π΅Π»ΠΈΡ‚ΡŒ Π½Π° 180 градусов (это 1 ΠΏΠΈ Ρ€Π°Π΄ΠΈΠ°Π½). Π£ нас получаСтся 9/180 = 1/20

ΠžΡ‚Π²Π΅Ρ‚: 9 градусов = 1/20 ΠΏΠΈ.

Бинус это Π²Π²Π΅Ρ€Ρ… ΠΈΠ»ΠΈ Π² сторону — синус — это Π²Π²Π΅Ρ€Ρ…, Π² сторону — это косинус.

ΠšΠΎΠΌΠΌΠ΅Π½Ρ‚Π°Ρ€ΠΈΠΈ ΠΊ этой ΡΡ‚Π°Ρ‚ΡŒΠ΅ Π·Π°ΠΏΡ€Π΅Ρ‰Π΅Π½Ρ‹. Из-Π·Π° ΠΎΠ³Ρ€ΠΎΠΌΠ½ΠΎΠ³ΠΎ ΠΈΡ… количСства ΠΌΠΎΠΈ ΠΎΡ‚Π²Π΅Ρ‚Ρ‹ Π½Π° ваши вопросы ΠΎ тригономСтричСском ΠΊΡ€ΡƒΠ³Π΅ ΡƒΠΆΠ΅ Π½Π΅ ΠΏΡƒΠ±Π»ΠΈΠΊΡƒΡŽΡ‚ΡΡ. Вопросы ΠΌΠΎΠΆΠ΅Ρ‚Π΅ Π·Π°Π΄Π°Π²Π°Ρ‚ΡŒ Π² коммСнтариях ΠΊ Π΄Ρ€ΡƒΠ³ΠΈΠΌ страницам. ΠŸΠΎΡΡ‚Π°Ρ€Π°ΡŽΡΡŒ Ρ€Π΅ΡˆΠΈΡ‚ΡŒ ΠΏΡ€ΠΎΠ±Π»Π΅ΠΌΡƒ Π·Π° счСт удалСния части ΠΊΠΎΠΌΠΌΠ΅Π½Ρ‚Π°Ρ€ΠΈΠ΅Π², Ρ‚Π΅ΠΌ самым освобоТу мСсто для Π½ΠΎΠ²Ρ‹Ρ….

ВригономСтричСскиС Ρ„ΡƒΠ½ΠΊΡ†ΠΈΠΈ ΡƒΠ³Π»Π° синус косинус тангСнс котангСнс основноС тригономСтричСскоС тоТдСство тригономСтричСский ΠΊΡ€ΡƒΠ³ числовая ΠΎΠΊΡ€ΡƒΠΆΠ½ΠΎΡΡ‚ΡŒ

Π‘ΠΎΠ΄Π΅Ρ€ΠΆΠ°Π½ΠΈΠ΅

ΠžΠΏΡ€Π΅Π΄Π΅Π»Π΅Π½ΠΈΠ΅ тригономСтричСских Ρ„ΡƒΠ½ΠΊΡ†ΠΈΠΉ ΠΏΡ€ΠΎΠΈΠ·Π²ΠΎΠ»ΡŒΠ½ΠΎΠ³ΠΎ ΡƒΠ³Π»Π°

Рассмотрим ΠΎΠΊΡ€ΡƒΠΆΠ½ΠΎΡΡ‚ΡŒ радиуса Β  R с Ρ†Π΅Π½Ρ‚Ρ€ΠΎΠΌ Π² Π½Π°Ρ‡Π°Π»Π΅ ΠΏΡ€ΡΠΌΠΎΡƒΠ³ΠΎΠ»ΡŒΠ½ΠΎΠΉ систСмой ΠΊΠΎΠΎΡ€Π΄ΠΈΠ½Π°Ρ‚ Oxy.

Рис.1

ΠŸΠΎΠ»ΠΎΠΆΠΈΡ‚Π΅Π»ΡŒΠ½Ρ‹ΠΌ считаСтся ΡƒΠ³ΠΎΠ» NOM, сторона OM ΠΊΠΎΡ‚ΠΎΡ€ΠΎΠ³ΠΎ ΠΏΠΎΠ»ΡƒΡ‡Π΅Π½Π° ΠΈΠ· ΠΏΠΎΠ»ΠΎΠΆΠΈΡ‚Π΅Π»ΡŒΠ½ΠΎΠΉ полуоси Ox Π² Ρ€Π΅Π·ΡƒΠ»ΡŒΡ‚Π°Ρ‚Π΅ ΠΏΠΎΠ²ΠΎΡ€ΠΎΡ‚Π°, осущСствляСмого Π² Π½Π°ΠΏΡ€Π°Π²Π»Π΅Π½ΠΈΠΈ двиТСния ΠΏΡ€ΠΎΡ‚ΠΈΠ² часовой стрСлки (рис.1).

Рис.2

ΠžΡ‚Ρ€ΠΈΡ†Π°Ρ‚Π΅Π»ΡŒΠ½Ρ‹ΠΌ считаСтся ΡƒΠ³ΠΎΠ» NOM, сторона OM ΠΊΠΎΡ‚ΠΎΡ€ΠΎΠ³ΠΎ ΠΏΠΎΠ»ΡƒΡ‡Π΅Π½Π° ΠΈΠ· ΠΏΠΎΠ»ΠΎΠΆΠΈΡ‚Π΅Π»ΡŒΠ½ΠΎΠΉ полуоси Ox Π² Ρ€Π΅Π·ΡƒΠ»ΡŒΡ‚Π°Ρ‚Π΅ ΠΏΠΎΠ²ΠΎΡ€ΠΎΡ‚Π°, осущСствляСмого Π² Π½Π°ΠΏΡ€Π°Π²Π»Π΅Π½ΠΈΠΈ, ΡΠΎΠ²ΠΏΠ°Π΄Π°ΡŽΡ‰Π΅ΠΌ с Π½Π°ΠΏΡ€Π°Π²Π»Π΅Π½ΠΈΠ΅ΠΌ двиТСния часовой стрСлки (рис. 2).

Если для ΠΊΠΎΠΎΡ€Π΄ΠΈΠ½Π°Ρ‚ Ρ‚ΠΎΡ‡ΠΊΠΈ Β  M0 , Π»Π΅ΠΆΠ°Ρ‰Π΅ΠΉ Π½Π° окруТности радиуса R с Ρ†Π΅Π½Ρ‚Ρ€ΠΎΠΌ Π² Π½Π°Ρ‡Π°Π»Π΅ ΠΊΠΎΠΎΡ€Π΄ΠΈΠ½Π°Ρ‚ O (рис. 3),

Рис.3

ввСсти ΠΎΠ±ΠΎΠ·Π½Π°Ρ‡Π΅Π½ΠΈΠ΅

M0 = ( x0 ; y0 ),

Ρ‚ΠΎ, Π² силу Ρ‚Π΅ΠΎΡ€Π΅ΠΌΡ‹ ΠŸΠΈΡ„Π°Π³ΠΎΡ€Π°, Π±ΡƒΠ΄Π΅Ρ‚ справСдливо равСнство:

x02 + y02 = R2,

ΠΈ ΠΌΠΎΠΆΠ½ΠΎ ΡΡ„ΠΎΡ€ΠΌΡƒΠ»ΠΈΡ€ΠΎΠ²Π°Ρ‚ΡŒ ΡΠ»Π΅Π΄ΡƒΡŽΡ‰Π΅Π΅ ΠΎΠ±Ρ‰Π΅Π΅ ΠΎΠΏΡ€Π΅Π΄Π΅Π»Π΅Π½ΠΈΠ΅ тригономСтричСских Ρ„ΡƒΠ½ΠΊΡ†ΠΈΠΉ ΠΏΡ€ΠΎΠΈΠ·Π²ΠΎΠ»ΡŒΠ½ΠΎΠ³ΠΎ ΡƒΠ³Π»Π°.

Бинусом, косинусом, тангСнсом ΠΈ котангСнсом ΠΏΡ€ΠΎΠΈΠ·Π²ΠΎΠ»ΡŒΠ½ΠΎΠ³ΠΎ ΡƒΠ³Π»Π° Ξ± Π½Π°Π·Ρ‹Π²Π°ΡŽΡ‚ числа, опрСдСляСмыС ΠΏΠΎ Ρ„ΠΎΡ€ΠΌΡƒΠ»Π°ΠΌ:

Π—ΠΠœΠ•Π§ΠΠΠ˜Π• 1. ΠžΡ‚ΠΌΠ΅Ρ‚ΠΈΠΌ ΡΠ»Π΅Π΄ΡƒΡŽΡ‰Π΅Π΅ Π²Π°ΠΆΠ½ΠΎΠ΅ свойство тригономСтричСских Ρ„ΡƒΠ½ΠΊΡ†ΠΈΠΉ синуса ΠΈ косинуса ΠΏΡ€ΠΎΠΈΠ·Π²ΠΎΠ»ΡŒΠ½ΠΎΠ³ΠΎ ΡƒΠ³Π»Π°:

Π—ΠΠœΠ•Π§ΠΠΠ˜Π• 2. ΠžΠΏΡ€Π΅Π΄Π΅Π»Π΅Π½ΠΈΠ΅ тригономСтричСских Ρ„ΡƒΠ½ΠΊΡ†ΠΈΠΉ ΠΏΡ€ΠΎΠΈΠ·Π²ΠΎΠ»ΡŒΠ½ΠΎΠ³ΠΎ ΡƒΠ³Π»Π° являСтся СстСствСнным ΠΎΠ±ΠΎΠ±Ρ‰Π΅Π½ΠΈΠ΅ΠΌ опрСдСлСния тригономСтричСских Ρ„ΡƒΠ½ΠΊΡ†ΠΈΠΉ острого ΡƒΠ³Π»Π°, Π΄Π°Π½Π½ΠΎΠ³ΠΎ Π² Ρ€Π°Π·Π΄Π΅Π»Π΅ справочника «Π’ригономСтричСскиС Ρ„ΡƒΠ½ΠΊΡ†ΠΈΠΈ острого ΡƒΠ³Π»Π°».

ОсновноС тригономСтричСскоС тоТдСство. ВригономСтричСский ΠΊΡ€ΡƒΠ³

Рассмотрим ΠΎΠΊΡ€ΡƒΠΆΠ½ΠΎΡΡ‚ΡŒ радиуса 1 с Ρ†Π΅Π½Ρ‚Ρ€ΠΎΠΌ Π² Π½Π°Ρ‡Π°Π»Π΅ ΠΊΠΎΠΎΡ€Π΄ΠΈΠ½Π°Ρ‚. Если для ΠΊΠΎΠΎΡ€Π΄ΠΈΠ½Π°Ρ‚ Ρ‚ΠΎΡ‡ΠΊΠΈ Β  M1 (рис. 4), Π»Π΅ΠΆΠ°Ρ‰Π΅ΠΉ Π½Π° этой окруТности,

Рис.4

ввСсти ΠΎΠ±ΠΎΠ·Π½Π°Ρ‡Π΅Π½ΠΈΠ΅

M1 = ( x1 ; y1 ) ,

Ρ‚ΠΎ, Π² силу Ρ‚Π΅ΠΎΡ€Π΅ΠΌΡ‹ ΠŸΠΈΡ„Π°Π³ΠΎΡ€Π°, Π±ΡƒΠ΄Π΅Ρ‚ справСдливо равСнство

x12 + y12 = 1 ,

Π° синус, косинус, тангСнс ΠΈ котангСнс ΡƒΠ³Π»Π° Ξ± Π±ΡƒΠ΄ΡƒΡ‚ Π²Ρ‹Ρ‡ΠΈΡΠ»ΡΡ‚ΡŒΡΡ ΠΏΠΎ Ρ„ΠΎΡ€ΠΌΡƒΠ»Π°ΠΌ

Из этих Ρ„ΠΎΡ€ΠΌΡƒΠ», Π² частности, Π²Ρ‹Ρ‚Π΅ΠΊΠ°Π΅Ρ‚ основноС тригономСтричСскоС тоТдСство:

sin2Ξ± + cos2Ξ± = 1 .

Π’Π°ΠΊΠΈΠΌ ΠΎΠ±Ρ€Π°Π·ΠΎΠΌ, основноС тригономСтричСскоС тоТдСство являСтся Ρ‚Π΅ΠΎΡ€Π΅ΠΌΠΎΠΉ ΠŸΠΈΡ„Π°Π³ΠΎΡ€Π°, сформулированной с ΠΏΠΎΠΌΠΎΡ‰ΡŒΡŽ тригономСтричСских Ρ„ΡƒΠ½ΠΊΡ†ΠΈΠΉ.

ΠžΠΊΡ€ΡƒΠΆΠ½ΠΎΡΡ‚ΡŒ радиуса 1, ΠΈΠ·ΠΎΠ±Ρ€Π°ΠΆΠ΅Π½Π½ΡƒΡŽ Π½Π° рисункС 4, Π½Π°Π·Ρ‹Π²Π°ΡŽΡ‚ тригономСтричСским ΠΊΡ€ΡƒΠ³ΠΎΠΌ ΠΈΠ»ΠΈ числовой ΠΎΠΊΡ€ΡƒΠΆΠ½ΠΎΡΡ‚ΡŒΡŽ.

ΠŸΡ€ΠΎ Ρ‚Ρ€ΠΈΠ³ΠΎΠ½ΠΎΠΌΠ΅Ρ‚Ρ€ΠΈΡŽ ⋆zagalina.ru

ВригономСтричСский ΠΊΡ€ΡƒΠ³β†’
Π‘ΠΎΠ΄Π΅Ρ€ΠΆΠ°Π½ΠΈΠ΅:

Β 

ΠŸΠΎΠ³ΠΎΠ²ΠΎΡ€ΠΈΠΌ ΠΎ Ρ‚Ρ€ΠΈΠ³ΠΎΠ½ΠΎΠΌΠ΅Ρ‚Ρ€ΠΈΠΈ

Вопрос:Β»Π§Ρ‚ΠΎ Ρ‚Π°ΠΊΠΎΠ΅ тригономСтрия?Β» Π‘Ρ€Π°Π·Ρƒ ΠΏΡ€Π΅Π΄ΡΡ‚Π°Π²Π»ΡΡŽ сСбС ΠΎΡ‚Π²Π΅Ρ‚: «Ну…., это ΠΊΠΎΠ³Π΄Π° синус ΠΈΠ»ΠΈ косинус…» «А Ρ‡Ρ‚ΠΎ Ρ‚Π°ΠΊΠΎΠ΅ синус ΠΈ косинус?Β» β€” «Ну…, ΠΎΡ‚Π½ΠΎΡˆΠ΅Π½ΠΈΠ΅ ΠΊΠ°Ρ‚Π΅Ρ‚ΠΎΠ² ΠΊ гипотСнузС…» Β«Π’ΠΎ Π΅ΡΡ‚ΡŒ β€” гСомСтрия?Β» β€” Β«???…» НСт, ΠΊΠΎΠ½Π΅Ρ‡Π½ΠΎ Π½Π΅ гСомСтрия! ΠŸΡ€Π΅Π΄ΡΡ‚Π°Π²ΡŒΡ‚Π΅ сСбС ΡƒΠ³ΠΎΠ» Π² 1000Β° . ΠŸΡ€Π΅Π΄ΡΡ‚Π°Π²ΠΈΠ»ΠΈ? НСт!.. А ΠΎΡ‚Ρ€ΠΈΡ†Π°Ρ‚Π΅Π»ΡŒΠ½Ρ‹ΠΉ ΡƒΠ³ΠΎΠ»? НСт Ρ‚Π°ΠΊΠΈΡ… ΡƒΠ³Π»ΠΎΠ²!!! На 90Β° заканчиваСтся ΠΏΡ€ΡΠΌΠΎΡƒΠ³ΠΎΠ»ΡŒΠ½Ρ‹Π΅ Ρ‚Ρ€Π΅ΡƒΠ³ΠΎΠ»ΡŒΠ½ΠΈΠΊΠΈ, ΠΊΠΎΡ‚ΠΎΡ€Ρ‹Π΅ ΠΈ Π΄Π°Π»ΠΈ ΠΎΠΏΡ€Π΅Π΄Π΅Π»Π΅Π½ΠΈΠ΅ для синусов, косинусов, тангСнсов ΠΈ котангСнсов. На 180Β° заканчиваСтся Ρ‚Ρ€Π΅ΡƒΠ³ΠΎΠ»ΡŒΠ½ΠΈΠΊ, Π½Π° 360Β° β€” планимСтрия. Π§Ρ‚ΠΎ ΠΆΠ΅ Ρ‚ΠΎΠ³Π΄Π° Ρ‚Π°ΠΊΠΎΠ΅ β€” тригономСтрия? РазбСрСмся… НачнСм с Π΄Π²ΡƒΡ… основных понятий β€” синуса ΠΈ косинуса. Π§Ρ‚ΠΎ Π²Ρ‹ ΠΎ Π½ΠΈΡ… Π·Π½Π°Π΅Ρ‚Π΅ ΠΊΡ€ΠΎΠΌΠ΅ Ρ‚ΠΎΠ³ΠΎ, Ρ‡Ρ‚ΠΎ это ΠΎΡ‚Π½ΠΎΡˆΠ΅Π½ΠΈΡ ΠΊΠ°Ρ‚Π΅Ρ‚ΠΎΠ² ΠΈ Π³ΠΈΠΏΠΎΡ‚Π΅Π½ΡƒΠ·Ρ‹? Вспомнили? сумма ΠΊΠ²Π°Π΄Ρ€Π°Ρ‚ΠΎΠ² синуса ΠΈ косинуса Ρ€Π°Π²Π½Ρ‹ Π΅Π΄ΠΈΠ½ΠΈΡ†Π΅.

SinΒ²Ξ± + CosΒ²Ξ± = 1

Β Β Β Β Β Β  А ΠΏΠΎΠΌΠ½ΠΈΡ‚Π΅, ΠΊΠ°ΠΊ выглядит ΡƒΡ€Π°Π²Π½Π΅Π½ΠΈΠ΅ окруТности с Ρ†Π΅Π½Ρ‚Ρ€ΠΎΠΌ Π² Π½Π°Ρ‡Π°Π»Π΅ ΠΊΠΎΠΎΡ€Π΄ΠΈΠ½Π°Ρ‚, радиус ΠΊΠΎΡ‚ΠΎΡ€ΠΎΠΉ R?

xΒ² + yΒ² = RΒ²

Β Β Β Β Β Β  А Ссли радиус R = 1, Ρ‚ΠΎ это ΡƒΡ€Π°Π²Π½Π΅Π½ΠΈΠ΅ Π±ΡƒΠ΄Π΅Ρ‚ Π²Ρ‹Π³Π»ΡΠ΄Π΅Ρ‚ΡŒΒ  xΒ² + yΒ² = 1. Π‘ΠΎΠ³Π»Π°ΡΠΈΡ‚Π΅ΡΡŒ, это ΠΎΡ‡Π΅Π½ΡŒ ΠΏΠΎΡ…ΠΎΠΆΠ΅ Π½Π° основноС тригономСтричСскоС ΡƒΡ€Π°Π²Π½Π΅Π½ΠΈΠ΅! А Ρ‡Ρ‚ΠΎ особСнно Π»ΡŽΠ±ΡΡ‚ школьники, Ρ€Π΅ΡˆΠ°Ρ уравнСния с Π½Π΅ΡƒΠ΄ΠΎΠ±Π½Ρ‹ΠΌΠΈ выраТСниями? Π—Π°ΠΌΠ΅Π½ΡΡ‚ΡŒ ΠΈΡ…, Π΄Π΅Π»Π°Ρ‚ΡŒ Ρ€Π°Π²Π½ΠΎΡΠΈΠ»ΡŒΠ½ΡƒΡŽ Π·Π°ΠΌΠ΅Π½Ρƒ Π½Π° ΠΊΠ°ΠΊΡƒΡŽ-Π½ΠΈΠ±ΡƒΠ΄ΡŒ Π±ΡƒΠΊΠ²Ρƒ. Π‘Π΄Π΅Π»Π°Π΅ΠΌ Ρ‚ΠΎΠΆΠ΅ Π·Π°ΠΌΠ΅Π½Ρƒ Π΄Π²ΡƒΡ… нСизвСстных xΒ² ΠΈ yΒ² Π½Π°Β  SinΒ²Ξ± ΠΈ CosΒ²Ξ±, ΠΏΠΎΠ»ΡƒΡ‡ΠΈΠΌ Ρ€Π°Π²Π½ΠΎΡΠΈΠ»ΡŒΠ½ΠΎΠ΅ ΡƒΡ€Π°Π²Π½Π΅Π½ΠΈΠ΅. ΠŸΠΎΠ»ΡƒΡ‡Π°Π΅Ρ‚ΡΡ, Ρ‡Ρ‚ΠΎ тригономСтрия β€” это Ρ€Π°Π·Π΄Π΅Π» ΠΌΠ°Ρ‚Π΅ΠΌΠ°Ρ‚ΠΈΠΊΠΈ, ΠΏΠΎΠ·Π²ΠΎΠ»ΡΡŽΡ‰ΠΈΠΉ Ρ€Π΅ΡˆΠ°Ρ‚ΡŒ Π½Π΅ΠΊΠΎΡ‚ΠΎΡ€Ρ‹Π΅ уравнСния Π³ΠΎΡ€Π°Π·Π΄ΠΎ ΠΏΡ€ΠΎΡ‰Π΅, Ссли ΠΈΡ… нСизвСстныС ΡƒΠ΄ΠΎΠ²Π»Π΅Ρ‚Π²ΠΎΡ€ΡΡŽΡ‚ условиям:

Π’ΠΎΠ³Π΄Π° Π½Π΅ΠΈΠ·Π²Π΅ΡΡ‚Π½ΡƒΡŽ ΠΌΠΎΠΆΠ½ΠΎ Π·Π°ΠΌΠ΅Π½ΠΈΡ‚ΡŒ Π½Π° синус ΠΈΠ»ΠΈ косинус.

Β Β Β Β Β Β  А ΠΏΡ€ΠΎΡ‰Π΅ ΠΏΠΎΡ‚ΠΎΠΌΡƒ, Ρ‡Ρ‚ΠΎ Π² Ρ‚Ρ€ΠΈΠ³ΠΎΠ½ΠΎΠΌΠ΅Ρ‚Ρ€ΠΈΠΈ ΠΏΠΎΡΠ²Π»ΡΡŽΡ‚ΡΡ Π΄ΠΎΠΏΠΎΠ»Π½ΠΈΡ‚Π΅Π»ΡŒΠ½Ρ‹Π΅ Ρ„ΠΎΡ€ΠΌΡƒΠ»Ρ‹ для расчСтов. НСизвСстными Π² Ρ‚Ρ€ΠΈΠ³ΠΎΠ½ΠΎΠΌΠ΅Ρ‚Ρ€ΠΈΠΈ становятся ΡƒΠΆΠ΅ Π½Π΅ ΠΎΡ‚Π΄Π΅Π»ΡŒΠ½Ρ‹Π΅ Π±ΡƒΠΊΠ²Ρ‹, Π° тригономСтричСскиС Ρ„ΡƒΠ½ΠΊΡ†ΠΈΠΈ: sin x, Β Β  cos x,Β  Β  tg x,Β Β Β  ctg x,

Π³Π΄Π΅ Β ,Β  Π°Β Β 

Β Β Β Β Β Β Β  Π’ΠΎΡ‚ Ρ‚Π°ΠΊ! Π­Ρ‚ΠΎ тригономСтрия!

Β 

ВригономСтричСскиС Ρ„ΡƒΠ½ΠΊΡ†ΠΈΠΈ

Β Β Β Β Β Β Β  ΠŸΠΎΡ‡Π΅ΠΌΡƒ Ρ„ΡƒΠ½ΠΊΡ†ΠΈΠΈ? Напомню, Ρ‡Ρ‚ΠΎ ΠΆΠ΅ Ρ‚Π°ΠΊΠΎΠ΅ β€” Ρ„ΡƒΠ½ΠΊΡ†ΠΈΠΈ. Ѐункция β€” это ΠΎΡ‚Π½ΠΎΡˆΠ΅Π½ΠΈΠ΅ ΠΌΠ΅ΠΆΠ΄Ρƒ двумя зависимыми Π΄Ρ€ΡƒΠ³ ΠΎΡ‚ Π΄Ρ€ΡƒΠ³Π° Π²Π΅Π»ΠΈΡ‡ΠΈΠ½Π°ΠΌΠΈ, ΠΊΠΎΡ‚ΠΎΡ€ΠΎΠ΅ записываСтся Π² Π²ΠΈΠ΄Π΅ матСматичСской Ρ„ΠΎΡ€ΠΌΡƒΠ»Ρ‹, ΠΎΠΏΡ€Π΅Π΄Π΅Π»ΡΡŽΡ‰Π΅ΠΉ эту Π·Π°Π²ΠΈΡΠΈΠΌΠΎΡΡ‚ΡŒ. ΠŸΡ€ΠΈ этом ΠΈΠ·ΠΌΠ΅Π½Π΅Π½ΠΈΠ΅ ΠΎΠ΄Π½ΠΎΠΉ Π²Π΅Π»ΠΈΡ‡ΠΈΠ½Ρ‹ Π²Π΅Π΄Π΅Ρ‚ ΠΊ измСнСнию Π΄Ρ€ΡƒΠ³ΠΎΠΉ Π²Π΅Π»ΠΈΡ‡ΠΈΠ½Ρ‹. ΠŸΠΎΠ΄Ρ€ΠΎΠ±Π½Π΅Π΅β†’

Β Β Β Β Β Β Β Β  Π’ случаС тригономСтричСских Ρ„ΡƒΠ½ΠΊΡ†ΠΈΠΉ, Π²Π΅Π»ΠΈΡ‡ΠΈΠ½Π° Ρƒ мСняСтся с ΠΈΠ·ΠΌΠ΅Π½Π΅Π½ΠΈΠ΅ΠΌ Π²Π΅Π»ΠΈΡ‡ΠΈΠ½Ρ‹ Ρ…, Ρ„ΠΎΡ€ΠΌΡƒΠ»Π° β€” это сами синус, косинус, тангСнс ΠΈ котангСнс.

Β Β Β Β Β Β Β Β  Π“Ρ€Π°Ρ„ΠΈΠΊΠΈ этих Ρ„ΡƒΠ½ΠΊΡ†ΠΈΠΉ ΠΏΡ€ΠΈΠ²Π΅Π΄Π΅Π½Ρ‹ Π½ΠΈΠΆΠ΅:

функция y(x) = sin x
функция y(x) = cos x
функция y(x) = tg x
функция y(x) = ctg x

Β Β Β Β Β Β Β Β  ΠŸΠΎΠ»ΡƒΡ‡Π°Π΅Ρ‚ΡΡ, Ρ‡Ρ‚ΠΎ синус, косинус, тангСнс ΠΈ котангСнс β€” это матСматичСскиС выраТСния, ΡΠ²ΡΠ·Ρ‹Π²Π°ΡŽΡ‰ΠΈΠ΅ Π΄Π²Π° нСизвСстных. А Π·Π½Π°Ρ‡ΠΈΡ‚, ΠΌΡ‹ ΠΌΠΎΠΆΠ΅ΠΌ ΠΈΡ…, эти нСизвСстныС, Π½Π°Ρ…ΠΎΠ΄ΠΈΡ‚ΡŒ. А ΠΏΠΎΠΌΠΎΠΆΠ΅Ρ‚ Π½Π°ΠΌ Π² этом вСликолСпная классичСская Β«ΡˆΠΏΠ°Ρ€Π³Π°Π»ΠΊΠ°Β» β€” тригономСтричСский ΠΊΡ€ΡƒΠ³.

Β Β Β Β Β Β Β Β  Но ΠΏΡ€Π΅ΠΆΠ΄Π΅, Ρ‡Π΅ΠΌ ΠΌΡ‹ ΠΏΠ΅Ρ€Π΅ΠΉΠ΄Π΅ΠΌ ΠΊ тригономСтричСскому ΠΊΡ€ΡƒΠ³Ρƒ, опрСдСлимся со значСниями Π°Ρ€Π³ΡƒΠΌΠ΅Π½Ρ‚Π° тригономСтричСской Ρ„ΡƒΠ½ΠΊΡ†ΠΈΠΈ Ρ…, ΠΊΠΎΡ‚ΠΎΡ€Ρ‹ΠΉ ΠΌΠΎΠΆΠ΅Ρ‚ ΠΌΠ΅Π½ΡΡ‚ΡŒΡΡ ΠΎΡ‚ минус бСсконСчности Π΄ΠΎ плюс бСсконСчности. Β 

x ∈ ( β€” ∞; + ∞).

Π’ этом случаС цСлСсообразно ΠΎΡ‚Π²Π»Π΅Ρ‡ΡŒΡΡ ΠΎΡ‚ градусной ΠΌΠ΅Ρ€Ρ‹ ΡƒΠ³Π»ΠΎΠ² ΠΈ Π²ΡΠΏΠΎΠΌΠ½ΠΈΡ‚ΡŒ, Ρ‡Ρ‚ΠΎ ΡƒΠ³Π»Ρ‹ ΠΈΠ·ΠΌΠ΅Ρ€ΡΡŽΡ‚ΡΡ Ρ‚Π°ΠΊΠΆΠ΅ Ρ€Π°Π΄ΠΈΠ°Π½Π°ΠΌΠΈ – просто числами, опрСдСляСмыми Ρ‡Π΅Ρ€Π΅Π· всСм извСстноС число Ο€, ΡΠΎΠΎΡ‚Π²Π΅Ρ‚ΡΡ‚Π²ΡƒΡŽΡ‰Π΅Π΅ 180Β°-Π½ΠΎΠΌΡƒ ΡƒΠ³Π»Ρƒ, ΠΈ Ο€ = 3,141582….. И Ρ‚ΠΎΠ³Π΄Π° x β€” это ΡƒΠΆΠ΅ Π½Π΅ ΡƒΠ³Π»Ρ‹, Π° числа .

Β 

Π‘Ρ‚Ρ€ΠΎΠΈΠΌ тригономСтричСский ΠΊΡ€ΡƒΠ³

Π§Π΅Ρ€Ρ‚ΠΈΠΌ Π½Π° плоскости Π΄Π΅ΠΊΠ°Ρ€Ρ‚ΠΎΠ²Ρƒ систСму ΠΊΠΎΠΎΡ€Π΄ΠΈΠ½Π°Ρ‚, Π° Π² Π½Π΅ΠΉ графичСски ΠΈΠ·ΠΎΠ±Ρ€Π°ΠΆΠ°Π΅ΠΌ ΡƒΡ€Π°Π²Π½Π΅Π½ΠΈΠ΅ :

Ρ…Β² + ΡƒΒ² = 1,

Ρ‚ΠΎ Π΅ΡΡ‚ΡŒ, Ρ‡Π΅Ρ€Ρ‚ΠΈΠΌ ΠΊΡ€ΡƒΠ³ радиусом, Ρ€Π°Π²Π½Ρ‹ΠΌ 1.

Β Β Β Β Β Β  Рассмотрим Ρ‚ΠΎΡ‡ΠΊΡƒ М(Ρ…ΠΎ, ΡƒΠΎ), ΠΏΡ€ΠΈΠ½Π°Π΄Π»Π΅ΠΆΠ°Ρ‰ΡƒΡŽ ΠΏΠ΅Ρ€Π²ΠΎΠΉ Ρ‡Π΅Ρ‚Π²Π΅Ρ€Ρ‚ΠΈ окруТности, ΠΈ Ρ‚Ρ€Π΅ΡƒΠ³ΠΎΠ»ΡŒΠ½ΠΈΠΊ, Π³Π΄Π΅ Ρ…ΠΎ, ΡƒΠΎ ΠΊΠΎΠΎΡ€Π΄ΠΈΠ½Π°Ρ‚Ρ‹ Ρ‚ΠΎΡ‡ΠΊΠΈ, Π° 0М – радиус окруТности, Ρ€Π°Π²Π½Ρ‹ΠΉ 1. Β ΠŸΠΎΠ»ΡƒΡ‡Π°Π΅Ρ‚ΡΡ, Ρ‡Ρ‚ΠΎ ΠΎΡ‚Π½ΠΎΡˆΠ΅Π½ΠΈΠ΅ ΡƒΠΎ Β ΠΊ радиусу Π΅ΡΡ‚ΡŒ Π½ΠΈΡ‡Ρ‚ΠΎ ΠΈΠ½ΠΎΠ΅, ΠΊΠ°ΠΊ синус ΡƒΠ³Π»Π° Ξ±, Π° ΠΎΡ‚Π½ΠΎΡˆΠ΅Π½ΠΈΠ΅ Ρ…ΠΎ ΠΊ радиусу – косинус ΡƒΠ³Π»Π° Ξ±. А это Π·Π½Π°Ρ‡ΠΈΡ‚, Ρ‡Ρ‚ΠΎ ΠΊΠΎΠΎΡ€Π΄ΠΈΠ½Π°Ρ‚Ρ‹ любой Ρ‚ΠΎΡ‡ΠΊΠΈ Π½Π° окруТности радиусом, Ρ€Π°Π²Π½Ρ‹ΠΌ 1 – это синус ΠΈ косинус ΡƒΠ³Π»Π°, ΠΎΠ±Ρ€Π°Π·ΠΎΠ²Π°Π½Π½ΠΎΠ³ΠΎ осью 0Ρ… ΠΈ радиус-Π²Π΅ΠΊΡ‚ΠΎΡ€ΠΎΠΌ Ρ‚ΠΎΡ‡ΠΊΠΈ М, Ρ‚ΠΎ Π΅ΡΡ‚ΡŒ ΠΎΡ‚Ρ€Π΅Π·ΠΊΠΎΠΌ ОМ.

Β Β Β Β Β Β  Π’ΠΎΠ³Π΄Π° наш ΠΊΡ€ΡƒΠ³ ΠΌΠΎΠΆΠ½ΠΎ ΠΏΡ€Π΅Π΄ΡΡ‚Π°Π²ΠΈΡ‚ΡŒ ΡΠ»Π΅Π΄ΡƒΡŽΡ‰ΠΈΠΌΒ  ΠΎΠ±Ρ€Π°Π·ΠΎΠΌ:β†’

Π’Ρ‹Ρ…ΠΎΠ΄ΠΈΡ‚, Ρ‡Ρ‚ΠΎ ΠΏΠΎ оси ΠžΡƒ ΠΏΠΎΠ»ΡƒΡ‡Π°Π΅ΠΌ значСния синусов Π°Ρ€Π³ΡƒΠΌΠ΅Π½Ρ‚Π°, Π½Π°Π·ΠΎΠ²Π΅ΠΌ Π΅Π΅ «ось синусов», Π° ΠΏΠΎ оси ΠžΡ… β€” значСния косинусов, ΠΏΠΎΠ»ΡƒΡ‡Π°Π΅ΠΌ ΠΊΠ°ΠΊ Π±Ρ‹ «ось косинусов». ΠŸΡ€ΠΈ этом, разумССтся, значСния синусов ΠΈ косинусов Π½Π΅ выходят Π·Π° ΠΏΡ€Π΅Π΄Π΅Π»Ρ‹ ΠΊΡ€ΡƒΠ³Π°. ΠŸΡ€ΠΎΡΡ‚ΠΎ, ΠΏΠΎΡ‚ΠΎΠΌΡƒ Ρ‡Ρ‚ΠΎ Π½Π΅ ΠΌΠΎΠ³ΡƒΡ‚ Π² силу своСй ограничСнности! Ну, Π° ΠΌΡ‹ с Π²Π°ΠΌΠΈ ΠΈΠΌΠ΅Π΅ΠΌ Π²ΠΎΠ·ΠΌΠΎΠΆΠ½ΠΎΡΡ‚ΡŒ опрСдСлятся с синусами ΠΈ косинусами ΡƒΠΆΠ΅ Π½Π΅ ΡƒΠ³Π»ΠΎΠ², Π° Π°Ρ€Π³ΡƒΠΌΠ΅Π½Ρ‚Π° Ρ… ∈ ( β€” ∞; + ∞). ΠŸΠΎΡ‡Π΅ΠΌΡƒ? А ΠΏΠΎΡ‚ΠΎΠΌΡƒ Ρ‡Ρ‚ΠΎ тригономСтричСскиС Ρ„ΡƒΠ½ΠΊΡ†ΠΈΠΈ β€” пСриодичСскиС, Ρ‚.Π΅. просто «вСртятся» ΠΏΠΎ ΠΊΡ€ΡƒΠ³Ρƒ. ΠŸΡ€ΠΈ этом значСния синусов ΠΈ косинусов ΠΎΠΏΡ€Π΅Π΄Π΅Π»ΡΡŽΡ‚ΡΡ значСниями ΠΊΠΎΠΎΡ€Π΄ΠΈΠ½Π°Ρ‚ Ρ‚ΠΎΡ‡Π΅ΠΊ Π½Π° окруТности со всСми ΠΈΡ… Π·Π½Π°ΠΊΠ°ΠΌΠΈ. НапримСр, согласно рисунку, ΠΊΠΎΠΎΡ€Π΄ΠΈΠ½Π°Ρ‚Π° Ρƒ Ρ‚ΠΎΡ‡ΠΊΠΈ Π½Π° окруТности, ΡΠΎΠΎΡ‚Π²Π΅Ρ‚ΡΡ‚Π²ΡƒΡŽΡ‰Π΅ΠΉ ΡƒΠ³Π»ΡƒΒ , опрСдСляСт  , Π° ΠΊΠΎΠΎΡ€Π΄ΠΈΠ½Π°Ρ‚Π° Ρ… опрСдСляСт  .

Β Β Β Β Β Β  Π‘ синусами ΠΈ косинусами Ρ€Π°Π·ΠΎΠ±Ρ€Π°Π»ΠΈΡΡŒ. А ΠΊΠ°ΠΊ ΠΆΠ΅ тангСнсы ΠΈ котангСнсы? Β  Β  Β  Π‘ Π½ΠΈΠΌΠΈ Ρ‚ΠΎΠΆΠ΅ Π½Π΅ Π²ΠΎΠ·Π½ΠΈΠΊΠ½Π΅Ρ‚ ΠΏΡ€ΠΎΠ±Π»Π΅ΠΌ. ВангСнс опрСдСляСтся:

Π’ΠΎΡ‚ ΠΈ ΠΏΡ€ΠΎΠ²ΠΎΠ΄ΠΈΠΌ «ось тангСнсов» Ρ‚Π°ΠΌ, Π³Π΄Π΅Β cos Ξ± Ρ€Π°Π²Π΅Π½ 1. Аналогично поступаСм ΠΈ с «осью котангСнсов» Π’ ΠΎΡ‚Π»ΠΈΡ‡ΠΈΠ΅ ΠΎΡ‚ синусов ΠΈ косинусов тангСнсам ΠΈ котангСнсам ΠΏΡ€ΡΠΌΡƒΡŽ ΠΏΡ€ΠΎΠ²ΠΎΠ΄ΠΈΠΌ Ρ‡Π΅Ρ€Π΅Π· Π½Π°Ρ‡Π°Π»ΠΎ ΠΊΠΎΠΎΡ€Π΄ΠΈΠ½Π°Ρ‚. Π’ΠΎ Π΅ΡΡ‚ΡŒ Π½Π° ΠΊΡ€ΡƒΠ³Π΅ Π΄Π²Π° ΡƒΠ³Π»Π° для тангСнса ΠΈ Π΄Π²Π° ΡƒΠ³Π»Π° для котангСнса. Оно ΠΈ понятно: ΠΏΠ΅Ρ€ΠΈΠΎΠ΄Β  Ρ„ΡƒΠ½ΠΊΡ†ΠΈΠΉ тангСнса ΠΈ котангСнса составляСт Ο€, Π° Π½Π΅ 2Ο€, ΠΊΠ°ΠΊ Ρƒ Ρ„ΡƒΠ½ΠΊΡ†ΠΈΠΉ синуса ΠΈ косинуса.

вся тригономСтрия Π½Π° ΠΎΠ΄Π½ΠΎΠΌ рисункС

ВригономСтричСский ΠΊΡ€ΡƒΠ³ β€” это самый простой способ Π½Π°Ρ‡Π°Ρ‚ΡŒ ΠΎΡΠ²Π°ΠΈΠ²Π°Ρ‚ΡŒ Ρ‚Ρ€ΠΈΠ³ΠΎΠ½ΠΎΠΌΠ΅Ρ‚Ρ€ΠΈΡŽ. Он Π»Π΅Π³ΠΊΠΎ запоминаСтся, ΠΈ Π½Π° Π½Ρ‘ΠΌ Π΅ΡΡ‚ΡŒ всё Π½Π΅ΠΎΠ±Ρ…ΠΎΠ΄ΠΈΠΌΠΎΠ΅. Он замСняСт дСсяток Ρ‚Π°Π±Π»ΠΈΡ†.

Бколько ΠΏΠΎΠ»Π΅Π·Π½ΠΎΠ³ΠΎ Π½Π° этом рисункС!

  1. ΠŸΠ΅Ρ€Π΅Π²ΠΎΠ΄ градусов Π² Ρ€Π°Π΄ΠΈΠ°Π½Ρ‹ ΠΈ Π½Π°ΠΎΠ±ΠΎΡ€ΠΎΡ‚. ΠŸΠΎΠ»Π½Ρ‹ΠΉ ΠΊΡ€ΡƒΠ³ содСрТит 360 градусов, ΠΈΠ»ΠΈ 2Ο€ Ρ€Π°Π΄ΠΈΠ°Π½.

  2. ЗначСния синусов ΠΈ косинусов основных ΡƒΠ³Π»ΠΎΠ². Помним, Ρ‡Ρ‚ΠΎ Π·Π½Π°Ρ‡Π΅Π½ΠΈΠ΅ косинуса ΡƒΠ³Π»Π° ΠΌΡ‹ Π½Π°Ρ…ΠΎΠ΄ΠΈΠΌ Π½Π° оси Π₯, Π° Π·Π½Π°Ρ‡Π΅Π½ΠΈΠ΅ синуса β€” Π½Π° оси Y.

  3. И синус, ΠΈ косинус ΠΏΡ€ΠΈΠ½ΠΈΠΌΠ°ΡŽΡ‚ значСния ΠΎΡ‚ βˆ’1 Π΄ΠΎ 1.

  1. Π—Π½Π°Ρ‡Π΅Π½ΠΈΠ΅ тангСнса ΡƒΠ³Π»Π° Ξ± Ρ‚ΠΎΠΆΠ΅ Π»Π΅Π³ΠΊΠΎ Π½Π°ΠΉΡ‚ΠΈ β€” ΠΏΠΎΠ΄Π΅Π»ΠΈΠ² sinΒ Ξ± Π½Π° cosΒ Ξ±. А чтобы Π½Π°ΠΉΡ‚ΠΈ котангСнс β€” Π½Π°ΠΎΠ±ΠΎΡ€ΠΎΡ‚, косинус Π΄Π΅Π»ΠΈΠΌ Π½Π° синус.

  2. Π—Π½Π°ΠΊΠΈ синуса, косинуса, тангСнса ΠΈ котангСнса.

  3. Бинус β€” функция нСчётная, косинус β€” чётная.

  4. ВригономСтричСский ΠΊΡ€ΡƒΠ³ ΠΏΠΎΠΌΠΎΠΆΠ΅Ρ‚ ΡƒΠ²ΠΈΠ΄Π΅Ρ‚ΡŒ, Ρ‡Ρ‚ΠΎ синус ΠΈ косинус β€” Ρ„ΡƒΠ½ΠΊΡ†ΠΈΠΈ пСриодичСскиС. ΠŸΠ΅Ρ€ΠΈΠΎΠ΄ Ρ€Π°Π²Π΅Π½ 2Ο€.

Если Π²Π°ΠΌ Ρ‡Ρ‚ΠΎ-Ρ‚ΠΎ нСпонятно β€” Ρ‡ΠΈΡ‚Π°ΠΉΡ‚Π΅ ΠΏΠΎΠ΄Ρ€ΠΎΠ±Π½Π΅Π΅:

Π’Π½Π΅ΡˆΠ½ΠΈΠΉ ΡƒΠ³ΠΎΠ» Ρ‚Ρ€Π΅ΡƒΠ³ΠΎΠ»ΡŒΠ½ΠΈΠΊΠ°. Бинус ΠΈ косинус внСшнСго ΡƒΠ³Π»Π°

Π’Β Π½Π΅ΠΊΠΎΡ‚ΠΎΡ€Ρ‹Ρ… Π·Π°Π΄Π°Ρ‡Π°Ρ… Π•Π“Π­ трСбуСтся Π½Π°ΠΉΡ‚ΠΈ синус, косинус ΠΈΠ»ΠΈ тангСнс внСшнСго ΡƒΠ³Π»Π° Ρ‚Ρ€Π΅ΡƒΠ³ΠΎΠ»ΡŒΠ½ΠΈΠΊΠ°. А что Ρ‚Π°ΠΊΠΎΠ΅ внСшний ΡƒΠ³ΠΎΠ» Ρ‚Ρ€Π΅ΡƒΠ³ΠΎΠ»ΡŒΠ½ΠΈΠΊΠ°?

Π”Π°Π²Π°ΠΉΡ‚Π΅ вспомним сначала, Ρ‡Ρ‚ΠΎ Ρ‚Π°ΠΊΠΎΠ΅ смСТныС ΡƒΠ³Π»Ρ‹. Π’ΠΎΡ‚ ΠΎΠ½ΠΈ, на рисункС. У смСТных ΡƒΠ³Π»ΠΎΠ² ΠΎΠ΄Π½Π° сторона общая, Π°Β Π΄Π²Π΅ Π΄Ρ€ΡƒΠ³ΠΈΠ΅ Π»Π΅ΠΆΠ°Ρ‚ Π½Π°Β ΠΎΠ΄Π½ΠΎΠΉ прямой. Π‘ΡƒΠΌΠΌΠ° смСТных ΡƒΠ³Π»ΠΎΠ² Ρ€Π°Π²Π½Π° 180Β°.

Π’ΠΎΠ·ΡŒΠΌΠ΅ΠΌ Ρ‚Ρ€Π΅ΡƒΠ³ΠΎΠ»ΡŒΠ½ΠΈΠΊ ΠΈΒ ΠΏΡ€ΠΎΠ΄ΠΎΠ»ΠΆΠΈΠΌ ΠΎΠ΄Π½Ρƒ ΠΈΠ·Β Π΅Π³ΠΎ сторон. Π’Π½Π΅ΡˆΠ½ΠΈΠΉ ΡƒΠ³ΠΎΠ» ΠΏΡ€ΠΈ Π²Π΅Ρ€ΡˆΠΈΠ½Π΅ А — это ΡƒΠ³ΠΎΠ», смСТный с углом А. Если угол А острый, то смСТный с ним ΡƒΠ³ΠΎΠ»Β β€” Ρ‚ΡƒΠΏΠΎΠΉ, ΠΈΒ Π½Π°ΠΎΠ±ΠΎΡ€ΠΎΡ‚.

ΠžΠ±Ρ€Π°Ρ‚ΠΈΡ‚Π΅ Π²Π½ΠΈΠΌΠ°Π½ΠΈΠ΅, Ρ‡Ρ‚ΠΎ:

sin (180Β°-Ξ±) = sin Ξ± cos (180Β°-Ξ±) = β€” cos Ξ± tgΒ (180Β°-Ξ±) = β€” tgΒ Ξ±

Π—Π°ΠΏΠΎΠΌΠ½ΠΈΡ‚Π΅ эти Π²Π°ΠΆΠ½Ρ‹Π΅ ΡΠΎΠΎΡ‚Π½ΠΎΡˆΠ΅Π½ΠΈΡ. БСйчас ΠΌΡ‹Β Π±Π΅Ρ€Π΅ΠΌ ΠΈΡ…Β Π±Π΅Π· Π΄ΠΎΠΊΠ°Π·Π°Ρ‚Π΅Π»ΡŒΡΡ‚Π². Π’Β Ρ€Π°Π·Π΄Π΅Π»Π΅ «ВригономСтрия», Π²Β Ρ‚Π΅ΠΌΠ΅ «ВригономСтричСский ΠΊΡ€ΡƒΠ³Β», мы вСрнСмся ΠΊΒ Π½ΠΈΠΌ.

Π›Π΅Π³ΠΊΠΎ Π΄ΠΎΠΊΠ°Π·Π°Ρ‚ΡŒ, Ρ‡Ρ‚ΠΎ внСшний ΡƒΠ³ΠΎΠ» Ρ‚Ρ€Π΅ΡƒΠ³ΠΎΠ»ΡŒΠ½ΠΈΠΊΠ° Ρ€Π°Π²Π΅Π½ суммС Π΄Π²ΡƒΡ… Π²Π½ΡƒΡ‚Ρ€Π΅Π½Π½ΠΈΡ… ΡƒΠ³Π»ΠΎΠ², нС смСТных с ним.

1. Π’Β Ρ‚Ρ€Π΅ΡƒΠ³ΠΎΠ»ΡŒΠ½ΠΈΠΊΠ΅ ABC ΡƒΠ³ΠΎΠ»Β C Ρ€Π°Π²Π΅Π½ 90Β°, . НайдитС тангСнс внСшнСго ΡƒΠ³Π»Π° ΠΏΡ€ΠΈ Π²Π΅Ρ€ΡˆΠΈΠ½Π΅ A.

ΠŸΡƒΡΡ‚ΡŒ β€” внСшний ΡƒΠ³ΠΎΠ» ΠΏΡ€ΠΈ Π²Π΅Ρ€ΡˆΠΈΠ½Π΅ А.

Зная cosΒ  , Π½Π°ΠΉΠ΄Π΅ΠΌ tgΒ  Β ΠΏΠΎΒ Ρ„ΠΎΡ€ΠΌΡƒΠ»Π΅

ΠŸΠΎΠ»ΡƒΡ‡ΠΈΠΌ:

2. Π’Β Ρ‚Ρ€Π΅ΡƒΠ³ΠΎΠ»ΡŒΠ½ΠΈΠΊΠ΅ ABC ΡƒΠ³ΠΎΠ»Β C Ρ€Π°Π²Π΅Π½Β 90Β°, cos A = 0,1. НайдитС синус внСшнСго ΡƒΠ³Π»Π° ΠΏΡ€ΠΈ Π²Π΅Ρ€ΡˆΠΈΠ½Π΅ B.

Π—Π°Π΄Π°Ρ‡Π° Ρ€Π΅ΡˆΠ°Π΅Ρ‚ΡΡ Π·Π°Β Ρ‡Π΅Ρ‚Ρ‹Ρ€Π΅ сСкунды. ΠŸΠΎΡΠΊΠΎΠ»ΡŒΠΊΡƒ сумма углов А ΠΈΒ Π’ Ρ€Π°Π²Π½Π°Β 90Β°, sin B = cos A = 0,1. Π’ΠΎΠ³Π΄Π° и синус внСшнСго ΡƒΠ³Π»Π° ΠΏΡ€ΠΈ Π²Π΅Ρ€ΡˆΠΈΠ½Π΅Β Π’ Ρ‚Π°ΠΊΠΆΠ΅ Ρ€Π°Π²Π΅Π½ 0,1.

Высота Π² ΠΏΡ€ΡΠΌΠΎΡƒΠ³ΠΎΠ»ΡŒΠ½ΠΎΠΌ Ρ‚Ρ€Π΅ΡƒΠ³ΠΎΠ»ΡŒΠ½ΠΈΠΊΠ΅

Вспомним, Ρ‡Ρ‚ΠΎ высота Ρ‚Ρ€Π΅ΡƒΠ³ΠΎΠ»ΡŒΠ½ΠΈΠΊΠ°Β β€” это пСрпСндикуляр, ΠΎΠΏΡƒΡ‰Π΅Π½Π½Ρ‹ΠΉ ΠΈΠ·Β Π΅Π³ΠΎ Π²Π΅Ρ€ΡˆΠΈΠ½Ρ‹ Π½Π°Β ΠΏΡ€ΠΎΡ‚ΠΈΠ²ΠΎΠΏΠΎΠ»ΠΎΠΆΠ½ΡƒΡŽ сторону.

Π’ ΠΏΡ€ΡΠΌΠΎΡƒΠ³ΠΎΠ»ΡŒΠ½ΠΎΠΌ Ρ‚Ρ€Π΅ΡƒΠ³ΠΎΠ»ΡŒΠ½ΠΈΠΊΠ΅ ΠΊΠ°Ρ‚Π΅Ρ‚Ρ‹ ΡΠ²Π»ΡΡŽΡ‚ΡΡ высотами Π΄Ρ€ΡƒΠ³ ΠΊ Π΄Ρ€ΡƒΠ³Ρƒ. Π“Π»Π°Π²Π½Ρ‹ΠΉ интСрСс прСдставляСт высота, провСдённая ΠΊ Π³ΠΈΠΏΠΎΡ‚Π΅Π½ΡƒΠ·Π΅.

Один ΠΈΠ·Β Ρ‚ΠΈΠΏΠΎΠ² экзамСнационных Π·Π°Π΄Π°Ρ‡ Π’6Β Π² Π±Π°Π½ΠΊΠ΅ Π·Π°Π΄Π°Π½ΠΈΠΉ Π€Π˜ΠŸΠ˜Β β€” Ρ‚Π°ΠΊΠΈΠ΅, Π³Π΄Π΅ Π²Β ΠΏΡ€ΡΠΌΠΎΡƒΠ³ΠΎΠ»ΡŒΠ½ΠΎΠΌ Ρ‚Ρ€Π΅ΡƒΠ³ΠΎΠ»ΡŒΠ½ΠΈΠΊΠ΅ высота ΠΏΡ€ΠΎΠ²Π΅Π΄Π΅Π½Π° ΠΈΠ·Β Π²Π΅Ρ€ΡˆΠΈΠ½Ρ‹ прямого ΡƒΠ³Π»Π°. ΠŸΠΎΡΠΌΠΎΡ‚Ρ€ΠΈΠΌ, Ρ‡Ρ‚ΠΎ получаСтся:

Высота ΠΏΡ€ΠΎΠ²Π΅Π΄Π΅Π½Π° ΠΊΒ Π³ΠΈΠΏΠΎΡ‚Π΅Π½ΡƒΠ·Π΅ АВ. Она Π΄Π΅Π»ΠΈΡ‚ Ρ‚Ρ€Π΅ΡƒΠ³ΠΎΠ»ΡŒΠ½ΠΈΠΊ АВБ Π½Π°Β Π΄Π²Π° ΠΏΡ€ΡΠΌΠΎΡƒΠ³ΠΎΠ»ΡŒΠ½Ρ‹Ρ… Ρ‚Ρ€Π΅ΡƒΠ³ΠΎΠ»ΡŒΠ½ΠΈΠΊΠ°Β β€” АБН и БНВ. Π‘ΠΌΠΎΡ‚Ρ€ΠΈΠΌ Π²Π½ΠΈΠΌΠ°Ρ‚Π΅Π»ΡŒΠ½ΠΎ на рисунок ΠΈΒ Π½Π°Ρ…ΠΎΠ΄ΠΈΠΌ Π½Π°Β Π½Π΅ΠΌ Ρ€Π°Π²Π½Ρ‹Π΅ ΡƒΠ³Π»Ρ‹. Π­Ρ‚ΠΎ ΠΈΒ Π΅ΡΡ‚ΡŒ ΠΊΠ»ΡŽΡ‡ ΠΊΒ Π·Π°Π΄Π°Ρ‡Π°ΠΌ ΠΏΠΎΒ Π³Π΅ΠΎΠΌΠ΅Ρ‚Ρ€ΠΈΠΈ, Π²Β ΠΊΠΎΡ‚ΠΎΡ€Ρ‹Ρ… высота ΠΎΠΏΡƒΡ‰Π΅Π½Π° Π½Π°Β Π³ΠΈΠΏΠΎΡ‚Π΅Π½ΡƒΠ·Ρƒ.

ΠœΡ‹Β ΠΏΠΎΠΌΠ½ΠΈΠΌ, Ρ‡Ρ‚ΠΎ сумма Π΄Π²ΡƒΡ… острых ΡƒΠ³Π»ΠΎΠ² ΠΏΡ€ΡΠΌΠΎΡƒΠ³ΠΎΠ»ΡŒΠ½ΠΎΠ³ΠΎ Ρ‚Ρ€Π΅ΡƒΠ³ΠΎΠ»ΡŒΠ½ΠΈΠΊΠ° Ρ€Π°Π²Π½Π° 90ΒΊ. Π—Π½Π°Ρ‡ΠΈΡ‚, ∠ АБН = 90ΒΊ β€” ∠ БАН, Ρ‚ΠΎΒ Π΅ΡΡ‚ΡŒ ΡƒΠ³ΠΎΠ» АБН Ρ€Π°Π²Π΅Π½ ΡƒΠ³Π»Ρƒ АВБ. Аналогично, ΡƒΠ³ΠΎΠ» БАВ Ρ€Π°Π²Π΅Π½ ΡƒΠ³Π»Ρƒ НБВ.

Π˜Π½Ρ‹ΠΌΠΈ словами, ΠΊΠ°ΠΆΠ΄Ρ‹ΠΉ ΠΈΠ·Β Ρ‚Ρ€Π΅Ρ… ΡƒΠ³Π»ΠΎΠ² Ρ‚Ρ€Π΅ΡƒΠ³ΠΎΠ»ΡŒΠ½ΠΈΠΊΠ° АВБ Ρ€Π°Π²Π΅Π½ ΠΎΠ΄Π½ΠΎΠΌΡƒ ΠΈΠ·Β ΡƒΠ³Π»ΠΎΠ² Ρ‚Ρ€Π΅ΡƒΠ³ΠΎΠ»ΡŒΠ½ΠΈΠΊΠ° АБН (ΠΈΒ Ρ‚Ρ€Π΅ΡƒΠ³ΠΎΠ»ΡŒΠ½ΠΈΠΊΠ° ВБН). Π’Ρ€Π΅ΡƒΠ³ΠΎΠ»ΡŒΠ½ΠΈΠΊΠΈ АВБ, АБН и ВБН Π½Π°Π·Ρ‹Π²Π°ΡŽΡ‚ΡΡ ΠΏΠΎΠ΄ΠΎΠ±Π½Ρ‹ΠΌΠΈ. Π”Π°Π²Π°ΠΉΡ‚Π΅ нарисуСм их рядом Π΄Ρ€ΡƒΠ³ с другом.

Они ΠΎΡ‚Π»ΠΈΡ‡Π°ΡŽΡ‚ΡΡ Ρ‚ΠΎΠ»ΡŒΠΊΠΎ Ρ€Π°Π·ΠΌΠ΅Ρ€Π°ΠΌΠΈ. Π‘Ρ‚ΠΎΡ€ΠΎΠ½Ρ‹ ΠΏΠΎΠ΄ΠΎΠ±Π½Ρ‹Ρ… Ρ‚Ρ€Π΅ΡƒΠ³ΠΎΠ»ΡŒΠ½ΠΈΠΊΠΎΠ² ΠΏΡ€ΠΎΠΏΠΎΡ€Ρ†ΠΈΠΎΠ½Π°Π»ΡŒΠ½Ρ‹. Π§Ρ‚ΠΎ это Π·Π½Π°Ρ‡ΠΈΡ‚?

Π’ΠΎΠ·ΡŒΠΌΠ΅ΠΌ Ρ‚Ρ€Π΅ΡƒΠ³ΠΎΠ»ΡŒΠ½ΠΈΠΊΠΈ АБН и АВБ. Π‘Ρ‚ΠΎΡ€ΠΎΠ½Ρ‹ Ρ‚Ρ€Π΅ΡƒΠ³ΠΎΠ»ΡŒΠ½ΠΈΠΊΠ° АВБ Π΄Π»ΠΈΠ½Π½Π΅Π΅, Ρ‡Π΅ΠΌ стороны Ρ‚Ρ€Π΅ΡƒΠ³ΠΎΠ»ΡŒΠ½ΠΈΠΊΠ° АБН Π²Β kΒ Ρ€Π°Π·:

ΠŸΡ€ΠΈ Ρ€Π΅ΡˆΠ΅Π½ΠΈΠΈ Π·Π°Π΄Π°Ρ‡ Π½Π°ΠΌ пригодится равСнство ΡƒΠ³Π»ΠΎΠ² Ρ‚Ρ€Π΅ΡƒΠ³ΠΎΠ»ΡŒΠ½ΠΈΠΊΠΎΠ² АВБ, АБН и ВБН, Π°Β Ρ‚Π°ΠΊΠΆΠ΅ ΠΏΡ€ΠΎΠΏΠΎΡ€Ρ†ΠΈΠΎΠ½Π°Π»ΡŒΠ½ΠΎΡΡ‚ΡŒ их сторон. ΠžΠ±Ρ€Π°Ρ‚ΠΈΡ‚Π΅ Ρ‚Π°ΠΊΠΆΠ΅ Π²Π½ΠΈΠΌΠ°Π½ΠΈΠ΅, Ρ‡Ρ‚ΠΎ ΠΏΠ»ΠΎΡ‰Π°Π΄ΡŒ Ρ‚Ρ€Π΅ΡƒΠ³ΠΎΠ»ΡŒΠ½ΠΈΠΊΠ° АВБ ΠΌΠΎΠΆΠ½ΠΎ Π·Π°ΠΏΠΈΡΠ°Ρ‚ΡŒ двумя Ρ€Π°Π·Π½Ρ‹ΠΌΠΈ способами: ΠΊΠ°ΠΊ ΠΏΠΎΠ»ΠΎΠ²ΠΈΠ½Ρƒ произвСдСния ΠΊΠ°Ρ‚Π΅Ρ‚ΠΎΠ² ΠΈΒ ΠΊΠ°ΠΊ ΠΏΠΎΠ»ΠΎΠ²ΠΈΠ½Ρƒ произвСдСния Π³ΠΈΠΏΠΎΡ‚Π΅Π½ΡƒΠ·Ρ‹ Π½Π°Β ΠΏΡ€ΠΎΠ²Π΅Π΄Π΅Π½Π½ΡƒΡŽ ΠΊΒ Π½Π΅ΠΉ высоту.

1. Π’ Ρ‚Ρ€Π΅ΡƒΠ³ΠΎΠ»ΡŒΠ½ΠΈΠΊΠ΅ ABC ΡƒΠ³ΠΎΠ»Β C Ρ€Π°Π²Π΅Π½ 90Β°, CHΒ β€” высота, Π’Π‘ = 3, . НайдитС AH.

Рассмотрим Ρ‚Ρ€Π΅ΡƒΠ³ΠΎΠ»ΡŒΠ½ΠΈΠΊ АВБ. Π’Β Π½Π΅ΠΌ извСстны косинус угла А ΠΈΒ ΠΏΡ€ΠΎΡ‚ΠΈΠ²ΠΎΠ»Π΅ΠΆΠ°Ρ‰ΠΈΠΉ ΠΊΠ°Ρ‚Π΅Ρ‚ Π’Π‘. Зная синус угла А, ΠΌΡ‹Β ΠΌΠΎΠ³Π»ΠΈΒ Π±Ρ‹ Π½Π°ΠΉΡ‚ΠΈ Π³ΠΈΠΏΠΎΡ‚Π΅Π½ΡƒΠ·Ρƒ АВ. Π’Π°ΠΊ Π΄Π°Π²Π°ΠΉΡ‚Π΅ Π½Π°ΠΉΠ΄Π΅ΠΌ sin A:

(ΠΏΠΎΡΠΊΠΎΠ»ΡŒΠΊΡƒ Π·Π½Π°Ρ‡Π΅Π½ΠΈΠ΅ синуса острого ΡƒΠ³Π»Π° ΠΏΠΎΠ»ΠΎΠΆΠΈΡ‚Π΅Π»ΡŒΠ½ΠΎ). Π’ΠΎΠ³Π΄Π°:

Рассмотрим ΠΏΡ€ΡΠΌΠΎΡƒΠ³ΠΎΠ»ΡŒΠ½Ρ‹ΠΉ Ρ‚Ρ€Π΅ΡƒΠ³ΠΎΠ»ΡŒΠ½ΠΈΠΊ ВБН, ∠ Н = 90Β°. ΠŸΠΎΡΠΊΠΎΠ»ΡŒΠΊΡƒ ∠ НБВ = ∠ А,

ΠžΡ‚ΡΡŽΠ΄Π°

ΠžΡ‚Π²Π΅Ρ‚: 16.

2. Π’ Ρ‚Ρ€Π΅ΡƒΠ³ΠΎΠ»ΡŒΠ½ΠΈΠΊΠ΅ ABC ΡƒΠ³ΠΎΠ»Β C Ρ€Π°Π²Π΅Π½ 90ΒΊ, АC = 8, sin A = 0,5. НайдитС высоту CH.

Π‘Π΄Π΅Π»Π°ΠΉΡ‚Π΅ Ρ‡Π΅Ρ€Ρ‚Π΅ΠΆ и рассмотритС ΠΏΡ€ΡΠΌΠΎΡƒΠ³ΠΎΠ»ΡŒΠ½Ρ‹ΠΉ Ρ‚Ρ€Π΅ΡƒΠ³ΠΎΠ»ΡŒΠ½ΠΈΠΊ АБН.

ΠžΡ‚Π²Π΅Ρ‚: 4.

3. Π’ Ρ‚Ρ€Π΅ΡƒΠ³ΠΎΠ»ΡŒΠ½ΠΈΠΊΠ΅ ABC ΡƒΠ³ΠΎΠ»Β C Ρ€Π°Π²Π΅Π½ 90ΒΊ, АВ = 13, . ΠšΒ Π³ΠΈΠΏΠΎΡ‚Π΅Π½ΡƒΠ·Π΅ ΠΏΡ€ΠΎΠ²Π΅Π΄Π΅Π½Π° высота CH. НайдитС AH. <.

Π­Ρ‚ΠΎ Ρ‡ΡƒΡ‚ΡŒ Π±ΠΎΠ»Π΅Π΅ слоТная Π·Π°Π΄Π°Ρ‡Π°. Π’Π΅Π΄ΡŒ Π²Π°ΠΌ нСизвСстны ΠΊΠ°Ρ‚Π΅Ρ‚Ρ‹ aΒ ΠΈΒ b.

Π—Π°Ρ‚ΠΎ ΠΌΠΎΠΆΠ½ΠΎ Π·Π°ΠΏΠΈΡΠ°Ρ‚ΡŒ Ρ‚Π΅ΠΎΡ€Π΅ΠΌΡƒ ΠŸΠΈΡ„Π°Π³ΠΎΡ€Π°: aΒ² + bΒ² = 13Β².

Нам извСстно Ρ‚Π°ΠΊΠΆΠ΅, Ρ‡Ρ‚ΠΎ:

РСшая эту систСму ΠΈΠ·Β Π΄Π²ΡƒΡ… ΡƒΡ€Π°Π²Π½Π΅Π½ΠΈΠΉ, Π½Π°ΠΉΠ΄Π΅ΠΌ:

;

Π—Π°ΠΏΠΈΡˆΠ΅ΠΌ ΠΏΠ»ΠΎΡ‰Π°Π΄ΡŒ Ρ‚Ρ€Π΅ΡƒΠ³ΠΎΠ»ΡŒΠ½ΠΈΠΊΠ° АВБ двумя способами:

и найдСм БН = 2,5.

Π‘ΡƒΠΌΠΌΠ° ΡƒΠ³Π»ΠΎΠ² Ρ‚Ρ€Π΅ΡƒΠ³ΠΎΠ»ΡŒΠ½ΠΈΠΊΠ°

Π‘ΡƒΠΌΠΌΠ° Ρ‚Ρ€Π΅ΡƒΠ³ΠΎΠ»ΡŒΠ½ΠΈΠΊΠ° Ρ€Π°Π²Π½Π° 180 градусов.

Π­Ρ‚ΠΎ Π»Π΅Π³ΠΊΠΎ Π΄ΠΎΠΊΠ°Π·Π°Ρ‚ΡŒ. НарисуйтС Ρ‚Ρ€Π΅ΡƒΠ³ΠΎΠ»ΡŒΠ½ΠΈΠΊ. Π§Π΅Ρ€Π΅Π· ΠΎΠ΄Π½Ρƒ ΠΈΠ·Β Π΅Π³ΠΎ Π²Π΅Ρ€ΡˆΠΈΠ½ ΠΏΡ€ΠΎΠ²Π΅Π΄ΠΈΡ‚Π΅ ΠΏΡ€ΡΠΌΡƒΡŽ, ΠΏΠ°Ρ€Π°Π»Π»Π΅Π»ΡŒΠ½ΡƒΡŽ ΠΏΡ€ΠΎΡ‚ΠΈΠ²ΠΎΠΏΠΎΠ»ΠΎΠΆΠ½ΠΎΠΉ сторонС, ΠΈΒ Π½Π°ΠΉΠ΄ΠΈΡ‚Π΅ на рисункС Ρ€Π°Π²Π½Ρ‹Π΅ ΡƒΠ³Π»Ρ‹. Π‘Ρ€Π°Π²Π½ΠΈΡ‚Π΅ ΡΒ Ρ€Π΅ΡˆΠ΅Π½ΠΈΠ΅ΠΌ Π²Β ΠΊΠΎΠ½Ρ†Π΅ ΡΡ‚Π°Ρ‚ΡŒΠΈ.

А мы разбСрСм Π·Π°Π΄Π°Ρ‡ΠΈ Π•Π“Π­, Π²Β ΠΊΠΎΡ‚ΠΎΡ€Ρ‹Ρ… Ρ„ΠΈΠ³ΡƒΡ€ΠΈΡ€ΡƒΠ΅Ρ‚ сумма ΡƒΠ³Π»ΠΎΠ² Ρ‚Ρ€Π΅ΡƒΠ³ΠΎΠ»ΡŒΠ½ΠΈΠΊΠ°.

1. Один ΠΈΠ·Β Π²Π½Π΅ΡˆΠ½ΠΈΡ… ΡƒΠ³Π»ΠΎΠ² Ρ‚Ρ€Π΅ΡƒΠ³ΠΎΠ»ΡŒΠ½ΠΈΠΊΠ° Ρ€Π°Π²Π΅Π½ 85ΒΊ. Π£Π³Π»Ρ‹, Π½Π΅ смСТныС с данным внСшним ΡƒΠ³Π»ΠΎΠΌ, относятся ΠΊΠ°ΠΊ 2:3. НайдитС наибольший ΠΈΠ·Β Π½ΠΈΡ…. ΠžΡ‚Π²Π΅Ρ‚ Π΄Π°ΠΉΡ‚Π΅ в градусах.

Π’Π½Π΅ΡˆΠ½ΠΈΠΉ ΡƒΠ³ΠΎΠ» Ρ‚Ρ€Π΅ΡƒΠ³ΠΎΠ»ΡŒΠ½ΠΈΠΊΠ° Ρ€Π°Π²Π΅Π½ суммС Π΄Π²ΡƒΡ… Π²Π½ΡƒΡ‚Ρ€Π΅Π½Π½ΠΈΡ… ΡƒΠ³Π»ΠΎΠ², нС смСТных с ним. Π‘Π»Π΅Π΄ΠΎΠ²Π°Ρ‚Π΅Π»ΡŒΠ½ΠΎ, сумма Π΄Π²ΡƒΡ… Π΄Ρ€ΡƒΠ³ΠΈΡ… ΡƒΠ³Π»ΠΎΠ² Ρ‚Ρ€Π΅ΡƒΠ³ΠΎΠ»ΡŒΠ½ΠΈΠΊΠ° Ρ€Π°Π²Π½Π°Β 85Β°, Π°Β ΠΈΡ…Β ΠΎΡ‚Π½ΠΎΡˆΠ΅Π½ΠΈΠ΅ Ρ€Π°Π²Π½ΠΎ 2:3. ΠŸΡƒΡΡ‚ΡŒ эти ΡƒΠ³Π»Ρ‹ Ρ€Π°Π²Π½Ρ‹ 2Ρ…Β ΠΈΒ 3Ρ…. ΠŸΠΎΠ»ΡƒΡ‡ΠΈΠΌ ΡƒΡ€Π°Π²Π½Π΅Π½ΠΈΠ΅ 2Ρ… + 3Ρ… = 85Β ΠΈΒ Π½Π°ΠΉΠ΄Π΅ΠΌ Ρ… = 17. Π’ΠΎΠ³Π΄Π° 3Ρ… = 51.

ΠžΡ‚Π²Π΅Ρ‚: 51.

2. Один ΠΈΠ·Β ΡƒΠ³Π»ΠΎΠ² Ρ€Π°Π²Π½ΠΎΠ±Π΅Π΄Ρ€Π΅Π½Π½ΠΎΠ³ΠΎ Ρ‚Ρ€Π΅ΡƒΠ³ΠΎΠ»ΡŒΠ½ΠΈΠΊΠ° Ρ€Π°Π²Π΅Π½ 98ΒΊ. НайдитС ΠΎΠ΄ΠΈΠ½ ΠΈΠ·Β Π΄Ρ€ΡƒΠ³ΠΈΡ… Π΅Π³ΠΎ ΡƒΠ³Π»ΠΎΠ². ΠžΡ‚Π²Π΅Ρ‚ Π΄Π°ΠΉΡ‚Π΅ в градусах.

Как Π²Ρ‹Β Π΄ΡƒΠΌΠ°Π΅Ρ‚Π΅, ΠΌΠΎΠΆΠ΅Ρ‚Β Π»ΠΈ Ρ€Π°Π²Π½ΠΎΠ±Π΅Π΄Ρ€Π΅Π½Π½Ρ‹ΠΉ Ρ‚Ρ€Π΅ΡƒΠ³ΠΎΠ»ΡŒΠ½ΠΈΠΊ ΠΈΠΌΠ΅Ρ‚ΡŒ Π΄Π²Π° ΡƒΠ³Π»Π° ΠΏΠΎΒ 98Β°?

НСт, ΠΊΠΎΠ½Π΅Ρ‡Π½ΠΎ! Π’Π΅Π΄ΡŒ сумма ΡƒΠ³Π»ΠΎΠ² Ρ‚Ρ€Π΅ΡƒΠ³ΠΎΠ»ΡŒΠ½ΠΈΠΊΠ° Ρ€Π°Π²Π½Π° 180Β°. Π—Π½Π°Ρ‡ΠΈΡ‚, ΠΎΠ΄ΠΈΠ½ ΠΈΠ·Β ΡƒΠ³Π»ΠΎΠ² Ρ‚Ρ€Π΅ΡƒΠ³ΠΎΠ»ΡŒΠ½ΠΈΠΊΠ° Ρ€Π°Π²Π΅Π½Β 98Β°, Π°Β Π΄Π²Π° Π΄Ρ€ΡƒΠ³ΠΈΡ… Ρ€Π°Π²Π½Ρ‹ .

ΠžΡ‚Π²Π΅Ρ‚: 41.

3. На рисункС ΡƒΠ³ΠΎΠ» 1Β Ρ€Π°Π²Π΅Π½ 46ΒΊ, ΡƒΠ³ΠΎΠ» 2Β Ρ€Π°Π²Π΅Π½ 30ΒΊ, ΡƒΠ³ΠΎΠ» 3Β Ρ€Π°Π²Π΅Π½ 44ΒΊ. НайдитС ΡƒΠ³ΠΎΠ» 4. ΠžΡ‚Π²Π΅Ρ‚ Π΄Π°ΠΉΡ‚Π΅ в градусах.

Π”Π°Π²Π°ΠΉΡ‚Π΅ ΠΎΡ‚ΠΌΠ΅Ρ‚ΠΈΠΌ Π½Π°Β Ρ‡Π΅Ρ€Ρ‚Π΅ΠΆΠ΅ Π΅Ρ‰Π΅ нСсколько ΡƒΠ³Π»ΠΎΠ². Они Π½Π°ΠΌ понадобятся.

Π‘Π½Π°Ρ‡Π°Π»Π° Π½Π°ΠΉΠ΄Π΅ΠΌ ΡƒΠ³ΠΎΠ» 5. ΠžΠ½Β Ρ€Π°Π²Π΅Π½ 180° — ∠1Β — ∠3 = 90Β° Π’ΠΎΠ³Π΄Π° ∠6 = 90Β° ∠7 = 180° — ∠2Β — ∠6 = 60Β°, Π£Π³ΠΎΠ»Β 4, смСТный с углом 7Β Ρ€Π°Π²Π΅Π½ 120Β°.

ΠžΡ‚Π²Π΅Ρ‚: 120.

Π—Π°ΠΌΠ΅Ρ‚ΠΈΠΌ, Ρ‡Ρ‚ΠΎ Ρ‚Π°ΠΊΠΎΠΉ способ Ρ€Π΅ΡˆΠ΅Π½ΠΈΡΒ β€” нС СдинствСнный. ΠŸΡ€ΠΎΡΡ‚ΠΎ Π½Π°Ρ…ΠΎΠ΄ΠΈΡ‚Π΅ ΠΈΒ ΠΎΡ‚ΠΌΠ΅Ρ‡Π°ΠΉΡ‚Π΅ Π½Π°Β Ρ‡Π΅Ρ€Ρ‚Π΅ΠΆΠ΅ всС ΡƒΠ³Π»Ρ‹, ΠΊΠΎΡ‚ΠΎΡ€Ρ‹Π΅ ΠΌΠΎΠΆΠ½ΠΎ Π½Π°ΠΉΡ‚ΠΈ.

4. Π£Π³Π»Ρ‹ Ρ‚Ρ€Π΅ΡƒΠ³ΠΎΠ»ΡŒΠ½ΠΈΠΊΠ° относятся ΠΊΠ°ΠΊ 2:3:4. НайдитС мСньший ΠΈΠ·Β Π½ΠΈΡ…. ΠžΡ‚Π²Π΅Ρ‚ Π΄Π°ΠΉΡ‚Π΅ в градусах.

ΠŸΡƒΡΡ‚ΡŒ ΡƒΠ³Π»Ρ‹ Ρ‚Ρ€Π΅ΡƒΠ³ΠΎΠ»ΡŒΠ½ΠΈΠΊΠ° Ρ€Π°Π²Π½Ρ‹ 2Ρ…, 3Ρ…Β ΠΈΒ 4Ρ…. Π—Π°ΠΏΠΈΡˆΠ΅ΠΌ, Ρ‡Π΅ΠΌΡƒ Ρ€Π°Π²Π½Π° сумма ΡƒΠ³Π»ΠΎΠ² этого Ρ‚Ρ€Π΅ΡƒΠ³ΠΎΠ»ΡŒΠ½ΠΈΠΊΠ°. 2Ρ… + 3Ρ… + 4Ρ… = 180Β° 9Ρ… = 180Β° Ρ… = 20Β° Π’ΠΎΠ³Π΄Π° 2Ρ… = 40Β°.

ΠžΡ‚Π²Π΅Ρ‚: 40.

Π£Π³Π»Ρ‹ ΠΏΡ€ΠΈ ΠΏΠ°Ρ€Π°Π»Π»Π΅Π»ΡŒΠ½Ρ‹Ρ… прямых и сСкущСй. Π’Π΅Ρ€Ρ‚ΠΈΠΊΠ°Π»ΡŒΠ½Ρ‹Π΅, смСТныС, односторонниС, соотвСтствСнныС, накрСст Π»Π΅ΠΆΠ°Ρ‰ΠΈΠ΅ ΡƒΠ³Π»Ρ‹

ΠŸΡƒΡΡ‚ΡŒ прямая с пСрСсСкаСт ΠΏΠ°Ρ€Π°Π»Π»Π΅Π»ΡŒΠ½Ρ‹Π΅ прямыС Π°Β ΠΈΒ b. ΠŸΡ€ΠΈ этом образуСтся восСмь ΡƒΠ³Π»ΠΎΠ². Π£Π³Π»Ρ‹ ΠΏΡ€ΠΈ ΠΏΠ°Ρ€Π°Π»Π»Π΅Π»ΡŒΠ½Ρ‹Ρ… прямых и сСкущСй Ρ‚Π°ΠΊ часто ΠΈΡΠΏΠΎΠ»ΡŒΠ·ΡƒΡŽΡ‚ΡΡ Π²Β Π·Π°Π΄Π°Ρ‡Π°Ρ…, Ρ‡Ρ‚ΠΎ Π²Β Π³Π΅ΠΎΠΌΠ΅Ρ‚Ρ€ΠΈΠΈ ΠΈΠΌΒ Π΄Π°Π½Ρ‹ ΡΠΏΠ΅Ρ†ΠΈΠ°Π»ΡŒΠ½Ρ‹Π΅ названия.

Π£Π³Π»Ρ‹ 1Β ΠΈΒ 3Β β€” Π²Π΅Ρ€Ρ‚ΠΈΠΊΠ°Π»ΡŒΠ½Ρ‹Π΅. ΠžΡ‡Π΅Π²ΠΈΠ΄Π½ΠΎ, Π²Π΅Ρ€Ρ‚ΠΈΠΊΠ°Π»ΡŒΠ½Ρ‹Π΅ ΡƒΠ³Π»Ρ‹ Ρ€Π°Π²Π½Ρ‹, Ρ‚ΠΎΒ Π΅ΡΡ‚ΡŒ ∠1 = ∠3, ∠2 = ∠4.

ΠšΠΎΠ½Π΅Ρ‡Π½ΠΎ, ΡƒΠ³Π»Ρ‹ 5Β ΠΈΒ 7, 6Β ΠΈΒ 8Β β€” Ρ‚ΠΎΠΆΠ΅ Π²Π΅Ρ€Ρ‚ΠΈΠΊΠ°Π»ΡŒΠ½Ρ‹Π΅.

Π£Π³Π»Ρ‹ 1Β ΠΈΒ 2Β β€” смСТныС, это ΠΌΡ‹Β ΡƒΠΆΠ΅ Π·Π½Π°Π΅ΠΌ. Π‘ΡƒΠΌΠΌΠ° смСТных ΡƒΠ³Π»ΠΎΠ² Ρ€Π°Π²Π½Π° 180ΒΊ.

Π£Π³Π»Ρ‹ 3Β ΠΈΒ 5Β (Π°Β Ρ‚Π°ΠΊΠΆΠ΅ 2Β ΠΈΒ 8, 1Β ΠΈΒ 7, 4Β ΠΈΒ 6)Β β€” накрСст Π»Π΅ΠΆΠ°Ρ‰ΠΈΠ΅. НакрСст Π»Π΅ΠΆΠ°Ρ‰ΠΈΠ΅ ΡƒΠ³Π»Ρ‹ Ρ€Π°Π²Π½Ρ‹. ∠3 = ∠5, ∠1 = ∠7, ∠2 = ∠8, ∠4 = ∠6.

Π£Π³Π»Ρ‹ 1Β ΠΈΒ 6Β β€” односторонниС. Они Π»Π΅ΠΆΠ°Ρ‚ ΠΏΠΎΒ ΠΎΠ΄Π½Ρƒ сторону от всСй «конструкции». Π£Π³Π»Ρ‹ 4Β ΠΈΒ 7Β β€” Ρ‚ΠΎΠΆΠ΅ односторонниС. Π‘ΡƒΠΌΠΌΠ° односторонних ΡƒΠ³Π»ΠΎΠ² Ρ€Π°Π²Π½Π° 180Β°, Ρ‚ΠΎΒ Π΅ΡΡ‚ΡŒ ∠1 + ∠6 = 180Β°, ∠4 + ∠7 = 180Β°.

Π£Π³Π»Ρ‹ 2Β ΠΈΒ 6Β (Π°Β Ρ‚Π°ΠΊΠΆΠ΅ 3Β ΠΈΒ 7, 1Β ΠΈΒ 5, 4Β ΠΈΒ 8) Π½Π°Π·Ρ‹Π²Π°ΡŽΡ‚ΡΡ соотвСтствСнными.

БоотвСтствСнныС ΡƒΠ³Π»Ρ‹ Ρ€Π°Π²Π½Ρ‹, Ρ‚ΠΎΒ Π΅ΡΡ‚ΡŒ ∠2 = ∠6, ∠3 = ∠7.

Π£Π³Π»Ρ‹ 3Β ΠΈΒ 5Β (Π°Β Ρ‚Π°ΠΊΠΆΠ΅ 2Β ΠΈΒ 8, 1Β ΠΈΒ 7, 4Β ΠΈΒ 6) Π½Π°Π·Ρ‹Π²Π°ΡŽΡ‚ накрСст Π»Π΅ΠΆΠ°Ρ‰ΠΈΠΌΠΈ.

НакрСст Π»Π΅ΠΆΠ°Ρ‰ΠΈΠ΅ ΡƒΠ³Π»Ρ‹ Ρ€Π°Π²Π½Ρ‹, Ρ‚ΠΎΒ Π΅ΡΡ‚ΡŒ ∠3 = ∠5, ∠1 = ∠7, ∠2 = ∠8, ∠4 = ∠6.

Π§Ρ‚ΠΎΠ±Ρ‹ ΠΏΡ€ΠΈΠΌΠ΅Π½ΡΡ‚ΡŒ всС эти Ρ„Π°ΠΊΡ‚Ρ‹ Π²Β Ρ€Π΅ΡˆΠ΅Π½ΠΈΠΈ Π·Π°Π΄Π°Ρ‡ Π•Π“Π­, Π½Π°Π΄ΠΎ Π½Π°ΡƒΡ‡ΠΈΡ‚ΡŒΡΡ Π²ΠΈΠ΄Π΅Ρ‚ΡŒ ΠΈΡ…Β Π½Π°Β Ρ‡Π΅Ρ€Ρ‚Π΅ΠΆΠ΅. НапримСр, глядя Π½Π°Β ΠΏΠ°Ρ€Π°Π»Π»Π΅Π»ΠΎΠ³Ρ€Π°ΠΌΠΌ ΠΈΠ»ΠΈ Ρ‚Ρ€Π°ΠΏΠ΅Ρ†ΠΈΡŽ, ΠΌΠΎΠΆΠ½ΠΎ ΡƒΠ²ΠΈΠ΄Π΅Ρ‚ΡŒ ΠΏΠ°Ρ€Ρƒ ΠΏΠ°Ρ€Π°Π»Π»Π΅Π»ΡŒΠ½Ρ‹Ρ… прямых ΠΈΒ ΡΠ΅ΠΊΡƒΡ‰ΡƒΡŽ, Π°Β Ρ‚Π°ΠΊΠΆΠ΅ односторонниС ΡƒΠ³Π»Ρ‹. ΠŸΡ€ΠΎΠ²Π΅Π΄Ρ диагональ ΠΏΠ°Ρ€Π°Π»Π»Π΅Π»ΠΎΠ³Ρ€Π°ΠΌΠΌΠ°, Π²ΠΈΠ΄ΠΈΠΌ накрСст Π»Π΅ΠΆΠ°Ρ‰ΠΈΠ΅ ΡƒΠ³Π»Ρ‹. Π­Ρ‚ΠΎΒ β€” ΠΎΠ΄ΠΈΠ½ из шагов, ΠΈΠ·Β ΠΊΠΎΡ‚ΠΎΡ€Ρ‹Ρ… и состоит Ρ€Π΅ΡˆΠ΅Π½ΠΈΠ΅.

1. БиссСктриса Ρ‚ΡƒΠΏΠΎΠ³ΠΎ ΡƒΠ³Π»Π° ΠΏΠ°Ρ€Π°Π»Π»Π΅Π»ΠΎΠ³Ρ€Π°ΠΌΠΌΠ° Π΄Π΅Π»ΠΈΡ‚ ΠΏΡ€ΠΎΡ‚ΠΈΠ²ΠΎΠΏΠΎΠ»ΠΎΠΆΠ½ΡƒΡŽ сторону Π²Β ΠΎΡ‚Π½ΠΎΡˆΠ΅Π½ΠΈΠΈ 3:4, считая ΠΎΡ‚Β Π²Π΅Ρ€ΡˆΠΈΠ½Ρ‹ Ρ‚ΡƒΠΏΠΎΠ³ΠΎ ΡƒΠ³Π»Π°. НайдитС Π±ΠΎΠ»ΡŒΡˆΡƒΡŽ сторону ΠΏΠ°Ρ€Π°Π»Π»Π΅Π»ΠΎΠ³Ρ€Π°ΠΌΠΌΠ°, Ссли Π΅Π³ΠΎ ΠΏΠ΅Ρ€ΠΈΠΌΠ΅Ρ‚Ρ€ Ρ€Π°Π²Π΅Π½ 88.

Напомним, Ρ‡Ρ‚ΠΎ биссСктриса ΡƒΠ³Π»Π°Β β€” это Π»ΡƒΡ‡, выходящий ΠΈΠ·Β Π²Π΅Ρ€ΡˆΠΈΠ½Ρ‹ ΡƒΠ³Π»Π° и дСлящий ΡƒΠ³ΠΎΠ» ΠΏΠΎΠΏΠΎΠ»Π°ΠΌ.

ΠŸΡƒΡΡ‚ΡŒ Π’ΠœΒ β€” биссСктриса Ρ‚ΡƒΠΏΠΎΠ³ΠΎ ΡƒΠ³Π»Π° Π’.Β ΠŸΠΎΒ ΡƒΡΠ»ΠΎΠ²ΠΈΡŽ, ΠΎΡ‚Ρ€Π΅Π·ΠΊΠΈ МD и АВ Ρ€Π°Π²Π½Ρ‹ 3Ρ…Β ΠΈΒ 4х соотвСтствСнно.

Рассмотрим ΡƒΠ³Π»Ρ‹ Π‘Π’Πœ ΠΈΒ Π’ΠœΠ. ΠŸΠΎΡΠΊΠΎΠ»ΡŒΠΊΡƒ АD ΠΈΒ Π’Π‘ ΠΏΠ°Ρ€Π°Π»Π»Π΅Π»ΡŒΠ½Ρ‹, Π’ΠœΒ β€” сСкущая, ΡƒΠ³Π»Ρ‹ Π‘Π’Πœ ΠΈΒ Π’ΠœΠ ΡΠ²Π»ΡΡŽΡ‚ΡΡ накрСст Π»Π΅ΠΆΠ°Ρ‰ΠΈΠΌΠΈ. ΠœΡ‹Β Π·Π½Π°Π΅ΠΌ, Ρ‡Ρ‚ΠΎ накрСст Π»Π΅ΠΆΠ°Ρ‰ΠΈΠ΅ ΡƒΠ³Π»Ρ‹ Ρ€Π°Π²Π½Ρ‹. Π—Π½Π°Ρ‡ΠΈΡ‚, Ρ‚Ρ€Π΅ΡƒΠ³ΠΎΠ»ΡŒΠ½ΠΈΠΊ ΠΠ’ΠœΒ β€” Ρ€Π°Π²Π½ΠΎΠ±Π΅Π΄Ρ€Π΅Π½Π½Ρ‹ΠΉ, ΡΠ»Π΅Π΄ΠΎΠ²Π°Ρ‚Π΅Π»ΡŒΠ½ΠΎ, АВ = АМ = 4Ρ….

ΠŸΠ΅Ρ€ΠΈΠΌΠ΅Ρ‚Ρ€ ΠΏΠ°Ρ€Π°Π»Π»Π΅Π»ΠΎΠ³Ρ€Π°ΠΌΠΌΠ°Β β€” это сумма всСх Π΅Π³ΠΎ сторон, Ρ‚ΠΎΒ Π΅ΡΡ‚ΡŒ 7Ρ… + 7Ρ… + 4Ρ… + 4Ρ… = 88. ΠžΡ‚ΡΡŽΠ΄Π° Ρ… = 4, 7Ρ… = 28.

ΠžΡ‚Π²Π΅Ρ‚: 28.

2. Π”ΠΈΠ°Π³ΠΎΠ½Π°Π»ΡŒ ΠΏΠ°Ρ€Π°Π»Π»Π΅Π»ΠΎΠ³Ρ€Π°ΠΌΠΌΠ° ΠΎΠ±Ρ€Π°Π·ΡƒΠ΅Ρ‚ с двумя Π΅Π³ΠΎ сторонами ΡƒΠ³Π»Ρ‹ 26ΒΊ ΠΈΒ 34ΒΊ. НайдитС больший ΡƒΠ³ΠΎΠ» ΠΏΠ°Ρ€Π°Π»Π»Π΅Π»ΠΎΠ³Ρ€Π°ΠΌΠΌΠ°. ΠžΡ‚Π²Π΅Ρ‚ Π΄Π°ΠΉΡ‚Π΅ Π² градусах.

НарисуйтС ΠΏΠ°Ρ€Π°Π»Π»Π΅Π»ΠΎΠ³Ρ€Π°ΠΌΠΌ ΠΈΒ Π΅Π³ΠΎ диагональ. Π—Π°ΠΌΠ΅Ρ‚ΠΈΠ² Π½Π°Β Ρ‡Π΅Ρ€Ρ‚Π΅ΠΆΠ΅ накрСст Π»Π΅ΠΆΠ°Ρ‰ΠΈΠ΅ ΡƒΠ³Π»Ρ‹ и односторонниС ΡƒΠ³Π»Ρ‹, Π²Ρ‹Β Π»Π΅Π³ΠΊΠΎ ΠΏΠΎΠ»ΡƒΡ‡ΠΈΡ‚Π΅ ΠΎΡ‚Π²Π΅Ρ‚: 120ΒΊ.

3. Π§Π΅ΠΌΡƒ Ρ€Π°Π²Π΅Π½ больший ΡƒΠ³ΠΎΠ» Ρ€Π°Π²Π½ΠΎΠ±Π΅Π΄Ρ€Π΅Π½Π½ΠΎΠΉ Ρ‚Ρ€Π°ΠΏΠ΅Ρ†ΠΈΠΈ, Ссли извСстно, Ρ‡Ρ‚ΠΎ Ρ€Π°Π·Π½ΠΎΡΡ‚ΡŒ ΠΏΡ€ΠΎΡ‚ΠΈΠ²ΠΎΠ»Π΅ΠΆΠ°Ρ‰ΠΈΡ… ΡƒΠ³Π»ΠΎΠ² Ρ€Π°Π²Π½Π° 50ΒΊ? ΠžΡ‚Π²Π΅Ρ‚ Π΄Π°ΠΉΡ‚Π΅ в градусах.

ΠœΡ‹Β Π·Π½Π°Π΅ΠΌ, Ρ‡Ρ‚ΠΎ Ρ€Π°Π²Π½ΠΎΠ±Π΅Π΄Ρ€Π΅Π½Π½ΠΎΠΉ (ΠΈΠ»ΠΈ Ρ€Π°Π²Π½ΠΎΠ±ΠΎΠΊΠΎΠΉ) называСтся трапСция, ΡƒΒ ΠΊΠΎΡ‚ΠΎΡ€ΠΎΠΉ Π±ΠΎΠΊΠΎΠ²Ρ‹Π΅ стороны Ρ€Π°Π²Π½Ρ‹. Π‘Π»Π΅Π΄ΠΎΠ²Π°Ρ‚Π΅Π»ΡŒΠ½ΠΎ, Ρ€Π°Π²Π½Ρ‹ ΡƒΠ³Π»Ρ‹ ΠΏΡ€ΠΈ Π²Π΅Ρ€Ρ…Π½Π΅ΠΌ основании, Π°Β Ρ‚Π°ΠΊΠΆΠ΅ ΡƒΠ³Π»Ρ‹ ΠΏΡ€ΠΈ Π½ΠΈΠΆΠ½Π΅ΠΌ основании.

Π”Π°Π²Π°ΠΉΡ‚Π΅ посмотрим Π½Π°Β Ρ‡Π΅Ρ€Ρ‚Π΅ΠΆ. ΠŸΠΎΒ ΡƒΡΠ»ΠΎΠ²ΠΈΡŽ, Ξ±Β — Ξ² = 50Β°, Ρ‚ΠΎΒ Π΅ΡΡ‚ΡŒ Ξ± = Ξ² + 50Β°.

Π£Π³Π»Ρ‹ Ξ±Β ΠΈΒ Ξ²Β β€” односторонниС ΠΏΡ€ΠΈ ΠΏΠ°Ρ€Π°Π»Π»Π΅Π»ΡŒΠ½Ρ‹Ρ… прямых и сСкущСй, ΡΠ»Π΅Π΄ΠΎΠ²Π°Ρ‚Π΅Π»ΡŒΠ½ΠΎ, Ξ± + Ξ² = 180Β°.

Π˜Ρ‚Π°ΠΊ, 2Ξ² + 50Β° = 180Β° Ξ² = 65Β°, Ρ‚ΠΎΠ³Π΄Π° Ξ± = 115Β°.

ΠžΡ‚Π²Π΅Ρ‚: 115.

Π­Π»Π΅ΠΌΠ΅Π½Ρ‚Ρ‹ Ρ‚Ρ€Π΅ΡƒΠ³ΠΎΠ»ΡŒΠ½ΠΈΠΊΠ°. Высоты, ΠΌΠ΅Π΄ΠΈΠ°Π½Ρ‹, биссСктрисы

Высоты, ΠΌΠ΅Π΄ΠΈΠ°Π½Ρ‹ и биссСктрисы Ρ‚Ρ€Π΅ΡƒΠ³ΠΎΠ»ΡŒΠ½ΠΈΠΊΠ° постоянно Π²ΡΡ‚Ρ€Π΅Ρ‡Π°ΡŽΡ‚ΡΡ Π½Π°ΠΌ Π²Β Π·Π°Π΄Π°Ρ‡Π°Ρ… ΠΏΠΎΒ Π³Π΅ΠΎΠΌΠ΅Ρ‚Ρ€ΠΈΠΈ. ΠœΡ‹Β Π½Π°Ρ‡Π½Π΅ΠΌ с таблицы, Π²Β ΠΊΠΎΡ‚ΠΎΡ€ΠΎΠΉ ΠΏΠΎΠΊΠ°Π·Π°Π½ΠΎ, Ρ‡Ρ‚ΠΎ Ρ‚Π°ΠΊΠΎΠ΅ высоты, ΠΌΠ΅Π΄ΠΈΠ°Π½Ρ‹ и биссСктрисы, ΠΈΒ ΠΊΠ°ΠΊΠΈΠΌΠΈ свойствами ΠΎΠ½ΠΈ ΠΎΠ±Π»Π°Π΄Π°ΡŽΡ‚. Π—Π°Ρ‚Π΅ΠΌΒ β€” ΠΏΠΎΠ΄Ρ€ΠΎΠ±Π½Ρ‹Π΅ объяснСния ΠΈΒ Ρ€Π΅ΡˆΠ΅Π½ΠΈΠ΅ Π·Π°Π΄Π°Ρ‡.

Напомним, Ρ‡Ρ‚ΠΎ высота Ρ‚Ρ€Π΅ΡƒΠ³ΠΎΠ»ΡŒΠ½ΠΈΠΊΠ°Β β€” это пСрпСндикуляр, ΠΎΠΏΡƒΡ‰Π΅Π½Π½Ρ‹ΠΉ ΠΈΠ·Β Π΅Π³ΠΎ Π²Π΅Ρ€ΡˆΠΈΠ½Ρ‹ Π½Π°Β ΠΏΡ€ΠΎΡ‚ΠΈΠ²ΠΎΠΏΠΎΠ»ΠΎΠΆΠ½ΡƒΡŽ сторону.

Π’Ρ€ΠΈ высоты Ρ‚Ρ€Π΅ΡƒΠ³ΠΎΠ»ΡŒΠ½ΠΈΠΊΠ° всСгда ΠΏΠ΅Ρ€Π΅ΡΠ΅ΠΊΠ°ΡŽΡ‚ΡΡ Π²Β ΠΎΠ΄Π½ΠΎΠΉ Ρ‚ΠΎΡ‡ΠΊΠ΅. Π’ΠΎΡ‚ ΠΊΠ°ΠΊ это выглядит в случаС ΠΎΡΡ‚Ρ€ΠΎΡƒΠ³ΠΎΠ»ΡŒΠ½ΠΎΠ³ΠΎ Ρ‚Ρ€Π΅ΡƒΠ³ΠΎΠ»ΡŒΠ½ΠΈΠΊΠ°.

ΠŸΠΎΠΏΡ€ΠΎΠ±ΡƒΠΉΡ‚Π΅ провСсти Ρ‚Ρ€ΠΈ высоты Π²Β Ρ‚ΡƒΠΏΠΎΡƒΠ³ΠΎΠ»ΡŒΠ½ΠΎΠΌ Ρ‚Ρ€Π΅ΡƒΠ³ΠΎΠ»ΡŒΠ½ΠΈΠΊΠ΅. ΠŸΠΎΠ»ΡƒΡ‡ΠΈΠ»ΠΎΡΡŒ? Π”Π°, Ρ€Π΅Π΄ΠΊΠΈΠΉ выпускник справляСтся с этим Π·Π°Π΄Π°Π½ΠΈΠ΅ΠΌ. Π”Π΅ΠΉΡΡ‚Π²ΠΈΡ‚Π΅Π»ΡŒΠ½ΠΎ, ΠΌΡ‹Β Π½Π΅Β ΠΌΠΎΠΆΠ΅ΠΌ ΠΎΠΏΡƒΡΡ‚ΠΈΡ‚ΡŒ пСрпСндикуляр из точки А Π½Π°Β ΠΎΡ‚Ρ€Π΅Π·ΠΎΠΊΒ Π’Π‘, Π·Π°Ρ‚ΠΎ ΠΌΠΎΠΆΠ΅ΠΌ ΠΎΠΏΡƒΡΡ‚ΠΈΡ‚ΡŒ Π΅Π³ΠΎ Π½Π°Β ΠΏΡ€ΡΠΌΡƒΡŽ Π’Π‘Β β€” Ρ‚ΠΎΒ Π΅ΡΡ‚ΡŒ Π½Π°Β ΠΏΡ€ΠΎΠ΄ΠΎΠ»ΠΆΠ΅Π½ΠΈΠ΅ стороны Π’Π‘.

В этом случаС Π²Β ΠΎΠ΄Π½ΠΎΠΉ Ρ‚ΠΎΡ‡ΠΊΠ΅ ΠΏΠ΅Ρ€Π΅ΡΠ΅ΠΊΠ°ΡŽΡ‚ΡΡ нС сами высоты, а их продолТСния.

А как выглядят Ρ‚Ρ€ΠΈ высоты Π²Β ΠΏΡ€ΡΠΌΠΎΡƒΠ³ΠΎΠ»ΡŒΠ½ΠΎΠΌ Ρ‚Ρ€Π΅ΡƒΠ³ΠΎΠ»ΡŒΠ½ΠΈΠΊΠ΅? Π’Β ΠΊΠ°ΠΊΠΎΠΉ Ρ‚ΠΎΡ‡ΠΊΠ΅ ΠΎΠ½ΠΈ ΠΏΠ΅Ρ€Π΅ΡΠ΅ΠΊΠ°ΡŽΡ‚ΡΡ?

МСдиана Ρ‚Ρ€Π΅ΡƒΠ³ΠΎΠ»ΡŒΠ½ΠΈΠΊΠ°Β β€” ΠΎΡ‚Ρ€Π΅Π·ΠΎΠΊ, ΡΠΎΠ΅Π΄ΠΈΠ½ΡΡŽΡ‰ΠΈΠΉ Π΅Π³ΠΎ Π²Π΅Ρ€ΡˆΠΈΠ½Ρƒ с сСрСдиной ΠΏΡ€ΠΎΡ‚ΠΈΠ²ΠΎΠΏΠΎΠ»ΠΎΠΆΠ½ΠΎΠΉ стороны.

Π’Ρ€ΠΈ ΠΌΠ΅Π΄ΠΈΠ°Π½Ρ‹ Ρ‚Ρ€Π΅ΡƒΠ³ΠΎΠ»ΡŒΠ½ΠΈΠΊΠ° ΠΏΠ΅Ρ€Π΅ΡΠ΅ΠΊΠ°ΡŽΡ‚ΡΡ Π²Β ΠΎΠ΄Π½ΠΎΠΉ Ρ‚ΠΎΡ‡ΠΊΠ΅ и дСлятся Π²Β Π½Π΅ΠΉ Π²Β ΠΎΡ‚Π½ΠΎΡˆΠ΅Π½ΠΈΠΈ 2:1, считая ΠΎΡ‚Β Π²Π΅Ρ€ΡˆΠΈΠ½Ρ‹.

БиссСктриса Ρ‚Ρ€Π΅ΡƒΠ³ΠΎΠ»ΡŒΠ½ΠΈΠΊΠ°Β β€” ΠΎΡ‚Ρ€Π΅Π·ΠΎΠΊ, ΡΠΎΠ΅Π΄ΠΈΠ½ΡΡŽΡ‰ΠΈΠΉ Π²Π΅Ρ€ΡˆΠΈΠ½Ρƒ Ρ‚Ρ€Π΅ΡƒΠ³ΠΎΠ»ΡŒΠ½ΠΈΠΊΠ° с точкой Π½Π°Β ΠΏΡ€ΠΎΡ‚ΠΈΠ²ΠΎΠΏΠΎΠ»ΠΎΠΆΠ½ΠΎΠΉ сторонС и дСлящий ΡƒΠ³ΠΎΠ» Ρ‚Ρ€Π΅ΡƒΠ³ΠΎΠ»ΡŒΠ½ΠΈΠΊΠ° ΠΏΠΎΠΏΠΎΠ»Π°ΠΌ.

У биссСктрисы ΡƒΠ³Π»Π° Π΅ΡΡ‚ΡŒ Π·Π°ΠΌΠ΅Ρ‡Π°Ρ‚Π΅Π»ΡŒΠ½ΠΎΠ΅ свойство — Ρ‚ΠΎΡ‡ΠΊΠΈ, ΠΏΡ€ΠΈΠ½Π°Π΄Π»Π΅ΠΆΠ°Ρ‰ΠΈΠ΅Β Π΅ΠΉ, Ρ€Π°Π²Π½ΠΎΡƒΠ΄Π°Π»Π΅Π½Ρ‹ от сторон ΡƒΠ³Π»Π°. ΠŸΠΎΡΡ‚ΠΎΠΌΡƒ Ρ‚Ρ€ΠΈ биссСктрисы Ρ‚Ρ€Π΅ΡƒΠ³ΠΎΠ»ΡŒΠ½ΠΈΠΊΠ° ΠΏΠ΅Ρ€Π΅ΡΠ΅ΠΊΠ°ΡŽΡ‚ΡΡ Π²Β ΠΎΠ΄Π½ΠΎΠΉ Ρ‚ΠΎΡ‡ΠΊΠ΅, Ρ€Π°Π²Π½ΠΎΡƒΠ΄Π°Π»Π΅Π½Π½ΠΎΠΉ от всСх сторон Ρ‚Ρ€Π΅ΡƒΠ³ΠΎΠ»ΡŒΠ½ΠΈΠΊΠ°. Π­Ρ‚Π° Ρ‚ΠΎΡ‡ΠΊΠ° являСтся Ρ†Π΅Π½Ρ‚Ρ€ΠΎΠΌ окруТности, вписанной Π²Β Ρ‚Ρ€Π΅ΡƒΠ³ΠΎΠ»ΡŒΠ½ΠΈΠΊ.

Π•Ρ‰Π΅ ΠΎΠ΄Π½ΠΎ свойство биссСктрисы пригодится Ρ‚Π΅ΠΌ, ΠΊΡ‚ΠΎ собираСтся Ρ€Π΅ΡˆΠ°Ρ‚ΡŒ Π·Π°Π΄Π°Ρ‡Ρƒ Π‘4. БиссСктриса Ρ‚Ρ€Π΅ΡƒΠ³ΠΎΠ»ΡŒΠ½ΠΈΠΊΠ° Π΄Π΅Π»ΠΈΡ‚ ΠΏΡ€ΠΎΡ‚ΠΈΠ²ΠΎΠΏΠΎΠ»ΠΎΠΆΠ½ΡƒΡŽ сторону Π²Β ΠΎΡ‚Π½ΠΎΡˆΠ΅Π½ΠΈΠΈ Π΄Π»ΠΈΠ½ ΠΏΡ€ΠΈΠ»Π΅ΠΆΠ°Ρ‰ΠΈΡ… сторон.

Π Π°Π·Π±Π΅Ρ€Π΅ΠΌ нСсколько Π·Π°Π΄Π°Ρ‡, Π²Β ΠΊΠΎΡ‚ΠΎΡ€Ρ‹Ρ… Ρ€Π΅Ρ‡ΡŒ ΠΈΠ΄Π΅Ρ‚ о высотах, ΠΌΠ΅Π΄ΠΈΠ°Π½Π°Ρ… и биссСктрисах Ρ‚Ρ€Π΅ΡƒΠ³ΠΎΠ»ΡŒΠ½ΠΈΠΊΠ°. ВсС Π·Π°Π΄Π°Ρ‡ΠΈ взяты ΠΈΠ·Β Π‘Π°Π½ΠΊΠ° Π·Π°Π΄Π°Π½ΠΈΠΉ ЀИПИ.

1. НайдитС острый ΡƒΠ³ΠΎΠ» ΠΌΠ΅ΠΆΠ΄Ρƒ биссСктрисами острых ΡƒΠ³Π»ΠΎΠ² ΠΏΡ€ΡΠΌΠΎΡƒΠ³ΠΎΠ»ΡŒΠ½ΠΎΠ³ΠΎ Ρ‚Ρ€Π΅ΡƒΠ³ΠΎΠ»ΡŒΠ½ΠΈΠΊΠ°. ΠžΡ‚Π²Π΅Ρ‚ Π΄Π°ΠΉΡ‚Π΅ в градусах.

ΠŸΡƒΡΡ‚ΡŒ биссСктрисы Ρ‚Ρ€Π΅ΡƒΠ³ΠΎΠ»ΡŒΠ½ΠΈΠΊΠ° АВБ ( Π²Β ΠΊΠΎΡ‚ΠΎΡ€ΠΎΠΌ ΡƒΠ³ΠΎΠ»Β Π‘ Ρ€Π°Π²Π΅Π½Β 90Β°) ΠΏΠ΅Ρ€Π΅ΡΠ΅ΠΊΠ°ΡŽΡ‚ΡΡ Π²Β Ρ‚ΠΎΡ‡ΠΊΠ΅ М.

Рассмотрим Ρ‚Ρ€Π΅ΡƒΠ³ΠΎΠ»ΡŒΠ½ΠΈΠΊ ΠΠ’Πœ.

∠ ΠœΠΠ’ = ∠ ВАБ,

∠ ΠΠ’Πœ = ∠ АВБ, Ρ‚ΠΎΠ³Π΄Π° ∠ ΠΠœΠ’ = 180° — ∠ ΠœΠΠ’Β — ∠ ΠΠ’Πœ = 180° — (∠ АВБ + ∠ ВАБ).

ΠžΡΡ‚Ρ€Ρ‹ΠΉ ΡƒΠ³ΠΎΠ» ΠΌΠ΅ΠΆΠ΄Ρƒ биссСктрисами на рисункС ΠΎΠ±ΠΎΠ·Π½Π°Ρ‡Π΅Π½Β Ο†.

Π£Π³ΠΎΠ» φ смСТный с углом ΠΠœΠ’, ΡΠ»Π΅Π΄ΠΎΠ²Π°Ρ‚Π΅Π»ΡŒΠ½ΠΎ, Ο† = (∠ АВБ + ∠ ВАБ).

ΠŸΠΎΡΠΊΠΎΠ»ΡŒΠΊΡƒ Ρ‚Ρ€Π΅ΡƒΠ³ΠΎΠ»ΡŒΠ½ΠΈΠΊ АВБ — ΠΏΡ€ΡΠΌΠΎΡƒΠ³ΠΎΠ»ΡŒΠ½Ρ‹ΠΉ, Ρ‚ΠΎΒ βˆ  АВБ + ∠ ВАБ = 90Β°.

Π’ΠΎΠ³Π΄Π° Ο† = (∠ АВБ + ∠ ВАБ) = 90Β° : 2 = 45Β°.

ΠžΡ‚Π²Π΅Ρ‚: 45.

2. ΠžΡΡ‚Ρ€Ρ‹Π΅ ΡƒΠ³Π»Ρ‹ ΠΏΡ€ΡΠΌΠΎΡƒΠ³ΠΎΠ»ΡŒΠ½ΠΎΠ³ΠΎ Ρ‚Ρ€Π΅ΡƒΠ³ΠΎΠ»ΡŒΠ½ΠΈΠΊΠ° Ρ€Π°Π²Π½Ρ‹ 29ΒΊ ΠΈΒ 61ΒΊ. НайдитС ΡƒΠ³ΠΎΠ» ΠΌΠ΅ΠΆΠ΄Ρƒ высотой ΠΈ биссСктрисой, ΠΏΡ€ΠΎΠ²Π΅Π΄Π΅Π½Π½Ρ‹ΠΌΠΈ ΠΈΠ·Β Π²Π΅Ρ€ΡˆΠΈΠ½Ρ‹ прямого ΡƒΠ³Π»Π°. ΠžΡ‚Π²Π΅Ρ‚ Π΄Π°ΠΉΡ‚Π΅ в градусах.

ΠŸΡƒΡΡ‚ΡŒ БН — высота, провСдСнная ΠΈΠ·Β Π²Π΅Ρ€ΡˆΠΈΠ½Ρ‹ прямого ΡƒΠ³Π»Π° Π‘, Π‘ΠšΒ β€” биссСктриса ΡƒΠ³Π»Π° Π‘.

Π’ΠΎΠ³Π΄Π° ∠ АБН = ∠ АВБ = 61Β°, ∠ АБК = 90Β° : 2 = 45Β°.

Π£Π³ΠΎΠ» ΠΌΠ΅ΠΆΠ΄Ρƒ высотой и биссСктрисой — это ΡƒΠ³ΠΎΠ» КБН.

∠ КБН = ∠ АБН — ∠ АБК = 61° — 45Β° = 16Β°

ΠžΡ‚Π²Π΅Ρ‚: 16.

3. Π”Π²Π° ΡƒΠ³Π»Π° Ρ‚Ρ€Π΅ΡƒΠ³ΠΎΠ»ΡŒΠ½ΠΈΠΊΠ° Ρ€Π°Π²Π½Ρ‹ 58ΒΊ ΠΈΒ 72ΒΊ. НайдитС Ρ‚ΡƒΠΏΠΎΠΉ ΡƒΠ³ΠΎΠ», ΠΊΠΎΡ‚ΠΎΡ€Ρ‹ΠΉ ΠΎΠ±Ρ€Π°Π·ΡƒΡŽΡ‚ высоты Ρ‚Ρ€Π΅ΡƒΠ³ΠΎΠ»ΡŒΠ½ΠΈΠΊΠ°, выходящиС ΠΈΠ·Β Π²Π΅Ρ€ΡˆΠΈΠ½ этих ΡƒΠ³Π»ΠΎΠ². ΠžΡ‚Π²Π΅Ρ‚ Π΄Π°ΠΉΡ‚Π΅ Π² градусах.

Π˜Π·Β Ρ‚Ρ€Π΅ΡƒΠ³ΠΎΠ»ΡŒΠ½ΠΈΠΊΠ° АВН (ΡƒΠ³ΠΎΠ» Н — прямой) Π½Π°ΠΉΠ΄Π΅ΠΌ ΡƒΠ³ΠΎΠ» ВАН. ΠžΠ½Β Ρ€Π°Π²Π΅Π½ 18Β°.

Π˜Π·Β Ρ‚Ρ€Π΅ΡƒΠ³ΠΎΠ»ΡŒΠ½ΠΈΠΊΠ° ΠΠ’Πš (ΡƒΠ³ΠΎΠ» ΠšΒ β€” прямой) Π½Π°ΠΉΠ΄Π΅ΠΌ ΡƒΠ³ΠΎΠ» ΠΠ’Πš. ΠžΠ½Β Ρ€Π°Π²Π΅Π½ 32Β°.

Π’Β Ρ‚Ρ€Π΅ΡƒΠ³ΠΎΠ»ΡŒΠ½ΠΈΠΊΠ΅ ΠΠžΠ’ извСстны Π΄Π²Π° ΡƒΠ³Π»Π°. НайдСм Ρ‚Ρ€Π΅Ρ‚ΠΈΠΉ, Ρ‚ΠΎΒ Π΅ΡΡ‚ΡŒ ΡƒΠ³ΠΎΠ» ΠΠžΠ’, ΠΊΠΎΡ‚ΠΎΡ€Ρ‹ΠΉ и являСтся Ρ‚ΡƒΠΏΡ‹ΠΌ ΡƒΠ³Π»ΠΎΠΌ ΠΌΠ΅ΠΆΠ΄Ρƒ высотами Ρ‚Ρ€Π΅ΡƒΠ³ΠΎΠ»ΡŒΠ½ΠΈΠΊΠ° АВБ:

∠ ΠΠžΠ’ = 180° — 18° — 32Β° = 130Β°.

ΠžΡ‚Π²Π΅Ρ‚: 130.

4. Π’Β Ρ‚Ρ€Π΅ΡƒΠ³ΠΎΠ»ΡŒΠ½ΠΈΠΊΠ΅ ABC ΡƒΠ³ΠΎΠ»Β C Ρ€Π°Π²Π΅Π½ 58ΒΊ, ADΒ ΠΈΒ BEΒ β€” биссСктрисы, ΠΏΠ΅Ρ€Π΅ΡΠ΅ΠΊΠ°ΡŽΡ‰ΠΈΠ΅ΡΡ Π²Β Ρ‚ΠΎΡ‡ΠΊΠ΅ O. НайдитС ΡƒΠ³ΠΎΠ» AOB. ΠžΡ‚Π²Π΅Ρ‚ Π΄Π°ΠΉΡ‚Π΅ в градусах.

ΠŸΡƒΡΡ‚ΡŒ Π²Β Ρ‚Ρ€Π΅ΡƒΠ³ΠΎΠ»ΡŒΠ½ΠΈΠΊΠ΅ АВБ ΡƒΠ³ΠΎΠ» ВАБ равСн А, ΡƒΠ³ΠΎΠ» АВБ Ρ€Π°Π²Π΅Π½ Π’.

Рассмотрим Ρ‚Ρ€Π΅ΡƒΠ³ΠΎΠ»ΡŒΠ½ΠΈΠΊ ΠΠžΠ’.

∠ ΠžΠΠ’ = ∠ А

∠ ΠΠ’Πž = ∠ Π’, Ρ‚ΠΎΠ³Π΄Π° ∠ ΠΠžΠ’ = 180° — (∠ А + ∠ Π’). Π˜Π·Β Ρ‚Ρ€Π΅ΡƒΠ³ΠΎΠ»ΡŒΠ½ΠΈΠΊΠ° АВБ ΠΏΠΎΠ»ΡƒΡ‡ΠΈΠΌ, Ρ‡Ρ‚ΠΎ ∠ А + ∠ Π’ = 180° — 58Β° = 122Β°.

Π’ΠΎΠ³Π΄Π° ∠ ΠΠžΠ’ = 180° — (∠ А + ∠ Π’) = 180° — 61Β° = 119Β°.

ΠžΡ‚Π²Π΅Ρ‚: 119Β°.

5. Π’Β Ρ‚Ρ€Π΅ΡƒΠ³ΠΎΠ»ΡŒΠ½ΠΈΠΊΠ΅ ABC ΡƒΠ³ΠΎΠ»Β A Ρ€Π°Π²Π΅Π½ 60ΒΊ, ΡƒΠ³ΠΎΠ»Β B Ρ€Π°Π²Π΅Π½ 82ΒΊ. AD, BEΒ ΠΈΒ CFΒ β€” биссСктрисы, ΠΏΠ΅Ρ€Π΅ΡΠ΅ΠΊΠ°ΡŽΡ‰ΠΈΠ΅ΡΡ Π²Β Ρ‚ΠΎΡ‡ΠΊΠ΅ O. НайдитС ΡƒΠ³ΠΎΠ» AOF. ΠžΡ‚Π²Π΅Ρ‚ Π΄Π°ΠΉΡ‚Π΅ в градусах.

НайдСм ΡƒΠ³ΠΎΠ» АБВ. ΠžΠ½Β Ρ€Π°Π²Π΅Π½ 38Β°.

Вогда ∠ АБF = ∠ ACB = 19°.

Π˜Π·Β Ρ‚Ρ€Π΅ΡƒΠ³ΠΎΠ»ΡŒΠ½ΠΈΠΊΠ° АБF Π½Π°ΠΉΠ΄Π΅ΠΌ ΡƒΠ³ΠΎΠ» AFC. ΠžΠ½Β Ρ€Π°Π²Π΅Π½ 101Β°.

Рассмотрим Ρ‚Ρ€Π΅ΡƒΠ³ΠΎΠ»ΡŒΠ½ΠΈΠΊ АОF.

∠ AFО = 101Β°, ∠ FAO = ∠ ВАБ = 30Β°. Π—Π½Π°Ρ‡ΠΈΡ‚, ∠ AOF = 49Β°.

ΠžΡ‚Π²Π΅Ρ‚: 49.

6. Π’Β Ρ‚Ρ€Π΅ΡƒΠ³ΠΎΠ»ΡŒΠ½ΠΈΠΊΠ΅ АВБ Π‘DΒ β€” ΠΌΠ΅Π΄ΠΈΠ°Π½Π°, ΡƒΠ³ΠΎΠ» ACB Ρ€Π°Π²Π΅Π½ 90ΒΊ, ΡƒΠ³ΠΎΠ»Β B Ρ€Π°Π²Π΅Π½ 58ΒΊ. НайдитС ΡƒΠ³ΠΎΠ» ACD. ΠžΡ‚Π²Π΅Ρ‚ Π΄Π°ΠΉΡ‚Π΅ в градусах.

Как Ρ€Π΅ΡˆΠ°Ρ‚ΡŒ эту Π·Π°Π΄Π°Ρ‡Ρƒ? Π£Β ΠΌΠ΅Π΄ΠΈΠ°Π½Ρ‹ ΠΏΡ€ΡΠΌΠΎΡƒΠ³ΠΎΠ»ΡŒΠ½ΠΎΠ³ΠΎ Ρ‚Ρ€Π΅ΡƒΠ³ΠΎΠ»ΡŒΠ½ΠΈΠΊΠ°, ΠΏΡ€ΠΎΠ²Π΅Π΄Π΅Π½Π½ΠΎΠΉ ΠΈΠ·Β Π²Π΅Ρ€ΡˆΠΈΠ½Ρ‹ прямого ΡƒΠ³Π»Π°, Π΅ΡΡ‚ΡŒ особоС свойство. ΠœΡ‹Β Π΄ΠΎΠΊΠ°ΠΆΠ΅ΠΌ Π΅Π³ΠΎ Π²Β Ρ‚Π΅ΠΌΠ΅ Β«ΠŸΡ€ΡΠΌΠΎΡƒΠ³ΠΎΠ»ΡŒΠ½ΠΈΠΊ ΠΈΒ Π΅Π³ΠΎ свойства».

Подсказка: Π‘Π΄Π΅Π»Π°ΠΉΡ‚Π΅ Ρ‡Π΅Ρ€Ρ‚Π΅ΠΆ, Π½Π°ΠΉΠ΄ΠΈΡ‚Π΅ Π½Π°Β Π½Π΅ΠΌ Ρ€Π°Π²Π½ΠΎΠ±Π΅Π΄Ρ€Π΅Π½Π½Ρ‹Π΅ Ρ‚Ρ€Π΅ΡƒΠ³ΠΎΠ»ΡŒΠ½ΠΈΠΊΠΈ ΠΈΒ Π΄ΠΎΠΊΠ°ΠΆΠΈΡ‚Π΅, Ρ‡Ρ‚ΠΎ ΠΎΠ½ΠΈ Ρ€Π°Π²Π½ΠΎΠ±Π΅Π΄Ρ€Π΅Π½Π½Ρ‹Π΅.

ΠŸΡ€Π°Π²ΠΈΠ»ΡŒΠ½Ρ‹ΠΉ ΠΎΡ‚Π²Π΅Ρ‚: 22.

Π§Π΅Ρ‚Ρ‹Ρ€Π΅Ρ…ΡƒΠ³ΠΎΠ»ΡŒΠ½ΠΈΠΊΠΈ. Π‘ΡƒΠΌΠΌΠ° ΡƒΠ³Π»ΠΎΠ² Ρ‡Π΅Ρ‚Ρ‹Ρ€Π΅Ρ…ΡƒΠ³ΠΎΠ»ΡŒΠ½ΠΈΠΊΠ°. ΠŸΠ°Ρ€Π°Π»Π»Π΅Π»ΠΎΠ³Ρ€Π°ΠΌΠΌ. Π’ΠΈΠ΄Ρ‹ ΠΏΠ°Ρ€Π°Π»Π»Π΅Π»ΠΎΠ³Ρ€Π°ΠΌΠΌΠΎΠ² и их свойства. Π ΠΎΠΌΠ±, ΠΏΡ€ΡΠΌΠΎΡƒΠ³ΠΎΠ»ΡŒΠ½ΠΈΠΊ, ΠΊΠ²Π°Π΄Ρ€Π°Ρ‚. ВрапСция и СС свойства

Π’Β ΡˆΠΊΠΎΠ»ΡŒΠ½Ρ‹Ρ… Π·Π°Π΄Π°Ρ‡Π°Ρ… ΠΏΠΎΒ Π³Π΅ΠΎΠΌΠ΅Ρ‚Ρ€ΠΈΠΈ ΠΌΡ‹Β ΠΎΠ±Ρ‹Ρ‡Π½ΠΎ рассматриваСм Π²Ρ‹ΠΏΡƒΠΊΠ»Ρ‹Π΅ Ρ‡Π΅Ρ‚Ρ‹Ρ€Π΅Ρ…ΡƒΠ³ΠΎΠ»ΡŒΠ½ΠΈΠΊΠΈ.

Π’Β Ρ‡Π΅ΠΌ Ρ€Π°Π·Π½ΠΈΡ†Π° ΠΌΠ΅ΠΆΠ΄Ρƒ Π½ΠΈΠΌΠΈ? Если Π»ΡŽΠ±Ρ‹Π΅ Π΄Π²Π΅ Ρ‚ΠΎΡ‡ΠΊΠΈ Π²Ρ‹ΠΏΡƒΠΊΠ»ΠΎΠ³ΠΎ ΠΌΠ½ΠΎΠ³ΠΎΡƒΠ³ΠΎΠ»ΡŒΠ½ΠΈΠΊΠ° ΡΠΎΠ΅Π΄ΠΈΠ½ΠΈΡ‚ΡŒ ΠΎΡ‚Ρ€Π΅Π·ΠΊΠΎΠΌΒ β€” вСсь ΠΎΡ‚Ρ€Π΅Π·ΠΎΠΊ Π±ΡƒΠ΄Π΅Ρ‚ Π»Π΅ΠΆΠ°Ρ‚ΡŒ Π²Π½ΡƒΡ‚Ρ€ΠΈ ΠΌΠ½ΠΎΠ³ΠΎΡƒΠ³ΠΎΠ»ΡŒΠ½ΠΈΠΊΠ°. Для Π½Π΅Π²Ρ‹ΠΏΡƒΠΊΠ»Ρ‹Ρ… Ρ„ΠΈΠ³ΡƒΡ€ это нС выполняСтся.

Π‘ΡƒΠΌΠΌΠ° ΡƒΠ³Π»ΠΎΠ² Π²Ρ‹ΠΏΡƒΠΊΠ»ΠΎΠ³ΠΎ Ρ‡Π΅Ρ‚Ρ‹Ρ€Π΅Ρ…ΡƒΠ³ΠΎΠ»ΡŒΠ½ΠΈΠΊΠ° Ρ€Π°Π²Π½Π° 360 градусов.

ΠŸΡ€ΠΎΠΈΠ·Π²ΠΎΠ»ΡŒΠ½Ρ‹Π΅ Ρ‡Π΅Ρ‚Ρ‹Ρ€Π΅Ρ…ΡƒΠ³ΠΎΠ»ΡŒΠ½ΠΈΠΊΠΈ Π²Β Π·Π°Π΄Π°Ρ‡Π°Ρ… ΠΏΠΎΒ Π³Π΅ΠΎΠΌΠ΅Ρ‚Ρ€ΠΈΠΈ Π²ΡΡ‚Ρ€Π΅Ρ‡Π°ΡŽΡ‚ΡΡ Ρ€Π΅Π΄ΠΊΠΎ. Намного Ρ‡Π°Ρ‰Π΅Β β€” Ρ‚Π°ΠΊΠΈΠ΅, ΡƒΒ ΠΊΠΎΡ‚ΠΎΡ€Ρ‹Ρ… Π΅ΡΡ‚ΡŒ ΠΏΠ°Ρ€Π°Π»Π»Π΅Π»ΡŒΠ½Ρ‹Π΅ стороны. Π­Ρ‚ΠΎ ΠΏΠ°Ρ€Π°Π»Π»Π΅Π»ΠΎΠ³Ρ€Π°ΠΌΠΌ, Ρ€ΠΎΠΌΠ±, ΠΊΠ²Π°Π΄Ρ€Π°Ρ‚, ΠΏΡ€ΡΠΌΠΎΡƒΠ³ΠΎΠ»ΡŒΠ½ΠΈΠΊ и трапСция. Π—Π΄Π΅ΡΡŒ Π²Β Ρ‚Π°Π±Π»ΠΈΡ†Π΅ собраны их опрСдСлСния и свойства.

Бинус ΠΈ косинус — тригономСтричСскиС Ρ„ΡƒΠ½ΠΊΡ†ΠΈΠΈ y=sin(x), y=cos(x). Бвойства, ΠΎΠ±Π»Π°ΡΡ‚ΡŒ опрСдСлСния, значСния, Ρ‡Π΅Ρ‚Π½ΠΎΡΡ‚ΡŒ, ΠΏΠ΅Ρ€ΠΈΠΎΠ΄Ρ‹, Π½ΡƒΠ»ΠΈ, ΠΏΡ€ΠΎΠΌΠ΅ΠΆΡƒΡ‚ΠΊΠΈ знакопостоянства, возрастаниС, ΡƒΠ±Ρ‹Π²Π°Π½ΠΈΠ΅, ΠΌΠΈΠ½ΠΈΠΌΡƒΠΌΡ‹, максимумы, основныС значСния, Π·Π½Π°ΠΊΠΈ, Ρ„ΠΎΡ€ΠΌΡƒΠ»Ρ‹ привСдСния





АдрСс этой страницы (Π²Π»ΠΎΠΆΠ΅Π½Π½ΠΎΡΡ‚ΡŒ) Π² справочникС dpva. ru:  главная страница  /Β / ВСхничСская информация /Β /Β ΠœΠ°Ρ‚Π΅ΠΌΠ°Ρ‚ΠΈΡ‡Π΅ΡΠΊΠΈΠΉ справочник /Β / ВригономСтричСскиС Ρ„ΡƒΠ½ΠΊΡ†ΠΈΠΈ, Ρ„ΠΎΡ€ΠΌΡƒΠ»Ρ‹ ΠΈ Π³Ρ€Π°Ρ„ΠΈΠΊΠΈ. sin, cos, tg, ctg….ЗначСния тригономСтричСских Ρ„ΡƒΠ½ΠΊΡ†ΠΈΠΉ. Π€ΠΎΡ€ΠΌΡƒΠ»Ρ‹ привСдСния тригономСтричСских Ρ„ΡƒΠ½ΠΊΡ†ΠΈΠΉ. ВригономСтричСскиС тоТдСства. Β /Β / Бинус ΠΈ косинус — тригономСтричСскиС Ρ„ΡƒΠ½ΠΊΡ†ΠΈΠΈ y=sin(x), y=cos(x). Бвойства, ΠΎΠ±Π»Π°ΡΡ‚ΡŒ опрСдСлСния, значСния, Ρ‡Π΅Ρ‚Π½ΠΎΡΡ‚ΡŒ, ΠΏΠ΅Ρ€ΠΈΠΎΠ΄Ρ‹, Π½ΡƒΠ»ΠΈ, ΠΏΡ€ΠΎΠΌΠ΅ΠΆΡƒΡ‚ΠΊΠΈ знакопостоянства, возрастаниС, ΡƒΠ±Ρ‹Π²Π°Π½ΠΈΠ΅, ΠΌΠΈΠ½ΠΈΠΌΡƒΠΌΡ‹, максимумы, основныС значСния, Π·Π½Π°ΠΊΠΈ, Ρ„ΠΎΡ€ΠΌΡƒΠ»Ρ‹ привСдСния

ΠŸΠΎΠ΄Π΅Π»ΠΈΡ‚ΡŒΡΡ:Β  Β 

Бинус (sin) ΠΈ косинус (cos) — тригономСтричСскиС Ρ„ΡƒΠ½ΠΊΡ†ΠΈΠΈ

y=sin(x), y=cos(x). Бвойства, ΠΎΠ±Π»Π°ΡΡ‚ΡŒ опрСдСлСния, ΠΎΠ±Π»Π°ΡΡ‚ΡŒ значСния, Ρ‡Π΅Ρ‚Π½ΠΎΡΡ‚ΡŒ, ΠΏΠ΅Ρ€ΠΈΠΎΠ΄Ρ‹, Π½ΡƒΠ»ΠΈ, ΠΏΡ€ΠΎΠΌΠ΅ΠΆΡƒΡ‚ΠΊΠΈ знакопостоянства, возрастаниС, ΡƒΠ±Ρ‹Π²Π°Π½ΠΈΠ΅, ΠΌΠΈΠ½ΠΈΠΌΡƒΠΌΡ‹, максимумы, основныС значСния, Π·Π½Π°ΠΊΠΈ ΠΏΠΎ чСтвСртям, Ρ„ΠΎΡ€ΠΌΡƒΠ»Ρ‹ привСдСния.

Бвойства, ΠΎΠ±Π»Π°ΡΡ‚ΡŒ опрСдСлСния, ΠΎΠ±Π»Π°ΡΡ‚ΡŒ значСния, Ρ‡Π΅Ρ‚Π½ΠΎΡΡ‚ΡŒ, ΠΏΠ΅Ρ€ΠΈΠΎΠ΄Ρ‹, Π½ΡƒΠ»ΠΈ, ΠΏΡ€ΠΎΠΌΠ΅ΠΆΡƒΡ‚ΠΊΠΈ знакопостоянства, возрастаниС, ΡƒΠ±Ρ‹Π²Π°Π½ΠΈΠ΅, ΠΌΠΈΠ½ΠΈΠΌΡƒΠΌΡ‹, максимумы, основныС значСния:

Бвойства, ΠΎΠ±Π»Π°ΡΡ‚ΡŒ опрСдСлСния, ΠΎΠ±Π»Π°ΡΡ‚ΡŒ значСния, Ρ‡Π΅Ρ‚Π½ΠΎΡΡ‚ΡŒ, ΠΏΠ΅Ρ€ΠΈΠΎΠ΄Ρ‹, Π½ΡƒΠ»ΠΈ, ΠΏΡ€ΠΎΠΌΠ΅ΠΆΡƒΡ‚ΠΊΠΈ знакопостоянства, возрастаниС, ΡƒΠ±Ρ‹Π²Π°Π½ΠΈΠ΅, ΠΌΠΈΠ½ΠΈΠΌΡƒΠΌΡ‹, максимумы, основныС значСния:

  • ΠžΠ±Π»Π°ΡΡ‚ΡŒ опрСдСлСния D(y):
  • ΠžΠ±Π»Π°ΡΡ‚ΡŒ Π·Π½Π°Ρ‡Π΅Π½ΠΈΠΉ E(x):
  • НаимСньший ΠΏΠΎΠ»ΠΎΠΆΠΈΡ‚Π΅Π»ΡŒΠ½Ρ‹ΠΉ ΠΏΠ΅Ρ€ΠΈΠΎΠ΄:
  • ΠšΠΎΠΎΡ€Π΄ΠΈΠ½Π°Ρ‚Ρ‹ Ρ‚ΠΎΡ‡Π΅ΠΊ пСрСсСчСния Π³Ρ€Π°Ρ„ΠΈΠΊΠ° Ρ„ΡƒΠ½ΠΊΡ†ΠΈΠΈ с осью:
  • ΠŸΡ€ΠΎΠΌΠ΅ΠΆΡƒΡ‚ΠΊΠΈ знакопостоянства —Β  Π½Π° ΠΊΠΎΡ‚ΠΎΡ€Ρ‹Ρ… функция ΠΏΡ€ΠΈΠ½ΠΈΠΌΠ°Π΅Ρ‚:
  • ΠŸΠΎΠ»ΠΎΠΆΠΈΡ‚Π΅Π»ΡŒΠ½Ρ‹Π΅ значСния:
  • ΠžΡ‚Ρ€ΠΈΡ†ΠΏΡ‚Π΅Π»ΡŒΠ½Ρ‹Π΅ значСния:
  • ΠŸΡ€ΠΎΠΌΠ΅ΠΆΡƒΡ‚ΠΊΠΈ возрастания:
  • ΠŸΡ€ΠΎΠΌΠ΅ΠΆΡƒΡ‚ΠΊΠΈ убывания:
  • Π’ΠΎΡ‡ΠΊΠΈ ΠΌΠΈΠ½ΠΈΠΌΡƒΠΌΠ°:
  • ΠœΠ½ΠΈΠΌΡƒΠΌΡ‹ Ρ„ΡƒΠ½ΠΊΡ†ΠΈΠΈ:
  • Π’ΠΎΡ‡ΠΊΠΈ максимума:
  • ΠœΠ°ΠΊΡΠΈΠΌΡƒΠΌΡ‹ Ρ„ΡƒΠ½ΠΊΡ†ΠΈΠΈ:

ΠŸΠ΅Ρ€Π΅Π²ΠΎΠ΄ градусной ΠΌΠ΅Ρ€Ρ‹ ΡƒΠ³Π»Π° Π² Ρ€Π°Π΄ΠΈΠ°Π½Π½ΡƒΡŽ ΠΈ ΠΎΠ±Ρ€Π°Ρ‚Π½ΠΎ

ΠΏΠΎΠ΄Ρ€ΠΎΠ±Π½Π΅Π΅:

ЗначСния тригономСтричСских Ρ„ΡƒΠ½ΠΊΡ†ΠΈΠΉ Π½Π΅ΠΊΠΎΡ‚ΠΎΡ€Ρ‹Ρ… ΡƒΠ³Π»ΠΎΠ²:

Π—Π½Π°ΠΊΠΈ Π·Π½Π°Ρ‡Π΅Π½ΠΈΠΉ тригономСтричСских Ρ„ΡƒΠ½ΠΊΡ†ΠΈΠΉ:

Π€ΠΎΡ€ΠΌΡƒΠ»Ρ‹ привСдСния тригономСтричСских Ρ„ΡƒΠ½ΠΊΡ†ΠΈΠΉ

ΠΏΠΎΠ΄Ρ€ΠΎΠ±Π½Π΅Π΅:
Поиск Π² ΠΈΠ½ΠΆΠ΅Π½Π΅Ρ€Π½ΠΎΠΌ справочникС DPVA. Π’Π²Π΅Π΄ΠΈΡ‚Π΅ свой запрос:
Поиск Π² ΠΈΠ½ΠΆΠ΅Π½Π΅Ρ€Π½ΠΎΠΌ справочникС DPVA. Π’Π²Π΅Π΄ΠΈΡ‚Π΅ свой запрос:

3 совСта экспСрта ΠΏΠΎ использованию Π΅Π΄ΠΈΠ½ΠΈΡ‡Π½ΠΎΠΉ окруТности

Если Π²Ρ‹ ΠΈΠ·ΡƒΡ‡Π°Π΅Ρ‚Π΅ Ρ‚Ρ€ΠΈΠ³ΠΎΠ½ΠΎΠΌΠ΅Ρ‚Ρ€ΠΈΡ‡Π΅ΡΠΊΡƒΡŽ систСму ΠΈΠ»ΠΈ ΠΌΠ°Ρ‚Π΅ΠΌΠ°Ρ‚ΠΈΠΊΡƒ — ΠΈΠ»ΠΈ Π³ΠΎΡ‚ΠΎΠ²ΠΈΡ‚Π΅ΡΡŒ ΠΊ этому — Π²Π°ΠΌ Π½ΡƒΠΆΠ½ΠΎ Π±ΡƒΠ΄Π΅Ρ‚ ΠΏΠΎΠ·Π½Π°ΠΊΠΎΠΌΠΈΡ‚ΡŒΡΡ с Π΅Π΄ΠΈΠ½ΠΈΡ‡Π½Ρ‹ΠΌ ΠΊΡ€ΡƒΠ³ΠΎΠΌ. Единичная ΠΎΠΊΡ€ΡƒΠΆΠ½ΠΎΡΡ‚ΡŒ — это , Π²Π°ΠΆΠ½Ρ‹ΠΉ инструмСнт, ΠΈΡΠΏΠΎΠ»ΡŒΠ·ΡƒΠ΅ΠΌΡ‹ΠΉ для нахоТдСния синуса, косинуса ΠΈ тангСнса ΡƒΠ³Π»Π°. Π° ΠΊΠ°ΠΊ это Ρ€Π°Π±ΠΎΡ‚Π°Π΅Ρ‚? И ΠΊΠ°ΠΊΡƒΡŽ ΠΈΠ½Ρ„ΠΎΡ€ΠΌΠ°Ρ†ΠΈΡŽ Π½ΡƒΠΆΠ½ΠΎ Π·Π½Π°Ρ‚ΡŒ, Ρ‡Ρ‚ΠΎΠ±Ρ‹ Сю ΠΏΠΎΠ»ΡŒΠ·ΠΎΠ²Π°Ρ‚ΡŒΡΡ?

Π’ этой ΡΡ‚Π°Ρ‚ΡŒΠ΅ ΠΌΡ‹ объясним, Ρ‡Ρ‚ΠΎ Ρ‚Π°ΠΊΠΎΠ΅ Π΅Π΄ΠΈΠ½ΠΈΡ‡Π½Ρ‹ΠΉ ΠΊΡ€ΡƒΠ³ ΠΈ ΠΏΠΎΡ‡Π΅ΠΌΡƒ Π²Ρ‹ Π΄ΠΎΠ»ΠΆΠ½Ρ‹ Π΅Π³ΠΎ Π·Π½Π°Ρ‚ΡŒ. ΠœΡ‹ Ρ‚Π°ΠΊΠΆΠ΅ Π΄Π°Π΅ΠΌ Π²Π°ΠΌ Ρ‚Ρ€ΠΈ совСта, ΠΊΠΎΡ‚ΠΎΡ€Ρ‹Π΅ ΠΏΠΎΠΌΠΎΠ³ΡƒΡ‚ Π²Π°ΠΌ Π·Π°ΠΏΠΎΠΌΠ½ΠΈΡ‚ΡŒ, ΠΊΠ°ΠΊ ΠΈΡΠΏΠΎΠ»ΡŒΠ·ΠΎΠ²Π°Ρ‚ΡŒ Π΅Π΄ΠΈΠ½ΠΈΡ‡Π½Ρ‹ΠΉ ΠΊΡ€ΡƒΠ³.

Π˜Π·ΠΎΠ±Ρ€Π°ΠΆΠ΅Π½ΠΈΠ΅ Ρ„ΡƒΠ½ΠΊΡ†ΠΈΠΈ: Gustavb / Wikimedia

Π•Π΄ΠΈΠ½ΠΈΡ‡Π½Ρ‹ΠΉ ΠΊΡ€ΡƒΠ³: Π±Π°Π·ΠΎΠ²ΠΎΠ΅ Π²Π²Π΅Π΄Π΅Π½ΠΈΠ΅

Π•Π΄ΠΈΠ½ΠΈΡ‡Π½Ρ‹ΠΉ ΠΊΡ€ΡƒΠ³ — это ΠΊΡ€ΡƒΠ³ с радиусом 1. Π­Ρ‚ΠΎ ΠΎΠ·Π½Π°Ρ‡Π°Π΅Ρ‚, Ρ‡Ρ‚ΠΎ для любой прямой Π»ΠΈΠ½ΠΈΠΈ, ΠΏΡ€ΠΎΠ²Π΅Π΄Π΅Π½Π½ΠΎΠΉ ΠΎΡ‚ Ρ†Π΅Π½Ρ‚Ρ€Π°Π»ΡŒΠ½ΠΎΠΉ Ρ‚ΠΎΡ‡ΠΊΠΈ ΠΊΡ€ΡƒΠ³Π° ΠΊ любой Ρ‚ΠΎΡ‡ΠΊΠ΅ Π½Π° ΠΊΡ€Π°ΡŽ ΠΊΡ€ΡƒΠ³Π°, Π΄Π»ΠΈΠ½Π° этой Π»ΠΈΠ½ΠΈΠΈ всСгда Π±ΡƒΠ΄Π΅Ρ‚ Ρ€Π°Π²Π½Π° 1. (Π­Ρ‚ΠΎ Ρ‚Π°ΠΊΠΆΠ΅ ΠΎΠ·Π½Π°Ρ‡Π°Π΅Ρ‚, Ρ‡Ρ‚ΠΎ Π΄ΠΈΠ°ΠΌΠ΅Ρ‚Ρ€ ΠΊΡ€ΡƒΠ³Π° Π±ΡƒΠ΄Π΅Ρ‚ Ρ€Π°Π²Π΅Π½ 2, ΠΏΠΎΡΠΊΠΎΠ»ΡŒΠΊΡƒ Π΄ΠΈΠ°ΠΌΠ΅Ρ‚Ρ€ Ρ€Π°Π²Π΅Π½ ΡƒΠ΄Π²ΠΎΠ΅Π½Π½ΠΎΠΉ Π΄Π»ΠΈΠ½Π΅ радиуса.)

Как ΠΏΡ€Π°Π²ΠΈΠ»ΠΎ, Ρ†Π΅Π½Ρ‚Ρ€Π°Π»ΡŒΠ½Π°Ρ Ρ‚ΠΎΡ‡ΠΊΠ° Π΅Π΄ΠΈΠ½ΠΈΡ‡Π½ΠΎΠΉ окруТности находится Ρ‚Π°ΠΌ, Π³Π΄Π΅ ΠΏΠ΅Ρ€Π΅ΡΠ΅ΠΊΠ°ΡŽΡ‚ΡΡ оси x ΠΈ y, ΠΈΠ»ΠΈ Π² ΠΊΠΎΠΎΡ€Π΄ΠΈΠ½Π°Ρ‚Π°Ρ… (0, 0):

Единичная ΠΎΠΊΡ€ΡƒΠΆΠ½ΠΎΡΡ‚ΡŒ, ΠΈΠ»ΠΈ триггСрная ΠΎΠΊΡ€ΡƒΠΆΠ½ΠΎΡΡ‚ΡŒ, ΠΊΠ°ΠΊ Π΅Π΅ Π΅Ρ‰Π΅ Π½Π°Π·Ρ‹Π²Π°ΡŽΡ‚, ΠΏΠΎΠ»Π΅Π·Π½ΠΎ Π·Π½Π°Ρ‚ΡŒ, ΠΏΠΎΡ‚ΠΎΠΌΡƒ Ρ‡Ρ‚ΠΎ позволяСт Π½Π°ΠΌ Π»Π΅Π³ΠΊΠΎ Π²Ρ‹Ρ‡ΠΈΡΠ»ΠΈΡ‚ΡŒ косинус, синус ΠΈ тангСнс любого ΡƒΠ³Π»Π° ΠΌΠ΅ΠΆΠ΄Ρƒ 0 Β° ΠΈ 360 Β° (ΠΈΠ»ΠΈ 0 ΠΈ 2Ο€ Ρ€Π°Π΄ΠΈΠ°Π½).

Как Π²Ρ‹ ΠΌΠΎΠΆΠ΅Ρ‚Π΅ Π²ΠΈΠ΄Π΅Ρ‚ΡŒ Π½Π° ΠΏΡ€ΠΈΠ²Π΅Π΄Π΅Π½Π½ΠΎΠΉ Π²Ρ‹ΡˆΠ΅ Π΄ΠΈΠ°Π³Ρ€Π°ΠΌΠΌΠ΅, нарисовав радиус ΠΏΠΎΠ΄ Π»ΡŽΠ±Ρ‹ΠΌ ΡƒΠ³Π»ΠΎΠΌ (ΠΎΡ‚ΠΌΠ΅Ρ‡Π΅Π½ Π½Π° ΠΈΠ·ΠΎΠ±Ρ€Π°ΠΆΠ΅Π½ΠΈΠΈ by), Π²Ρ‹ создадитС ΠΏΡ€ΡΠΌΠΎΡƒΠ³ΠΎΠ»ΡŒΠ½Ρ‹ΠΉ Ρ‚Ρ€Π΅ΡƒΠ³ΠΎΠ»ΡŒΠ½ΠΈΠΊ. На этом Ρ‚Ρ€Π΅ΡƒΠ³ΠΎΠ»ΡŒΠ½ΠΈΠΊΠ΅ косинус — это Π³ΠΎΡ€ΠΈΠ·ΠΎΠ½Ρ‚Π°Π»ΡŒΠ½Π°Ρ линия, Π° синус — это Π²Π΅Ρ€Ρ‚ΠΈΠΊΠ°Π»ΡŒΠ½Π°Ρ линия. Π”Ρ€ΡƒΠ³ΠΈΠΌΠΈ словами, косинус = x-ΠΊΠΎΠΎΡ€Π΄ΠΈΠ½Π°Ρ‚Π°, Π° синус = y-ΠΊΠΎΠΎΡ€Π΄ΠΈΠ½Π°Ρ‚Π°. (Бамая длинная линия Ρ‚Ρ€Π΅ΡƒΠ³ΠΎΠ»ΡŒΠ½ΠΈΠΊΠ°, ΠΈΠ»ΠΈ Π³ΠΈΠΏΠΎΡ‚Π΅Π½ΡƒΠ·Π°, являСтся радиусом ΠΈ поэтому Ρ€Π°Π²Π½Π° 1.)

ΠŸΠΎΡ‡Π΅ΠΌΡƒ всС это Ρ‚Π°ΠΊ Π²Π°ΠΆΠ½ΠΎ? ΠŸΠΎΠΌΠ½ΠΈΡ‚Π΅, Ρ‡Ρ‚ΠΎ Π²Ρ‹ ΠΌΠΎΠΆΠ΅Ρ‚Π΅ Π½Π°ΠΉΡ‚ΠΈ Π΄Π»ΠΈΠ½Ρ‹ сторон Ρ‚Ρ€Π΅ΡƒΠ³ΠΎΠ»ΡŒΠ½ΠΈΠΊΠ°, ΠΈΡΠΏΠΎΠ»ΡŒΠ·ΡƒΡ Ρ‚Π΅ΠΎΡ€Π΅ΠΌΡƒ ΠŸΠΈΡ„Π°Π³ΠΎΡ€Π° , ΠΈΠ»ΠΈ $ a ^ 2 + b ^ 2 = c ^ 2 $ (Π³Π΄Π΅ a ΠΈ b — Π΄Π»ΠΈΠ½Ρ‹ сторон Ρ‚Ρ€Π΅ΡƒΠ³ΠΎΠ»ΡŒΠ½ΠΈΠΊΠ°, Π° c — Π΄Π»ΠΈΠ½Π° Π³ΠΈΠΏΠΎΡ‚Π΅Π½ΡƒΠ·Ρ‹). 2ΞΈ = 1 $$

Π˜ΠΌΠ΅ΠΉΡ‚Π΅ Π² Π²ΠΈΠ΄Ρƒ, Ρ‡Ρ‚ΠΎ эти значСния ΠΌΠΎΠ³ΡƒΡ‚ Π±Ρ‹Ρ‚ΡŒ ΠΎΡ‚Ρ€ΠΈΡ†Π°Ρ‚Π΅Π»ΡŒΠ½Ρ‹ΠΌΠΈ Π² зависимости ΠΎΡ‚ ΠΎΠ±Ρ€Π°Π·ΠΎΠ²Π°Π½Π½ΠΎΠ³ΠΎ ΡƒΠ³Π»Π° ΠΈ Π² ΠΊΠ°ΠΊΠΎΠΉ ΠΊΠ²Π°Π΄Ρ€Π°Π½Ρ‚ ΠΏΠΎΠΏΠ°Π΄Π°ΡŽΡ‚ ΠΊΠΎΠΎΡ€Π΄ΠΈΠ½Π°Ρ‚Ρ‹ x ΠΈ y (Π±ΠΎΠ»Π΅Π΅ ΠΏΠΎΠ΄Ρ€ΠΎΠ±Π½ΠΎ я объясню это ΠΏΠΎΠ·ΠΆΠ΅).

Π’ΠΎΡ‚ ΠΎΠ±Π·ΠΎΡ€ всСх основных ΡƒΠ³Π»ΠΎΠ² Π² градусах ΠΈ Ρ€Π°Π΄ΠΈΠ°Π½Π°Ρ… Π½Π° Π΅Π΄ΠΈΠ½ΠΈΡ‡Π½ΠΎΠΉ окруТности:

Π•Π΄ΠΈΠ½ΠΈΡ‡Π½Ρ‹ΠΉ ΠΊΡ€ΡƒΠ³ — градусы

Π•Π΄ΠΈΠ½ΠΈΡ‡Π½Ρ‹ΠΉ ΠΊΡ€ΡƒΠ³ — Ρ€Π°Π΄ΠΈΠ°Π½Ρ‹

А Ρ‡Ρ‚ΠΎ, Ссли Ρ‚Ρ€Π΅ΡƒΠ³ΠΎΠ»ΡŒΠ½ΠΈΠΊ Π½Π΅ образовался? Π”Π°Π²Π°ΠΉΡ‚Π΅ посмотрим Π½Π° , Ρ‡Ρ‚ΠΎ происходит, ΠΊΠΎΠ³Π΄Π° ΡƒΠ³ΠΎΠ» Ρ€Π°Π²Π΅Π½ 0 Β°, образуя Π³ΠΎΡ€ΠΈΠ·ΠΎΠ½Ρ‚Π°Π»ΡŒΠ½ΡƒΡŽ ΠΏΡ€ΡΠΌΡƒΡŽ линию вдоль оси x:

Π’ этой строкС ΠΊΠΎΠΎΡ€Π΄ΠΈΠ½Π°Ρ‚Π° x Ρ€Π°Π²Π½Π° 1, Π° ΠΊΠΎΠΎΡ€Π΄ΠΈΠ½Π°Ρ‚Π° y Ρ€Π°Π²Π½Π° 0.ΠœΡ‹ Π·Π½Π°Π΅ΠΌ, Ρ‡Ρ‚ΠΎ косинус Ρ€Π°Π²Π΅Π½ ΠΊΠΎΠΎΡ€Π΄ΠΈΠ½Π°Ρ‚Π΅ x, Π° синус Ρ€Π°Π²Π΅Π½ ΠΊΠΎΠΎΡ€Π΄ΠΈΠ½Π°Ρ‚Π΅ y, , поэтому ΠΌΡ‹ ΠΌΠΎΠΆΠ΅ΠΌ Π·Π°ΠΏΠΈΡΠ°Ρ‚ΡŒ это:

Π§Ρ‚ΠΎ Ссли ΡƒΠ³ΠΎΠ» Ρ€Π°Π²Π΅Π½ 90 Β° ΠΈ ΠΎΠ±Ρ€Π°Π·ΡƒΠ΅Ρ‚ идСально Π²Π΅Ρ€Ρ‚ΠΈΠΊΠ°Π»ΡŒΠ½ΡƒΡŽ линию вдоль оси y?

Π—Π΄Π΅ΡΡŒ ΠΌΡ‹ Π²ΠΈΠ΄ΠΈΠΌ, Ρ‡Ρ‚ΠΎ ΠΊΠΎΠΎΡ€Π΄ΠΈΠ½Π°Ρ‚Π° x Ρ€Π°Π²Π½Π° 0, Π° ΠΊΠΎΠΎΡ€Π΄ΠΈΠ½Π°Ρ‚Π° y Ρ€Π°Π²Π½Π° 1. Π­Ρ‚ΠΎ Π΄Π°Π΅Ρ‚ Π½Π°ΠΌ ΡΠ»Π΅Π΄ΡƒΡŽΡ‰ΠΈΠ΅ значСния для синуса ΠΈ косинуса:

  • $ \ cos90 Β° = 0 $
  • $ \ sin90 Β° = 1 $

Π­Ρ‚ΠΎΡ‚ слоган ΠΎΠΏΡ€Π΅Π΄Π΅Π»Π΅Π½Π½ΠΎ ΠΏΠΎΠ΄Ρ…ΠΎΠ΄ΠΈΡ‚, Ссли Π²Ρ‹ Π½Π΅ Π»ΡŽΠ±ΠΈΡ‚Π΅Π»ΡŒ ΠΌΠ°Ρ‚Π΅ΠΌΠ°Ρ‚ΠΈΠΊΠΈ.

ΠŸΠΎΡ‡Π΅ΠΌΡƒ Π²Ρ‹ Π΄ΠΎΠ»ΠΆΠ½Ρ‹ Π·Π½Π°Ρ‚ΡŒ Π΅Π΄ΠΈΠ½ΠΈΡ‡Π½ΡƒΡŽ ΠΎΠΊΡ€ΡƒΠΆΠ½ΠΎΡΡ‚ΡŒ

Как сказано Π²Ρ‹ΡˆΠ΅, Сдиничная ΠΎΠΊΡ€ΡƒΠΆΠ½ΠΎΡΡ‚ΡŒ ΠΏΠΎΠ»Π΅Π·Π½Π°, ΠΏΠΎΡ‚ΠΎΠΌΡƒ Ρ‡Ρ‚ΠΎ позволяСт Π½Π°ΠΌ Π»Π΅Π³ΠΊΠΎ Π½Π°ΠΉΡ‚ΠΈ синус, косинус ΠΈΠ»ΠΈ тангСнс любого градуса ΠΈΠ»ΠΈ Ρ€Π°Π΄ΠΈΠ°Π½Π°. ОсобСнно ΠΏΠΎΠ»Π΅Π·Π½ΠΎ Π·Π½Π°Ρ‚ΡŒ ΠΊΡ€ΡƒΠ³ΠΎΠ²ΡƒΡŽ Π΄ΠΈΠ°Π³Ρ€Π°ΠΌΠΌΡƒ Π΅Π΄ΠΈΠ½ΠΈΡ†, Ссли Π²Π°ΠΌ Π½ΡƒΠΆΠ½ΠΎ Ρ€Π΅ΡˆΠΈΡ‚ΡŒ ΠΎΠΏΡ€Π΅Π΄Π΅Π»Π΅Π½Π½Ρ‹Π΅ Ρ‚Ρ€ΠΈΠ³Π³Π΅Ρ€Π½Ρ‹Π΅ значСния для домашнСго задания ΠΏΠΎ ΠΌΠ°Ρ‚Π΅ΠΌΠ°Ρ‚ΠΈΠΊΠ΅ ΠΈΠ»ΠΈ Ссли Π²Ρ‹ Π³ΠΎΡ‚ΠΎΠ²ΠΈΡ‚Π΅ΡΡŒ ΠΊ ΠΈΠ·ΡƒΡ‡Π΅Π½ΠΈΡŽ матСматичСского Π°Π½Π°Π»ΠΈΠ·Π°.

Но ΠΊΠ°ΠΊ ΠΈΠΌΠ΅Π½Π½ΠΎ Π·Π½Π°Π½ΠΈΠ΅ Π΅Π΄ΠΈΠ½ΠΈΡ‡Π½ΠΎΠ³ΠΎ ΠΊΡ€ΡƒΠ³Π° ΠΌΠΎΠΆΠ΅Ρ‚ Π²Π°ΠΌ ΠΏΠΎΠΌΠΎΡ‡ΡŒ? Допустим, Π²Π°ΠΌ Π·Π°Π΄Π°Π»ΠΈ ΡΠ»Π΅Π΄ΡƒΡŽΡ‰ΡƒΡŽ Π·Π°Π΄Π°Ρ‡Ρƒ Π½Π° тСстС ΠΏΠΎ ΠΌΠ°Ρ‚Π΅ΠΌΠ°Ρ‚ΠΈΠΊΠ΅, ΠΈ нСльзя ΠΈΡΠΏΠΎΠ»ΡŒΠ·ΠΎΠ²Π°Ρ‚ΡŒ для Π΅Π΅ Ρ€Π΅ΡˆΠ΅Π½ΠΈΡ с ΠΏΠΎΠΌΠΎΡ‰ΡŒΡŽ ΠΊΠ°Π»ΡŒΠΊΡƒΠ»ΡΡ‚ΠΎΡ€Π° :

$$ \ sin30 Β° $$

Π‘ Ρ‡Π΅Π³ΠΎ Π½Π°Ρ‡Π°Ρ‚ΡŒ? Π”Π°Π²Π°ΠΉΡ‚Π΅ снова посмотрим Π½Π° Π΄ΠΈΠ°Π³Ρ€Π°ΠΌΠΌΡƒ Π΅Π΄ΠΈΠ½ΠΈΡ‡Π½ΠΎΠ³ΠΎ ΠΊΡ€ΡƒΠ³Π° — Π½Π° этот Ρ€Π°Π· со всСми большими ΡƒΠ³Π»Π°ΠΌΠΈ (ΠΊΠ°ΠΊ Π² градусах, Ρ‚Π°ΠΊ ΠΈ Π² Ρ€Π°Π΄ΠΈΠ°Π½Π°Ρ…) ΠΈ ΠΈΡ… ΡΠΎΠΎΡ‚Π²Π΅Ρ‚ΡΡ‚Π²ΡƒΡŽΡ‰ΠΈΠΌΠΈ ΠΊΠΎΠΎΡ€Π΄ΠΈΠ½Π°Ρ‚Π°ΠΌΠΈ:

Π”ΠΆΠΈΠΌ. belk / Π’ΠΈΠΊΠΈΠΌΠ΅Π΄ΠΈΠ°

НС Ρ€Π°ΡΡΡ‚Ρ€Π°ΠΈΠ²Π°ΠΉΡ‚Π΅ΡΡŒ! ΠŸΠΎΠΌΠ½ΠΈΡ‚Π΅, всС, Ρ‡Ρ‚ΠΎ Π²Π°ΠΌ Π½ΡƒΠΆΠ½ΠΎ, это $ \ sin30 Β° $. ΠŸΠΎΡΠΌΠΎΡ‚Ρ€Π΅Π² Π½Π° этот Π³Ρ€Π°Ρ„ΠΈΠΊ, ΠΌΡ‹ Π²ΠΈΠ΄ΠΈΠΌ, Ρ‡Ρ‚ΠΎ , ΠΊΠΎΠΎΡ€Π΄ΠΈΠ½Π°Ρ‚Π° y Ρ€Π°Π²Π½Π° $ 1/2 $ ΠΏΡ€ΠΈ 30 Β°. А ΠΏΠΎΡΠΊΠΎΠ»ΡŒΠΊΡƒ ΠΊΠΎΠΎΡ€Π΄ΠΈΠ½Π°Ρ‚Π° y Ρ€Π°Π²Π½Π° синусу, наш ΠΎΡ‚Π²Π΅Ρ‚ Π±ΡƒΠ΄Π΅Ρ‚ ΡΠ»Π΅Π΄ΡƒΡŽΡ‰ΠΈΠΌ:

$$ \ sin30 Β° = 1/2 $$

Но Ρ‡Ρ‚ΠΎ, Ссли Π²Ρ‹ ΡΡ‚ΠΎΠ»ΠΊΠ½Π΅Ρ‚Π΅ΡΡŒ с ΠΏΡ€ΠΎΠ±Π»Π΅ΠΌΠΎΠΉ, Π² ΠΊΠΎΡ‚ΠΎΡ€ΠΎΠΉ вмСсто градусов ΠΈΡΠΏΠΎΠ»ΡŒΠ·ΡƒΡŽΡ‚ΡΡ Ρ€Π°Π΄ΠΈΠ°Π½Ρ‹? ΠŸΡ€ΠΎΡ†Π΅ΡΡ Π΅Π΅ Ρ€Π΅ΡˆΠ΅Π½ΠΈΡ остаСтся ΠΏΡ€Π΅ΠΆΠ½ΠΈΠΌ. Π‘ΠΊΠ°ΠΆΠ΅ΠΌ, Π½Π°ΠΏΡ€ΠΈΠΌΠ΅Ρ€, Ρƒ вас Π²ΠΎΠ·Π½ΠΈΠΊΠ»Π° ΠΏΡ€ΠΎΠ±Π»Π΅ΠΌΠ°, которая выглядит Ρ‚Π°ΠΊ:

$$ \ cos {{3Ο€} / 4} $$

ΠžΠΏΡΡ‚ΡŒ ΠΆΠ΅, ΠΈΡΠΏΠΎΠ»ΡŒΠ·ΡƒΡ Π΄ΠΈΠ°Π³Ρ€Π°ΠΌΠΌΡƒ Π²Ρ‹ΡˆΠ΅, ΠΌΡ‹ ΠΌΠΎΠΆΠ΅ΠΌ Π²ΠΈΠ΄Π΅Ρ‚ΡŒ, Ρ‡Ρ‚ΠΎ ΠΊΠΎΠΎΡ€Π΄ΠΈΠ½Π°Ρ‚Π° x (ΠΈΠ»ΠΈ косинус) для $ {3Ο€} / 4 $ (ΠΊΠΎΡ‚ΠΎΡ€Ρ‹ΠΉ Ρ€Π°Π²Π΅Π½ 135 Β°) составляСт $ — {√2} / 2 $.Π’ΠΎΡ‚ ΠΊΠ°ΠΊ Π±Ρ‹ Ρ‚ΠΎΠ³Π΄Π° выглядСл наш ΠΎΡ‚Π²Π΅Ρ‚ Π½Π° эту ΠΏΡ€ΠΎΠ±Π»Π΅ΠΌΡƒ:

$$ \ cos ({3Ο€} / 4) = — {√2} / 2 $$

ВсС это довольно просто, Ссли Ρƒ вас Π΅ΡΡ‚ΡŒ привСдСнная Π²Ρ‹ΡˆΠ΅ круговая Π΄ΠΈΠ°Π³Ρ€Π°ΠΌΠΌΠ° Π΅Π΄ΠΈΠ½ΠΈΡ† для использования Π² качСствС справочной ΠΈΠ½Ρ„ΠΎΡ€ΠΌΠ°Ρ†ΠΈΠΈ. Но Π² Π±ΠΎΠ»ΡŒΡˆΠΈΠ½ΡΡ‚Π²Π΅ случаСв (Ссли Π½Π΅ всСгда) это Π½Π΅ Ρ‚Π°ΠΊ, ΠΈ Π²Ρ‹ Π΄ΠΎΠ»ΠΆΠ½Ρ‹ Π±ΡƒΠ΄Π΅Ρ‚Π΅ ΠΎΡ‚Π²Π΅Ρ‡Π°Ρ‚ΡŒ Π½Π° Ρ‚Π°ΠΊΠΈΠ΅ матСматичСскиС вопросы, ΠΈΡΠΏΠΎΠ»ΡŒΠ·ΡƒΡ Ρ‚ΠΎΠ»ΡŒΠΊΠΎ свой ΠΌΠΎΠ·Π³.

Π’Π°ΠΊ ΠΊΠ°ΠΊ ΠΆΠ΅ ΠΌΠΎΠΆΠ½ΠΎ Π·Π°ΠΏΠΎΠΌΠ½ΠΈΡ‚ΡŒ Π΅Π΄ΠΈΠ½ΠΈΡ‡Π½Ρ‹ΠΉ ΠΊΡ€ΡƒΠ³? Π§ΠΈΡ‚Π°ΠΉΡ‚Π΅ наши Π³Π»Π°Π²Π½Ρ‹Π΅ совСты!

Как Π·Π°ΠΏΠΎΠΌΠ½ΠΈΡ‚ΡŒ Π΅Π΄ΠΈΠ½ΠΈΡ‡Π½Ρ‹ΠΉ ΠΊΡ€ΡƒΠ³: 3 основных совСта

Π’ этом Ρ€Π°Π·Π΄Π΅Π»Π΅ ΠΌΡ‹ Π΄Π°Π΅ΠΌ Π²Π°ΠΌ нСсколько основных совСтов ΠΏΠΎ запоминанию Ρ‚Ρ€ΠΈΠ³Π³Π΅Ρ€Π½ΠΎΠ³ΠΎ ΠΊΡ€ΡƒΠ³Π°, Ρ‡Ρ‚ΠΎΠ±Ρ‹ Π²Ρ‹ ΠΌΠΎΠ³Π»ΠΈ Π»Π΅Π³ΠΊΠΎ ΠΈΡΠΏΠΎΠ»ΡŒΠ·ΠΎΠ²Π°Ρ‚ΡŒ Π΅Π³ΠΎ для Ρ€Π΅ΡˆΠ΅Π½ΠΈΡ любой матСматичСской Π·Π°Π΄Π°Ρ‡ΠΈ, которая Π² Π½Π΅ΠΌ Ρ‚Ρ€Π΅Π±ΡƒΠ΅Ρ‚.

Π― Π±Ρ‹ Π½Π΅ Ρ€Π΅ΠΊΠΎΠΌΠ΅Π½Π΄ΠΎΠ²Π°Π» ΠΏΡ€Π°ΠΊΡ‚ΠΈΠΊΠΎΠ²Π°Ρ‚ΡŒ Π΅Π΄ΠΈΠ½ΠΈΡ‡Π½Ρ‹ΠΉ ΠΊΡ€ΡƒΠ³ с ΠΊΠ»Π΅ΠΉΠΊΠΈΠΌΠΈ Π»Π΅Π½Ρ‚Π°ΠΌΠΈ, Π½ΠΎ, эй, это Ρ‚ΠΎΠ»ΡŒΠΊΠΎ Π½Π°Ρ‡Π°Π»ΠΎ.

# 1: Π—Π°ΠΏΠΎΠΌΠ½ΠΈΡ‚Π΅ ΠΎΠ±Ρ‰ΠΈΠ΅ ΡƒΠ³Π»Ρ‹ ΠΈ ΠΊΠΎΠΎΡ€Π΄ΠΈΠ½Π°Ρ‚Ρ‹

Π§Ρ‚ΠΎΠ±Ρ‹ эффСктивно ΠΈΡΠΏΠΎΠ»ΡŒΠ·ΠΎΠ²Π°Ρ‚ΡŒ Π΅Π΄ΠΈΠ½ΠΈΡ‡Π½Ρ‹ΠΉ ΠΊΡ€ΡƒΠ³, Π²Π°ΠΌ Π½ΡƒΠΆΠ½ΠΎ Π·Π°ΠΏΠΎΠΌΠ½ΠΈΡ‚ΡŒ Π½Π°ΠΈΠ±ΠΎΠ»Π΅Π΅ распространСнныС ΡƒΠ³Π»Ρ‹ (ΠΊΠ°ΠΊ Π² градусах, Ρ‚Π°ΠΊ ΠΈ Π² Ρ€Π°Π΄ΠΈΠ°Π½Π°Ρ…), Π° Ρ‚Π°ΠΊΠΆΠ΅ ΠΈΡ… ΡΠΎΠΎΡ‚Π²Π΅Ρ‚ΡΡ‚Π²ΡƒΡŽΡ‰ΠΈΠ΅ ΠΊΠΎΠΎΡ€Π΄ΠΈΠ½Π°Ρ‚Ρ‹ x ΠΈ y.

Π”ΠΈΠ°Π³Ρ€Π°ΠΌΠΌΠ° Π²Ρ‹ΡˆΠ΅ прСдставляСт собой ΠΏΠΎΠ»Π΅Π·Π½ΡƒΡŽ Π΄ΠΈΠ°Π³Ρ€Π°ΠΌΠΌΡƒ Π΅Π΄ΠΈΠ½ΠΈΡ‡Π½ΠΎΠ³ΠΎ ΠΊΡ€ΡƒΠ³Π°, Π½Π° ΠΊΠΎΡ‚ΠΎΡ€ΡƒΡŽ стоит ΠΎΠ±Ρ€Π°Ρ‚ΠΈΡ‚ΡŒ Π²Π½ΠΈΠΌΠ°Π½ΠΈΠ΅, ΠΏΠΎΡΠΊΠΎΠ»ΡŒΠΊΡƒ ΠΎΠ½Π° Π²ΠΊΠ»ΡŽΡ‡Π°Π΅Ρ‚ всС основныС ΡƒΠ³Π»Ρ‹ Π² градусах ΠΈ Ρ€Π°Π΄ΠΈΠ°Π½Π°Ρ… Π² Π΄ΠΎΠΏΠΎΠ»Π½Π΅Π½ΠΈΠ΅ ΠΊ ΠΈΡ… ΡΠΎΠΎΡ‚Π²Π΅Ρ‚ΡΡ‚Π²ΡƒΡŽΡ‰ΠΈΠΌ ΠΊΠΎΠΎΡ€Π΄ΠΈΠ½Π°Ρ‚Π½Ρ‹ΠΌ Ρ‚ΠΎΡ‡ΠΊΠ°ΠΌ ΠΏΠΎ осям x ΠΈ y.

Π’ΠΎΡ‚ Π΄ΠΈΠ°Π³Ρ€Π°ΠΌΠΌΠ°, содСрТащая Ρ‚Ρƒ ΠΆΠ΅ ΠΈΠ½Ρ„ΠΎΡ€ΠΌΠ°Ρ†ΠΈΡŽ Π² Π²ΠΈΠ΄Π΅ Ρ‚Π°Π±Π»ΠΈΡ†Ρ‹:

Π£Π³ΠΎΠ» (градусы)

Π£Π³ΠΎΠ» (Ρ€Π°Π΄ΠΈΠ°Π½Ρ‹)

ΠšΠΎΠΎΡ€Π΄ΠΈΠ½Π°Ρ‚Ρ‹ Ρ‚ΠΎΡ‡ΠΊΠΈ Π½Π° окруТности

0 Β° / 360 Β°

0 / 2Ο€

(1, 0)

30 Β°

$ Ο€ / 6 $

долл. БША ({√3} / 2, 1/2)

долл. БША

45 Β°

$ Ο€ / 4 $

долл. БША ({√2} / 2, {√2} / 2)

долл. БША

60 Β°

$ Ο€ / 3 $

долл. БША (1/2, {√3} / 2)

долл. БША

90 Β°

$ Ο€ / 2 $

(0, 1)

120 Β°

$ {2Ο€} / 3 $

долл. БША (- 1/2, {√3} / 2)

долл. БША

135 Β°

$ {3Ο€} / 4 $

долл. БША (- {√2} / 2, {√2} / 2)

долл. БША

150 Β°

$ {5Ο€} / 6 $

$ (- {√3} / 2, 1/2)

$

180 Β°

Ο€

(-1, 0)

210 Β°

$ {7} / 6 $

$ (- {√3} / 2, -1/2)

$

225 Β°

$ {5Ο€} / 4 $

$ (- {√2} / 2, — {√2} / 2)

$

240 Β°

$ {4Ο€} / 3 $

$ (- 1/2, — {√3} / 2)

$

270 Β°

$ {3Ο€} / 2 $

(0, -1)

300 Β°

$ {5Ο€} / 3 $

Π΄ΠΎΠ»Π». БША (1/2, — {√3} / 2)

долл. БША

315 Β°

$ {7Ο€} / 4 $

$ ({√2} / 2, — {√2} / 2)

$

330 Β°

$ {11Ο€} / 6 $

долл. БША ({√3} / 2, -1/2)

долл. БША

Π’Π΅ΠΏΠ΅Ρ€ΡŒ, хотя Π²Ρ‹ ΠΌΠΎΠΆΠ΅Ρ‚Π΅ ΠΏΠΎΠΏΡ‹Ρ‚Π°Ρ‚ΡŒΡΡ Π·Π°ΠΏΠΎΠΌΠ½ΠΈΡ‚ΡŒ всС эти ΠΊΠΎΠΎΡ€Π΄ΠΈΠ½Π°Ρ‚Ρ‹ ΠΈ ΡƒΠ³Π»Ρ‹, это ΠΌΠ½ΠΎΠ³ΠΎ, Π²Π΅Ρ‰Π΅ΠΉ, ΠΊΠΎΡ‚ΠΎΡ€Ρ‹Π΅ Π½ΡƒΠΆΠ½ΠΎ Π·Π°ΠΏΠΎΠΌΠ½ΠΈΡ‚ΡŒ.

К ΡΡ‡Π°ΡΡ‚ΡŒΡŽ, Π΅ΡΡ‚ΡŒ ΠΏΡ€ΠΈΠ΅ΠΌ, ΠΊΠΎΡ‚ΠΎΡ€Ρ‹ΠΉ ΠΏΠΎΠΌΠΎΠΆΠ΅Ρ‚ Π²Π°ΠΌ Π·Π°ΠΏΠΎΠΌΠ½ΠΈΡ‚ΡŒ самыС Π²Π°ΠΆΠ½Ρ‹Π΅ части Π΅Π΄ΠΈΠ½ΠΈΡ‡Π½ΠΎΠ³ΠΎ ΠΊΡ€ΡƒΠ³Π°.

ΠŸΠΎΡΠΌΠΎΡ‚Ρ€ΠΈΡ‚Π΅ Π½Π° ΠΊΠΎΠΎΡ€Π΄ΠΈΠ½Π°Ρ‚Ρ‹ Π²Ρ‹ΡˆΠ΅, ΠΈ Π²Ρ‹ Π·Π°ΠΌΠ΅Ρ‚ΠΈΡ‚Π΅ Ρ‡Π΅Ρ‚ΠΊΡƒΡŽ Π·Π°ΠΊΠΎΠ½ΠΎΠΌΠ΅Ρ€Π½ΠΎΡΡ‚ΡŒ: всС Ρ‚ΠΎΡ‡ΠΊΠΈ (Π·Π° ΠΈΡΠΊΠ»ΡŽΡ‡Π΅Π½ΠΈΠ΅ΠΌ Ρ‚ΠΎΡ‡Π΅ΠΊ ΠΏΠΎΠ΄ 0 Β°, 90 Β°, 270 Β° ΠΈ 360 Β°) Ρ‡Π΅Ρ€Π΅Π΄ΡƒΡŽΡ‚ΡΡ Ρ‚ΠΎΠ»ΡŒΠΊΠΎ ΠΌΠ΅ΠΆΠ΄Ρƒ трСмя значСниями (ΠΏΠΎΠ»ΠΎΠΆΠΈΡ‚Π΅Π»ΡŒΠ½Ρ‹ΠΌΠΈ ΠΈΠ»ΠΈ ΠΎΡ‚Ρ€ΠΈΡ†Π°Ρ‚Π΅Π»ΡŒΠ½Ρ‹ΠΌΠΈ):

  • $ 1/2
  • $ {√2} / 2
  • $
  • $ {√3} / 2
  • $

ΠšΠ°ΠΆΠ΄ΠΎΠΌΡƒ Π·Π½Π°Ρ‡Π΅Π½ΠΈΡŽ соотвСтствуСт короткая, срСдняя ΠΈΠ»ΠΈ длинная линия для косинуса ΠΈ синуса:

Π’ΠΎΡ‚ Ρ‡Ρ‚ΠΎ ΠΎΠ·Π½Π°Ρ‡Π°ΡŽΡ‚ эти Π΄Π»ΠΈΠ½Ρ‹:

  • ΠšΠΎΡ€ΠΎΡ‚ΠΊΠ°Ρ Π³ΠΎΡ€ΠΈΠ·ΠΎΠ½Ρ‚Π°Π»ΡŒΠ½Π°Ρ ΠΈΠ»ΠΈ Π²Π΅Ρ€Ρ‚ΠΈΠΊΠ°Π»ΡŒΠ½Π°Ρ линия = 1/2 Π΄ΠΎΠ»Π»Π°Ρ€Π° БША
  • БрСдняя Π³ΠΎΡ€ΠΈΠ·ΠΎΠ½Ρ‚Π°Π»ΡŒΠ½Π°Ρ ΠΈΠ»ΠΈ Π²Π΅Ρ€Ρ‚ΠΈΠΊΠ°Π»ΡŒΠ½Π°Ρ линия = $ {√2} / 2 $
  • Длинная Π³ΠΎΡ€ΠΈΠ·ΠΎΠ½Ρ‚Π°Π»ΡŒΠ½Π°Ρ ΠΈΠ»ΠΈ Π²Π΅Ρ€Ρ‚ΠΈΠΊΠ°Π»ΡŒΠ½Π°Ρ линия = $ {√3} / 2 $

НапримСр, Ссли Π²Ρ‹ ΠΏΡ‹Ρ‚Π°Π΅Ρ‚Π΅ΡΡŒ Ρ€Π΅ΡˆΠΈΡ‚ΡŒ $ \ cos {Ο€ / 3} $, Π²Ρ‹ Π΄ΠΎΠ»ΠΆΠ½Ρ‹ сразу Π·Π½Π°Ρ‚ΡŒ, Ρ‡Ρ‚ΠΎ этот ΡƒΠ³ΠΎΠ» (Ρ€Π°Π²Π½Ρ‹ΠΉ 60 Β°) ΡƒΠΊΠ°Π·Ρ‹Π²Π°Π΅Ρ‚ Π½Π° ΠΊΠΎΡ€ΠΎΡ‚ΠΊΡƒΡŽ Π³ΠΎΡ€ΠΈΠ·ΠΎΠ½Ρ‚Π°Π»ΡŒΠ½ΡƒΡŽ линию Π½Π° Π΅Π΄ΠΈΠ½ΠΈΡ‡Π½ΠΎΠΉ окруТности. Π‘Π»Π΅Π΄ΠΎΠ²Π°Ρ‚Π΅Π»ΡŒΠ½ΠΎ, Π΅Π³ΠΎ ΡΠΎΠΎΡ‚Π²Π΅Ρ‚ΡΡ‚Π²ΡƒΡŽΡ‰Π°Ρ ΠΊΠΎΠΎΡ€Π΄ΠΈΠ½Π°Ρ‚Π° x Π΄ΠΎΠ»ΠΆΠ½Π° Ρ€Π°Π²Π½ΡΡ‚ΡŒΡΡ $ 1/2 $ (ΠΏΠΎΠ»ΠΎΠΆΠΈΡ‚Π΅Π»ΡŒΠ½ΠΎΠ΅ Π·Π½Π°Ρ‡Π΅Π½ΠΈΠ΅, ΠΏΠΎΡΠΊΠΎΠ»ΡŒΠΊΡƒ $ Ο€ / 3 $ создаСт Ρ‚ΠΎΡ‡ΠΊΡƒ Π² ΠΏΠ΅Ρ€Π²ΠΎΠΌ ΠΊΠ²Π°Π΄Ρ€Π°Π½Ρ‚Π΅ систСмы ΠΊΠΎΠΎΡ€Π΄ΠΈΠ½Π°Ρ‚).

НаконСц, хотя ΠΏΠΎΠ»Π΅Π·Π½ΠΎ Π·Π°ΠΏΠΎΠΌΠ½ΠΈΡ‚ΡŒ всС ΡƒΠ³Π»Ρ‹ Π² Ρ‚Π°Π±Π»ΠΈΡ†Π΅ Π²Ρ‹ΡˆΠ΅, ΠΎΠ±Ρ€Π°Ρ‚ΠΈΡ‚Π΅ Π²Π½ΠΈΠΌΠ°Π½ΠΈΠ΅, Ρ‡Ρ‚ΠΎ , бСзусловно, самыС Π²Π°ΠΆΠ½Ρ‹Π΅ ΡƒΠ³Π»Ρ‹, ΠΊΠΎΡ‚ΠΎΡ€Ρ‹Π΅ слСдуСт Π·Π°ΠΏΠΎΠΌΠ½ΠΈΡ‚ΡŒ, ΡΠ»Π΅Π΄ΡƒΡŽΡ‰ΠΈΠ΅:

  • 30 Β° / $ Ο€ / 6 $
  • 45 Β° / $ Ο€ / 4 $
  • 60 Β° / $ Ο€ / 3 $

ΠžΠ±Ρ€Π°Ρ‰Π°ΠΉΡ‚Π΅ΡΡŒ со своими Π½Π΅Π³Π°Ρ‚ΠΈΠ²Π°ΠΌΠΈ ΠΈ ΠΏΠΎΠ·ΠΈΡ‚ΠΈΠ²Π°ΠΌΠΈ, ΠΊΠ°ΠΊ с кабСлями, ΠΊΠΎΡ‚ΠΎΡ€Ρ‹Π΅ ΠΏΠΎΡ‚Π΅Π½Ρ†ΠΈΠ°Π»ΡŒΠ½ΠΎ ΠΌΠΎΠ³ΡƒΡ‚ ΡƒΠ±ΠΈΡ‚ΡŒ вас ΠΏΡ€ΠΈ Π½Π΅ΠΏΡ€Π°Π²ΠΈΠ»ΡŒΠ½ΠΎΠΌ ΠΏΠΎΠ΄ΠΊΠ»ΡŽΡ‡Π΅Π½ΠΈΠΈ.

β„– 2: Π£Π·Π½Π°ΠΉΡ‚Π΅, Ρ‡Ρ‚ΠΎ Π΅ΡΡ‚ΡŒ ΠΎΡ‚Ρ€ΠΈΡ†Π°Ρ‚Π΅Π»ΡŒΠ½ΠΎΠ³ΠΎ ΠΈ Ρ‡Ρ‚ΠΎ ΠΏΠΎΠ»ΠΎΠΆΠΈΡ‚Π΅Π»ΡŒΠ½ΠΎΠ³ΠΎ

ΠžΡ‡Π΅Π½ΡŒ Π²Π°ΠΆΠ½ΠΎ ΡƒΠΌΠ΅Ρ‚ΡŒ Ρ€Π°Π·Π»ΠΈΡ‡Π°Ρ‚ΡŒ ΠΏΠΎΠ»ΠΎΠΆΠΈΡ‚Π΅Π»ΡŒΠ½Ρ‹Π΅ ΠΈ ΠΎΡ‚Ρ€ΠΈΡ†Π°Ρ‚Π΅Π»ΡŒΠ½Ρ‹Π΅ ΠΊΠΎΠΎΡ€Π΄ΠΈΠ½Π°Ρ‚Ρ‹ x ΠΈ y, Ρ‡Ρ‚ΠΎΠ±Ρ‹ Π²Ρ‹ ΠΌΠΎΠ³Π»ΠΈ Π½Π°ΠΉΡ‚ΠΈ ΠΏΡ€Π°Π²ΠΈΠ»ΡŒΠ½ΠΎΠ΅ Π·Π½Π°Ρ‡Π΅Π½ΠΈΠ΅ для Ρ‚Ρ€ΠΈΠ³Π³Π΅Ρ€Π½ΠΎΠΉ ΠΏΡ€ΠΎΠ±Π»Π΅ΠΌΡ‹. НапоминаСм, Ρ‡Ρ‚ΠΎ w , Π±ΡƒΠ΄Π΅Ρ‚ Π»ΠΈ ΠΊΠΎΠΎΡ€Π΄ΠΈΠ½Π°Ρ‚Π° Π½Π° Π΅Π΄ΠΈΠ½ΠΈΡ‡Π½ΠΎΠΉ окруТности ΠΏΠΎΠ»ΠΎΠΆΠΈΡ‚Π΅Π»ΡŒΠ½ΠΎΠΉ ΠΈΠ»ΠΈ ΠΎΡ‚Ρ€ΠΈΡ†Π°Ρ‚Π΅Π»ΡŒΠ½ΠΎΠΉ, зависит ΠΎΡ‚ , Π² ΠΊΠ°ΠΊΠΎΠΉ ΠΊΠ²Π°Π΄Ρ€Π°Π½Ρ‚ (I, II, III ΠΈΠ»ΠΈ IV) ΠΏΠΎΠΏΠ°Π΄Π°Π΅Ρ‚ Ρ‚ΠΎΡ‡ΠΊΠ°:

Π’ΠΎΡ‚ Π΄ΠΈΠ°Π³Ρ€Π°ΠΌΠΌΠ°, ΠΏΠΎΠΊΠ°Π·Ρ‹Π²Π°ΡŽΡ‰Π°Ρ, Π±ΡƒΠ΄Π΅Ρ‚ Π»ΠΈ ΠΊΠΎΠΎΡ€Π΄ΠΈΠ½Π°Ρ‚Π° ΠΏΠΎΠ»ΠΎΠΆΠΈΡ‚Π΅Π»ΡŒΠ½ΠΎΠΉ ΠΈΠ»ΠΈ ΠΎΡ‚Ρ€ΠΈΡ†Π°Ρ‚Π΅Π»ΡŒΠ½ΠΎΠΉ Π² зависимости ΠΎΡ‚ ΠΊΠ²Π°Π΄Ρ€Π°Π½Ρ‚Π°, Π² ΠΊΠΎΡ‚ΠΎΡ€ΠΎΠΌ находится ΠΊΠΎΠ½ΠΊΡ€Π΅Ρ‚Π½Ρ‹ΠΉ ΡƒΠ³ΠΎΠ» (Π² градусах ΠΈΠ»ΠΈ Ρ€Π°Π΄ΠΈΠ°Π½Π°Ρ…):

ΠšΠ²Π°Π΄Ρ€Π°Π½Ρ‚

ΠšΠΎΠΎΡ€Π΄ΠΈΠ½Π°Ρ‚Π° X (косинус)

ΠšΠΎΠΎΡ€Π΄ΠΈΠ½Π°Ρ‚Π° Y (синус)

Π―

+

+

II

–

+

III

–

–

IV

+

–

Допустим, Π²Π°ΠΌ Π·Π°Π΄Π°Π»ΠΈ ΡΠ»Π΅Π΄ΡƒΡŽΡ‰ΡƒΡŽ Π·Π°Π΄Π°Ρ‡Ρƒ Π½Π° тСстС ΠΏΠΎ ΠΌΠ°Ρ‚Π΅ΠΌΠ°Ρ‚ΠΈΠΊΠ΅:

$$ \ cos210 Β° $$

ΠŸΡ€Π΅ΠΆΠ΄Π΅ Ρ‡Π΅ΠΌ Π²Ρ‹ Π΄Π°ΠΆΠ΅ ΠΏΠΎΠΏΡ‹Ρ‚Π°Π΅Ρ‚Π΅ΡΡŒ Π΅Π΅ Ρ€Π΅ΡˆΠΈΡ‚ΡŒ, Π²Ρ‹ Π΄ΠΎΠ»ΠΆΠ½Ρ‹ Π±Ρ‹Ρ‚ΡŒ Π² состоянии Ρ€Π°ΡΠΏΠΎΠ·Π½Π°Ρ‚ΡŒ, Ρ‡Ρ‚ΠΎ ΠΎΡ‚Π²Π΅Ρ‚ΠΎΠΌ Π±ΡƒΠ΄Π΅Ρ‚ ΠΎΡ‚Ρ€ΠΈΡ†Π°Ρ‚Π΅Π»ΡŒΠ½ΠΎΠ΅ число , ΠΏΠΎΡΠΊΠΎΠ»ΡŒΠΊΡƒ ΡƒΠ³ΠΎΠ» 210 Β° ΠΏΠΎΠΏΠ°Π΄Π°Π΅Ρ‚ Π² ΠΊΠ²Π°Π΄Ρ€Π°Π½Ρ‚ III (Π³Π΄Π΅ x-ΠΊΠΎΠΎΡ€Π΄ΠΈΠ½Π°Ρ‚Ρ‹ всСгда ΠΎΡ‚Ρ€ΠΈΡ†Π°Ρ‚Π΅Π»ΡŒΠ½Ρ‹).

Π’Π΅ΠΏΠ΅Ρ€ΡŒ, ΠΈΡΠΏΠΎΠ»ΡŒΠ·ΡƒΡ ΡƒΠ»ΠΎΠ²ΠΊΡƒ, ΠΊΠΎΡ‚ΠΎΡ€ΡƒΡŽ ΠΌΡ‹ ΠΈΠ·ΡƒΡ‡ΠΈΠ»ΠΈ Π² совСтС 1, Π²Ρ‹ ΠΌΠΎΠΆΠ΅Ρ‚Π΅ Π²Ρ‹ΡΡΠ½ΠΈΡ‚ΡŒ, Ρ‡Ρ‚ΠΎ ΡƒΠ³ΠΎΠ» 210 Β° создаСт Π΄Π»ΠΈΠ½Π½ΠΎΠΉ Π³ΠΎΡ€ΠΈΠ·ΠΎΠ½Ρ‚Π°Π»ΡŒΠ½ΠΎΠΉ Π»ΠΈΠ½ΠΈΠΈ. Π‘Π»Π΅Π΄ΠΎΠ²Π°Ρ‚Π΅Π»ΡŒΠ½ΠΎ, наш ΠΎΡ‚Π²Π΅Ρ‚ Ρ‚Π°ΠΊΠΎΠ²:

$$ \ cos210 Β° = — {√3} / 2 $$

# 3: Π£ΠΌΠ΅ΠΉΡ‚Π΅ Π½Π°Ρ…ΠΎΠ΄ΠΈΡ‚ΡŒ ΠΊΠ°ΡΠ°Ρ‚Π΅Π»ΡŒΠ½ΡƒΡŽ

НаконСц, Π²Π°ΠΆΠ½ΠΎ Π·Π½Π°Ρ‚ΡŒ, ΠΊΠ°ΠΊ ΠΈΡΠΏΠΎΠ»ΡŒΠ·ΠΎΠ²Π°Ρ‚ΡŒ всю эту ΠΈΠ½Ρ„ΠΎΡ€ΠΌΠ°Ρ†ΠΈΡŽ ΠΎ Ρ‚Ρ€ΠΈΠ³Π³Π΅Ρ€Π½ΠΎΠΉ окруТности, синусС ΠΈ косинусС, Ρ‡Ρ‚ΠΎΠ±Ρ‹ ΠΈΠΌΠ΅Ρ‚ΡŒ Π²ΠΎΠ·ΠΌΠΎΠΆΠ½ΠΎΡΡ‚ΡŒ Π½Π°ΠΉΡ‚ΠΈ тангСнс ΡƒΠ³Π»Π°.

Π’ Ρ‚Ρ€ΠΈΠ³Π³Π΅Ρ€Π΅, Ρ‡Ρ‚ΠΎΠ±Ρ‹ Π½Π°ΠΉΡ‚ΠΈ тангСнс ΡƒΠ³Π»Π° ΞΈ (Π² градусах ΠΈΠ»ΠΈ Ρ€Π°Π΄ΠΈΠ°Π½Π°Ρ…), Π²Ρ‹ просто Π΄Π΅Π»ΠΈΡ‚Π΅ синус Π½Π° косинус:

$$ \ tanΞΈ = {\ sinΞΈ} / {\ cosΞΈ} $$

НапримСр, Π²Ρ‹ ΠΏΡ‹Ρ‚Π°Π΅Ρ‚Π΅ΡΡŒ Ρ€Π΅ΡˆΠΈΡ‚ΡŒ эту ΠΏΡ€ΠΎΠ±Π»Π΅ΠΌΡƒ:

$$ \ tan300 Β° $$

ΠŸΠ΅Ρ€Π²Ρ‹ΠΉ шаг — ΡΠΎΡΡ‚Π°Π²ΠΈΡ‚ΡŒ ΡƒΡ€Π°Π²Π½Π΅Π½ΠΈΠ΅ Π² Ρ‚Π΅Ρ€ΠΌΠΈΠ½Π°Ρ… синуса ΠΈ косинуса:

$$ \ tan300 Β° = {\ sin300 Β°} / {\ cos300 Β°} $$

Π’Π΅ΠΏΠ΅Ρ€ΡŒ, Ρ‡Ρ‚ΠΎΠ±Ρ‹ Π½Π°ΠΉΡ‚ΠΈ ΠΊΠ°ΡΠ°Ρ‚Π΅Π»ΡŒΠ½ΡƒΡŽ, Π½Π°ΠΌ Π½ΡƒΠΆΠ½ΠΎ Π½Π°ΠΉΡ‚ΠΈ синус ΠΈ косинус 300 Β°.Π’Ρ‹ Π΄ΠΎΠ»ΠΆΠ½Ρ‹ быстро Ρ€Π°ΡΠΏΠΎΠ·Π½Π°Ρ‚ΡŒ, Ρ‡Ρ‚ΠΎ ΡƒΠ³ΠΎΠ» 300 Β° ΠΏΠΎΠΏΠ°Π΄Π°Π΅Ρ‚ Π² Ρ‡Π΅Ρ‚Π²Π΅Ρ€Ρ‚Ρ‹ΠΉ ΠΊΠ²Π°Π΄Ρ€Π°Π½Ρ‚, Π° это ΠΎΠ·Π½Π°Ρ‡Π°Π΅Ρ‚, Ρ‡Ρ‚ΠΎ косинус ΠΈΠ»ΠΈ ΠΊΠΎΠΎΡ€Π΄ΠΈΠ½Π°Ρ‚Π° x Π±ΡƒΠ΄Π΅Ρ‚ ΠΏΠΎΠ»ΠΎΠΆΠΈΡ‚Π΅Π»ΡŒΠ½Ρ‹ΠΌ, Π° синус ΠΈΠ»ΠΈ ΠΊΠΎΠΎΡ€Π΄ΠΈΠ½Π°Ρ‚Π° y — ΠΎΡ‚Ρ€ΠΈΡ†Π°Ρ‚Π΅Π»ΡŒΠ½Ρ‹ΠΌ.

Π’Ρ‹ Ρ‚Π°ΠΊΠΆΠ΅ Π΄ΠΎΠ»ΠΆΠ½Ρ‹ сразу Π·Π½Π°Ρ‚ΡŒ, Ρ‡Ρ‚ΠΎ ΡƒΠ³ΠΎΠ» 300 Β° создаСт ΠΊΠΎΡ€ΠΎΡ‚ΠΊΡƒΡŽ Π³ΠΎΡ€ΠΈΠ·ΠΎΠ½Ρ‚Π°Π»ΡŒΠ½ΡƒΡŽ линию ΠΈ Π΄Π»ΠΈΠ½Π½ΡƒΡŽ Π²Π΅Ρ€Ρ‚ΠΈΠΊΠ°Π»ΡŒΠ½ΡƒΡŽ линию. Π‘Π»Π΅Π΄ΠΎΠ²Π°Ρ‚Π΅Π»ΡŒΠ½ΠΎ, косинус (Π³ΠΎΡ€ΠΈΠ·ΠΎΠ½Ρ‚Π°Π»ΡŒΠ½Π°Ρ линия) Π±ΡƒΠ΄Π΅Ρ‚ Ρ€Π°Π²Π΅Π½ $ 1/2 $, Π° синус (Π²Π΅Ρ€Ρ‚ΠΈΠΊΠ°Π»ΡŒΠ½Π°Ρ линия) Π±ΡƒΠ΄Π΅Ρ‚ Ρ€Π°Π²Π΅Π½ $ — {√3} / 2 $ (ΠΎΡ‚Ρ€ΠΈΡ†Π°Ρ‚Π΅Π»ΡŒΠ½ΠΎΠ΅ Π·Π½Π°Ρ‡Π΅Π½ΠΈΠ΅ y, ΠΏΠΎΡΠΊΠΎΠ»ΡŒΠΊΡƒ эта Ρ‚ΠΎΡ‡ΠΊΠ° находится Π² ΠΊΠ²Π°Π΄Ρ€Π°Π½Ρ‚Π΅ IV ).

Π’Π΅ΠΏΠ΅Ρ€ΡŒ, Ρ‡Ρ‚ΠΎΠ±Ρ‹ Π½Π°ΠΉΡ‚ΠΈ ΠΊΠ°ΡΠ°Ρ‚Π΅Π»ΡŒΠ½ΡƒΡŽ, всС, Ρ‡Ρ‚ΠΎ Π²Π°ΠΌ Π½ΡƒΠΆΠ½ΠΎ ΡΠ΄Π΅Π»Π°Ρ‚ΡŒ, это ΠΏΠΎΠ΄ΠΊΠ»ΡŽΡ‡ΠΈΡ‚ΡŒ ΠΈ Ρ€Π΅ΡˆΠΈΡ‚ΡŒ:

$$ \ tan300 ° = {- {√3} / 2} / {1/2} $$

$$ \ tan300 ° = -√3 $$

ΠŸΠΎΡ€Π° ΠΌΡƒΡ€Π»Ρ‹ΠΊΠ°Ρ‚ΡŒ ΠΏΠΎ своим матСматичСским способностям!

Набор практичСских вопросов Β«Π•Π΄ΠΈΠ½ΠΈΡ‡Π½Ρ‹ΠΉ ΠΊΡ€ΡƒΠ³Β»

Π’Π΅ΠΏΠ΅Ρ€ΡŒ, ΠΊΠΎΠ³Π΄Π° Π²Ρ‹ Π·Π½Π°Π΅Ρ‚Π΅, ΠΊΠ°ΠΊ выглядит Π΅Π΄ΠΈΠ½ΠΈΡ‡Π½Ρ‹ΠΉ ΠΊΡ€ΡƒΠ³ ΠΈ ΠΊΠ°ΠΊ Π΅Π³ΠΎ ΠΈΡΠΏΠΎΠ»ΡŒΠ·ΠΎΠ²Π°Ρ‚ΡŒ, Π΄Π°Π²Π°ΠΉΡ‚Π΅ ΠΏΡ€ΠΎΠ²Π΅Ρ€ΠΈΠΌ, Ρ‡Ρ‚ΠΎ Π²Ρ‹ ΡƒΠ·Π½Π°Π»ΠΈ, с ΠΏΠΎΠΌΠΎΡ‰ΡŒΡŽ Π½Π΅ΡΠΊΠΎΠ»ΡŒΠΊΠΈΡ… практичСских Π·Π°Π΄Π°Ρ‡.

Вопросы

  1. $ \ sin45 Β° $
  2. $ \ cos240 Β° $
  3. $ \ cos {5Ο€} / 3 $
  4. $ \ tan {2Ο€} / 3 $

ΠΎΡ‚Π²Π΅Ρ‚Ρ‹

  1. $ {√2} / 2
  2. долл. БША
  3. $ -1 / 2 $
  4. $ 1/2
  5. $ -√3 $

ΠžΡ‚Π²Π΅Ρ‚ объяснСния

# 1: $ \ sin45 Β° $

ΠŸΡ€ΠΈ этой ΠΏΡ€ΠΎΠ±Π»Π΅ΠΌΠ΅ Π΅ΡΡ‚ΡŒ Π΄Π²Π° элСмСнта ΠΈΠ½Ρ„ΠΎΡ€ΠΌΠ°Ρ†ΠΈΠΈ, ΠΊΠΎΡ‚ΠΎΡ€Ρ‹Π΅ Π²Ρ‹ Π΄ΠΎΠ»ΠΆΠ½Ρ‹ сразу ΠΎΠΏΡ€Π΅Π΄Π΅Π»ΠΈΡ‚ΡŒ:

  • ΠžΡ‚Π²Π΅Ρ‚ Π±ΡƒΠ΄Π΅Ρ‚ ΠΏΠΎΠ»ΠΎΠΆΠΈΡ‚Π΅Π»ΡŒΠ½Ρ‹ΠΌ, , ΠΏΠΎΡΠΊΠΎΠ»ΡŒΠΊΡƒ ΡƒΠ³ΠΎΠ» 45 Β° находится Π² ΠΊΠ²Π°Π΄Ρ€Π°Π½Ρ‚Π΅ I, Π° синус ΡƒΠ³Π»Π° Ρ€Π°Π²Π΅Π½ ΠΊΠΎΠΎΡ€Π΄ΠΈΠ½Π°Ρ‚Π΅ y
  • Π£Π³ΠΎΠ» 45 Β° ΠΎΠ±Ρ€Π°Π·ΡƒΠ΅Ρ‚ Π²Π΅Ρ€Ρ‚ΠΈΠΊΠ°Π»ΡŒΠ½ΡƒΡŽ линию срСднСй Π΄Π»ΠΈΠ½Ρ‹ (для синуса)

ΠŸΠΎΡΠΊΠΎΠ»ΡŒΠΊΡƒ 45 Β° ΠΎΠ·Π½Π°Ρ‡Π°Π΅Ρ‚ ΠΏΠΎΠ»ΠΎΠΆΠΈΡ‚Π΅Π»ΡŒΠ½ΡƒΡŽ линию срСднСй Π΄Π»ΠΈΠ½Ρ‹, ΠΏΡ€Π°Π²ΠΈΠ»ΡŒΠ½Ρ‹ΠΉ ΠΎΡ‚Π²Π΅Ρ‚: $ {√2} / 2 $.

Если Π²Ρ‹ Π½Π΅ Π·Π½Π°Π΅Ρ‚Π΅, ΠΊΠ°ΠΊ это ΠΏΠΎΠ½ΡΡ‚ΡŒ, нарисуйтС Π΄ΠΈΠ°Π³Ρ€Π°ΠΌΠΌΡƒ, которая ΠΏΠΎΠΌΠΎΠΆΠ΅Ρ‚ Π²Π°ΠΌ ΠΎΠΏΡ€Π΅Π΄Π΅Π»ΠΈΡ‚ΡŒ, Π±ΡƒΠ΄Π΅Ρ‚ Π»ΠΈ Π΄Π»ΠΈΠ½Π° Π»ΠΈΠ½ΠΈΠΈ ΠΊΠΎΡ€ΠΎΡ‚ΠΊΠΎΠΉ, срСднСй ΠΈΠ»ΠΈ Π΄Π»ΠΈΠ½Π½ΠΎΠΉ.

# 2: $ \ cos240 Β° $

Как ΠΈ Π² Π·Π°Π΄Π°Ρ‡Π΅ β„– 1 Π²Ρ‹ΡˆΠ΅, Π΅ΡΡ‚ΡŒ Π΄Π²Π΅ части ΠΈΠ½Ρ„ΠΎΡ€ΠΌΠ°Ρ†ΠΈΠΈ, ΠΊΠΎΡ‚ΠΎΡ€ΡƒΡŽ Π²Ρ‹ Π΄ΠΎΠ»ΠΆΠ½Ρ‹ быстро ΠΏΠΎΠ½ΡΡ‚ΡŒ ΠΏΡ€ΠΈ Ρ€Π΅ΡˆΠ΅Π½ΠΈΠΈ этой ΠΏΡ€ΠΎΠ±Π»Π΅ΠΌΡ‹:

  • ΠžΡ‚Π²Π΅Ρ‚ Π±ΡƒΠ΄Π΅Ρ‚ ΠΎΡ‚Ρ€ΠΈΡ†Π°Ρ‚Π΅Π»ΡŒΠ½Ρ‹ΠΌ, , ΠΏΠΎΡΠΊΠΎΠ»ΡŒΠΊΡƒ ΡƒΠ³ΠΎΠ» 240 Β° находится Π² ΠΊΠ²Π°Π΄Ρ€Π°Π½Ρ‚Π΅ III, Π° косинус ΡƒΠ³Π»Π° Ρ€Π°Π²Π΅Π½ ΠΊΠΎΠΎΡ€Π΄ΠΈΠ½Π°Ρ‚Π΅ x
  • Π£Π³ΠΎΠ» 240 Β° ΠΎΠ±Ρ€Π°Π·ΡƒΠ΅Ρ‚ ΠΊΠΎΡ€ΠΎΡ‚ΠΊΡƒΡŽ Π³ΠΎΡ€ΠΈΠ·ΠΎΠ½Ρ‚Π°Π»ΡŒΠ½ΡƒΡŽ линию (для косинуса)

ΠŸΠΎΡΠΊΠΎΠ»ΡŒΠΊΡƒ 240 Β° ΠΎΠ·Π½Π°Ρ‡Π°Π΅Ρ‚ ΠΊΠΎΡ€ΠΎΡ‚ΠΊΡƒΡŽ ΠΎΡ‚Ρ€ΠΈΡ†Π°Ρ‚Π΅Π»ΡŒΠ½ΡƒΡŽ линию, ΠΏΡ€Π°Π²ΠΈΠ»ΡŒΠ½Ρ‹ΠΉ ΠΎΡ‚Π²Π΅Ρ‚: $ -1 / 2 $.

# 3: $ \ cos {5Ο€} / 3 $

Π’ ΠΎΡ‚Π»ΠΈΡ‡ΠΈΠ΅ ΠΎΡ‚ Π·Π°Π΄Π°Ρ‡, описанных Π²Ρ‹ΡˆΠ΅, Π² этой Π·Π°Π΄Π°Ρ‡Π΅ вмСсто градусов ΠΈΡΠΏΠΎΠ»ΡŒΠ·ΡƒΠ΅Ρ‚ΡΡ Ρ€Π°Π΄ΠΈΠ°Π½ . Π₯отя это ΠΌΠΎΠΆΠ΅Ρ‚ ΡΠ΄Π΅Π»Π°Ρ‚ΡŒ ΠΏΡ€ΠΎΠ±Π»Π΅ΠΌΡƒ Π±ΠΎΠ»Π΅Π΅ слоТной для Ρ€Π΅ΡˆΠ΅Π½ΠΈΡ, Π½Π° самом Π΄Π΅Π»Π΅ здСсь ΠΈΡΠΏΠΎΠ»ΡŒΠ·ΡƒΡŽΡ‚ΡΡ Ρ‚Π΅ ΠΆΠ΅ основныС шаги, Ρ‡Ρ‚ΠΎ ΠΈ для Π΄Π²ΡƒΡ… Π΄Ρ€ΡƒΠ³ΠΈΡ… Π·Π°Π΄Π°Ρ‡.

Π’ΠΎ-ΠΏΠ΅Ρ€Π²Ρ‹Ρ…, Π²Ρ‹ Π΄ΠΎΠ»ΠΆΠ½Ρ‹ ΠΏΠΎΠ½ΡΡ‚ΡŒ, Ρ‡Ρ‚ΠΎ ΡƒΠ³ΠΎΠ» $ {5Ο€} / 3 $ находится Π² ΠΊΠ²Π°Π΄Ρ€Π°Π½Ρ‚Π΅ IV, поэтому ΠΊΠΎΠΎΡ€Π΄ΠΈΠ½Π°Ρ‚Π° x ΠΈΠ»ΠΈ косинус Π±ΡƒΠ΄Π΅Ρ‚ ΠΏΠΎΠ»ΠΎΠΆΠΈΡ‚Π΅Π»ΡŒΠ½Ρ‹ΠΌ числом. Π’Ρ‹ Ρ‚Π°ΠΊΠΆΠ΅ Π΄ΠΎΠ»ΠΆΠ½Ρ‹ ΡƒΠΌΠ΅Ρ‚ΡŒ ΡΠΊΠ°Π·Π°Ρ‚ΡŒ, Ρ‡Ρ‚ΠΎ $ {5Ο€} / 3 $ создаСт ΠΊΠΎΡ€ΠΎΡ‚ΠΊΡƒΡŽ Π³ΠΎΡ€ΠΈΠ·ΠΎΠ½Ρ‚Π°Π»ΡŒΠ½ΡƒΡŽ линию.

Π­Ρ‚ΠΎ Π΄Π°Π΅Ρ‚ Π²Π°ΠΌ достаточно ΠΈΠ½Ρ„ΠΎΡ€ΠΌΠ°Ρ†ΠΈΠΈ, Ρ‡Ρ‚ΠΎΠ±Ρ‹ ΠΎΠΏΡ€Π΅Π΄Π΅Π»ΠΈΡ‚ΡŒ, Ρ‡Ρ‚ΠΎ ΠΎΡ‚Π²Π΅Ρ‚ составляСт $ 1/2 $.

# 4: $ \ tan {2Ο€} / 3 $

Π’ этой Π·Π°Π΄Π°Ρ‡Π΅ ΠΈΡΠΏΠΎΠ»ΡŒΠ·ΡƒΠ΅Ρ‚ΡΡ тангСнс вмСсто синуса ΠΈΠ»ΠΈ косинуса, Π° это Π·Π½Π°Ρ‡ΠΈΡ‚, Ρ‡Ρ‚ΠΎ с нашСй стороны потрСбуСтся Π½Π΅ΠΌΠ½ΠΎΠ³ΠΎ большС матСматичСских вычислСний. ΠŸΡ€Π΅ΠΆΠ΄Π΅ всСго, вспомнитС ΠΎΡΠ½ΠΎΠ²Π½ΡƒΡŽ Ρ„ΠΎΡ€ΠΌΡƒΠ»Ρƒ для нахоТдСния тангСнса:

$$ \ tan ΞΈ = {\ sin ΞΈ} / {\ cos ΞΈ} $$

Π’Π΅ΠΏΠ΅Ρ€ΡŒ Π΄Π°Π²Π°ΠΉΡ‚Π΅ возьмСм ΠΏΠΎΠ»ΡƒΡ‡Π΅Π½Π½ΡƒΡŽ ΡΡ‚Π΅ΠΏΠ΅Π½ΡŒ — $ {2Ο€} / 3 $ — ΠΈ подставим Π΅Π΅ Π² это ΡƒΡ€Π°Π²Π½Π΅Π½ΠΈΠ΅:

$$ \ tan {2Ο€} / 3 = {\ sin {2Ο€} / 3} / {\ cos {2Ο€} / 3} $$

Π’Π΅ΠΏΠ΅Ρ€ΡŒ Π²Ρ‹ Π΄ΠΎΠ»ΠΆΠ½Ρ‹ ΠΈΠΌΠ΅Ρ‚ΡŒ Π²ΠΎΠ·ΠΌΠΎΠΆΠ½ΠΎΡΡ‚ΡŒ Ρ€Π΅ΡˆΠ°Ρ‚ΡŒ для синуса ΠΈ косинуса ΠΎΡ‚Π΄Π΅Π»ΡŒΠ½ΠΎ, ΠΈΡΠΏΠΎΠ»ΡŒΠ·ΡƒΡ Ρ‚ΠΎ, Ρ‡Ρ‚ΠΎ Π²Ρ‹ Π·Π°ΠΏΠΎΠΌΠ½ΠΈΠ»ΠΈ ΠΎ Π΅Π΄ΠΈΠ½ΠΈΡ‡Π½ΠΎΠΉ окруТности.ΠŸΠΎΡΠΊΠΎΠ»ΡŒΠΊΡƒ ΡƒΠ³ΠΎΠ» $ {2Ο€} / 3 $ находится Π² ΠΊΠ²Π°Π΄Ρ€Π°Π½Ρ‚Π΅ II, ΠΊΠΎΠΎΡ€Π΄ΠΈΠ½Π°Ρ‚Π° x (ΠΈΠ»ΠΈ косинус) Π±ΡƒΠ΄Π΅Ρ‚ ΠΎΡ‚Ρ€ΠΈΡ†Π°Ρ‚Π΅Π»ΡŒΠ½ΠΎΠΉ, Π° ΠΊΠΎΠΎΡ€Π΄ΠΈΠ½Π°Ρ‚Π° y (ΠΈΠ»ΠΈ синусом) Π±ΡƒΠ΄Π΅Ρ‚ ΠΏΠΎΠ»ΠΎΠΆΠΈΡ‚Π΅Π»ΡŒΠ½ΠΎΠΉ.

Π—Π°Ρ‚Π΅ΠΌ Π²Ρ‹ смоТСтС ΠΎΠΏΡ€Π΅Π΄Π΅Π»ΠΈΡ‚ΡŒ Π½Π° основС ΠΎΠ΄Π½ΠΎΠ³ΠΎ Ρ‚ΠΎΠ»ΡŒΠΊΠΎ ΡƒΠ³Π»Π°, Ρ‡Ρ‚ΠΎ Π³ΠΎΡ€ΠΈΠ·ΠΎΠ½Ρ‚Π°Π»ΡŒΠ½Π°Ρ линия — это короткая линия, , Π° Π²Π΅Ρ€Ρ‚ΠΈΠΊΠ°Π»ΡŒΠ½Π°Ρ линия — это длинная линия. Π­Ρ‚ΠΎ ΠΎΠ·Π½Π°Ρ‡Π°Π΅Ρ‚, Ρ‡Ρ‚ΠΎ косинус Ρ€Π°Π²Π΅Π½ $ -1 / 2 $, Π° синус Ρ€Π°Π²Π΅Π½ $ {√3} / 2 $.

Π’Π΅ΠΏΠ΅Ρ€ΡŒ, ΠΊΠΎΠ³Π΄Π° ΠΌΡ‹ вычислили эти значСния, всС, Ρ‡Ρ‚ΠΎ Π½Π°ΠΌ Π½ΡƒΠΆΠ½ΠΎ ΡΠ΄Π΅Π»Π°Ρ‚ΡŒ, это Π²ΡΡ‚Π°Π²ΠΈΡ‚ΡŒ ΠΈΡ… Π² нашС исходноС ΡƒΡ€Π°Π²Π½Π΅Π½ΠΈΠ΅ ΠΈ Π½Π°ΠΉΡ‚ΠΈ тангСнс:

$$ \ tan {2Ο€} / 3 = {{√3} / 2} / {- 1/2} $$

$$ \ tan {2Ο€} / 3 = -√3 $$

Π§Ρ‚ΠΎ дальшС?

Если Π²Ρ‹ Π² блиТайшСС врСмя Π±ΡƒΠ΄Π΅Ρ‚Π΅ ΡΠ΄Π°Π²Π°Ρ‚ΡŒ SAT ΠΈΠ»ΠΈ ACT, Π²Π°ΠΌ Π½ΡƒΠΆΠ½ΠΎ Π±ΡƒΠ΄Π΅Ρ‚ Π·Π½Π°Ρ‚ΡŒ Π½Π΅ΠΊΠΎΡ‚ΠΎΡ€Ρ‹Π΅ Ρ‚Ρ€ΠΈΠ³Π³Π΅Ρ€Ρ‹, Ρ‡Ρ‚ΠΎΠ±Ρ‹ Π²Ρ‹ ΠΌΠΎΠ³Π»ΠΈ ΠΏΡ€Π΅ΡƒΡΠΏΠ΅Ρ‚ΡŒ Π² Ρ€Π°Π·Π΄Π΅Π»Π΅ ΠΌΠ°Ρ‚Π΅ΠΌΠ°Ρ‚ΠΈΠΊΠΈ. ВзглянитС Π½Π° наши экспСртныС руководства ΠΏΠΎ Ρ‚Π΅ΡΡ‚ΠΈΡ€ΠΎΠ²Π°Π½ΠΈΡŽ SAT ΠΈ ACT, Ρ‡Ρ‚ΠΎΠ±Ρ‹ Π²Ρ‹ ΠΌΠΎΠ³Π»ΠΈ Ρ‚ΠΎΡ‡Π½ΠΎ ΡƒΠ·Π½Π°Ρ‚ΡŒ, Ρ‡Ρ‚ΠΎ Π²Π°ΠΌ Π½ΡƒΠΆΠ½ΠΎ Π·Π½Π°Ρ‚ΡŒ Π² дСнь тСста!

Помимо запоминания Π΅Π΄ΠΈΠ½ΠΈΡ‡Π½ΠΎΠ³ΠΎ ΠΊΡ€ΡƒΠ³Π°, Π½Π΅ΠΏΠ»ΠΎΡ…ΠΎ Π½Π°ΡƒΡ‡ΠΈΡ‚ΡŒΡΡ Π²ΡΡ‚Π°Π²Π»ΡΡ‚ΡŒ числа ΠΈ ΠΎΡ‚Π²Π΅Ρ‚Ρ‹. ΠŸΡ€ΠΎΡ‡Ρ‚ΠΈΡ‚Π΅ наши руководства, Ρ‡Ρ‚ΠΎΠ±Ρ‹ ΡƒΠ·Π½Π°Ρ‚ΡŒ всС ΠΎΠ± этих Π΄Π²ΡƒΡ… ΠΏΠΎΠ»Π΅Π·Π½Ρ‹Ρ… стратСгиях, ΠΊΠΎΡ‚ΠΎΡ€Ρ‹Π΅ Π²Ρ‹ ΠΌΠΎΠΆΠ΅Ρ‚Π΅ ΠΈΡΠΏΠΎΠ»ΡŒΠ·ΠΎΠ²Π°Ρ‚ΡŒ Π² любом тСстС ΠΏΠΎ ΠΌΠ°Ρ‚Π΅ΠΌΠ°Ρ‚ΠΈΠΊΠ΅, Π²ΠΊΠ»ΡŽΡ‡Π°Ρ SAT ΠΈ ACT!

Π€Π°ΠΉΠ»: Circle cos sin.gif — Wikimedia Commons

Английский: Π£ нас Π΅ΡΡ‚ΡŒ Π΅Π΄ΠΈΠ½ΠΈΡ‡Π½Ρ‹ΠΉ ΠΊΡ€ΡƒΠ³ (с радиусом = 1) Π·Π΅Π»Π΅Π½ΠΎΠ³ΠΎ Ρ†Π²Π΅Ρ‚Π°, располоТСнный Π² Π½Π°Ρ‡Π°Π»Π΅ ΠΊΠΎΠΎΡ€Π΄ΠΈΠ½Π°Ρ‚ Π² ΠΏΡ€Π°Π²ΠΎΠΌ Π½ΠΈΠΆΠ½Π΅ΠΌ ΡƒΠ³Π»Ρƒ.

Π’ сСрСдинС этого ΠΊΡ€ΡƒΠ³Π° ΠΆΠ΅Π»Ρ‚Ρ‹ΠΌ Ρ†Π²Π΅Ρ‚ΠΎΠΌ прСдставлСн ΡƒΠ³ΠΎΠ» Ρ‚Π΅Ρ‚Π° (ΞΈ). Π­Ρ‚ΠΎΡ‚ ΡƒΠ³ΠΎΠ» прСдставляСт собой Π²Π΅Π»ΠΈΡ‡ΠΈΠ½Ρƒ вращСния ΠΏΡ€ΠΎΡ‚ΠΈΠ² часовой стрСлки ΠΏΠΎ ΠΊΡ€ΡƒΠ³Ρƒ, начиная справа ΠΏΠΎ оси x, ΠΊΠ°ΠΊ ΠΏΠΎΠΊΠ°Π·Π°Π½ΠΎ Π½Π° рисункС. Вочная копия этого нСбольшого ΡƒΠ³Π»Π° ΠΏΠΎΠΊΠ°Π·Π°Π½Π° Π²Π²Π΅Ρ€Ρ…Ρƒ справа ΠΊΠ°ΠΊ наглядная ΠΈΠ»Π»ΡŽΡΡ‚Ρ€Π°Ρ†ΠΈΡ опрСдСлСния ΞΈ.

Под этим ΡƒΠ³Π»ΠΎΠΌ, начиная с Π½Π°Ρ‡Π°Π»Π° ΠΊΠΎΠΎΡ€Π΄ΠΈΠ½Π°Ρ‚, Ρ€Π°Π΄ΠΈΠ°Π»ΡŒΠ½ΠΎ Π½Π°Ρ€ΡƒΠΆΡƒ проводится (тусклая) зСлСная линия. Π­Ρ‚Π° линия пСрСсСкаСт Π΅Π΄ΠΈΠ½ΠΈΡ‡Π½Ρ‹ΠΉ ΠΊΡ€ΡƒΠ³ Π² ΠΎΠ΄Π½ΠΎΠΉ Ρ‚ΠΎΡ‡ΠΊΠ΅, которая прСдставляСт собой Π·Π΅Π»Π΅Π½ΡƒΡŽ Ρ‚ΠΎΡ‡ΠΊΡƒ, Π²Ρ€Π°Ρ‰Π°ΡŽΡ‰ΡƒΡŽΡΡ с постоянной ΡΠΊΠΎΡ€ΠΎΡΡ‚ΡŒΡŽ ΠΏΡ€ΠΈ ΠΈΠ·ΠΌΠ΅Π½Π΅Π½ΠΈΠΈ ΡƒΠ³Π»Π° ΞΈ, Ρ‚Π°ΠΊΠΆΠ΅ с постоянной ΡΠΊΠΎΡ€ΠΎΡΡ‚ΡŒΡŽ.

Π’Π΅Ρ€Ρ‚ΠΈΠΊΠ°Π»ΡŒΠ½ΠΎΠ΅ ΠΏΠΎΠ»ΠΎΠΆΠ΅Π½ΠΈΠ΅ этой Ρ‚ΠΎΡ‡ΠΊΠΈ проСцируСтся прямо (вдоль слабой красной Π»ΠΈΠ½ΠΈΠΈ) Π½Π° Π³Ρ€Π°Ρ„ΠΈΠΊ слСва ΠΎΡ‚ ΠΊΡ€ΡƒΠ³Π°. Π­Ρ‚ΠΎ ΠΏΡ€ΠΈΠ²ΠΎΠ΄ΠΈΡ‚ ΠΊ красной Ρ‚ΠΎΡ‡ΠΊΠ΅. ΠšΠΎΠΎΡ€Π΄ΠΈΠ½Π°Ρ‚Π° Y этой красной Ρ‚ΠΎΡ‡ΠΊΠΈ (такая ΠΆΠ΅, ΠΊΠ°ΠΊ ΠΊΠΎΠΎΡ€Π΄ΠΈΠ½Π°Ρ‚Π° Y Π·Π΅Π»Π΅Π½ΠΎΠΉ Ρ‚ΠΎΡ‡ΠΊΠΈ) являСтся Π·Π½Π°Ρ‡Π΅Π½ΠΈΠ΅ΠΌ ΡΠΈΠ½ΡƒΡΠΎΠΈΠ΄Π°Π»ΡŒΠ½ΠΎΠΉ Ρ„ΡƒΠ½ΠΊΡ†ΠΈΠΈ, вычислСнной ΠΏΠΎΠ΄ ΡƒΠ³Π»ΠΎΠΌ ΞΈ, Ρ‚ΠΎ Π΅ΡΡ‚ΡŒ:

ΠšΠΎΠΎΡ€Π΄ΠΈΠ½Π°Ρ‚Π° y Π·Π΅Π»Π΅Π½ΠΎΠΉ Ρ‚ΠΎΡ‡ΠΊΠΈ = sin ΞΈ

ΠŸΡ€ΠΈ ΠΈΠ·ΠΌΠ΅Π½Π΅Π½ΠΈΠΈ ΡƒΠ³Π»Π° ΞΈ красная Ρ‚ΠΎΡ‡ΠΊΠ° пСрСмСщаСтся Π²Π²Π΅Ρ€Ρ… ΠΈ Π²Π½ΠΈΠ·, отслСТивая красный Π³Ρ€Π°Ρ„ΠΈΠΊ. Π­Ρ‚ΠΎ Π³Ρ€Π°Ρ„ΠΈΠΊ ΡΠΈΠ½ΡƒΡΠΎΠΈΠ΄Π°Π»ΡŒΠ½ΠΎΠΉ Ρ„ΡƒΠ½ΠΊΡ†ΠΈΠΈ.Π‘Π»Π°Π±Ρ‹Π΅ Π²Π΅Ρ€Ρ‚ΠΈΠΊΠ°Π»ΡŒΠ½Ρ‹Π΅ Π»ΠΈΠ½ΠΈΠΈ, проходящиС слСва, ΠΎΡ‚ΠΌΠ΅Ρ‡Π°ΡŽΡ‚ ΠΊΠ°ΠΆΠ΄Ρ‹ΠΉ ΠΊΠ²Π°Π΄Ρ€Π°Π½Ρ‚ ΠΏΠΎ окруТности, Ρ‚ΠΎ Π΅ΡΡ‚ΡŒ ΠΏΠΎΠ΄ ΠΊΠ°ΠΆΠ΄Ρ‹ΠΌ ΡƒΠ³Π»ΠΎΠΌ 90 Β° ΠΈΠ»ΠΈ Ο€ / 2 Ρ€Π°Π΄ΠΈΠ°Π½. ΠžΠ±Ρ€Π°Ρ‚ΠΈΡ‚Π΅ Π²Π½ΠΈΠΌΠ°Π½ΠΈΠ΅, ΠΊΠ°ΠΊ синусоида ΠΈΠ΄Π΅Ρ‚ ΠΎΡ‚ 1 ΠΊ Π½ΡƒΠ»ΡŽ, ΠΊ -1, Π° Π·Π°Ρ‚Π΅ΠΌ ΠΎΠ±Ρ€Π°Ρ‚Π½ΠΎ ΠΊ Π½ΡƒΠ»ΡŽ, ΠΈΠΌΠ΅Π½Π½ΠΎ Π½Π° этих линиях. Π­Ρ‚ΠΎ ΠΎΡ‚Ρ€Π°ΠΆΠ°Π΅Ρ‚ Ρ‚ΠΎΡ‚ Ρ„Π°ΠΊΡ‚, Ρ‡Ρ‚ΠΎ sin (0) = 0, sin (Ο€ / 2) = 1, sin (Ο€) = 0 ΠΈ sin (3Ο€ / 2) -1.

Аналогичный процСсс выполняСтся с ΠΊΠΎΠΎΡ€Π΄ΠΈΠ½Π°Ρ‚ΠΎΠΉ x Π·Π΅Π»Π΅Π½ΠΎΠΉ Ρ‚ΠΎΡ‡ΠΊΠΈ. Однако, ΠΏΠΎΡΠΊΠΎΠ»ΡŒΠΊΡƒ ΠΊΠΎΠΎΡ€Π΄ΠΈΠ½Π°Ρ‚Π° x ΠΎΡ‚ΠΊΠ»ΠΎΠ½Π΅Π½Π° ΠΎΡ‚ ΠΎΠ±Ρ‹Ρ‡Π½ΠΎΠ³ΠΎ соглашСния для построСния Π³Ρ€Π°Ρ„ΠΈΠΊΠΎΠ² (Π³Π΄Π΅ y = f (x), с Π²Π΅Ρ€Ρ‚ΠΈΠΊΠ°Π»ΡŒΠ½Ρ‹ΠΌ y ΠΈ x Π³ΠΎΡ€ΠΈΠ·ΠΎΠ½Ρ‚Π°Π»ΡŒΠ½Ρ‹ΠΌ), Π±Ρ‹Π»Π° Π²Ρ‹ΠΏΠΎΠ»Π½Π΅Π½Π° опСрация Β«untiltΒ», Ρ‡Ρ‚ΠΎΠ±Ρ‹ ΠΏΠΎΠ²Ρ‚ΠΎΡ€ΠΈΡ‚ΡŒ процСсс снова Π² Ρ‚ΠΎΠΌ ΠΆΠ΅ ориСнтация, Π° Π½Π΅ Π²Π΅Ρ€Ρ‚ΠΈΠΊΠ°Π»ΡŒΠ½Π°Ρ.Π­Ρ‚ΠΎ Π±Ρ‹Π»ΠΎ прСдставлСно Π² Π²ΠΈΠ΄Π΅ Β«ΠΈΠ·Π³ΠΈΠ±Π°Β», ΠΏΠΎΠΊΠ°Π·Π°Π½Π½ΠΎΠ³ΠΎ Π² ΠΏΡ€Π°Π²ΠΎΠΌ Π²Π΅Ρ€Ρ…Π½Π΅ΠΌ ΡƒΠ³Π»Ρƒ.

И снова зСлСная Ρ‚ΠΎΡ‡ΠΊΠ° проСцируСтся Π²Π²Π΅Ρ€Ρ… (вдоль слабой синСй Π»ΠΈΠ½ΠΈΠΈ), ΠΈ эта «изогнутая» проСкция заканчиваСтся Ρƒ самого ΠΏΡ€Π°Π²ΠΎΠ³ΠΎ края Π²Π΅Ρ€Ρ…Π½Π΅Π³ΠΎ Π³Ρ€Π°Ρ„ΠΈΠΊΠ°, Π² синСй Ρ‚ΠΎΡ‡ΠΊΠ΅. ΠšΠΎΠΎΡ€Π΄ΠΈΠ½Π°Ρ‚Π° y этой синСй Ρ‚ΠΎΡ‡ΠΊΠΈ (которая ΠΈΠ·-Π·Π° Β«ΠΈΠ·Π³ΠΈΠ±Π°Β» ΠΏΡ€ΠΎΠ΅ΠΊΡ†ΠΈΠΈ совпадаСт с ΠΊΠΎΠΎΡ€Π΄ΠΈΠ½Π°Ρ‚ΠΎΠΉ x Π·Π΅Π»Π΅Π½ΠΎΠΉ Ρ‚ΠΎΡ‡ΠΊΠΈ) — это Π·Π½Π°Ρ‡Π΅Π½ΠΈΠ΅ Ρ„ΡƒΠ½ΠΊΡ†ΠΈΠΈ косинуса, вычислСнноС ΠΏΠΎΠ΄ ΡƒΠ³Π»ΠΎΠΌ ΞΈ, Ρ‚ΠΎ Π΅ΡΡ‚ΡŒ:

x ΠΊΠΎΠΎΡ€Π΄ΠΈΠ½Π°Ρ‚Π° Π·Π΅Π»Π΅Π½ΠΎΠΉ Ρ‚ΠΎΡ‡ΠΊΠΈ = cos ΞΈ
Биняя кривая, очСрчСнная этой Ρ‚ΠΎΡ‡ΠΊΠΎΠΉ, ΠΊΠΎΠ³Π΄Π° ΠΎΠ½Π° двиТСтся Π²Π²Π΅Ρ€Ρ… ΠΈ Π²Π½ΠΈΠ· с ΠΈΠ·ΠΌΠ΅Π½Π΅Π½ΠΈΠ΅ΠΌ ΞΈ, являСтся Π³Ρ€Π°Ρ„ΠΈΠΊΠΎΠΌ Ρ„ΡƒΠ½ΠΊΡ†ΠΈΠΈ косинуса.Π•Ρ‰Π΅ Ρ€Π°Π· ΠΎΠ±Ρ€Π°Ρ‚ΠΈΡ‚Π΅ Π²Π½ΠΈΠΌΠ°Π½ΠΈΠ΅ Π½Π° Ρ‚ΠΎ, ΠΊΠ°ΠΊ ΠΎΠ½ Π²Π΅Π΄Π΅Ρ‚ сСбя ΠΏΡ€ΠΈ пСрСсСчСнии ΠΊΠ°ΠΆΠ΄ΠΎΠ³ΠΎ ΠΊΠ²Π°Π΄Ρ€Π°Π½Ρ‚Π°, отраТая Ρ‚ΠΎΡ‚ Ρ„Π°ΠΊΡ‚, Ρ‡Ρ‚ΠΎ cos (0) = 1, cos (Ο€ / 2) = 0, cos (Ο€) = -1 ΠΈ cos (3Ο€ / 2) = 0.

Π•Π΄ΠΈΠ½ΠΈΡ‡Π½Ρ‹ΠΉ ΠΊΡ€ΡƒΠ³ ΠžΠΏΡ€Π΅Π΄Π΅Π»Π΅Π½ΠΈΠ΅ Ρ„ΡƒΠ½ΠΊΡ†ΠΈΠΉ синуса ΠΈ косинуса


Π•Π΄ΠΈΠ½ΠΈΡ†Π° ΠšΡ€ΡƒΠ³ΠΎΠ²ΠΎΠ΅ ΠΎΠΏΡ€Π΅Π΄Π΅Π»Π΅Π½ΠΈΠ΅ Ρ„ΡƒΠ½ΠΊΡ†ΠΈΠΉ синуса ΠΈ косинуса

ВригономСтричСскиС Ρ„ΡƒΠ½ΠΊΡ†ΠΈΠΈ ΠΌΠΎΠ³ΡƒΡ‚ Π±Ρ‹Ρ‚ΡŒ опрСдСляСтся Π² Ρ‚Π΅Ρ€ΠΌΠΈΠ½Π°Ρ… Π΅Π΄ΠΈΠ½ΠΈΡ‡Π½ΠΎΠ³ΠΎ ΠΊΡ€ΡƒΠ³Π°, Ρ‚. Π΅. ΠΊΡ€ΡƒΠ³ радиуса ΠΎΠ΄ΠΈΠ½.

The sin / cos Π’Ρ€Π΅ΡƒΠ³ΠΎΠ»ΡŒΠ½ΠΈΠΊ

Если Π΅Π΄ΠΈΠ½ΠΈΡ‡Π½Ρ‹ΠΉ ΠΊΡ€ΡƒΠ³ ΠΏΠΎΠΌΠ΅Ρ‰Π΅Π½ Π² Π½Π°Ρ‡Π°Π»ΠΎ ΠΏΡ€ΡΠΌΠΎΡƒΠ³ΠΎΠ»ΡŒΠ½ΠΎΠΉ систСмы ΠΊΠΎΠΎΡ€Π΄ΠΈΠ½Π°Ρ‚ с ΡƒΠ³Π»ΠΎΠΌ q, отсчитываСмым ΠΎΡ‚ ΠΏΠΎΠ»ΠΎΠΆΠΈΡ‚Π΅Π»ΡŒΠ½Π°Ρ ось x ΠΊ сторонС Π²Ρ‹Π²ΠΎΠ΄Π°, Π·Π°Ρ‚Π΅ΠΌ Ρ‚ΠΎΡ‡ΠΊΠ° Π½Π° устройствС ΠΊΡ€ΡƒΠ³, Π³Π΄Π΅ конСчная сторона пСрСсСкаСт Π΅Π΄ΠΈΠ½ΠΈΡ‡Π½Ρ‹ΠΉ ΠΊΡ€ΡƒΠ³, опрСдСляСтся ΠΊΠ°ΠΊ Π±Ρ‹Ρ‚ΡŒ (cos q, sin q), Ρ‚.Π΅. пСрвая ΠΊΠΎΠΎΡ€Π΄ΠΈΠ½Π°Ρ‚Π° Ρ‚ΠΎΡ‡ΠΊΠΈ Π½Π° устройствС ΠΊΡ€ΡƒΠ³ — это cos q, Π° вторая ΠΊΠΎΠΎΡ€Π΄ΠΈΠ½Π°Ρ‚Π° — sin q.

The tan / sec Π’Ρ€Π΅ΡƒΠ³ΠΎΠ»ΡŒΠ½ΠΈΠΊ

Tan q ΠΈ sec q ΠΎΠΏΡ€Π΅Π΄Π΅Π»Π΅Π½Ρ‹ Ρ‚Ρ€Π΅ΡƒΠ³ΠΎΠ»ΡŒΠ½ΠΈΠΊΠΎΠΌ, высота ΠΊΠΎΡ‚ΠΎΡ€ΠΎΠ³ΠΎ касаСтся Π΅Π΄ΠΈΠ½ΠΈΡ‡Π½ΠΎΠΉ окруТности Π² Ρ‚ΠΎΡ‡ΠΊΠ΅ (1, 0), Π³ΠΈΠΏΠΎΡ‚Π΅Π½ΡƒΠ·Π° ΠΊΠΎΡ‚ΠΎΡ€ΠΎΠ³ΠΎ находится Π½Π° ΠΊΠΎΠ½Π΅Ρ‡Π½ΠΎΠΉ сторонС ΡƒΠ³Π»Π°.

The дСтская ΠΊΡ€ΠΎΠ²Π°Ρ‚ΠΊΠ° / csc Triangle

ДСтская ΠΊΡ€ΠΎΠ²Π°Ρ‚ΠΊΠ° q ΠΈ csc q ΠΎΠΏΡ€Π΅Π΄Π΅Π»Π΅Π½Ρ‹ Ρ‚Ρ€Π΅ΡƒΠ³ΠΎΠ»ΡŒΠ½ΠΈΠΊΠΎΠΌ, высота ΠΊΠΎΡ‚ΠΎΡ€ΠΎΠ³ΠΎ Ρ€Π°Π²Π½Π° Π΅Π΄ΠΈΠ½ΠΈΡ†Π΅, Π° Π³ΠΈΠΏΠΎΡ‚Π΅Π½ΡƒΠ·Π° находится Π½Π° конСчная сторона ΡƒΠ³ΠΎΠ»ΠΊΠ°.

The sin / cos, tan / sec ΠΈ cot / csc Π’Ρ€Π΅ΡƒΠ³ΠΎΠ»ΡŒΠ½ΠΈΠΊΠΈ

ВсС Ρ‚Ρ€ΠΈ Ρ‚Ρ€Π΅ΡƒΠ³ΠΎΠ»ΡŒΠ½ΠΈΠΊΠΈ, ΠΈΡΠΏΠΎΠ»ΡŒΠ·ΡƒΠ΅ΠΌΡ‹Π΅ для опрСдСлСния тригономСтричСских Ρ„ΡƒΠ½ΠΊΡ†ΠΈΠΉ, ΠΏΠΎΠΊΠ°Π·Π°Π½Ρ‹ Π½Π° рисункС Π½ΠΈΠΆΠ΅.

Π˜ΡΠΏΠΎΠ»ΡŒΠ·ΡƒΡ sin / cos, tan / sec ΠΈ cot / csc Π’Ρ€Π΅ΡƒΠ³ΠΎΠ»ΡŒΠ½ΠΈΠΊΠΈ для установлСния Π±Π°Π·ΠΎΠ²ΠΎΠ³ΠΎ ВригономСтричСскиС ΠΈΠ΄Π΅Π½Ρ‚ΠΈΡ„ΠΈΠΊΠ°Ρ†ΠΈΠΎΠ½Π½Ρ‹Π΅ Π΄Π°Π½Π½Ρ‹Π΅

Π’Ρ€ΠΈ ΠΎΠ΄ΠΈΠ½Π°ΠΊΠΎΠ²Ρ‹Ρ… Ρ‚Ρ€Π΅ΡƒΠ³ΠΎΠ»ΡŒΠ½ΠΈΠΊΠ° sin / cos, tan / sec ΠΈ cot / csc ΠΈΠ·Π²Π»Π΅ΠΊΠ°ΡŽΡ‚ΡΡ ΠΈΠ· рисунка.Π§Π΅Ρ‚Π²Π΅Ρ€Ρ‚Ρ‹ΠΉ Π°Π½Π°Π»ΠΎΠ³ΠΈΡ‡Π½Ρ‹ΠΉ ΠΏΠΎΠΊΠ°Π·Π°Π½ Ρ‚Ρ€Π΅ΡƒΠ³ΠΎΠ»ΡŒΠ½ΠΈΠΊ со смСТными, ΠΏΡ€ΠΎΡ‚ΠΈΠ²ΠΎΠΏΠΎΠ»ΠΎΠΆΠ½Ρ‹ΠΌΠΈ сторонами ΠΈ сторонами Π³ΠΈΠΏΠΎΡ‚Π΅Π½ΡƒΠ·Ρ‹. ΠΏΠΎΠΌΠ΅Ρ‡Π΅Π½Ρ‹.


ΠžΠΏΡ€Π΅Π΄Π΅Π»Π΅Π½ΠΈΠ΅ ΡˆΠ΅ΡΡ‚ΠΈ тригономСтричСскиС Ρ„ΡƒΠ½ΠΊΡ†ΠΈΠΈ ΠΈ Π΄Ρ€ΡƒΠ³ΠΈΠ΅ ΠΏΠΎΠ»Π΅Π·Π½Ρ‹Π΅ тоТдСства ΡΠ»Π΅Π΄ΡƒΡŽΡ‚ ΠΈΠ· использования Ρ‚ΠΎΡ‚ Ρ„Π°ΠΊΡ‚, Ρ‡Ρ‚ΠΎ ΠΎΡ‚Π½ΠΎΡˆΠ΅Π½ΠΈΠ΅ ΡΠΎΠΎΡ‚Π²Π΅Ρ‚ΡΡ‚Π²ΡƒΡŽΡ‰ΠΈΡ… сторон ΠΎΠ΄ΠΈΠ½Π°ΠΊΠΎΠ²Ρ‹Ρ… Ρ‚Ρ€Π΅ΡƒΠ³ΠΎΠ»ΡŒΠ½ΠΈΠΊΠΎΠ² Π΄ΠΎΠ»ΠΆΠ½ΠΎ Π±Ρ‹Ρ‚ΡŒ Ρ€Π°Π²Π½Ρ‹ΠΌ. Π Π΅Π·ΡƒΠ»ΡŒΡ‚Π°Ρ‚Ρ‹:

Π˜ΡΠΏΠΎΠ»ΡŒΠ·ΡƒΡ sin / cos, tan / sec ΠΈ cot / csc Π’Ρ€Π΅ΡƒΠ³ΠΎΠ»ΡŒΠ½ΠΈΠΊΠΈ для опрСдСлСния пифагорСйского Π˜Π΄Π΅Π½Ρ‚ΠΈΡ„ΠΈΠΊΠ°Ρ†ΠΈΠΎΠ½Π½Ρ‹Π΅ Π΄Π°Π½Π½Ρ‹Π΅

Π’Π΅ΠΎΡ€Π΅ΠΌΠ° ΠŸΠΈΡ„Π°Π³ΠΎΡ€Π° ΡƒΡ‚Π²Π΅Ρ€ΠΆΠ΄Π°Π΅Ρ‚: Π² любом ΠΏΡ€ΡΠΌΠΎΡƒΠ³ΠΎΠ»ΡŒΠ½Ρ‹ΠΉ Ρ‚Ρ€Π΅ΡƒΠ³ΠΎΠ»ΡŒΠ½ΠΈΠΊ, сумма ΠΊΠ²Π°Π΄Ρ€Π°Ρ‚ΠΎΠ² Π΄Π»ΠΈΠ½ стороны, содСрТащиС прямой ΡƒΠ³ΠΎΠ», Ρ€Π°Π²Π½Ρ‹ ΠΊΠ²Π°Π΄Ρ€Π°Ρ‚Ρƒ Π³ΠΈΠΏΠΎΡ‚Π΅Π½ΡƒΠ·Π°.ΠšΠΎΡ€ΠΎΡ‡Π΅ c 2 = a 2 + b 2 .

ΠŸΡ€ΠΈΠΌΠ΅Π½Π΅Π½ΠΈΠ΅ ΠΏΠΈΡ„Π°Π³ΠΎΡ€Π° Π’Π΅ΠΎΡ€Π΅ΠΌΠ° для Ρ‚Ρ€Π΅ΡƒΠ³ΠΎΠ»ΡŒΠ½ΠΈΠΊΠΎΠ² sin / cos, tan / sec ΠΈ cot / csc Π΄Π°Π΅Ρ‚:

Π•Π΄ΠΈΠ½ΠΈΡ‡Π½Ρ‹ΠΉ ΠΊΡ€ΡƒΠ³ | Purplemath

Purplemath

Когда Π²Ρ‹ Ρ€Π°Π±ΠΎΡ‚Π°Π΅Ρ‚Π΅ с ΡƒΠ³Π»Π°ΠΌΠΈ Π²ΠΎ всСх Ρ‡Π΅Ρ‚Ρ‹Ρ€Π΅Ρ… ΠΊΠ²Π°Π΄Ρ€Π°Π½Ρ‚Π°Ρ…, Ρ‚Ρ€ΠΈΠ³Π³Π΅Ρ€Π½Ρ‹Π΅ ΠΎΡ‚Π½ΠΎΡˆΠ΅Π½ΠΈΡ для этих ΡƒΠ³Π»ΠΎΠ² Π²Ρ‹Ρ‡ΠΈΡΠ»ΡΡŽΡ‚ΡΡ Π² Ρ‚Π΅Ρ€ΠΌΠΈΠ½Π°Ρ… Π·Π½Π°Ρ‡Π΅Π½ΠΈΠΉ x , y ΠΈ r , Π³Π΄Π΅ r — радиус окруТности, ΡΠΎΠΎΡ‚Π²Π΅Ρ‚ΡΡ‚Π²ΡƒΡŽΡ‰Π΅ΠΉ ΠΊ Π³ΠΈΠΏΠΎΡ‚Π΅Π½ΡƒΠ·Π΅ ΠΏΡ€ΡΠΌΠΎΡƒΠ³ΠΎΠ»ΡŒΠ½ΠΎΠ³ΠΎ Ρ‚Ρ€Π΅ΡƒΠ³ΠΎΠ»ΡŒΠ½ΠΈΠΊΠ° для вашСго ΡƒΠ³Π»Π°.На рисункС Π½ΠΈΠΆΠ΅ ΡƒΠ³ΠΎΠ» заканчиваСтся Π²ΠΎ Π²Ρ‚ΠΎΡ€ΠΎΠΌ ΠΊΠ²Π°Π΄Ρ€Π°Π½Ρ‚Π΅, ΠΊΠ°ΠΊ ΠΏΠΎΠΊΠ°Π·Π°Π½ΠΎ диагональной Π»ΠΈΠ½ΠΈΠ΅ΠΉ:

MathHelp.com

Π›ΡŽΠ±Ρ‹Π΅ Π΄Π²Π° ΠΏΡ€ΡΠΌΠΎΡƒΠ³ΠΎΠ»ΡŒΠ½Ρ‹Ρ… Ρ‚Ρ€Π΅ΡƒΠ³ΠΎΠ»ΡŒΠ½ΠΈΠΊΠ° с ΠΎΠ΄ΠΈΠ½Π°ΠΊΠΎΠ²Ρ‹ΠΌ ΡƒΠ³Π»ΠΎΠΌ Π² основании ΞΈ («Ρ‚Π΅Ρ‚Π°», произносится ΠΊΠ°ΠΊ THAY-tuh) Π±ΡƒΠ΄ΡƒΡ‚ ΠΏΠΎΡ…ΠΎΠΆΠΈ Π² тСхничСском смыслС, имСя ΠΏΡ€ΠΎΠΏΠΎΡ€Ρ†ΠΈΠΎΠ½Π°Π»ΡŒΠ½Ρ‹Π΅ стороны.Π­Ρ‚ΠΎ сходство Π±ΠΎΠ»Π΅Π΅ ΠΎΡ‡Π΅Π²ΠΈΠ΄Π½ΠΎ, ΠΊΠΎΠ³Π΄Π° Ρ‚Ρ€Π΅ΡƒΠ³ΠΎΠ»ΡŒΠ½ΠΈΠΊΠΈ Π²Π»ΠΎΠΆΠ΅Π½Ρ‹ Π΄Ρ€ΡƒΠ³ Π² Π΄Ρ€ΡƒΠ³Π°:

Бходство (ΠΈ, ΡΠ»Π΅Π΄ΠΎΠ²Π°Ρ‚Π΅Π»ΡŒΠ½ΠΎ, ΠΏΡ€ΠΎΠΏΠΎΡ€Ρ†ΠΈΠΎΠ½Π°Π»ΡŒΠ½ΠΎΡΡ‚ΡŒ) ΠΎΠ·Π½Π°Ρ‡Π°Π΅Ρ‚, Ρ‡Ρ‚ΠΎ Ρ‚Ρ€ΠΈΠ³Π³Π΅Ρ€Π½Ρ‹Π΅ ΠΎΡ‚Π½ΠΎΡˆΠ΅Π½ΠΈΡ для Π΄Π²ΡƒΡ… Π²Π»ΠΎΠΆΠ΅Π½Π½Ρ‹Ρ… Ρ‚Ρ€Π΅ΡƒΠ³ΠΎΠ»ΡŒΠ½ΠΈΠΊΠΎΠ², ΠΏΠΎΠΊΠ°Π·Π°Π½Π½Ρ‹Ρ… Π²Ρ‹ΡˆΠ΅, Π±ΡƒΠ΄ΡƒΡ‚ ΠΎΠ΄ΠΈΠ½Π°ΠΊΠΎΠ²Ρ‹ΠΌΠΈ, ΠΊΠ°ΠΊ Π²Ρ‹ ΠΌΠΎΠΆΠ΅Ρ‚Π΅ Π²ΠΈΠ΄Π΅Ρ‚ΡŒ ΠΈΠ· вычислСний Π½ΠΈΠΆΠ΅ для ΠΊΠ°ΠΆΠ΄ΠΎΠ³ΠΎ ΠΈΠ· Π΄Π²ΡƒΡ… Ρ‚Ρ€Π΅ΡƒΠ³ΠΎΠ»ΡŒΠ½ΠΈΠΊΠΎΠ² Π²Ρ‹ΡˆΠ΅:

Π’Ρ€ΠΈΠ³Π³Π΅Ρ€Π½Ρ‹Π΅ ΠΎΡ‚Π½ΠΎΡˆΠ΅Π½ΠΈΡ для ΡƒΠ³Π»Π° ΞΈ ΠΎΠ΄ΠΈΠ½Π°ΠΊΠΎΠ²ΠΎΠ³ΠΎ Ρ€Π°Π·ΠΌΠ΅Ρ€Π° ΠΎΠ΄ΠΈΠ½Π°ΠΊΠΎΠ²Ρ‹ (ΠΊΠ°ΠΊ Π²Ρ‹ ΠΌΠΎΠΆΠ΅Ρ‚Π΅ Π²ΠΈΠ΄Π΅Ρ‚ΡŒ Π²Ρ‹ΡˆΠ΅), Π΄Π°ΠΆΠ΅ нСсмотря Π½Π° Ρ‚ΠΎ, Ρ‡Ρ‚ΠΎ ΠΊΠΎΠ½ΠΊΡ€Π΅Ρ‚Π½Ρ‹Π΅ числа ΠΈΠ· Π½Π°Π±ΠΎΡ€ΠΎΠ² сторон Π΄Π²ΡƒΡ… Ρ‚Ρ€Π΅ΡƒΠ³ΠΎΠ»ΡŒΠ½ΠΈΠΊΠΎΠ² Ρ€Π°Π·Π»ΠΈΡ‡Π°ΡŽΡ‚ΡΡ.Π­Ρ‚ΠΎ ΠΏΠΎΠ΄Ρ‡Π΅Ρ€ΠΊΠΈΠ²Π°Π΅Ρ‚, Ρ‡Ρ‚ΠΎ для тригономСтричСских ΡΠΎΠΎΡ‚Π½ΠΎΡˆΠ΅Π½ΠΈΠΉ ΠΈΠΌΠ΅Π΅Ρ‚ Π·Π½Π°Ρ‡Π΅Π½ΠΈΠ΅ ΡƒΠ³ΠΎΠ» ΞΈ, Π° Π½Π΅ ΠΊΠΎΠ½ΠΊΡ€Π΅Ρ‚Π½Ρ‹ΠΉ Ρ‚Ρ€Π΅ΡƒΠ³ΠΎΠ»ΡŒΠ½ΠΈΠΊ, ΠΈΠ· ΠΊΠΎΡ‚ΠΎΡ€ΠΎΠ³ΠΎ Π²Ρ‹ ΠΏΠΎΠ»ΡƒΡ‡ΠΈΠ»ΠΈ этот ΡƒΠ³ΠΎΠ».

Для упрощСния вычислСний ΠΌΠ°Ρ‚Π΅ΠΌΠ°Ρ‚ΠΈΠΊΠΈ Π»ΡŽΠ±ΡΡ‚ ΠΏΠΎΠΌΠ΅Ρ‰Π°Ρ‚ΡŒ ΡƒΠ³Π»ΠΎΠ²ΠΎΠΉ Ρ‚Ρ€Π΅ΡƒΠ³ΠΎΠ»ΡŒΠ½ΠΈΠΊ Π² ΠΊΡ€ΡƒΠ³ с радиусом r = 1. ΠŸΠΎΡΠΊΠΎΠ»ΡŒΠΊΡƒ число 1 Π² ΠΌΠ°Ρ‚Π΅ΠΌΠ°Ρ‚ΠΈΠΊΠ΅ называСтся Β«Π΅Π΄ΠΈΠ½ΠΈΡ†Π΅ΠΉΒ», ΠΊΡ€ΡƒΠ³ с радиусом Π΄Π»ΠΈΠ½Ρ‹ 1 называСтся Β«Π΅Π΄ΠΈΠ½ΠΈΡ‡Π½Ρ‹ΠΌ ΠΊΡ€ΡƒΠ³ΠΎΠΌΒ». «. Как Ρ‚ΠΎΠ»ΡŒΠΊΠΎ Π³ΠΈΠΏΠΎΡ‚Π΅Π½ΡƒΠ·Π° ΠΈΠΌΠ΅Π΅Ρ‚ Ρ„ΠΈΠΊΡΠΈΡ€ΠΎΠ²Π°Π½Π½ΡƒΡŽ Π΄Π»ΠΈΠ½Ρƒ r = 1, Ρ‚ΠΎΠ³Π΄Π° значСния Ρ‚Ρ€ΠΈΠ³Π³Π΅Ρ€Π½Ρ‹Ρ… ΠΎΡ‚Π½ΠΎΡˆΠ΅Π½ΠΈΠΉ Π±ΡƒΠ΄ΡƒΡ‚ Π·Π°Π²ΠΈΡΠ΅Ρ‚ΡŒ Ρ‚ΠΎΠ»ΡŒΠΊΠΎ ΠΎΡ‚ x ΠΈ y , ΠΏΠΎΡΠΊΠΎΠ»ΡŒΠΊΡƒ ΡƒΠΌΠ½ΠΎΠΆΠ΅Π½ΠΈΠ΅ ΠΈΠ»ΠΈ Π΄Π΅Π»Π΅Π½ΠΈΠ΅ Π½Π° r = 1 Π½ΠΈΡ‡Π΅Π³ΠΎ Π½Π΅ ΠΈΠ·ΠΌΠ΅Π½ΠΈΡ‚.Волько значСния x ΠΈ y Π±ΡƒΠ΄ΡƒΡ‚ ΠΈΠΌΠ΅Ρ‚ΡŒ Π·Π½Π°Ρ‡Π΅Π½ΠΈΠ΅.


Π•Π΄ΠΈΠ½ΠΈΡ‡Π½Ρ‹ΠΉ ΠΊΡ€ΡƒΠ³

Бмысл Π΅Π΄ΠΈΠ½ΠΈΡ‡Π½ΠΎΠ³ΠΎ ΠΊΡ€ΡƒΠ³Π° Π² Ρ‚ΠΎΠΌ, Ρ‡Ρ‚ΠΎ ΠΎΠ½ ΡƒΠΏΡ€ΠΎΡ‰Π°Π΅Ρ‚ ΠΈ ΡƒΠΏΡ€ΠΎΡ‰Π°Π΅Ρ‚ Π΄Ρ€ΡƒΠ³ΠΈΠ΅ части ΠΌΠ°Ρ‚Π΅ΠΌΠ°Ρ‚ΠΈΠΊΠΈ. НапримСр, Π² Π΅Π΄ΠΈΠ½ΠΈΡ‡Π½ΠΎΠΌ ΠΊΡ€ΡƒΠ³Π΅ для любого ΡƒΠ³Π»Π° ΞΈ Ρ‚Ρ€ΠΈΠ³Π³Π΅Ρ€Π½Ρ‹Π΅ значСния для синуса ΠΈ косинуса явно Π½Π΅ Π±ΠΎΠ»Π΅Π΅ Ρ‡Π΅ΠΌ sin (ΞΈ) = y ΠΈ cos (ΞΈ) = x .Π˜ΡΡ…ΠΎΠ΄Ρ ΠΈΠ· этого, Π²Ρ‹ ΠΌΠΎΠΆΠ΅Ρ‚Π΅ ΠΏΡ€ΠΈΠ½ΡΡ‚ΡŒ Ρ‚ΠΎΡ‚ Ρ„Π°ΠΊΡ‚, Ρ‡Ρ‚ΠΎ тангСнс ΠΎΠΏΡ€Π΅Π΄Π΅Π»Π΅Π½ ΠΊΠ°ΠΊ тангСнс (ΞΈ) = y / x , Π° Π·Π°Ρ‚Π΅ΠΌ Π·Π°ΠΌΠ΅Π½ΠΈΡ‚ΡŒ x ΠΈ y , Ρ‡Ρ‚ΠΎΠ±Ρ‹ Π»Π΅Π³ΠΊΠΎ Π΄ΠΎΠΊΠ°Π·Π°Ρ‚ΡŒ, Ρ‡Ρ‚ΠΎ Π·Π½Π°Ρ‡Π΅Π½ΠΈΠ΅ tan (ΞΈ) Ρ‚Π°ΠΊΠΆΠ΅ Π΄ΠΎΠ»ΠΆΠ΅Π½ Π±Ρ‹Ρ‚ΡŒ Ρ€Π°Π²Π΅Π½ ΠΎΡ‚Π½ΠΎΡˆΠ΅Π½ΠΈΡŽ sin (ΞΈ) / cos (ΞΈ).

Π•Ρ‰Π΅ ΠΎΠ΄Π½Π° Π²Π΅Ρ‰ΡŒ, ΠΊΠΎΡ‚ΠΎΡ€ΡƒΡŽ Π²Ρ‹ ΠΌΠΎΠΆΠ΅Ρ‚Π΅ ΡƒΠ²ΠΈΠ΄Π΅Ρ‚ΡŒ ΠΈΠ· Π΅Π΄ΠΈΠ½ΠΈΡ‡Π½ΠΎΠ³ΠΎ ΠΊΡ€ΡƒΠ³Π°, — это Ρ‚ΠΎ, Ρ‡Ρ‚ΠΎ значСния синуса ΠΈ косинуса Π½ΠΈΠΊΠΎΠ³Π΄Π° Π½Π΅ Π±ΡƒΠ΄ΡƒΡ‚ большС 1 ΠΈΠ»ΠΈ мСньшС -1, ΠΏΠΎΡΠΊΠΎΠ»ΡŒΠΊΡƒ x ΠΈ y Π½ΠΈΠΊΠΎΠ³Π΄Π° Π½Π΅ ΠΏΡ€ΠΈΠ½ΠΈΠΌΠ°ΡŽΡ‚ Π·Π½Π°Ρ‡Π΅Π½ΠΈΠΉ Π·Π° ΠΏΡ€Π΅Π΄Π΅Π»Π°ΠΌΠΈ этого ΠΈΠ½Ρ‚Π΅Ρ€Π²Π°Π»Π°.ΠšΡ€ΠΎΠΌΠ΅ Ρ‚ΠΎΠ³ΠΎ, ΠΏΠΎΡΠΊΠΎΠ»ΡŒΠΊΡƒ ΠΊΠ°ΡΠ°Ρ‚Π΅Π»ΡŒΠ½Π°Ρ Π²ΠΊΠ»ΡŽΡ‡Π°Π΅Ρ‚ Π΄Π΅Π»Π΅Π½ΠΈΠ΅ Π½Π° x , ΠΈ ΠΏΠΎΡΠΊΠΎΠ»ΡŒΠΊΡƒ x = 0, ΠΊΠΎΠ³Π΄Π° Π²Ρ‹ Π½Π° Ρ‡Π΅Ρ‚Π²Π΅Ρ€Ρ‚ΡŒ ΠΈ Ρ‚Ρ€ΠΈ Ρ‡Π΅Ρ‚Π²Π΅Ρ€Ρ‚ΠΈ ΠΏΡƒΡ‚ΠΈ ΠΏΠΎ окруТности (Ρ‚ΠΎ Π΅ΡΡ‚ΡŒ, ΠΊΠΎΠ³Π΄Π° Π²Ρ‹ Π½Π°Ρ…ΠΎΠ΄ΠΈΡ‚Π΅ΡΡŒ ΠΏΠΎΠ΄ ΡƒΠ³Π»ΠΎΠΌ 90 Β° ΠΈ 270 Β° ), ΠΊΠ°ΡΠ°Ρ‚Π΅Π»ΡŒΠ½Π°Ρ Π½Π΅ Π±ΡƒΠ΄Π΅Ρ‚ ΠΎΠΏΡ€Π΅Π΄Π΅Π»ΡΡ‚ΡŒΡΡ для этих ΡƒΠ³Π»ΠΎΠ²Ρ‹Ρ… ΠΌΠ΅Ρ€.

НСкоторыС ΡƒΠ³Π»Ρ‹ ΠΈΠΌΠ΅ΡŽΡ‚ «Ρ…ΠΎΡ€ΠΎΡˆΠΈΠ΅» Ρ‚Ρ€ΠΈΠ³Π³Π΅Ρ€Π½Ρ‹Π΅ значСния. Π­Ρ‚ΠΈ ΡƒΠ³Π»Ρ‹ Π² ΠΏΠ΅Ρ€Π²ΠΎΠΌ ΠΊΠ²Π°Π΄Ρ€Π°Π½Ρ‚Π΅ (ΡΠ²Π»ΡΡŽΡ‰ΠΈΠ΅ΡΡ Β«ΠΎΠΏΠΎΡ€Π½Ρ‹ΠΌΠΈΒ» ΡƒΠ³Π»Π°ΠΌΠΈ) ΡΠΎΡΡ‚Π°Π²Π»ΡΡŽΡ‚ 0 Β°, 30 Β°, 45 Β°, 60 Β° ΠΈ 90 Β°. (Π‘Ρ‚Ρ€ΠΎΠ³ΠΎ говоря, 0deg ΠΈ 90deg Π½Π΅ находятся «Π²» ΠΊΠ°ΠΊΠΎΠΌ-Π»ΠΈΠ±ΠΎ ΠΊΠ²Π°Π΄Ρ€Π°Π½Ρ‚Π΅, Π½ΠΎ ΠΌΡ‹ Π±ΡƒΠ΄Π΅ΠΌ Ρ€Π°Π±ΠΎΡ‚Π°Ρ‚ΡŒ с Π½ΠΈΠΌΠΈ, ΠΊΠ°ΠΊ Ссли Π±Ρ‹ ΠΎΠ½ΠΈ Π±Ρ‹Π»ΠΈ Π² ΠΏΠ΅Ρ€Π²ΠΎΠΌ ΠΊΠ²Π°Π΄Ρ€Π°Π½Ρ‚Π΅.Π’Π°ΠΊ ΠΏΡ€ΠΎΡ‰Π΅.) Π’Π°ΠΊ Ρ‡Ρ‚ΠΎ Π²Ρ‹, вСроятно, Π΄ΠΎΠ»ΠΆΠ½Ρ‹ Π±ΡƒΠ΄Π΅Ρ‚Π΅ Π·Π°ΠΏΠΎΠΌΠ½ΠΈΡ‚ΡŒ значСния Ρ‚Ρ€ΠΈΠ³Π³Π΅Ρ€Π½Ρ‹Ρ… Ρ„ΡƒΠ½ΠΊΡ†ΠΈΠΉ для этих ΡƒΠ³Π»ΠΎΠ². ВСроятно, Ρ‚Π΅ΠΏΠ΅Ρ€ΡŒ Π²Π°ΠΌ Ρ‚Π°ΠΊΠΆΠ΅ Π±ΡƒΠ΄Π΅Ρ‚ прСдоставлСн ΠΊΡ€ΡƒΠ³ с ΠΎΡ‚ΠΌΠ΅Ρ‡Π΅Π½Π½Ρ‹ΠΌΠΈ ΡƒΠ³Π»Π°ΠΌΠΈ. Π’ ΠΏΠ΅Ρ€Π²ΠΎΠΌ ΠΊΠ²Π°Π΄Ρ€Π°Π½Ρ‚Π΅ ΠΈΠΌΠ΅Π΅ΠΌ:

(Π’Ρ‹, Π²ΠΎΠ·ΠΌΠΎΠΆΠ½ΠΎ, Π·Π°ΠΌΠ΅Ρ‚ΠΈΠ»ΠΈ Ρ€Π°Π΄ΠΈΠΊΠ°Π»Ρ‹ Ρƒ 1 Π² ΠΏΡ€ΠΈΠ²Π΅Π΄Π΅Π½Π½ΠΎΠΌ Π²Ρ‹ΡˆΠ΅ ΠΏΡ€ΠΈΠΌΠ΅Ρ€Π΅. Π”Π°, ΠΎΠ½ΠΈ ΡƒΠΏΡ€ΠΎΡ‰Π°ΡŽΡ‚ΡΡ Π΄ΠΎ 1, Ρ‚Π°ΠΊ Ρ‡Ρ‚ΠΎ Π²Ρ‹ Ρ‚ΠΎΠΆΠ΅ ΠΌΠΎΠΆΠ΅Ρ‚Π΅ ΠΏΠΈΡΠ°Ρ‚ΡŒ Ρ‚Π°ΠΊ, ΠΈ Π²Π°ΠΌ, бСзусловно, слСдуСт ΡΠ΄Π΅Π»Π°Ρ‚ΡŒ ΡƒΠΏΡ€ΠΎΡ‰Π΅Π½ΠΈΠ΅ Π² своСм ΠΎΠΊΠΎΠ½Ρ‡Π°Ρ‚Π΅Π»ΡŒΠ½ΠΎΠΌ ΠΎΡ‚Π²Π΅Ρ‚Π΅.Но ΠΎΠ±Ρ€Π°Ρ‚ΠΈΡ‚Π΅ Π²Π½ΠΈΠΌΠ°Π½ΠΈΠ΅, Ρ‡Ρ‚ΠΎ всС Π·Π½Π°ΠΌΠ΅Π½Π°Ρ‚Π΅Π»ΠΈ Ρ€Π°Π²Π½Ρ‹ Π΄Π²ΠΎΠΉΠΊΠ°ΠΌ, Π° числитСли ΠΈΠ΄ΡƒΡ‚ Π²Π²Π΅Ρ€Ρ… ΠΈΠ»ΠΈ Π²Π½ΠΈΠ·: 1, 2, 3. Π­Ρ‚ΠΎ ΠΌΠΎΠΆΠ΅Ρ‚ Π±Ρ‹Ρ‚ΡŒ ΠΏΠΎΠ»Π΅Π·Π½ΠΎ для запоминания Π·Π½Π°Ρ‡Π΅Π½ΠΈΠΉ Ρ‚Ρ€ΠΈΠ³Π³Π΅Ρ€ΠΎΠ².)

Π’Π°ΠΌ ΠΌΠΎΠΆΠ΅Ρ‚ Π±Ρ‹Ρ‚ΡŒ Π΄Π°Π½ ΠΏΠΎΠ»Π½Ρ‹ΠΉ Π΅Π΄ΠΈΠ½ΠΈΡ‡Π½Ρ‹ΠΉ ΠΊΡ€ΡƒΠ³ со значСниями ΡƒΠ³Π»ΠΎΠ² Π² Ρ‚Ρ€Π΅Ρ… Π΄Ρ€ΡƒΠ³ΠΈΡ… ΠΊΠ²Π°Π΄Ρ€Π°Π½Ρ‚Π°Ρ…. Но Π²Π°ΠΌ Π½ΡƒΠΆΠ½ΠΎ Ρ‚ΠΎΠ»ΡŒΠΊΠΎ Π·Π½Π°Ρ‚ΡŒ значСния Π² ΠΏΠ΅Ρ€Π²ΠΎΠΌ ΠΊΠ²Π°Π΄Ρ€Π°Π½Ρ‚Π΅. Как Ρ‚ΠΎΠ»ΡŒΠΊΠΎ Π²Ρ‹ ΠΈΡ… ΡƒΠ·Π½Π°Π΅Ρ‚Π΅, ΠΈ ΠΏΠΎΡΠΊΠΎΠ»ΡŒΠΊΡƒ значСния ΠΏΠΎΠ²Ρ‚ΠΎΡ€ΡΡŽΡ‚ΡΡ (ΠΊΡ€ΠΎΠΌΠ΅ Π·Π½Π°ΠΊΠ°) Π² Π΄Ρ€ΡƒΠ³ΠΈΡ… ΠΊΠ²Π°Π΄Ρ€Π°Π½Ρ‚Π°Ρ…, Π²Ρ‹ Π·Π½Π°Π΅Ρ‚Π΅ всС, Ρ‡Ρ‚ΠΎ Π²Π°ΠΌ Π½ΡƒΠΆΠ½ΠΎ Π·Π½Π°Ρ‚ΡŒ ΠΎ Π΅Π΄ΠΈΠ½ΠΈΡ‡Π½ΠΎΠΌ ΠΊΡ€ΡƒΠ³Π΅.


  • ΠŸΠΎΠ΄Ρ‚Π²Π΅Ρ€Π΄ΠΈΡ‚Π΅, Ρ‡Ρ‚ΠΎ Ρ‚ΠΎΡ‡ΠΊΠ° (15/113, –112/113) являСтся Ρ‚ΠΎΡ‡ΠΊΠΎΠΉ Π½Π° Π΅Π΄ΠΈΠ½ΠΈΡ‡Π½ΠΎΠΉ окруТности. НайдитС синус ΠΈ котангСнс ΡƒΠ³Π»Π° A, ΠΈΠΌΠ΅ΡŽΡ‰Π΅Π³ΠΎ эту Ρ‚ΠΎΡ‡ΠΊΡƒ Π½Π° своСй ΠΊΠΎΠ½Π΅Ρ‡Π½ΠΎΠΉ сторонС.

Π›ΡŽΠ±Π°Ρ Ρ‚ΠΎΡ‡ΠΊΠ° Π½Π° Π΅Π΄ΠΈΠ½ΠΈΡ‡Π½ΠΎΠΉ окруТности Π±ΡƒΠ΄Π΅Ρ‚ Π½Π°Ρ…ΠΎΠ΄ΠΈΡ‚ΡŒΡΡ Π½Π° расстоянии ΠΎΠ΄Π½ΠΎΠΉ Π΅Π΄ΠΈΠ½ΠΈΡ†Ρ‹ ΠΎΡ‚ Ρ†Π΅Π½Ρ‚Ρ€Π°; это ΠΎΠΏΡ€Π΅Π΄Π΅Π»Π΅Π½ΠΈΠ΅ Π΅Π΄ΠΈΠ½ΠΈΡ‡Π½ΠΎΠ³ΠΎ ΠΊΡ€ΡƒΠ³Π°. Π§Ρ‚ΠΎΠ±Ρ‹ Β«ΠΏΠΎΠ΄Ρ‚Π²Π΅Ρ€Π΄ΠΈΡ‚ΡŒΒ», Ρ‡Ρ‚ΠΎ Ρ‚ΠΎΡ‡ΠΊΠ°, ΠΊΠΎΡ‚ΠΎΡ€ΡƒΡŽ ΠΎΠ½ΠΈ ΠΌΠ½Π΅ Π΄Π°Π»ΠΈ, являСтся Ρ‚ΠΎΡ‡ΠΊΠΎΠΉ Π½Π° Π΅Π΄ΠΈΠ½ΠΈΡ‡Π½ΠΎΠΉ окруТности, я ΠΌΠΎΠ³Ρƒ ΠΏΡ€ΠΈΠΌΠ΅Π½ΠΈΡ‚ΡŒ Ρ‚Π΅ΠΎΡ€Π΅ΠΌΡƒ ΠŸΠΈΡ„Π°Π³ΠΎΡ€Π°, Ρ‡Ρ‚ΠΎΠ±Ρ‹ Π½Π°ΠΉΡ‚ΠΈ Π΄Π»ΠΈΠ½Ρƒ радиуса ΠΏΡ€ΡΠΌΠΎΡƒΠ³ΠΎΠ»ΡŒΠ½ΠΎΠ³ΠΎ Ρ‚Ρ€Π΅ΡƒΠ³ΠΎΠ»ΡŒΠ½ΠΈΠΊΠ°, ΠΎΠ±Ρ€Π°Π·ΠΎΠ²Π°Π½Π½ΠΎΠ³ΠΎ ΠΏΡƒΡ‚Π΅ΠΌ опускания пСрпСндикуляра с оси x Π²Π½ΠΈΠ·. ΠΊ Ρ‚ΠΎΡ‡ΠΊΠ΅.Мой пСрпСндикуляр — ярко-синяя пунктирная линия:

Если Ρ‚Π΅ΠΎΡ€Π΅ΠΌΠ° ΠŸΠΈΡ„Π°Π³ΠΎΡ€Π° Π΄Π°Π΅Ρ‚ ΠΌΠ½Π΅ Π·Π½Π°Ρ‡Π΅Π½ΠΈΠ΅ радиуса 1, Ρ‚ΠΎ я Β«ΠΏΠΎΠ΄Ρ‚Π²Π΅Ρ€ΠΆΠ΄Π°ΡŽΒ», Ρ‡Ρ‚ΠΎ Ρ‚ΠΎΡ‡ΠΊΠ° находится Π½Π° Π΅Π΄ΠΈΠ½ΠΈΡ‡Π½ΠΎΠΉ окруТности.

Π’ΠΎΠ³Π΄Π° Π΄Π»ΠΈΠ½Π° Ρ‚Ρ€Π΅Ρ‚ΡŒΠ΅ΠΉ стороны ΠΏΡ€ΡΠΌΠΎΡƒΠ³ΠΎΠ»ΡŒΠ½ΠΎΠ³ΠΎ Ρ‚Ρ€Π΅ΡƒΠ³ΠΎΠ»ΡŒΠ½ΠΈΠΊΠ°, которая Ρ‚Π°ΠΊΠΆΠ΅ являСтся Π΄Π»ΠΈΠ½ΠΎΠΉ радиуса ΠΊΡ€ΡƒΠ³Π°, Ρ€Π°Π²Π½Π° 1. Π’Π°ΠΊΠΈΠΌ ΠΎΠ±Ρ€Π°Π·ΠΎΠΌ, эта Ρ‚ΠΎΡ‡ΠΊΠ° Π΄Π΅ΠΉΡΡ‚Π²ΠΈΡ‚Π΅Π»ΡŒΠ½ΠΎ находится Π½Π° Π΅Π΄ΠΈΠ½ΠΈΡ‡Π½ΠΎΠΉ окруТности.

Π’Π΅ΠΏΠ΅Ρ€ΡŒ ΠΎΠ½ΠΈ хотят, Ρ‡Ρ‚ΠΎΠ±Ρ‹ я нашСл синус ΠΈ котангСнс основного ΡƒΠ³Π»Π°.Бинус — это Π·Π½Π°Ρ‡Π΅Π½ΠΈΠ΅ ΠΈ . (МнС Π½Π΅ Π½ΡƒΠΆΠ½ΠΎ Π±Π΅ΡΠΏΠΎΠΊΠΎΠΈΡ‚ΡŒΡΡ ΠΎ Π³ΠΈΠΏΠΎΡ‚Π΅Π½ΡƒΠ·Π΅, ΠΏΠΎΡ‚ΠΎΠΌΡƒ Ρ‡Ρ‚ΠΎ ΠΎΠ½Π° всСгда Ρ€Π°Π²Π½Π° 1 Π² Π΅Π΄ΠΈΠ½ΠΈΡ‡Π½ΠΎΠΉ окруТности.) Π˜Ρ‚Π°ΠΊ, синус Π½ΠΈΠΆΠ΅Π»Π΅ΠΆΠ°Ρ‰Π΅Π³ΠΎ ΡƒΠ³Π»Π°:

ΠšΠΎΡ‚Π°Π½Π³Π΅Π½Ρ — это Π²Π΅Π»ΠΈΡ‡ΠΈΠ½Π°, обратная ΠΊΠ°ΡΠ°Ρ‚Π΅Π»ΡŒΠ½ΠΎΠΉ. ΠšΠ°ΡΠ°Ρ‚Π΅Π»ΡŒΠ½Π°Ρ являСтся Β«ΠΏΡ€ΠΎΡ‚ΠΈΠ²ΠΎΠΏΠΎΠ»ΠΎΠΆΠ½ΠΎΠΉ ΠΏΠΎ сосСдству» ΠΈΠ»ΠΈ, Π² Π΄Π°Π½Π½ΠΎΠΌ контСкстС, Β« y Π½Π°Π΄ x Β». Π’ΠΎΠ³Π΄Π° котангСнс ΠΎΠ±Ρ€Π°Ρ‚Π΅Π½ этому:

дСтская ΠΊΡ€ΠΎΠ²Π°Ρ‚ΠΊΠ° (A) = 15 / (- 112) = –15/112

Π’ ΠΏΠ΅Ρ€Π²ΠΎΠΉ части этого упраТнСния я ΠΏΠΎΠΊΠ°Π·Π°Π», Ρ‡Ρ‚ΠΎ радиус Ρ€Π°Π²Π΅Π½ 1.ΠžΡΡ‚Π°Π»ΡŒΠ½Π°Ρ Ρ‡Π°ΡΡ‚ΡŒ ΠΌΠΎΠ΅Π³ΠΎ Ρ€ΡƒΡ‡Π½ΠΎΠ³ΠΎ ΠΎΡ‚Π²Π΅Ρ‚Π°:

sin (A) = –112

дСтская ΠΊΡ€ΠΎΠ²Π°Ρ‚ΠΊΠ° (A) = –15/112


URL: https://www.purplemath.com/modules/unitcirc.htm

Π•Π΄ΠΈΠ½ΠΈΡ‡Π½Ρ‹ΠΉ ΠΊΡ€ΡƒΠ³ — Π£Ρ€Π°Π²Π½Π΅Π½ΠΈΠ΅ Π΅Π΄ΠΈΠ½ΠΈΡ‡Π½ΠΎΠΉ окруТности

Π•Π΄ΠΈΠ½ΠΈΡ‡Π½Ρ‹ΠΉ ΠΊΡ€ΡƒΠ³ ΠΈΠ· самого названия опрСдСляСт ΠΊΡ€ΡƒΠ³ с Π΅Π΄ΠΈΠ½ΠΈΡ‡Π½Ρ‹ΠΌ радиусом.ΠšΡ€ΡƒΠ³ — это замкнутая гСомСтричСская Ρ„ΠΈΠ³ΡƒΡ€Π° Π±Π΅Π· сторон ΠΈ ΡƒΠ³Π»ΠΎΠ². Π•Π΄ΠΈΠ½ΠΈΡ‡Π½Ρ‹ΠΉ ΠΊΡ€ΡƒΠ³ ΠΎΠ±Π»Π°Π΄Π°Π΅Ρ‚ всСми свойствами ΠΊΡ€ΡƒΠ³Π°, ΠΈ Π΅Π³ΠΎ ΡƒΡ€Π°Π²Π½Π΅Π½ΠΈΠ΅ Ρ‚Π°ΠΊΠΆΠ΅ выводится ΠΈΠ· уравнСния ΠΊΡ€ΡƒΠ³Π°. ΠšΡ€ΠΎΠΌΠ΅ Ρ‚ΠΎΠ³ΠΎ, Π΅Π΄ΠΈΠ½ΠΈΡ‡Π½Ρ‹ΠΉ ΠΊΡ€ΡƒΠ³ ΠΏΠΎΠ»Π΅Π·Π΅Π½ для получСния стандартных ΡƒΠ³Π»ΠΎΠ²Ρ‹Ρ… Π·Π½Π°Ρ‡Π΅Π½ΠΈΠΉ всСх тригономСтричСских ΡΠΎΠΎΡ‚Π½ΠΎΡˆΠ΅Π½ΠΈΠΉ.

Π—Π΄Π΅ΡΡŒ ΠΌΡ‹ ΠΈΠ·ΡƒΡ‡ΠΈΠΌ ΡƒΡ€Π°Π²Π½Π΅Π½ΠΈΠ΅ Π΅Π΄ΠΈΠ½ΠΈΡ‡Π½ΠΎΠΉ окруТности ΠΈ ΠΏΠΎΠΉΠΌΠ΅ΠΌ, ΠΊΠ°ΠΊ ΠΏΡ€Π΅Π΄ΡΡ‚Π°Π²ΠΈΡ‚ΡŒ ΠΊΠ°ΠΆΠ΄ΡƒΡŽ ΠΈΠ· Ρ‚ΠΎΡ‡Π΅ΠΊ Π½Π° окруТности Π΅Π΄ΠΈΠ½ΠΈΡ‡Π½ΠΎΠΉ окруТности с ΠΏΠΎΠΌΠΎΡ‰ΡŒΡŽ тригономСтричСских ΡΠΎΠΎΡ‚Π½ΠΎΡˆΠ΅Π½ΠΈΠΉ cosΞΈ ΠΈ sinΞΈ.

Π§Ρ‚ΠΎ Ρ‚Π°ΠΊΠΎΠ΅ Π΅Π΄ΠΈΠ½ΠΈΡ‡Π½Ρ‹ΠΉ ΠΊΡ€ΡƒΠ³?

Единичная ΠΎΠΊΡ€ΡƒΠΆΠ½ΠΎΡΡ‚ΡŒ — это ΠΎΠΊΡ€ΡƒΠΆΠ½ΠΎΡΡ‚ΡŒ с радиусом Π² 1 Π΅Π΄ΠΈΠ½ΠΈΡ†Ρƒ. Π•Π΄ΠΈΠ½ΠΈΡ‡Π½Ρ‹ΠΉ ΠΊΡ€ΡƒΠ³ ΠΎΠ±Ρ‹Ρ‡Π½ΠΎ прСдставлСн Π² Π΄Π΅ΠΊΠ°Ρ€Ρ‚ΠΎΠ²ΠΎΠΉ ΠΊΠΎΠΎΡ€Π΄ΠΈΠ½Π°Ρ‚Π½ΠΎΠΉ плоскости. Π•Π΄ΠΈΠ½ΠΈΡ‡Π½Ρ‹ΠΉ ΠΊΡ€ΡƒΠ³ алгСбраичСски прСдставлСн с ΠΏΠΎΠΌΠΎΡ‰ΡŒΡŽ уравнСния Π²Ρ‚ΠΎΡ€ΠΎΠΉ стСпСни с двумя ΠΏΠ΅Ρ€Π΅ΠΌΠ΅Π½Π½Ρ‹ΠΌΠΈ x ΠΈ y. Единичная ΠΎΠΊΡ€ΡƒΠΆΠ½ΠΎΡΡ‚ΡŒ примСняСтся Π² Ρ‚Ρ€ΠΈΠ³ΠΎΠ½ΠΎΠΌΠ΅Ρ‚Ρ€ΠΈΠΈ ΠΈ ΠΏΠΎΠΌΠΎΠ³Π°Π΅Ρ‚ Π½Π°Ρ…ΠΎΠ΄ΠΈΡ‚ΡŒ значСния тригономСтричСских ΡΠΎΠΎΡ‚Π½ΠΎΡˆΠ΅Π½ΠΈΠΉ синуса, косинуса ΠΈ тангСнса.

ΠžΠΏΡ€Π΅Π΄Π΅Π»Π΅Π½ΠΈΠ΅ Π΅Π΄ΠΈΠ½ΠΈΡ‡Π½ΠΎΠ³ΠΎ ΠΊΡ€ΡƒΠ³Π°

ГСомСтричСскоС мСсто Ρ‚ΠΎΡ‡ΠΊΠΈ, находящСйся Π½Π° расстоянии ΠΎΠ΄Π½ΠΎΠΉ Π΅Π΄ΠΈΠ½ΠΈΡ†Ρ‹ ΠΎΡ‚ фиксированной Ρ‚ΠΎΡ‡ΠΊΠΈ, называСтся Π΅Π΄ΠΈΠ½ΠΈΡ‡Π½ΠΎΠΉ ΠΎΠΊΡ€ΡƒΠΆΠ½ΠΎΡΡ‚ΡŒΡŽ.

Π£Ρ€Π°Π²Π½Π΅Π½ΠΈΠ΅ Π΅Π΄ΠΈΠ½ΠΈΡ‡Π½ΠΎΠΉ окруТности

ΠžΠ±Ρ‰Π΅Π΅ ΡƒΡ€Π°Π²Π½Π΅Π½ΠΈΠ΅ ΠΊΡ€ΡƒΠ³Π°: (x — a) 2 + (y — b) 2 = r 2 , Ρ‡Ρ‚ΠΎ прСдставляСт собой ΠΊΡ€ΡƒΠ³ с Ρ†Π΅Π½Ρ‚Ρ€ΠΎΠΌ (a, b) ΠΈ радиусом r. Π­Ρ‚ΠΎ ΡƒΡ€Π°Π²Π½Π΅Π½ΠΈΠ΅ ΠΊΡ€ΡƒΠ³Π° ΡƒΠΏΡ€ΠΎΡ‰Π΅Π½ΠΎ ΠΈ прСдставляСт собой ΡƒΡ€Π°Π²Π½Π΅Π½ΠΈΠ΅ Π΅Π΄ΠΈΠ½ΠΈΡ‡Π½ΠΎΠ³ΠΎ ΠΊΡ€ΡƒΠ³Π°. Единичная ΠΎΠΊΡ€ΡƒΠΆΠ½ΠΎΡΡ‚ΡŒ образуСтся с Ρ†Π΅Π½Ρ‚Ρ€ΠΎΠΌ Π² Ρ‚ΠΎΡ‡ΠΊΠ΅ (0, 0), которая являСтся Π½Π°Ρ‡Π°Π»ΠΎΠΌ ΠΊΠΎΠΎΡ€Π΄ΠΈΠ½Π°Ρ‚Π½Ρ‹Ρ… осСй. ΠΈ радиусом 1 Π΅Π΄. Π‘Π»Π΅Π΄ΠΎΠ²Π°Ρ‚Π΅Π»ΡŒΠ½ΠΎ, ΡƒΡ€Π°Π²Π½Π΅Π½ΠΈΠ΅ Π΅Π΄ΠΈΠ½ΠΈΡ‡Π½ΠΎΠΉ окруТности: (x — 0) 2 + (y — 0) 2 = 1 2 .Π­Ρ‚ΠΎ упрощаСтся, Ρ‡Ρ‚ΠΎΠ±Ρ‹ ΠΏΠΎΠ»ΡƒΡ‡ΠΈΡ‚ΡŒ ΡƒΡ€Π°Π²Π½Π΅Π½ΠΈΠ΅ Π΅Π΄ΠΈΠ½ΠΈΡ‡Π½ΠΎΠΉ окруТности.

Π£Ρ€Π°Π²Π½Π΅Π½ΠΈΠ΅ Π΅Π΄ΠΈΠ½ΠΈΡ‡Π½ΠΎΠΉ окруТности: x 2 + y 2 = 1

Π—Π΄Π΅ΡΡŒ для Π΅Π΄ΠΈΠ½ΠΈΡ‡Π½ΠΎΠΉ окруТности Ρ†Π΅Π½Ρ‚Ρ€ Π»Π΅ΠΆΠΈΡ‚ Π² Ρ‚ΠΎΡ‡ΠΊΠ΅ (0,0), Π° радиус Ρ€Π°Π²Π΅Π½ 1 Π΅Π΄ΠΈΠ½ΠΈΡ†Π΅. Π’Ρ‹ΡˆΠ΅ΡƒΠΏΠΎΠΌΡΠ½ΡƒΡ‚ΠΎΠ΅ ΡƒΡ€Π°Π²Π½Π΅Π½ΠΈΠ΅ удовлСтворяСт всСм Ρ‚ΠΎΡ‡ΠΊΠ°ΠΌ, Π»Π΅ΠΆΠ°Ρ‰ΠΈΠΌ Π½Π° окруТности Π² Ρ‡Π΅Ρ‚Ρ‹Ρ€Π΅Ρ… ΠΊΠ²Π°Π΄Ρ€Π°Π½Ρ‚Π°Ρ….

НахоТдСниС тригономСтричСских Ρ„ΡƒΠ½ΠΊΡ†ΠΈΠΉ ΠΏΠΎ Π΅Π΄ΠΈΠ½ΠΈΡ‡Π½ΠΎΠΉ окруТности

ΠœΡ‹ ΠΌΠΎΠΆΠ΅ΠΌ Π²Ρ‹Ρ‡ΠΈΡΠ»ΠΈΡ‚ΡŒ тригономСтричСскиС Ρ„ΡƒΠ½ΠΊΡ†ΠΈΠΈ синуса, косинуса ΠΈ тангСнса, ΠΈΡΠΏΠΎΠ»ΡŒΠ·ΡƒΡ Π΅Π΄ΠΈΠ½ΠΈΡ‡Π½ΡƒΡŽ ΠΎΠΊΡ€ΡƒΠΆΠ½ΠΎΡΡ‚ΡŒ.Π”Π°Π²Π°ΠΉΡ‚Π΅ ΠΏΡ€ΠΈΠΌΠ΅Π½ΠΈΠΌ Ρ‚Π΅ΠΎΡ€Π΅ΠΌΡƒ ΠŸΠΈΡ„Π°Π³ΠΎΡ€Π° Π² Π΅Π΄ΠΈΠ½ΠΈΡ‡Π½ΠΎΠΌ ΠΊΡ€ΡƒΠ³Π΅, Ρ‡Ρ‚ΠΎΠ±Ρ‹ ΠΏΠΎΠ½ΡΡ‚ΡŒ тригономСтричСскиС Ρ„ΡƒΠ½ΠΊΡ†ΠΈΠΈ. Рассмотрим ΠΏΡ€ΡΠΌΠΎΡƒΠ³ΠΎΠ»ΡŒΠ½Ρ‹ΠΉ Ρ‚Ρ€Π΅ΡƒΠ³ΠΎΠ»ΡŒΠ½ΠΈΠΊ, ΠΏΠΎΠΌΠ΅Ρ‰Π΅Π½Π½Ρ‹ΠΉ Π² Π΅Π΄ΠΈΠ½ΠΈΡ‡Π½Ρ‹ΠΉ ΠΊΡ€ΡƒΠ³ Π² Π΄Π΅ΠΊΠ°Ρ€Ρ‚ΠΎΠ²ΠΎΠΉ ΠΊΠΎΠΎΡ€Π΄ΠΈΠ½Π°Ρ‚Π½ΠΎΠΉ плоскости. Радиус ΠΊΡ€ΡƒΠ³Π° прСдставляСт собой Π³ΠΈΠΏΠΎΡ‚Π΅Π½ΡƒΠ·Ρƒ ΠΏΡ€ΡΠΌΠΎΡƒΠ³ΠΎΠ»ΡŒΠ½ΠΎΠ³ΠΎ Ρ‚Ρ€Π΅ΡƒΠ³ΠΎΠ»ΡŒΠ½ΠΈΠΊΠ°. Радиус-Π²Π΅ΠΊΡ‚ΠΎΡ€ составляСт ΡƒΠ³ΠΎΠ» ΞΈ с ΠΏΠΎΠ»ΠΎΠΆΠΈΡ‚Π΅Π»ΡŒΠ½ΠΎΠΉ осью x, Π° ΠΊΠΎΠΎΡ€Π΄ΠΈΠ½Π°Ρ‚Ρ‹ ΠΊΠΎΠ½Π΅Ρ‡Π½ΠΎΠΉ Ρ‚ΠΎΡ‡ΠΊΠΈ радиус-Π²Π΅ΠΊΡ‚ΠΎΡ€Π° Ρ€Π°Π²Π½Ρ‹ (x, y). Π—Π΄Π΅ΡΡŒ значСния x ΠΈ y — Π΄Π»ΠΈΠ½Π° основания ΠΈ высота ΠΏΡ€ΡΠΌΠΎΡƒΠ³ΠΎΠ»ΡŒΠ½ΠΎΠ³ΠΎ Ρ‚Ρ€Π΅ΡƒΠ³ΠΎΠ»ΡŒΠ½ΠΈΠΊΠ°. Π’Π΅ΠΏΠ΅Ρ€ΡŒ Ρƒ нас Π΅ΡΡ‚ΡŒ ΠΏΡ€ΡΠΌΠΎΡƒΠ³ΠΎΠ»ΡŒΠ½Ρ‹ΠΉ Ρ‚Ρ€Π΅ΡƒΠ³ΠΎΠ»ΡŒΠ½ΠΈΠΊ со сторонами 1, x, y.ΠŸΡ€ΠΈΠΌΠ΅Π½ΡΡ это Π² Ρ‚Ρ€ΠΈΠ³ΠΎΠ½ΠΎΠΌΠ΅Ρ‚Ρ€ΠΈΠΈ, ΠΌΡ‹ ΠΌΠΎΠΆΠ΅ΠΌ Π½Π°ΠΉΡ‚ΠΈ значСния тригономСтричСского ΠΎΡ‚Π½ΠΎΡˆΠ΅Π½ΠΈΡ ΡΠ»Π΅Π΄ΡƒΡŽΡ‰ΠΈΠΌ ΠΎΠ±Ρ€Π°Π·ΠΎΠΌ:

  • sinΞΈ = Высота / Π“ΠΈΠΏΠΎΡ‚Π΅Π·Π° = y / 1
  • cosΞΈ = Π‘Π°Π·Π° / Π“ΠΈΠΏΠΎΡ‚Π΅Π½ΡƒΠ·Π° = x / 1

Π’Π΅ΠΏΠ΅Ρ€ΡŒ Ρƒ нас Π΅ΡΡ‚ΡŒ sinΞΈ = y, cosΞΈ = x, ΠΈ, ΠΈΡΠΏΠΎΠ»ΡŒΠ·ΡƒΡ это, ΠΌΡ‹ Ρ‚Π΅ΠΏΠ΅Ρ€ΡŒ ΠΈΠΌΠ΅Π΅ΠΌ tanΞΈ = y / x. Π’ΠΎΡ‡Π½ΠΎ Ρ‚Π°ΠΊ ΠΆΠ΅ ΠΌΡ‹ ΠΌΠΎΠΆΠ΅ΠΌ ΠΏΠΎΠ»ΡƒΡ‡ΠΈΡ‚ΡŒ значСния Π΄Ρ€ΡƒΠ³ΠΈΡ… тригономСтричСских ΡΠΎΠΎΡ‚Π½ΠΎΡˆΠ΅Π½ΠΈΠΉ, ΠΈΡΠΏΠΎΠ»ΡŒΠ·ΡƒΡ ΠΏΡ€ΡΠΌΠΎΡƒΠ³ΠΎΠ»ΡŒΠ½Ρ‹ΠΉ Ρ‚Ρ€Π΅ΡƒΠ³ΠΎΠ»ΡŒΠ½ΠΈΠΊ Π²Π½ΡƒΡ‚Ρ€ΠΈ Π΅Π΄ΠΈΠ½ΠΈΡ‡Π½ΠΎΠΉ окруТности. Π’Π°ΠΊΠΆΠ΅, измСняя значСния ΞΈ, ΠΌΡ‹ ΠΌΠΎΠΆΠ΅ΠΌ ΠΏΠΎΠ»ΡƒΡ‡ΠΈΡ‚ΡŒ Π³Π»Π°Π²Π½Ρ‹Π΅ значСния этих тригономСтричСских ΡΠΎΠΎΡ‚Π½ΠΎΡˆΠ΅Π½ΠΈΠΉ.

Π•Π΄ΠΈΠ½ΠΈΡ‡Π½Ρ‹ΠΉ ΠΊΡ€ΡƒΠ³ с Sin Cos ΠΈ Tan

Π›ΡŽΠ±Π°Ρ Ρ‚ΠΎΡ‡ΠΊΠ° Π½Π° Π΅Π΄ΠΈΠ½ΠΈΡ‡Π½ΠΎΠΉ окруТности ΠΈΠΌΠ΅Π΅Ρ‚ ΠΊΠΎΠΎΡ€Π΄ΠΈΠ½Π°Ρ‚Ρ‹ (x, y), ΠΊΠΎΡ‚ΠΎΡ€Ρ‹Π΅ Ρ€Π°Π²Π½Ρ‹ тригономСтричСским тоТдСствам (cosΞΈ, sinΞΈ). Для Π»ΡŽΠ±Ρ‹Ρ… Π·Π½Π°Ρ‡Π΅Π½ΠΈΠΉ ΞΈ, ΠΎΠ±Ρ€Π°Π·ΠΎΠ²Π°Π½Π½Ρ‹Ρ… Π»ΠΈΠ½ΠΈΠ΅ΠΉ радиуса с ΠΏΠΎΠ»ΠΎΠΆΠΈΡ‚Π΅Π»ΡŒΠ½ΠΎΠΉ осью x, ΠΊΠΎΠΎΡ€Π΄ΠΈΠ½Π°Ρ‚Ρ‹ ΠΊΠΎΠ½Π΅Ρ‡Π½ΠΎΠΉ Ρ‚ΠΎΡ‡ΠΊΠΈ радиуса ΠΏΡ€Π΅Π΄ΡΡ‚Π°Π²Π»ΡΡŽΡ‚ собой косинус ΠΈ синус Π·Π½Π°Ρ‡Π΅Π½ΠΈΠΉ ΞΈ. Π—Π΄Π΅ΡΡŒ ΠΌΡ‹ ΠΈΠΌΠ΅Π΅ΠΌ cosΞΈ = x ΠΈ sinΞΈ = y, ΠΈ эти значСния ΠΏΠΎΠΌΠΎΠ³Π°ΡŽΡ‚ Π²Ρ‹Ρ‡ΠΈΡΠ»ΠΈΡ‚ΡŒ Π΄Ρ€ΡƒΠ³ΠΈΠ΅ значСния тригономСтричСского ΠΎΡ‚Π½ΠΎΡˆΠ΅Π½ΠΈΡ. ΠŸΡ€ΠΈΠΌΠ΅Π½ΡΡ это Π΄Π°Π»Π΅Π΅, ΠΌΡ‹ ΠΈΠΌΠ΅Π΅ΠΌ tanΞΈ = sinΞΈ / cosΞΈ ΠΈΠ»ΠΈ tanΞΈ = y / x.

Π•Ρ‰Π΅ ΠΎΠ΄ΠΈΠ½ Π²Π°ΠΆΠ½Ρ‹ΠΉ ΠΌΠΎΠΌΠ΅Π½Ρ‚, ΠΊΠΎΡ‚ΠΎΡ€Ρ‹ΠΉ слСдуСт ΠΏΠΎΠ½ΡΡ‚ΡŒ, это Ρ‚ΠΎ, Ρ‡Ρ‚ΠΎ значСния sinΞΈ ΠΈ cosΞΈ всСгда Π»Π΅ΠΆΠ°Ρ‚ ΠΌΠ΅ΠΆΠ΄Ρƒ 1 ΠΈ -1, Π° Π·Π½Π°Ρ‡Π΅Π½ΠΈΠ΅ радиуса Ρ€Π°Π²Π½ΠΎ 1, ΠΈ ΠΎΠ½ΠΎ ΠΈΠΌΠ΅Π΅Ρ‚ Π·Π½Π°Ρ‡Π΅Π½ΠΈΠ΅ -1 Π½Π° ΠΎΡ‚Ρ€ΠΈΡ†Π°Ρ‚Π΅Π»ΡŒΠ½ΠΎΠΉ оси x. Π’Π΅ΡΡŒ ΠΊΡ€ΡƒΠ³ прСдставляСт собой ΠΏΠΎΠ»Π½Ρ‹ΠΉ ΡƒΠ³ΠΎΠ» Π² 360ΒΊ, Π° Ρ‡Π΅Ρ‚Ρ‹Ρ€Π΅ ΠΊΠ²Π°Π΄Ρ€Π°Π½Ρ‚Π° ΠΊΡ€ΡƒΠ³Π° ΠΎΠ±Ρ€Π°Π·ΡƒΡŽΡ‚ ΡƒΠ³Π»Ρ‹ 90ΒΊ, 180ΒΊ, 270ΒΊ, 360ΒΊ (0ΒΊ). ΠŸΡ€ΠΈ 90ΒΊ ΠΈ 270ΒΊ Π·Π½Π°Ρ‡Π΅Π½ΠΈΠ΅ cosΞΈ Ρ€Π°Π²Π½ΠΎ 0 ΠΈ, ΡΠ»Π΅Π΄ΠΎΠ²Π°Ρ‚Π΅Π»ΡŒΠ½ΠΎ, значСния тангСнса ΡƒΠ³Π»Π° Π½Π°ΠΊΠ»ΠΎΠ½Π° ΠΏΡ€ΠΈ этих ΡƒΠ³Π»Π°Ρ… Π½Π΅ ΠΎΠΏΡ€Π΅Π΄Π΅Π»Π΅Π½Ρ‹.

ΠŸΡ€ΠΈΠΌΠ΅Ρ€: НайдитС Π·Π½Π°Ρ‡Π΅Π½ΠΈΠ΅ тангСнса ΡƒΠ³Π»Π° 45ΒΊ, ΠΈΡΠΏΠΎΠ»ΡŒΠ·ΡƒΡ значСния sin ΠΈ cos ΠΈΠ· Π΅Π΄ΠΈΠ½ΠΈΡ‡Π½ΠΎΠΉ окруТности.

РСшСниС:

ΠœΡ‹ Π·Π½Π°Π΅ΠΌ, Ρ‡Ρ‚ΠΎ tan 45 Β° = sin 45 Β° / cos 45 Β°

Π˜ΡΠΏΠΎΠ»ΡŒΠ·ΡƒΡ ΠΊΡ€ΡƒΠ³ΠΎΠ²ΡƒΡŽ Π΄ΠΈΠ°Π³Ρ€Π°ΠΌΠΌΡƒ Π΅Π΄ΠΈΠ½ΠΈΡ†:
Π³Ρ€Π΅Ρ… 45 Β° = 1 / √2
cos 45 ° = 1 / √2

Π‘Π»Π΅Π΄ΠΎΠ²Π°Ρ‚Π΅Π»ΡŒΠ½ΠΎ, tan 45 Β° = sin 45 Β° / cos 45 Β°
= (1 / √2) / (1 / √2)
= 1

ΠžΡ‚Π²Π΅Ρ‚: Π‘Π»Π΅Π΄ΠΎΠ²Π°Ρ‚Π΅Π»ΡŒΠ½ΠΎ, tan 45 Β° = 1

ΠšΡ€ΡƒΠ³ΠΎΠ²Π°Ρ Π΄ΠΈΠ°Π³Ρ€Π°ΠΌΠΌΠ° Π΅Π΄ΠΈΠ½ΠΈΡ† Π² Ρ€Π°Π΄ΠΈΠ°Π½Π°Ρ…

Π•Π΄ΠΈΠ½ΠΈΡ‡Π½Ρ‹ΠΉ ΠΊΡ€ΡƒΠ³ прСдставляСт собой ΠΏΠΎΠ»Π½Ρ‹ΠΉ ΡƒΠ³ΠΎΠ» Π² 2Ο€ Ρ€Π°Π΄ΠΈΠ°Π½. А Π΅Π΄ΠΈΠ½ΠΈΡ‡Π½Ρ‹ΠΉ ΠΊΡ€ΡƒΠ³ Ρ€Π°Π·Π΄Π΅Π»Π΅Π½ Π½Π° Ρ‡Π΅Ρ‚Ρ‹Ρ€Π΅ ΠΊΠ²Π°Π΄Ρ€Π°Π½Ρ‚Π° ΠΏΠΎΠ΄ ΡƒΠ³Π»Π°ΠΌΠΈ Ο€ / 2, Ο€.3Ο€ / 2 ΠΈ 2Ο€ соотвСтствСнно. Π”Π°Π»Π΅Π΅ Π² ΠΏΡ€Π΅Π΄Π΅Π»Π°Ρ… ΠΏΠ΅Ρ€Π²ΠΎΠ³ΠΎ ΠΊΠ²Π°Π΄Ρ€Π°Π½Ρ‚Π° ΠΏΠΎΠ΄ ΡƒΠ³Π»Π°ΠΌΠΈ 0, Ο€ / 6, Ο€ / 4, Ο€ / 3, Ο€ / 2 — стандартныС значСния, ΠΊΠΎΡ‚ΠΎΡ€Ρ‹Π΅ ΠΏΡ€ΠΈΠΌΠ΅Π½ΠΈΠΌΡ‹ ΠΊ тригономСтричСским ΠΎΡ‚Π½ΠΎΡˆΠ΅Π½ΠΈΡΠΌ. Π’ΠΎΡ‡ΠΊΠΈ Π½Π° Π΅Π΄ΠΈΠ½ΠΈΡ‡Π½ΠΎΠΌ ΠΊΡ€ΡƒΠ³Π΅ для этих ΡƒΠ³Π»ΠΎΠ² ΠΏΡ€Π΅Π΄ΡΡ‚Π°Π²Π»ΡΡŽΡ‚ стандартныС ΡƒΠ³Π»ΠΎΠ²Ρ‹Π΅ значСния ΠΎΡ‚Π½ΠΎΡˆΠ΅Π½ΠΈΠΉ косинуса ΠΈ синуса. ΠŸΡ€ΠΈ Π²Π½ΠΈΠΌΠ°Ρ‚Π΅Π»ΡŒΠ½ΠΎΠΌ рассмотрСнии рисунка Π½ΠΈΠΆΠ΅ значСния ΠΏΠΎΠ²Ρ‚ΠΎΡ€ΡΡŽΡ‚ΡΡ Π² Ρ‡Π΅Ρ‚Ρ‹Ρ€Π΅Ρ… ΠΊΠ²Π°Π΄Ρ€Π°Π½Ρ‚Π°Ρ…, Π½ΠΎ с ΠΈΠ·ΠΌΠ΅Π½Π΅Π½ΠΈΠ΅ΠΌ Π·Π½Π°ΠΊΠ°. Π­Ρ‚ΠΎ ΠΈΠ·ΠΌΠ΅Π½Π΅Π½ΠΈΠ΅ Π·Π½Π°ΠΊΠ° происходит ΠΈΠ·-Π·Π° ΠΎΠΏΠΎΡ€Π½Ρ‹Ρ… осСй x ΠΈ y, ΠΊΠΎΡ‚ΠΎΡ€Ρ‹Π΅ ΠΏΠΎΠ»ΠΎΠΆΠΈΡ‚Π΅Π»ΡŒΠ½Ρ‹ с ΠΎΠ΄Π½ΠΎΠΉ стороны ΠΈ ΠΎΡ‚Ρ€ΠΈΡ†Π°Ρ‚Π΅Π»ΡŒΠ½Ρ‹ с Π΄Ρ€ΡƒΠ³ΠΎΠΉ стороны ΠΎΡ‚ Π½Π°Ρ‡Π°Π»Π° ΠΊΠΎΠΎΡ€Π΄ΠΈΠ½Π°Ρ‚.Π’Π΅ΠΏΠ΅Ρ€ΡŒ с ΠΏΠΎΠΌΠΎΡ‰ΡŒΡŽ этого ΠΌΡ‹ ΠΌΠΎΠΆΠ΅ΠΌ Π»Π΅Π³ΠΊΠΎ Π½Π°ΠΉΡ‚ΠΈ значСния тригономСтричСских ΡΠΎΠΎΡ‚Π½ΠΎΡˆΠ΅Π½ΠΈΠΉ стандартных ΡƒΠ³Π»ΠΎΠ² Π² Ρ‡Π΅Ρ‚Ρ‹Ρ€Π΅Ρ… ΠΊΠ²Π°Π΄Ρ€Π°Π½Ρ‚Π°Ρ… Π΅Π΄ΠΈΠ½ΠΈΡ‡Π½ΠΎΠΉ окруТности.

Π•Π΄ΠΈΠ½ΠΈΡ‡Π½Ρ‹ΠΉ ΠΊΡ€ΡƒΠ³ ΠΈ тригономСтричСскиС идСнтичности

Π˜Π΄Π΅Π½Ρ‚ΠΈΡ‡Π½ΠΎΡΡ‚ΠΈ Π΅Π΄ΠΈΠ½ΠΈΡ‡Π½ΠΎΠΉ окруТности синуса, косСканса ΠΈ тангСнса ΠΌΠΎΠ³ΡƒΡ‚ Π±Ρ‹Ρ‚ΡŒ Π΄ΠΎΠΏΠΎΠ»Π½ΠΈΡ‚Π΅Π»ΡŒΠ½ΠΎ ΠΈΡΠΏΠΎΠ»ΡŒΠ·ΠΎΠ²Π°Π½Ρ‹ для получСния Π΄Ρ€ΡƒΠ³ΠΈΡ… тригономСтричСских ΠΈΠ΄Π΅Π½Ρ‚ΠΈΡ„ΠΈΠΊΠ°Ρ‚ΠΎΡ€ΠΎΠ², Ρ‚Π°ΠΊΠΈΡ… ΠΊΠ°ΠΊ котангСнс, сСканс ΠΈ косСканс. Π˜Π΄Π΅Π½Ρ‚ΠΈΡ„ΠΈΠΊΠ°Ρ‚ΠΎΡ€Ρ‹ Π΅Π΄ΠΈΠ½ΠΈΡ‡Π½ΠΎΠΉ окруТности, Ρ‚Π°ΠΊΠΈΠ΅ ΠΊΠ°ΠΊ косСканс, сСканс, котангСнс, ΡΠ²Π»ΡΡŽΡ‚ΡΡ ΡΠΎΠΎΡ‚Π²Π΅Ρ‚ΡΡ‚Π²ΡƒΡŽΡ‰ΠΈΠΌΠΈ ΠΎΠ±Ρ€Π°Ρ‚Π½Ρ‹ΠΌΠΈ Π²Π΅Π»ΠΈΡ‡ΠΈΠ½Π°ΠΌΠΈ синуса, косинуса ΠΈ тангСнса.ΠšΡ€ΠΎΠΌΠ΅ Ρ‚ΠΎΠ³ΠΎ, ΠΌΡ‹ ΠΌΠΎΠΆΠ΅ΠΌ ΠΏΠΎΠ»ΡƒΡ‡ΠΈΡ‚ΡŒ Π·Π½Π°Ρ‡Π΅Π½ΠΈΠ΅ tanΞΈ, Ρ€Π°Π·Π΄Π΅Π»ΠΈΠ² sinΞΈ Π½Π° cosΞΈ, ΠΈ ΠΌΡ‹ ΠΌΠΎΠΆΠ΅ΠΌ ΠΏΠΎΠ»ΡƒΡ‡ΠΈΡ‚ΡŒ Π·Π½Π°Ρ‡Π΅Π½ΠΈΠ΅ cotΞΈ, Ρ€Π°Π·Π΄Π΅Π»ΠΈΠ² cosΞΈ Π½Π° sinΞΈ.

Для ΠΏΡ€ΡΠΌΠΎΡƒΠ³ΠΎΠ»ΡŒΠ½ΠΎΠ³ΠΎ Ρ‚Ρ€Π΅ΡƒΠ³ΠΎΠ»ΡŒΠ½ΠΈΠΊΠ°, ΠΏΠΎΠΌΠ΅Ρ‰Π΅Π½Π½ΠΎΠ³ΠΎ Π² Π΅Π΄ΠΈΠ½ΠΈΡ‡Π½Ρ‹ΠΉ ΠΊΡ€ΡƒΠ³ Π² Π΄Π΅ΠΊΠ°Ρ€Ρ‚ΠΎΠ²ΠΎΠΉ ΠΊΠΎΠΎΡ€Π΄ΠΈΠ½Π°Ρ‚Π½ΠΎΠΉ плоскости, с Π³ΠΈΠΏΠΎΡ‚Π΅Π½ΡƒΠ·ΠΎΠΉ, основаниСм ΠΈ высотой, ΠΈΠ·ΠΌΠ΅Ρ€ΡΡŽΡ‰ΠΈΠΌΠΈ Π΅Π΄ΠΈΠ½ΠΈΡ†Ρ‹, x, y соотвСтствСнно, тоТдСства Π΅Π΄ΠΈΠ½ΠΈΡ‡Π½ΠΎΠ³ΠΎ ΠΊΡ€ΡƒΠ³Π° ΠΌΠΎΠ³ΡƒΡ‚ Π±Ρ‹Ρ‚ΡŒ Π·Π°Π΄Π°Π½Ρ‹ ΠΊΠ°ΠΊ,

  • sinΞΈ = y / 1
  • cosΞΈ = x / 1
  • tanΞΈ = sinΞΈ / cosΞΈ = y / x
  • с (ΞΈ = 1 / x
  • csc (ΞΈ) = 1 / y
  • дСтская ΠΊΡ€ΠΎΠ²Π°Ρ‚ΠΊΠ° (ΞΈ) = cosΞΈ / sinΞΈ = x / y

Π•Π΄ΠΈΠ½ΠΈΡ‡Π½Ρ‹ΠΉ ΠΊΡ€ΡƒΠ³ ΠŸΠΈΡ„Π°Π³ΠΎΡ€Π΅ΠΉΡΠΊΠΈΠ΅ тоТдСства

Π’Ρ€ΠΈ Π²Π°ΠΆΠ½Ρ‹Ρ… ΠΏΠΈΡ„Π°Π³ΠΎΡ€ΠΎΠ²Ρ‹Ρ… тоТдСства тригономСтричСских ΡΠΎΠΎΡ‚Π½ΠΎΡˆΠ΅Π½ΠΈΠΉ ΠΌΠΎΠΆΠ½ΠΎ Π»Π΅Π³ΠΊΠΎ ΠΏΠΎΠ½ΡΡ‚ΡŒ ΠΈ Π΄ΠΎΠΊΠ°Π·Π°Ρ‚ΡŒ с ΠΏΠΎΠΌΠΎΡ‰ΡŒΡŽ Π΅Π΄ΠΈΠ½ΠΈΡ‡Π½ΠΎΠΉ окруТности.Π’Π΅ΠΎΡ€Π΅ΠΌΠ° ΠŸΠΈΡ„Π°Π³ΠΎΡ€Π° ΡƒΡ‚Π²Π΅Ρ€ΠΆΠ΄Π°Π΅Ρ‚, Ρ‡Ρ‚ΠΎ Π² ΠΏΡ€ΡΠΌΠΎΡƒΠ³ΠΎΠ»ΡŒΠ½ΠΎΠΌ Ρ‚Ρ€Π΅ΡƒΠ³ΠΎΠ»ΡŒΠ½ΠΈΠΊΠ΅ ΠΊΠ²Π°Π΄Ρ€Π°Ρ‚ Π³ΠΈΠΏΠΎΡ‚Π΅Π½ΡƒΠ·Ρ‹ Ρ€Π°Π²Π΅Π½ суммС ΠΊΠ²Π°Π΄Ρ€Π°Ρ‚ΠΎΠ² Π΄Π²ΡƒΡ… Π΄Ρ€ΡƒΠ³ΠΈΡ… сторон. Π’Ρ€ΠΈ ΠΏΠΈΡ„Π°Π³ΠΎΡ€ΠΎΠ²Π° тоТдСства Π² Ρ‚Ρ€ΠΈΠ³ΠΎΠ½ΠΎΠΌΠ΅Ρ‚Ρ€ΠΈΠΈ Π·Π°ΠΊΠ»ΡŽΡ‡Π°ΡŽΡ‚ΡΡ Π² ΡΠ»Π΅Π΄ΡƒΡŽΡ‰Π΅ΠΌ.

  • sin 2 ΞΈ + cos 2 ΞΈ = 1
  • 1 + Π·Π°Π³Π°Ρ€ 2 ΞΈ = сСк 2 ΞΈ
  • 1 + дСтская ΠΊΡ€ΠΎΠ²Π°Ρ‚ΠΊΠ° 2 ΞΈ = cosec 2 ΞΈ

Π—Π΄Π΅ΡΡŒ ΠΌΡ‹ попытаСмся Π΄ΠΎΠΊΠ°Π·Π°Ρ‚ΡŒ ΠΏΠ΅Ρ€Π²ΠΎΠ΅ тоТдСство с ΠΏΠΎΠΌΠΎΡ‰ΡŒΡŽ Ρ‚Π΅ΠΎΡ€Π΅ΠΌΡ‹ ΠŸΠΈΡ„Π°Π³ΠΎΡ€Π°.Π’ΠΎΠ·ΡŒΠΌΠ΅ΠΌ x ΠΈ y Π² качСствС ΠΊΠ°Ρ‚Π΅Ρ‚ΠΎΠ² ΠΏΡ€ΡΠΌΠΎΡƒΠ³ΠΎΠ»ΡŒΠ½ΠΎΠ³ΠΎ Ρ‚Ρ€Π΅ΡƒΠ³ΠΎΠ»ΡŒΠ½ΠΈΠΊΠ° с Π΅Π΄ΠΈΠ½ΠΈΡ†Π΅ΠΉ Π³ΠΈΠΏΠΎΡ‚Π΅Π½ΡƒΠ·Ρ‹ 1. ΠŸΡ€ΠΈΠΌΠ΅Π½ΡΡ Ρ‚Π΅ΠΎΡ€Π΅ΠΌΡƒ ΠŸΠΈΡ„Π°Π³ΠΎΡ€Π°, ΠΌΡ‹ ΠΏΠΎΠ»ΡƒΡ‡Π°Π΅ΠΌ x 2 + y 2 = 1, Ρ‡Ρ‚ΠΎ прСдставляСт собой ΡƒΡ€Π°Π²Π½Π΅Π½ΠΈΠ΅ Π΅Π΄ΠΈΠ½ΠΈΡ‡Π½ΠΎΠΉ окруТности. Π’Π°ΠΊΠΆΠ΅ Π² Π΅Π΄ΠΈΠ½ΠΈΡ‡Π½ΠΎΠΌ ΠΊΡ€ΡƒΠ³Π΅ ΠΈΠΌΠ΅Π΅ΠΌ x = cosΞΈ ΠΈ y = sinΞΈ, ΠΈ примСняя это Π² ΠΏΡ€ΠΈΠ²Π΅Π΄Π΅Π½Π½ΠΎΠΌ Π²Ρ‹ΡˆΠ΅ ΡƒΡ‚Π²Π΅Ρ€ΠΆΠ΄Π΅Π½ΠΈΠΈ Ρ‚Π΅ΠΎΡ€Π΅ΠΌΡ‹ ΠŸΠΈΡ„Π°Π³ΠΎΡ€Π°, ΠΌΡ‹ ΠΈΠΌΠ΅Π΅ΠΌ cos 2 ΞΈ + sin 2 ΞΈ = 1. Π’Π°ΠΊΠΈΠΌ ΠΎΠ±Ρ€Π°Π·ΠΎΠΌ, ΠΌΡ‹ ΠΈΠΌΠ΅Π΅ΠΌ ΡƒΡΠΏΠ΅ΡˆΠ½ΠΎ Π΄ΠΎΠΊΠ°Π·Π°Π» ΠΏΠ΅Ρ€Π²ΠΎΠ΅ тоТдСство с ΠΏΠΎΠΌΠΎΡ‰ΡŒΡŽ Ρ‚Π΅ΠΎΡ€Π΅ΠΌΡ‹ ΠŸΠΈΡ„Π°Π³ΠΎΡ€Π°. Π”Π°Π»Π΅Π΅ Π² ΠΏΡ€Π΅Π΄Π΅Π»Π°Ρ… Π΅Π΄ΠΈΠ½ΠΈΡ‡Π½ΠΎΠ³ΠΎ ΠΊΡ€ΡƒΠ³Π° ΠΌΡ‹ Ρ‚Π°ΠΊΠΆΠ΅ ΠΌΠΎΠΆΠ΅ΠΌ Π΄ΠΎΠΊΠ°Π·Π°Ρ‚ΡŒ Π΄Π²Π° Π΄Ρ€ΡƒΠ³ΠΈΡ… тоТдСства ΠŸΠΈΡ„Π°Π³ΠΎΡ€Π°.

Единичная ΠΎΠΊΡ€ΡƒΠΆΠ½ΠΎΡΡ‚ΡŒ ΠΈ тригономСтричСскиС значСния

Π Π°Π·Π»ΠΈΡ‡Π½Ρ‹Π΅ тригономСтричСскиС тоТдСства ΠΈ ΠΈΡ… значСния Π³Π»Π°Π²Π½Ρ‹Ρ… ΡƒΠ³Π»ΠΎΠ² ΠΌΠΎΠ³ΡƒΡ‚ Π±Ρ‹Ρ‚ΡŒ вычислСны с ΠΏΠΎΠΌΠΎΡ‰ΡŒΡŽ Π΅Π΄ΠΈΠ½ΠΈΡ‡Π½ΠΎΠΉ окруТности. Π’ Π΅Π΄ΠΈΠ½ΠΈΡ‡Π½ΠΎΠΌ ΠΊΡ€ΡƒΠ³Π΅ косинус являСтся ΠΊΠΎΠΎΡ€Π΄ΠΈΠ½Π°Ρ‚ΠΎΠΉ x, Π° синус — ΠΊΠΎΠΎΡ€Π΄ΠΈΠ½Π°Ρ‚ΠΎΠΉ y. Π”Π°Π²Π°ΠΉΡ‚Π΅ Ρ‚Π΅ΠΏΠ΅Ρ€ΡŒ Π½Π°ΠΉΠ΄Π΅ΠΌ ΠΈΡ… ΡΠΎΠΎΡ‚Π²Π΅Ρ‚ΡΡ‚Π²ΡƒΡŽΡ‰ΠΈΠ΅ значСния для ΞΈ = 0 Β° ΠΈ ΞΈ = 90ΒΊ.

Для ΞΈ = 0 Β° ΠΊΠΎΠΎΡ€Π΄ΠΈΠ½Π°Ρ‚Π° x Ρ€Π°Π²Π½Π° 1, Π° ΠΊΠΎΠΎΡ€Π΄ΠΈΠ½Π°Ρ‚Π° y Ρ€Π°Π²Π½Π° 0. Π‘Π»Π΅Π΄ΠΎΠ²Π°Ρ‚Π΅Π»ΡŒΠ½ΠΎ, ΠΌΡ‹ ΠΈΠΌΠ΅Π΅ΠΌ cos0ΒΊ = 1 ΠΈ sin0ΒΊ = 0.Π”Π°Π²Π°ΠΉΡ‚Π΅ посмотрим Π½Π° Π΄Ρ€ΡƒΠ³ΠΎΠΉ ΡƒΠ³ΠΎΠ» 90ΒΊ. Π—Π΄Π΅ΡΡŒ Π·Π½Π°Ρ‡Π΅Π½ΠΈΠ΅ cos90ΒΊ = 1 ΠΈ sin90ΒΊ = 1. Π”Π°Π»Π΅Π΅, Π΄Π°Π²Π°ΠΉΡ‚Π΅ Π²ΠΎΡΠΏΠΎΠ»ΡŒΠ·ΡƒΠ΅ΠΌΡΡ этой Π΅Π΄ΠΈΠ½ΠΈΡ‡Π½ΠΎΠΉ ΠΎΠΊΡ€ΡƒΠΆΠ½ΠΎΡΡ‚ΡŒΡŽ ΠΈ Π½Π°ΠΉΠ΄Π΅ΠΌ Π²Π°ΠΆΠ½Ρ‹Π΅ значСния тригономСтричСской Ρ„ΡƒΠ½ΠΊΡ†ΠΈΠΈ ΞΈ, Ρ‚Π°ΠΊΠΈΠ΅ ΠΊΠ°ΠΊ 30ΒΊ, 45ΒΊ, 60ΒΊ. ΠšΡ€ΠΎΠΌΠ΅ Ρ‚ΠΎΠ³ΠΎ, ΠΌΡ‹ Ρ‚Π°ΠΊΠΆΠ΅ ΠΌΠΎΠΆΠ΅ΠΌ ΠΈΠ·ΠΌΠ΅Ρ€ΠΈΡ‚ΡŒ эти значСния ΞΈ Π² Ρ€Π°Π΄ΠΈΠ°Π½Π°Ρ…. ΠœΡ‹ Π·Π½Π°Π΅ΠΌ, Ρ‡Ρ‚ΠΎ 360 Β° = 2Ο€ Ρ€Π°Π΄ΠΈΠ°Π½. Π’Π΅ΠΏΠ΅Ρ€ΡŒ ΠΌΡ‹ ΠΌΠΎΠΆΠ΅ΠΌ ΠΏΡ€Π΅ΠΎΠ±Ρ€Π°Π·ΠΎΠ²Π°Ρ‚ΡŒ ΡƒΠ³Π»ΠΎΠ²Ρ‹Π΅ ΠΌΠ΅Ρ€Ρ‹ Π² Ρ€Π°Π΄ΠΈΠ°Π½Ρ‹ ΠΈ Π²Ρ‹Ρ€Π°Π·ΠΈΡ‚ΡŒ ΠΈΡ… Π² Ρ€Π°Π΄ΠΈΠ°Π½Π°Ρ….

Π•Π΄ΠΈΠ½ΠΈΡ‡Π½Ρ‹ΠΉ ΠΊΡ€ΡƒΠ³ΠΎΠ²ΠΎΠΉ стол:

Π’Π°Π±Π»ΠΈΡ†Π° Π΅Π΄ΠΈΠ½ΠΈΡ‡Π½ΠΎΠΉ окруТности ΠΈΡΠΏΠΎΠ»ΡŒΠ·ΡƒΠ΅Ρ‚ΡΡ для пСрСчислСния ΠΊΠΎΠΎΡ€Π΄ΠΈΠ½Π°Ρ‚ Ρ‚ΠΎΡ‡Π΅ΠΊ Π΅Π΄ΠΈΠ½ΠΈΡ‡Π½ΠΎΠΉ окруТности, ΡΠΎΠΎΡ‚Π²Π΅Ρ‚ΡΡ‚Π²ΡƒΡŽΡ‰ΠΈΡ… ΠΎΠ±Ρ‰ΠΈΠΌ ΡƒΠ³Π»Π°ΠΌ, с ΠΏΠΎΠΌΠΎΡ‰ΡŒΡŽ тригономСтричСских ΡΠΎΠΎΡ‚Π½ΠΎΡˆΠ΅Π½ΠΈΠΉ.

Π£Π³ΠΎΠ» ΞΈ Π Π°Π΄ΠΈΠ°Π½Ρ‹ SinΞΈ CosΞΈ TanΞΈ = SinΞΈ / CosΞΈ ΠšΠΎΠΎΡ€Π΄ΠΈΠ½Π°Ρ‚Ρ‹
0 Β° 0 0 1 0 (1, 0)
30 Β° Ο€ / 6 1/2 √3 / 2 1 / √3 (√3 / 2, 1/2)
45 Β° Ο€ / 4 1 / √2 1 / √2 1 (1 / √2, 1 / √2)
60 Β° Ο€ / 3 √3 / 2 1/2 √3 (1/2, √3 / 2)
90 Β° Ο€ / 2 1 0 undefined (0,1)

ΠœΡ‹ ΠΌΠΎΠΆΠ΅ΠΌ Π½Π°ΠΉΡ‚ΠΈ Ρ„ΡƒΠ½ΠΊΡ†ΠΈΠΈ сСканса, косСканса ΠΈ котангСнса, Ρ‚Π°ΠΊΠΆΠ΅ ΠΈΡΠΏΠΎΠ»ΡŒΠ·ΡƒΡ ΡΠ»Π΅Π΄ΡƒΡŽΡ‰ΠΈΠ΅ Ρ„ΠΎΡ€ΠΌΡƒΠ»Ρ‹:

  • сСк ΞΈ = 1 / cosΞΈ
  • косСкунд ΞΈ = 1 / sinΞΈ
  • cotΞΈ = 1 / tanΞΈ

ΠœΡ‹ обсудили Π΅Π΄ΠΈΠ½ΠΈΡ‡Π½Ρ‹ΠΉ ΠΊΡ€ΡƒΠ³ для ΠΏΠ΅Ρ€Π²ΠΎΠ³ΠΎ ΠΊΠ²Π°Π΄Ρ€Π°Π½Ρ‚Π°.Π’ΠΎΡ‡Π½ΠΎ Ρ‚Π°ΠΊ ΠΆΠ΅ ΠΌΡ‹ ΠΌΠΎΠΆΠ΅ΠΌ Ρ€Π°ΡΡˆΠΈΡ€ΠΈΡ‚ΡŒ ΠΈ Π½Π°ΠΉΡ‚ΠΈ Ρ€Π°Π΄ΠΈΠ°Π½Ρ‹ для всСх ΠΊΠ²Π°Π΄Ρ€Π°Π½Ρ‚ΠΎΠ² Π΅Π΄ΠΈΠ½ΠΈΡ‡Π½ΠΎΠΉ окруТности. Π¦ΠΈΡ„Ρ€Ρ‹ 1/2, 1 / √2, √3 / 2, 0, 1 ΠΏΠΎΠ²Ρ‚ΠΎΡ€ΡΡŽΡ‚ΡΡ вмСстС со Π·Π½Π°ΠΊΠΎΠΌ Π²ΠΎ всСх 4 ΠΊΠ²Π°Π΄Ρ€Π°Π½Ρ‚Π°Ρ….

Π•Π΄ΠΈΠ½ΠΈΡ‡Π½Ρ‹ΠΉ ΠΊΡ€ΡƒΠ³ Π² слоТной плоскости

Π•Π΄ΠΈΠ½ΠΈΡ‡Π½Ρ‹ΠΉ ΠΊΡ€ΡƒΠ³ состоит ΠΈΠ· всСх комплСксных чисСл, Π°Π±ΡΠΎΠ»ΡŽΡ‚Π½ΠΎΠ΅ Π·Π½Π°Ρ‡Π΅Π½ΠΈΠ΅ ΠΊΠΎΡ‚ΠΎΡ€Ρ‹Ρ… Ρ€Π°Π²Π½ΠΎ 1. Π‘Π»Π΅Π΄ΠΎΠ²Π°Ρ‚Π΅Π»ΡŒΠ½ΠΎ, ΠΎΠ½ΠΎ ΠΈΠΌΠ΅Π΅Ρ‚ ΡƒΡ€Π°Π²Π½Π΅Π½ΠΈΠ΅ | z | = 1. Π›ΡŽΠ±ΠΎΠ΅ комплСксноС число z = x + \ (i \) y Π±ΡƒΠ΄Π΅Ρ‚ Π»Π΅ΠΆΠ°Ρ‚ΡŒ Π½Π° Π΅Π΄ΠΈΠ½ΠΈΡ‡Π½ΠΎΠΉ окруТности с ΡƒΡ€Π°Π²Π½Π΅Π½ΠΈΠ΅ΠΌ, Π·Π°Π΄Π°Π½Π½Ρ‹ΠΌ ΠΊΠ°ΠΊ x 2 + y 2 = 1.

Π•Π΄ΠΈΠ½ΠΈΡ‡Π½Ρ‹ΠΉ ΠΊΡ€ΡƒΠ³ ΠΌΠΎΠΆΠ½ΠΎ Ρ€Π°ΡΡΠΌΠ°Ρ‚Ρ€ΠΈΠ²Π°Ρ‚ΡŒ ΠΊΠ°ΠΊ Π΅Π΄ΠΈΠ½ΠΈΡ‡Π½Ρ‹Π΅ комплСксныС числа Π½Π° комплСксной плоскости, Ρ‚.Π΅.Π΅., Π½Π°Π±ΠΎΡ€ комплСксных чисСл z, Π·Π°Π΄Π°Π½Π½Ρ‹ΠΉ Ρ„ΠΎΡ€ΠΌΠΎΠΉ,

z = e \ (i \) t = cos t + \ (i \) sin t = cis (t)

ΠŸΡ€ΠΈΠ²Π΅Π΄Π΅Π½Π½ΠΎΠ΅ Π²Ρ‹ΡˆΠ΅ ΡΠΎΠΎΡ‚Π½ΠΎΡˆΠ΅Π½ΠΈΠ΅ прСдставляСт собой Ρ„ΠΎΡ€ΠΌΡƒΠ»Ρƒ Π­ΠΉΠ»Π΅Ρ€Π°.

Часто Π·Π°Π΄Π°Π²Π°Π΅ΠΌΡ‹Π΅ вопросы ΠΎ Unit Circle

Π§Ρ‚ΠΎ Ρ‚Π°ΠΊΠΎΠ΅ Π΅Π΄ΠΈΠ½ΠΈΡ‡Π½Ρ‹ΠΉ ΠΊΡ€ΡƒΠ³ Π² ΠΌΠ°Ρ‚Π΅ΠΌΠ°Ρ‚ΠΈΠΊΠ΅?

Π•Π΄ΠΈΠ½ΠΈΡ‡Π½Ρ‹ΠΉ ΠΊΡ€ΡƒΠ³ — это ΠΊΡ€ΡƒΠ³ с радиусом Π² ΠΎΠ΄Π½Ρƒ Π΅Π΄ΠΈΠ½ΠΈΡ†Ρƒ. ΠžΠ±Ρ‹Ρ‡Π½ΠΎ Π΅Π΄ΠΈΠ½ΠΈΡ‡Π½Ρ‹ΠΉ ΠΊΡ€ΡƒΠ³ прСдставлСн Π² ΠΊΠΎΠΎΡ€Π΄ΠΈΠ½Π°Ρ‚Π½ΠΎΠΉ плоскости с Ρ†Π΅Π½Ρ‚Ρ€ΠΎΠΌ Π² Π½Π°Ρ‡Π°Π»Π΅ ΠΊΠΎΠΎΡ€Π΄ΠΈΠ½Π°Ρ‚. Π£Ρ€Π°Π²Π½Π΅Π½ΠΈΠ΅ Π΅Π΄ΠΈΠ½ΠΈΡ‡Π½ΠΎΠΉ окруТности с радиусом ΠΎΠ΄Π½Π° Π΅Π΄ΠΈΠ½ΠΈΡ†Π° ΠΈ Ρ†Π΅Π½Ρ‚Ρ€ΠΎΠΌ Π² Ρ‚ΠΎΡ‡ΠΊΠ΅ (0, 0): x 2 + y 2 = 1.ΠšΡ€ΠΎΠΌΠ΅ Ρ‚ΠΎΠ³ΠΎ, Сдиничная ΠΎΠΊΡ€ΡƒΠΆΠ½ΠΎΡΡ‚ΡŒ примСняСтся Π² Ρ‚Ρ€ΠΈΠ³ΠΎΠ½ΠΎΠΌΠ΅Ρ‚Ρ€ΠΈΠΈ ΠΈ ΠΈΡΠΏΠΎΠ»ΡŒΠ·ΡƒΠ΅Ρ‚ΡΡ для нахоТдСния Π³Π»Π°Π²Π½Ρ‹Ρ… Π·Π½Π°Ρ‡Π΅Π½ΠΈΠΉ тригономСтричСских ΡΠΎΠΎΡ‚Π½ΠΎΡˆΠ΅Π½ΠΈΠΉ синуса ΠΈ косинуса.

Как Π½Π°ΠΉΡ‚ΠΈ Π³Ρ€Π΅Ρ… ΠΈ Ρ€Π°Π·ΡƒΠΌ, ΠΈΡΠΏΠΎΠ»ΡŒΠ·ΡƒΡ Π΅Π΄ΠΈΠ½ΠΈΡ‡Π½Ρ‹ΠΉ ΠΊΡ€ΡƒΠ³?

Π•Π΄ΠΈΠ½ΠΈΡ‡Π½Ρ‹ΠΉ ΠΊΡ€ΡƒΠ³ ΠΌΠΎΠΆΠ½ΠΎ ΠΈΡΠΏΠΎΠ»ΡŒΠ·ΠΎΠ²Π°Ρ‚ΡŒ для опрСдСлСния Π·Π½Π°Ρ‡Π΅Π½ΠΈΠΉ sinΞΈ ΠΈ CosΞΈ. Π’ Π΅Π΄ΠΈΠ½ΠΈΡ‡Π½ΠΎΠΌ ΠΊΡ€ΡƒΠ³Π΅ радиуса 1 Π΅Π΄ΠΈΠ½ΠΈΡ†Π° ΠΈ с Ρ†Π΅Π½Ρ‚Ρ€ΠΎΠΌ Π² (0, 0), Π΄Π°Π²Π°ΠΉΡ‚Π΅ возьмСм радиус, Π½Π°ΠΊΠ»ΠΎΠ½Π΅Π½Π½Ρ‹ΠΉ ΠΊ ΠΏΠΎΠ»ΠΎΠΆΠΈΡ‚Π΅Π»ΡŒΠ½ΠΎΠΉ оси x ΠΏΠΎΠ΄ ΡƒΠ³Π»ΠΎΠΌ ΞΈ, ΠΈ ΠΊΠΎΠ½Π΅Ρ‡Π½ΡƒΡŽ Ρ‚ΠΎΡ‡ΠΊΡƒ радиуса ΠΊΠ°ΠΊ (x, y). НарисуйтС пСрпСндикуляр ΠΎΡ‚ ΠΊΠΎΠ½Ρ†Π° радиуса ΠΊ оси x, ΠΈ ΠΎΠ½ ΠΎΠ±Ρ€Π°Π·ΡƒΠ΅Ρ‚ ΠΏΡ€ΡΠΌΠΎΡƒΠ³ΠΎΠ»ΡŒΠ½Ρ‹ΠΉ Ρ‚Ρ€Π΅ΡƒΠ³ΠΎΠ»ΡŒΠ½ΠΈΠΊ с радиусом Π² качСствС Π³ΠΈΠΏΠΎΡ‚Π΅Π½ΡƒΠ·Ρ‹.БосСдняя сторона этого Ρ‚Ρ€Π΅ΡƒΠ³ΠΎΠ»ΡŒΠ½ΠΈΠΊΠ° — это Π·Π½Π°Ρ‡Π΅Π½ΠΈΠ΅ x, противополоТная сторона Ρ‚Ρ€Π΅ΡƒΠ³ΠΎΠ»ΡŒΠ½ΠΈΠΊΠ° — Π·Π½Π°Ρ‡Π΅Π½ΠΈΠ΅ y, Π° Π³ΠΈΠΏΠΎΡ‚Π΅Π½ΡƒΠ·Π° Ρ€Π°Π²Π½Π° 1 Π΅Π΄ΠΈΠ½ΠΈΡ†Π΅. Π”Π°Π»Π΅Π΅, ΠΈΡΠΏΠΎΠ»ΡŒΠ·ΡƒΡ Ρ„ΠΎΡ€ΠΌΡƒΠ»Ρƒ тригономСтричСского ΠΎΡ‚Π½ΠΎΡˆΠ΅Π½ΠΈΡ, ΠΈΠΌΠ΅Π΅ΠΌ sinΞΈ = Opp / Hyp = y / 1 ΠΈ cosΞΈ = Adj / Hyp = x / 1. Π’Π°ΠΊΠΈΠΌ ΠΎΠ±Ρ€Π°Π·ΠΎΠΌ, sinΞΈ = y ΠΈ cosΞΈ = x.

Π§Ρ‚ΠΎ Ρ‚Π°ΠΊΠΎΠ΅ ΠΎΠΏΡ€Π΅Π΄Π΅Π»Π΅Π½ΠΈΠ΅ Ρ‚Ρ€ΠΈΠ³Π³Π΅Ρ€Π½Ρ‹Ρ… Ρ„ΡƒΠ½ΠΊΡ†ΠΈΠΉ с ΠΏΠΎΠΌΠΎΡ‰ΡŒΡŽ Π΅Π΄ΠΈΠ½ΠΈΡ‡Π½ΠΎΠΉ окруТности?

Π’Ρ€ΠΈΠ³ΠΎΠ½ΠΎΠΌΠ΅Ρ‚Ρ€ΠΈΡ‡Π΅ΡΠΊΡƒΡŽ Ρ„ΡƒΠ½ΠΊΡ†ΠΈΡŽ ΠΌΠΎΠΆΠ½ΠΎ Π²Ρ‹Ρ‡ΠΈΡΠ»ΠΈΡ‚ΡŒ для Π³Π»Π°Π²Π½Ρ‹Ρ… Π·Π½Π°Ρ‡Π΅Π½ΠΈΠΉ с ΠΏΠΎΠΌΠΎΡ‰ΡŒΡŽ Π΅Π΄ΠΈΠ½ΠΈΡ‡Π½ΠΎΠΉ окруТности. Для Π΅Π΄ΠΈΠ½ΠΈΡ‡Π½ΠΎΠ³ΠΎ ΠΊΡ€ΡƒΠ³Π°, ΠΈΠΌΠ΅ΡŽΡ‰Π΅Π³ΠΎ Ρ†Π΅Π½Ρ‚Ρ€ Π² Π½Π°Ρ‡Π°Π»Π΅ ΠΊΠΎΠΎΡ€Π΄ΠΈΠ½Π°Ρ‚ (0, 0), радиус 1 Π΅Π΄ΠΈΠ½ΠΈΡ†Ρƒ, Ссли радиус Π½Π°ΠΊΠ»ΠΎΠ½Π΅Π½ ΠΏΠΎΠ΄ ΡƒΠ³Π»ΠΎΠΌ ΞΈ ΠΈ конСчная Ρ‚ΠΎΡ‡ΠΊΠ° радиус-Π²Π΅ΠΊΡ‚ΠΎΡ€Π° Ρ€Π°Π²Π½Π° (x, y), Ρ‚ΠΎ cosΞΈ = x ΠΈ sinΞΈ = y.ΠšΡ€ΠΎΠΌΠ΅ Ρ‚ΠΎΠ³ΠΎ, всС ΠΎΡΡ‚Π°Π»ΡŒΠ½Ρ‹Π΅ тригономСтричСскиС ΠΎΡ‚Π½ΠΎΡˆΠ΅Π½ΠΈΡ ΠΌΠΎΠ³ΡƒΡ‚ Π±Ρ‹Ρ‚ΡŒ рассчитаны Π½Π° основС этих Π΄Π²ΡƒΡ… Π·Π½Π°Ρ‡Π΅Π½ΠΈΠΉ. ΠšΡ€ΠΎΠΌΠ΅ Ρ‚ΠΎΠ³ΠΎ, основныС значСния ΠΌΠΎΠΆΠ½ΠΎ Π²Ρ‹Ρ‡ΠΈΡΠ»ΠΈΡ‚ΡŒ, ΠΈΠ·ΠΌΠ΅Π½ΠΈΠ² Π·Π½Π°Ρ‡Π΅Π½ΠΈΠ΅ ΞΈ.

Как Π½Π°ΠΉΡ‚ΠΈ ΠΊΠΎΠ½Π΅Ρ‡Π½ΡƒΡŽ Ρ‚ΠΎΡ‡ΠΊΡƒ Π½Π° Π΅Π΄ΠΈΠ½ΠΈΡ‡Π½ΠΎΠΉ окруТности?

ΠšΠΎΠ½Π΅Ρ‡Π½Π°Ρ Ρ‚ΠΎΡ‡ΠΊΠ° Π½Π° Π΅Π΄ΠΈΠ½ΠΈΡ‡Π½ΠΎΠΉ окруТности ΠΌΠΎΠΆΠ΅Ρ‚ Π±Ρ‹Ρ‚ΡŒ Π½Π°ΠΉΠ΄Π΅Π½Π° с ΠΏΠΎΠΌΠΎΡ‰ΡŒΡŽ уравнСния Π΅Π΄ΠΈΠ½ΠΈΡ‡Π½ΠΎΠΉ окруТности x 2 + y 2 = 1. Если данная Ρ‚ΠΎΡ‡ΠΊΠ° удовлСтворяСт этому ΡƒΡ€Π°Π²Π½Π΅Π½ΠΈΡŽ, Ρ‚ΠΎ это Ρ‚ΠΎΡ‡ΠΊΠ°, лСТащая Π½Π° Π΅Π΄ΠΈΠ½ΠΈΡ‡Π½ΠΎΠΉ окруТности. . ΠšΡ€ΠΎΠΌΠ΅ Ρ‚ΠΎΠ³ΠΎ, ΠΊΠΎΠ½Π΅Ρ‡Π½ΡƒΡŽ Ρ‚ΠΎΡ‡ΠΊΡƒ Π½Π° Π΅Π΄ΠΈΠ½ΠΈΡ‡Π½ΠΎΠΌ Π·Π½Π°Ρ‡Π΅Π½ΠΈΠΈ ΠΌΠΎΠΆΠ½ΠΎ Π½Π°ΠΉΡ‚ΠΈ для значСния ΞΈ, найдя значСния cosΞΈ ΠΈ sinΞΈ.

Π§Ρ‚ΠΎ Ρ‚Π°ΠΊΠΎΠ΅ ΡƒΡ€Π°Π²Π½Π΅Π½ΠΈΠ΅ Π΅Π΄ΠΈΠ½ΠΈΡ‡Π½ΠΎΠΉ окруТности?

Π£Ρ€Π°Π²Π½Π΅Π½ΠΈΠ΅ Π΅Π΄ΠΈΠ½ΠΈΡ‡Π½ΠΎΠΉ окруТности: x 2 + y 2 = 1. Π—Π΄Π΅ΡΡŒ считаСтся, Ρ‡Ρ‚ΠΎ Сдиничная ΠΎΠΊΡ€ΡƒΠΆΠ½ΠΎΡΡ‚ΡŒ ΠΈΠΌΠ΅Π΅Ρ‚ Ρ†Π΅Π½Ρ‚Ρ€ Π² Π½Π°Ρ‡Π°Π»Π΅ (0, 0) ΠΊΠΎΠΎΡ€Π΄ΠΈΠ½Π°Ρ‚Π½Ρ‹Ρ… осСй ΠΈ ΠΈΠΌΠ΅Π΅Ρ‚ радиус 1. Π‘Π»ΠΎΠΊ. Π­Ρ‚ΠΎ ΡƒΡ€Π°Π²Π½Π΅Π½ΠΈΠ΅ Π΅Π΄ΠΈΠ½ΠΈΡ‡Π½ΠΎΠΉ окруТности Π±Ρ‹Π»ΠΎ ΠΏΠΎΠ»ΡƒΡ‡Π΅Π½ΠΎ с ΠΏΠΎΠΌΠΎΡ‰ΡŒΡŽ Ρ„ΠΎΡ€ΠΌΡƒΠ»Ρ‹ расстояния.

Как вывСсти ΡƒΡ€Π°Π²Π½Π΅Π½ΠΈΠ΅ Π΅Π΄ΠΈΠ½ΠΈΡ‡Π½ΠΎΠΉ окруТности?

Π£Ρ€Π°Π²Π½Π΅Π½ΠΈΠ΅ Π΅Π΄ΠΈΠ½ΠΈΡ‡Π½ΠΎΠΉ окруТности ΠΌΠΎΠΆΠ΅Ρ‚ Π±Ρ‹Ρ‚ΡŒ вычислСно с использованиСм Ρ„ΠΎΡ€ΠΌΡƒΠ»Ρ‹ расстояния ΠΊΠΎΠΎΡ€Π΄ΠΈΠ½Π°Ρ‚Π½ΠΎΠΉ Π³Π΅ΠΎΠΌΠ΅Ρ‚Ρ€ΠΈΠΈ.Для ΠΊΡ€ΡƒΠ³Π°, ΠΈΠΌΠ΅ΡŽΡ‰Π΅Π³ΠΎ Ρ†Π΅Π½Ρ‚Ρ€ Π² Π½Π°Ρ‡Π°Π»Π΅ ΠΊΠΎΠΎΡ€Π΄ΠΈΠ½Π°Ρ‚ (0, 0), радиус 1 Π΅Π΄ΠΈΠ½ΠΈΡ†Ρƒ, любая Ρ‚ΠΎΡ‡ΠΊΠ° Π½Π° окруТности ΠΌΠΎΠΆΠ΅Ρ‚ Π±Ρ‹Ρ‚ΡŒ принята ΠΊΠ°ΠΊ (x, y). ΠŸΡ€ΠΈΠΌΠ΅Π½ΡΡ ΠΎΠΏΡ€Π΅Π΄Π΅Π»Π΅Π½ΠΈΠ΅ ΠΊΡ€ΡƒΠ³Π° ΠΈ ΠΈΡΠΏΠΎΠ»ΡŒΠ·ΡƒΡ Ρ„ΠΎΡ€ΠΌΡƒΠ»Ρƒ расстояния, ΠΌΡ‹ ΠΈΠΌΠ΅Π΅ΠΌ (x — 0) 2 + (y — 0) 2 = 1, Ρ‡Ρ‚ΠΎ ΠΌΠΎΠΆΠ½ΠΎ ΡƒΠΏΡ€ΠΎΡΡ‚ΠΈΡ‚ΡŒ ΠΊΠ°ΠΊ x 2 + y 2 = 1

Когда Π·Π°Π³Π°Ρ€ Π½Π΅ опрСдСляСтся Π½Π° Π΅Π΄ΠΈΠ½ΠΈΡ‡Π½ΠΎΠΌ ΠΊΡ€ΡƒΠ³Π΅?

Π•Π΄ΠΈΠ½ΠΈΡ‡Π½Ρ‹ΠΉ ΠΊΡ€ΡƒΠ³, ΠΈΠΌΠ΅ΡŽΡ‰ΠΈΠΉ ΡƒΡ€Π°Π²Π½Π΅Π½ΠΈΠ΅ x 2 + y 2 = 1, ΠΏΠΎΠΌΠΎΠ³Π°Π΅Ρ‚ Π½Π°ΠΉΡ‚ΠΈ тригономСтричСскиС ΠΎΡ‚Π½ΠΎΡˆΠ΅Π½ΠΈΡ sinΞΈ = y ΠΈ cosΞΈ = x.Π˜ΡΠΏΠΎΠ»ΡŒΠ·ΡƒΡ эти значСния, ΠΌΡ‹ ΠΌΠΎΠΆΠ΅ΠΌ ΡƒΠ΄ΠΎΠ±Π½ΠΎ Π½Π°ΠΉΡ‚ΠΈ Π·Π½Π°Ρ‡Π΅Π½ΠΈΠ΅ tanΞΈ = sinΞΈ / cosΞΈ = y / x. TanΞΈ Π±ΡƒΠ΄Π΅Ρ‚ Π½Π΅ΠΎΠΏΡ€Π΅Π΄Π΅Π»Π΅Π½Π½Ρ‹ΠΌ для cosΞΈ = 0, Ρ‚.Π΅. ΠΊΠΎΠ³Π΄Π° ΞΈ Ρ€Π°Π²Π½ΠΎ 90 Β° ΠΈ 270 Β°.

Какая связь ΠΌΠ΅ΠΆΠ΄Ρƒ ΠΏΡ€ΡΠΌΠΎΡƒΠ³ΠΎΠ»ΡŒΠ½Ρ‹ΠΌΠΈ Ρ‚Ρ€Π΅ΡƒΠ³ΠΎΠ»ΡŒΠ½ΠΈΠΊΠ°ΠΌΠΈ ΠΈ Π΅Π΄ΠΈΠ½ΠΈΡ‡Π½ΠΎΠΉ ΠΎΠΊΡ€ΡƒΠΆΠ½ΠΎΡΡ‚ΡŒΡŽ?

ΠŸΡ€Π°Π²Ρ‹Π΅ Ρ‚Ρ€Π΅ΡƒΠ³ΠΎΠ»ΡŒΠ½ΠΈΠΊΠΈ ΠΈ Сдиничная ΠΎΠΊΡ€ΡƒΠΆΠ½ΠΎΡΡ‚ΡŒ ΠΎΠ΄Π½ΠΎΠ·Π½Π°Ρ‡Π½ΠΎ связаны. Π›ΡŽΠ±ΡƒΡŽ Ρ‚ΠΎΡ‡ΠΊΡƒ Π½Π° Π΅Π΄ΠΈΠ½ΠΈΡ‡Π½ΠΎΠΌ ΠΊΡ€ΡƒΠ³Π΅ ΠΌΠΎΠΆΠ½ΠΎ ΠΏΡ€Π΅Π΄ΡΡ‚Π°Π²ΠΈΡ‚ΡŒ ΠΊΠ°ΠΊ ΠΏΡ€ΡΠΌΠΎΡƒΠ³ΠΎΠ»ΡŒΠ½Ρ‹ΠΉ Ρ‚Ρ€Π΅ΡƒΠ³ΠΎΠ»ΡŒΠ½ΠΈΠΊ с радиусом ΠΊΠ°ΠΊ Π³ΠΈΠΏΠΎΡ‚Π΅Π½ΡƒΠ·Ρƒ ΠΏΡ€ΡΠΌΠΎΡƒΠ³ΠΎΠ»ΡŒΠ½ΠΎΠ³ΠΎ Ρ‚Ρ€Π΅ΡƒΠ³ΠΎΠ»ΡŒΠ½ΠΈΠΊΠ° ΠΈ ΠΊΠΎΠΎΡ€Π΄ΠΈΠ½Π°Ρ‚Ρ‹ Ρ‚ΠΎΡ‡ΠΊΠΈ ΠΊΠ°ΠΊ Π΄Π²Π΅ Π΄Ρ€ΡƒΠ³ΠΈΠ΅ стороны ΠΏΡ€ΡΠΌΠΎΡƒΠ³ΠΎΠ»ΡŒΠ½ΠΎΠ³ΠΎ Ρ‚Ρ€Π΅ΡƒΠ³ΠΎΠ»ΡŒΠ½ΠΈΠΊΠ°.Π£Ρ€Π°Π²Π½Π΅Π½ΠΈΠ΅ окруТности x 2 + y 2 = 1 ΠΏΠΎΠ»Π½ΠΎΡΡ‚ΡŒΡŽ удовлСтворяСт Ρ‚Π΅ΠΎΡ€Π΅ΠΌΠ΅ ΠŸΠΈΡ„Π°Π³ΠΎΡ€Π°, относящСйся ΠΊ ΠΏΡ€ΡΠΌΠΎΡƒΠ³ΠΎΠ»ΡŒΠ½ΠΎΠΌΡƒ Ρ‚Ρ€Π΅ΡƒΠ³ΠΎΠ»ΡŒΠ½ΠΈΠΊΡƒ. ΠšΡ€ΠΎΠΌΠ΅ Ρ‚ΠΎΠ³ΠΎ, ΠΏΡ€ΡΠΌΠΎΡƒΠ³ΠΎΠ»ΡŒΠ½Ρ‹ΠΉ Ρ‚Ρ€Π΅ΡƒΠ³ΠΎΠ»ΡŒΠ½ΠΈΠΊ Π²Π½ΡƒΡ‚Ρ€ΠΈ Π΅Π΄ΠΈΠ½ΠΈΡ‡Π½ΠΎΠ³ΠΎ ΠΊΡ€ΡƒΠ³Π° ΠΏΠΎΠΌΠΎΠ³Π°Π΅Ρ‚ ΠΏΠΎΠ»ΡƒΡ‡ΠΈΡ‚ΡŒ значСния тригономСтричСского ΠΎΡ‚Π½ΠΎΡˆΠ΅Π½ΠΈΡ.

Для Ρ‡Π΅Π³ΠΎ ΠΈΡΠΏΠΎΠ»ΡŒΠ·ΡƒΠ΅Ρ‚ΡΡ Π΅Π΄ΠΈΠ½ΠΈΡ‡Π½Ρ‹ΠΉ ΠΊΡ€ΡƒΠ³?

Π•Π΄ΠΈΠ½ΠΈΡ‡Π½Ρ‹ΠΉ ΠΊΡ€ΡƒΠ³ особСнно ΠΏΠΎΠ»Π΅Π·Π΅Π½ Π² Ρ‚Ρ€ΠΈΠ³ΠΎΠ½ΠΎΠΌΠ΅Ρ‚Ρ€ΠΈΠΈ. Для тригономСтричСских ΡΠΎΠΎΡ‚Π½ΠΎΡˆΠ΅Π½ΠΈΠΉ sinΞΈ, cosΞΈ, tanΞΈ ΠΈΡ… значСния Π³Π»Π°Π²Π½Ρ‹Ρ… ΡƒΠ³Π»ΠΎΠ² 0ΒΊ, 30ΒΊ, 45ΒΊ, 60ΒΊ, 90ΒΊ ΠΌΠΎΠ³ΡƒΡ‚ Π±Ρ‹Ρ‚ΡŒ Π»Π΅Π³ΠΊΠΎ вычислСны с использованиСм Π΅Π΄ΠΈΠ½ΠΈΡ‡Π½ΠΎΠΉ окруТности.ΠšΡ€ΠΎΠΌΠ΅ Ρ‚ΠΎΠ³ΠΎ, Π΅Π΄ΠΈΠ½ΠΈΡ‡Π½Ρ‹ΠΉ ΠΊΡ€ΡƒΠ³ ΠΏΠΎΠ»Π΅Π·Π΅Π½ для прСдставлСния комплСксных чисСл Π½Π° плоскости Π°Ρ€Π³Π°Π½Π΄Π°.

ΠšΠ°ΠΊΠΎΠ²Ρ‹ ΠΊΠ²Π°Π΄Ρ€Π°Π½Ρ‚Ρ‹ Π΅Π΄ΠΈΠ½ΠΈΡ‡Π½ΠΎΠ³ΠΎ ΠΊΡ€ΡƒΠ³Π°?

Π•Π΄ΠΈΠ½ΠΈΡ‡Π½Ρ‹ΠΉ ΠΊΡ€ΡƒΠ³ состоит ΠΈΠ· Ρ‡Π΅Ρ‚Ρ‹Ρ€Π΅Ρ… ΠΊΠ²Π°Π΄Ρ€Π°Π½Ρ‚ΠΎΠ², Π°Π½Π°Π»ΠΎΠ³ΠΈΡ‡Π½Ρ‹Ρ… ΠΊΠ²Π°Π΄Ρ€Π°Π½Ρ‚Π°ΠΌ Π² систСмС ΠΊΠΎΠΎΡ€Π΄ΠΈΠ½Π°Ρ‚. Π§Π΅Ρ‚Ρ‹Ρ€Π΅ ΠΊΠ²Π°Π΄Ρ€Π°Π½Ρ‚Π° ΠΈΠΌΠ΅ΡŽΡ‚ Ρ€Π°Π²Π½ΡƒΡŽ ΠΏΠ»ΠΎΡ‰Π°Π΄ΡŒ ΠΈ ΡΠΎΡΡ‚Π°Π²Π»ΡΡŽΡ‚ ΠΎΠ΄Π½Ρƒ Ρ‡Π΅Ρ‚Π²Π΅Ρ€Ρ‚ΡƒΡŽ ΠΏΠ»ΠΎΡ‰Π°Π΄ΠΈ ΠΊΡ€ΡƒΠ³Π°. ΠšΠ°ΠΆΠ΄Ρ‹ΠΉ ΠΈΠ· ΠΊΠ²Π°Π΄Ρ€Π°Π½Ρ‚ΠΎΠ² ΠΎΠ±Ρ€Π°Π·ΡƒΠ΅Ρ‚ ΡƒΠ³ΠΎΠ» 90 Β° ΠΈΠ»ΠΈ прямой ΡƒΠ³ΠΎΠ» Π² Ρ†Π΅Π½Ρ‚Ρ€Π΅ ΠΊΡ€ΡƒΠ³Π°.

Как ΠΎΠΏΠΈΡΠ°Ρ‚ΡŒ Π΅Π΄ΠΈΠ½ΠΈΡ‡Π½Ρ‹ΠΉ ΠΊΡ€ΡƒΠ³ комплСксными числами?

Π•Π΄ΠΈΠ½ΠΈΡ‡Π½Ρ‹ΠΉ ΠΊΡ€ΡƒΠ³ состоит ΠΈΠ· всСх комплСксных чисСл с Π°Π±ΡΠΎΠ»ΡŽΡ‚Π½Ρ‹ΠΌ Π·Π½Π°Ρ‡Π΅Π½ΠΈΠ΅ΠΌ ΠΊΠ°ΠΊ 1, Ρ‚Π°ΠΊΠΈΠΌ ΠΎΠ±Ρ€Π°Π·ΠΎΠΌ, любоС комплСксноС число z = x + \ (i \) y Π±ΡƒΠ΄Π΅Ρ‚ Π»Π΅ΠΆΠ°Ρ‚ΡŒ Π½Π° Π΅Π΄ΠΈΠ½ΠΈΡ‡Π½ΠΎΠΌ ΠΊΡ€ΡƒΠ³Π΅ с ΡƒΡ€Π°Π²Π½Π΅Π½ΠΈΠ΅ΠΌ, Π·Π°Π΄Π°Π½Π½Ρ‹ΠΌ ΠΊΠ°ΠΊ x 2 + y 2 = 1 .Π‘Π»Π΅Π΄ΠΎΠ²Π°Ρ‚Π΅Π»ΡŒΠ½ΠΎ, ΡƒΡ€Π°Π²Π½Π΅Π½ΠΈΠ΅ Π΅Π΄ΠΈΠ½ΠΈΡ‡Π½ΠΎΠΉ окруТности ΠΌΠΎΠΆΠ½ΠΎ ΠΏΡ€Π΅Π΄ΡΡ‚Π°Π²ΠΈΡ‚ΡŒ ΠΊΠ°ΠΊ | z | = 1.

10 сСкрСтных Ρ‚Ρ€ΠΈΠ³Π³Π΅Ρ€Π½Ρ‹Ρ… Ρ„ΡƒΠ½ΠΊΡ†ΠΈΠΉ, ΠΊΠΎΡ‚ΠΎΡ€Ρ‹ΠΌ вас Π½ΠΈΠΊΠΎΠ³Π΄Π° Π½Π΅ ΡƒΡ‡ΠΈΠ»ΠΈ учитСля ΠΌΠ°Ρ‚Π΅ΠΌΠ°Ρ‚ΠΈΠΊΠΈ

Π’ понСдСльник The Onion сообщил, Ρ‡Ρ‚ΠΎ Β«Π½Π°Ρ†ΠΈΠΎΠ½Π°Π»ΡŒΠ½Ρ‹Π΅ учитСля ΠΌΠ°Ρ‚Π΅ΠΌΠ°Ρ‚ΠΈΠΊΠΈ вводят 27 Π½ΠΎΠ²Ρ‹Ρ… Ρ‚Ρ€ΠΈΠ³Π³Π΅Ρ€Π½Ρ‹Ρ… Ρ„ΡƒΠ½ΠΊΡ†ΠΈΠΉΒ». Π­Ρ‚ΠΎ Π·Π°Π±Π°Π²Π½ΠΎΠ΅ Ρ‡Ρ‚Π΅Π½ΠΈΠ΅. Гамсин, Π½Π΅Π³Ρ‚Π°Π½ ΠΈ cosvnx ΠΈΠ· ΡΡ‚Π°Ρ‚ΡŒΠΈ ΠΎ Π›ΡƒΠΊΠ΅ Π²Ρ‹ΠΌΡ‹ΡˆΠ»Π΅Π½Ρ‹, Π½ΠΎ Π² этой ΡΡ‚Π°Ρ‚ΡŒΠ΅ Π΅ΡΡ‚ΡŒ доля ΠΏΡ€Π°Π²Π΄Ρ‹: Π΅ΡΡ‚ΡŒ 10 сСкрСтных Ρ‚Ρ€ΠΈΠ³Π³Π΅Ρ€Π½Ρ‹Ρ… Ρ„ΡƒΠ½ΠΊΡ†ΠΈΠΉ, ΠΎ ΠΊΠΎΡ‚ΠΎΡ€Ρ‹Ρ… Π²Ρ‹ Π½ΠΈΠΊΠΎΠ³Π΄Π° Π½Π΅ ΡΠ»Ρ‹ΡˆΠ°Π»ΠΈ, ΠΈ Ρƒ Π½ΠΈΡ… Π΅ΡΡ‚ΡŒ Π²ΠΎΡΡ…ΠΈΡ‚ΠΈΡ‚Π΅Π»ΡŒΠ½Ρ‹Π΅ названия, Ρ‚Π°ΠΊΠΈΠ΅ ΠΊΠ°ΠΊ «гавСрсинус» ΠΈ «эксСканс».

Π”ΠΈΠ°Π³Ρ€Π°ΠΌΠΌΠ° с Π΅Π΄ΠΈΠ½ΠΈΡ‡Π½Ρ‹ΠΌ ΠΊΡ€ΡƒΠ³ΠΎΠΌ ΠΈ большим количСством Ρ‚Ρ€ΠΈΠ³Π³Π΅Ρ€Π½Ρ‹Ρ… Ρ„ΡƒΠ½ΠΊΡ†ΠΈΠΉ, Ρ‡Π΅ΠΌ Π²Ρ‹ ΠΌΠΎΠΆΠ΅Ρ‚Π΅ потрясти ΠΏΠ°Π»ΠΊΠΎΠΉ.(Π₯ΠΎΡ€ΠΎΡˆΠΎ извСстно, Ρ‡Ρ‚ΠΎ Π²Ρ‹ ΠΌΠΎΠΆΠ΅Ρ‚Π΅ Π²ΡΡ‚Ρ€ΡΡ…Π½ΡƒΡ‚ΡŒ ΠΏΠ°Π»ΠΊΡƒ максимум ΠΏΡ€ΠΈ 8 Ρ‚Ρ€ΠΈΠ³Π³Π΅Ρ€Π½Ρ‹Ρ… функциях.) Π—Π½Π°ΠΊΠΎΠΌΡ‹Π΅ синус, косинус ΠΈ тангСнс Π²Ρ‹Π΄Π΅Π»Π΅Π½Ρ‹ красным, синим ΠΈ, Π½Ρƒ, ΠΆΠ΅Π»Ρ‚ΠΎΠ²Π°Ρ‚ΠΎ-ΠΊΠΎΡ€ΠΈΡ‡Π½Π΅Π²Ρ‹ΠΌ, соотвСтствСнно. ВСрсин Π²Ρ‹Π΄Π΅Π»Π΅Π½ Π·Π΅Π»Π΅Π½Ρ‹ΠΌ рядом с косинусом, Π° эксСканс Ρ€ΠΎΠ·ΠΎΠ²Ρ‹ΠΌ справа ΠΎΡ‚ вСрсина. Excosecant ΠΈ coverine Ρ‚Π°ΠΊΠΆΠ΅ ΠΏΡ€ΠΈΡΡƒΡ‚ΡΡ‚Π²ΡƒΡŽΡ‚ Π½Π° ΠΈΠ·ΠΎΠ±Ρ€Π°ΠΆΠ΅Π½ΠΈΠΈ. НС ΠΈΠ·ΠΎΠ±Ρ€Π°ΠΆΠ΅Π½Ρ‹: Π²Π΅Ρ€ΠΊΠΎΠ·ΠΈΠ½, ΠΊΠ²Π΅Ρ€ΠΊΠΎΠ·ΠΈΠ½ ΠΈ Ρ…Π°Π²Π΅Ρ€ — Ρ‡Ρ‚ΠΎ ΡƒΠ³ΠΎΠ΄Π½ΠΎ. Π˜Π·ΠΎΠ±Ρ€Π°ΠΆΠ΅Π½ΠΈΠ΅: Π›ΠΈΠΌΠ°Π½Π΅Ρ€ ΠΈ Π‘Ρ‚ΠΈΠ²Π΅Π½ ДТонсон, Ρ‡Π΅Ρ€Π΅Π· Wikimedia Commons.

Π₯ΠΎΡ‚ΠΈΡ‚Π΅ Π»ΠΈ Π²Ρ‹ ΠΌΡƒΡ‡ΠΈΡ‚ΡŒ студСнтов с Π½ΠΈΠΌΠΈ ΠΈΠ»ΠΈ Π²ΠΎΠ²Π»Π΅Ρ‡ΡŒ ΠΈΡ… Π² Ρ€Π°Π·Π³ΠΎΠ²ΠΎΡ€, Ρ‡Ρ‚ΠΎΠ±Ρ‹ ΠΏΠΎΠΊΠ°Π·Π°Ρ‚ΡŒΡΡ эрудированным ΠΈ / ΠΈΠ»ΠΈ нСвыносимым, Π²ΠΎΡ‚ опрСдСлСния всСх «потСрянных Ρ‚Ρ€ΠΈΠ³Π³Π΅Ρ€Π½Ρ‹Ρ… Ρ„ΡƒΠ½ΠΊΡ†ΠΈΠΉΒ» , ΠΊΠΎΡ‚ΠΎΡ€Ρ‹Π΅ я нашСл Π² своСм ΠΈΡΡ‡Π΅Ρ€ΠΏΡ‹Π²Π°ΡŽΡ‰Π΅ΠΌ исслСдовании ΠΎΡ€ΠΈΠ³ΠΈΠ½Π°Π»ΡŒΠ½Ρ‹Ρ… историчСских тСкстов ВикипСдия рассказала я ΠΎ.

ВСрсия: вСрсия (ΞΈ) = 1-cos (ΞΈ)
Π’Π΅Ρ€ΠΊΠΎΠ·ΠΈΠ½: Π²Π΅Ρ€ΠΊΠΎΠ·ΠΈΠ½ (ΞΈ) = 1 + cos (ΞΈ)
Coversine: ΠΎΡ…Π²Π°Ρ‚Ρ‹Π²Π°Π΅Ρ‚ (ΞΈ) = 1-sin (ΞΈ)
Covercosine: covercosine (ΞΈ) = 1 + sin (ΞΈ)
. ГавСрсин: гавСрсин (ΞΈ) = вСрсСн (ΞΈ) / 2
Π₯Π°Π²Π΅Ρ€ΠΊΠΎΠ·ΠΈΠ½: Ρ…Π°Π²Π΅Ρ€ΠΊΠΎΠ·ΠΈΠ½ (ΞΈ) = Π²Π΅Ρ€ΠΊΠΎΠ·ΠΈΠ½ (ΞΈ) / 2
Π₯аковСрсин: hacoversin (ΞΈ) = Coversin (ΞΈ) / 2
Π“Π°ΠΊΠΎΠ²Π΅Ρ€ΠΊΠΎΠ·ΠΈΠ½: Π³Π°ΠΊΠΎΠ²Π΅Ρ€ΠΊΠΎΠ·ΠΈΠ½ (ΞΈ) = ΠΊΠ²Π΅Ρ€ΠΊΠΎΠ·ΠΈΠ½ (ΞΈ) / 2
Exsecant: exsec (ΞΈ) = sec (ΞΈ) -1
Excosecant: excsc (ΞΈ) = csc (ΞΈ) -1

Π”ΠΎΠ»ΠΆΠ΅Π½ ΠΏΡ€ΠΈΠ·Π½Π°Ρ‚ΡŒΡΡ, я Π±Ρ‹Π» Π½Π΅ΠΌΠ½ΠΎΠ³ΠΎ Ρ€Π°Π·ΠΎΡ‡Π°Ρ€ΠΎΠ²Π°Π½, ΠΊΠΎΠ³Π΄Π° посмотрСл Π½Π° Π½ΠΈΡ….ВсС ΠΎΠ½ΠΈ ΠΏΡ€Π΅Π΄ΡΡ‚Π°Π²Π»ΡΡŽΡ‚ собой простыС ΠΊΠΎΠΌΠ±ΠΈΠ½Π°Ρ†ΠΈΠΈ Π΄ΠΎΡ€ΠΎΠ³ΠΎΠ³ΠΎ старого синуса ΠΈ косинуса. ΠŸΠΎΡ‡Π΅ΠΌΡƒ ΠΎΠ½ΠΈ Π²ΠΎΠΎΠ±Ρ‰Π΅ ΠΏΠΎΠ»ΡƒΡ‡ΠΈΠ»ΠΈ ΠΈΠΌΠ΅Π½Π° ?! Из Ρ‚ΠΎΠ³ΠΎ мСста ΠΈ Π²Ρ€Π΅ΠΌΠ΅Π½ΠΈ, ΠΊΠΎΠ³Π΄Π° я ΠΌΠΎΠ³Ρƒ ΡΠΈΠ΄Π΅Ρ‚ΡŒ Π½Π° Π΄ΠΈΠ²Π°Π½Π΅ ΠΈ ΠΏΠΎΡ‡Ρ‚ΠΈ ΠΌΠ³Π½ΠΎΠ²Π΅Π½Π½ΠΎ Π½Π°Ρ…ΠΎΠ΄ΠΈΡ‚ΡŒ синус любого ΡƒΠ³Π»Π° с Ρ‚ΠΎΡ‡Π½ΠΎΡΡ‚ΡŒΡŽ Π΄ΠΎ 100 дСсятичных Π·Π½Π°ΠΊΠΎΠ² с ΠΏΠΎΠΌΠΎΡ‰ΡŒΡŽ ΠΎΠ½Π»Π°ΠΉΠ½-ΠΊΠ°Π»ΡŒΠΊΡƒΠ»ΡΡ‚ΠΎΡ€Π°, Π² ВСрсинС Π½Π΅Ρ‚ нСобходимости. Но эти, казалось Π±Ρ‹, лишниС Ρ„ΡƒΠ½ΠΊΡ†ΠΈΠΈ восполнили потрСбности Π² ΠΌΠΈΡ€Π΅ ΠΏΡ€Π΅Π΄Π²Π°Ρ€ΠΈΡ‚Π΅Π»ΡŒΠ½Ρ‹Ρ… ΠΊΠ°Π»ΡŒΠΊΡƒΠ»ΡΡ‚ΠΎΡ€ΠΎΠ².

Numberphile Π½Π΅Π΄Π°Π²Π½ΠΎ ΠΎΠΏΡƒΠ±Π»ΠΈΠΊΠΎΠ²Π°Π» Π²ΠΈΠ΄Π΅ΠΎ ΠΎ Ρ‚Π°Π±Π»ΠΈΡ†Π°Ρ… ΠΆΡƒΡ€Π½Π°Π»ΠΎΠ², Π² ΠΊΠΎΡ‚ΠΎΡ€ΠΎΠΌ ΠΎΠ±ΡŠΡΡΠ½ΡΠ΅Ρ‚ΡΡ, ΠΊΠ°ΠΊ люди использовали Π»ΠΎΠ³Π°Ρ€ΠΈΡ„ΠΌΡ‹ для умноТСния Π±ΠΎΠ»ΡŒΡˆΠΈΡ… чисСл Π² Ρ‚Π΅ΠΌΠ½Ρ‹Π΅ Π΄Π½ΠΈ, ΠΊΠΎΠ³Π΄Π° Π΅Ρ‰Π΅ Π½Π΅ Π±Ρ‹Π»ΠΎ ΠΊΠ°Π»ΡŒΠΊΡƒΠ»ΡΡ‚ΠΎΡ€Π°.Π’ΠΎ-ΠΏΠ΅Ρ€Π²Ρ‹Ρ…, напомню ΠΎ Π»ΠΎΠ³Π°Ρ€ΠΈΡ„ΠΌΠ°Ρ…. Π£Ρ€Π°Π²Π½Π΅Π½ΠΈΠ΅ log b x = y ΠΎΠ·Π½Π°Ρ‡Π°Π΅Ρ‚, Ρ‡Ρ‚ΠΎ b y = x. НапримСр, 10 2 = 100, поэтому log 10 100 = 2. ΠŸΠΎΠ»Π΅Π·Π½Ρ‹ΠΉ Ρ„Π°ΠΊΡ‚ ΠΎ Π»ΠΎΠ³Π°Ρ€ΠΈΡ„ΠΌΠ°Ρ… Π·Π°ΠΊΠ»ΡŽΡ‡Π°Π΅Ρ‚ΡΡ Π² Ρ‚ΠΎΠΌ, Ρ‡Ρ‚ΠΎ log b (c Γ— d) = log b c + log b d. Π”Ρ€ΡƒΠ³ΠΈΠΌΠΈ словами, Π»ΠΎΠ³Π°Ρ€ΠΈΡ„ΠΌΡ‹ ΠΏΡ€Π΅Π²Ρ€Π°Ρ‰Π°ΡŽΡ‚ ΡƒΠΌΠ½ΠΎΠΆΠ΅Π½ΠΈΠ΅ Π² слоТСниС. Если Π²Ρ‹ Ρ…ΠΎΡ‚ΠΈΡ‚Π΅ ΡƒΠΌΠ½ΠΎΠΆΠΈΡ‚ΡŒ Π΄Π²Π° числа вмСстС с ΠΏΠΎΠΌΠΎΡ‰ΡŒΡŽ Ρ‚Π°Π±Π»ΠΈΡ†Ρ‹ ΠΆΡƒΡ€Π½Π°Π»Π°, Π²Ρ‹ Π΄ΠΎΠ»ΠΆΠ½Ρ‹ Π½Π°ΠΉΡ‚ΠΈ Π»ΠΎΠ³Π°Ρ€ΠΈΡ„ΠΌ ΠΎΠ±ΠΎΠΈΡ… чисСл, Π° Π·Π°Ρ‚Π΅ΠΌ ΡΠ»ΠΎΠΆΠΈΡ‚ΡŒ Π»ΠΎΠ³Π°Ρ€ΠΈΡ„ΠΌΡ‹. Π—Π°Ρ‚Π΅ΠΌ Π²Ρ‹ Π΄ΠΎΠ»ΠΆΠ½Ρ‹ ΠΈΡΠΏΠΎΠ»ΡŒΠ·ΠΎΠ²Π°Ρ‚ΡŒ свою Ρ‚Π°Π±Π»ΠΈΡ†Ρƒ ΠΆΡƒΡ€Π½Π°Π»Π°, Ρ‡Ρ‚ΠΎΠ±Ρ‹ ΡƒΠ·Π½Π°Ρ‚ΡŒ, ΠΊΠ°ΠΊΠΎΠ΅ число ΠΈΠΌΠ΅Π΅Ρ‚ этот Π»ΠΎΠ³Π°Ρ€ΠΈΡ„ΠΌ, ΠΈ это Π±Ρ‹Π» ваш ΠΎΡ‚Π²Π΅Ρ‚.БСйчас это Π·Π²ΡƒΡ‡ΠΈΡ‚ Π³Ρ€ΠΎΠΌΠΎΠ·Π΄ΠΊΠΎ, Π½ΠΎ ΡƒΠΌΠ½ΠΎΠΆΠ΅Π½ΠΈΠ΅ Π²Ρ€ΡƒΡ‡Π½ΡƒΡŽ Ρ‚Ρ€Π΅Π±ΡƒΠ΅Ρ‚ Π³ΠΎΡ€Π°Π·Π΄ΠΎ большС ΠΎΠΏΠ΅Ρ€Π°Ρ†ΠΈΠΉ, Ρ‡Π΅ΠΌ слоТСниС. Когда каТдая опСрация Π·Π°Π½ΠΈΠΌΠ°Π΅Ρ‚ Π½Π΅Ρ‚Ρ€ΠΈΠ²ΠΈΠ°Π»ΡŒΠ½ΠΎΠ΅ количСство Π²Ρ€Π΅ΠΌΠ΅Π½ΠΈ (ΠΈ ΠΏΠΎΠ΄Π²Π΅Ρ€ΠΆΠ΅Π½Π° Π½Π΅Ρ‚Ρ€ΠΈΠ²ΠΈΠ°Π»ΡŒΠ½ΠΎΠΌΡƒ количСству ошибок), ΠΏΡ€ΠΎΡ†Π΅Π΄ΡƒΡ€Π°, ΠΏΠΎΠ·Π²ΠΎΠ»ΡΡŽΡ‰Π°Ρ ΠΏΡ€Π΅ΠΎΠ±Ρ€Π°Π·ΠΎΠ²Π°Ρ‚ΡŒ ΡƒΠΌΠ½ΠΎΠΆΠ΅Π½ΠΈΠ΅ Π² слоТСниС, Π² Ρ€Π΅Π°Π»ΡŒΠ½ΠΎΠΌ Π²Ρ€Π΅ΠΌΠ΅Π½ΠΈ экономит врСмя ΠΈ ΠΏΠΎΠΌΠΎΠ³Π°Π΅Ρ‚ ΠΏΠΎΠ²Ρ‹ΡΠΈΡ‚ΡŒ Ρ‚ΠΎΡ‡Π½ΠΎΡΡ‚ΡŒ.

Π‘Π΅ΠΊΡ€Π΅Ρ‚Π½Ρ‹Π΅ Ρ‚Ρ€ΠΈΠ³Π³Π΅Ρ€Π½Ρ‹Π΅ Ρ„ΡƒΠ½ΠΊΡ†ΠΈΠΈ, Ρ‚Π°ΠΊΠΈΠ΅ ΠΊΠ°ΠΊ Π»ΠΎΠ³Π°Ρ€ΠΈΡ„ΠΌΡ‹, ΡƒΠΏΡ€ΠΎΡ‰Π°ΡŽΡ‚ вычислСния. Π§Π°Ρ‰Π΅ всСго использовались вСрсин ΠΈ гавСрсин. Π’Π±Π»ΠΈΠ·ΠΈ ΡƒΠ³Π»Π° ΞΈ = 0 cos (ΞΈ) ΠΎΡ‡Π΅Π½ΡŒ Π±Π»ΠΈΠ·ΠΊΠΎ ΠΊ 1.Если Π²Ρ‹ выполняли вычислСниС, Π² ΠΊΠΎΡ‚ΠΎΡ€ΠΎΠΌ Π±Ρ‹Π»ΠΎ 1-cos (ΞΈ), вашС вычислСниС ΠΌΠΎΠ³Π»ΠΎ Π±Ρ‹ Π±Ρ‹Ρ‚ΡŒ Ρ€Π°Π·Ρ€ΡƒΡˆΠ΅Π½ΠΎ, Ссли Π² вашСй Ρ‚Π°Π±Π»ΠΈΡ†Π΅ косинусов Π½Π΅ Π±Ρ‹Π»ΠΎ достаточно Π·Π½Π°Ρ‡Π°Ρ‰ΠΈΡ… Ρ†ΠΈΡ„Ρ€. Для ΠΈΠ»Π»ΡŽΡΡ‚Ρ€Π°Ρ†ΠΈΠΈ косинус 5 градусов Ρ€Π°Π²Π΅Π½ 0,996194698, Π° косинус 1 градуса Ρ€Π°Π²Π΅Π½ 0,999847695. Π Π°Π·Π½ΠΈΡ†Π° cos (1 Β°) -cos (5 Β°) составляСт 0,003652997. Если Π±Ρ‹ Ρƒ вас Π±Ρ‹Π»ΠΎ Ρ‚Ρ€ΠΈ Π·Π½Π°Ρ‡Π°Ρ‰ΠΈΡ… Ρ†ΠΈΡ„Ρ€Ρ‹ Π² вашСй Ρ‚Π°Π±Π»ΠΈΡ†Π΅ косинусов, Π²Ρ‹ Π±Ρ‹ ΠΏΠΎΠ»ΡƒΡ‡ΠΈΠ»ΠΈ Ρ‚ΠΎΠ»ΡŒΠΊΠΎ ΠΎΠ΄Π½Ρƒ Π·Π½Π°Ρ‡Π°Ρ‰ΡƒΡŽ Ρ†ΠΈΡ„Ρ€Ρƒ Π² своСм ΠΎΡ‚Π²Π΅Ρ‚Π΅ ΠΈΠ·-Π·Π° Π²Π΅Π΄ΡƒΡ‰ΠΈΡ… Π½ΡƒΠ»Π΅ΠΉ Π² Ρ€Π°Π·Π½ΠΈΡ†Π΅. А Ρ‚Π°Π±Π»ΠΈΡ†Π° Ρ‚ΠΎΠ»ΡŒΠΊΠΎ с трСмя Π·Π½Π°Ρ‡Π°Ρ‰ΠΈΠΌΠΈ Ρ†ΠΈΡ„Ρ€Π°ΠΌΠΈ точности Π½Π΅ смоТСт Ρ€Π°Π·Π»ΠΈΡ‡ΠΈΡ‚ΡŒ ΡƒΠ³Π»Ρ‹ 0 ΠΈ 1 градус.Π’ΠΎ ΠΌΠ½ΠΎΠ³ΠΈΡ… случаях это Π½Π΅ ΠΈΠΌΠ΅Π΅Ρ‚ значСния, Π½ΠΎ ΠΌΠΎΠΆΠ΅Ρ‚ ΡΡ‚Π°Ρ‚ΡŒ ΠΏΡ€ΠΎΠ±Π»Π΅ΠΌΠΎΠΉ, Ссли ошибки Π½Π°ΠΊΠ°ΠΏΠ»ΠΈΠ²Π°ΡŽΡ‚ΡΡ Π² Ρ…ΠΎΠ΄Π΅ вычислСний.

БонусныС Ρ‚Ρ€ΠΈΠ³Π³Π΅Ρ€Π½Ρ‹Π΅ Ρ„ΡƒΠ½ΠΊΡ†ΠΈΠΈ Ρ‚Π°ΠΊΠΆΠ΅ ΠΈΠΌΠ΅ΡŽΡ‚ Ρ‚ΠΎ прСимущСство, Ρ‡Ρ‚ΠΎ ΠΎΠ½ΠΈ Π½ΠΈΠΊΠΎΠ³Π΄Π° Π½Π΅ Π±Ρ‹Π²Π°ΡŽΡ‚ ΠΎΡ‚Ρ€ΠΈΡ†Π°Ρ‚Π΅Π»ΡŒΠ½Ρ‹ΠΌΠΈ. ВСрсина находится Π² Π΄ΠΈΠ°ΠΏΠ°Π·ΠΎΠ½Π΅ ΠΎΡ‚ 0 Π΄ΠΎ 2, поэтому, Ссли Π²Ρ‹ ΠΈΡΠΏΠΎΠ»ΡŒΠ·ΡƒΠ΅Ρ‚Π΅ Ρ‚Π°Π±Π»ΠΈΡ†Ρ‹ ΠΆΡƒΡ€Π½Π°Π»Π° для умноТСния Π½Π° вСрсину, Π²Π°ΠΌ Π½Π΅ Π½ΡƒΠΆΠ½ΠΎ Π±Π΅ΡΠΏΠΎΠΊΠΎΠΈΡ‚ΡŒΡΡ ΠΎ Ρ‚ΠΎΠΌ, Ρ‡Ρ‚ΠΎ Π»ΠΎΠ³Π°Ρ€ΠΈΡ„ΠΌ Π½Π΅ ΠΎΠΏΡ€Π΅Π΄Π΅Π»Π΅Π½ для ΠΎΡ‚Ρ€ΠΈΡ†Π°Ρ‚Π΅Π»ΡŒΠ½Ρ‹Ρ… чисСл. (Он Ρ‚Π°ΠΊΠΆΠ΅ Π½Π΅ ΠΎΠΏΡ€Π΅Π΄Π΅Π»Π΅Π½ для 0, Π½ΠΎ с этим Π»Π΅Π³ΠΊΠΎ ΠΈΠΌΠ΅Ρ‚ΡŒ Π΄Π΅Π»ΠΎ.) Π•Ρ‰Π΅ ΠΎΠ΄Π½ΠΎ прСимущСство вСрсин ΠΈ гавСрсин состоит Π² Ρ‚ΠΎΠΌ, Ρ‡Ρ‚ΠΎ ΠΎΠ½ΠΈ ΠΌΠΎΠ³ΡƒΡ‚ ΡƒΠ΄Π΅Ρ€ΠΆΠΈΠ²Π°Ρ‚ΡŒ вас ΠΎΡ‚ нСобходимости Ρ‡Ρ‚ΠΎ-Ρ‚ΠΎ Π²ΠΎΠ·Π²ΠΎΠ΄ΠΈΡ‚ΡŒ Π² ΠΊΠ²Π°Π΄Ρ€Π°Ρ‚.НСмного тригономСтричСского Π²ΠΎΠ»ΡˆΠ΅Π±ΡΡ‚Π²Π° (Ρ‚Π°ΠΊΠΆΠ΅ извСстного ΠΊΠ°ΠΊ Π·Π°ΠΏΠΎΠΌΠΈΠ½Π°Π½ΠΈΠ΅ ΠΎΠ΄Π½ΠΎΠΉ ΠΈΠ· бСсконСчного списка тригономСтричСских Ρ„ΠΎΡ€ΠΌΡƒΠ», ΠΊΠΎΡ‚ΠΎΡ€Ρ‹Π΅ Π²Ρ‹ Π²Ρ‹ΡƒΡ‡ΠΈΠ»ΠΈ Π² ΡΡ‚Π°Ρ€ΡˆΠ΅ΠΉ школС) ΠΏΠΎΠΊΠ°Π·Ρ‹Π²Π°Π΅Ρ‚, Ρ‡Ρ‚ΠΎ 1-cos (ΞΈ) = 2sin 2 (ΞΈ / 2). Π’Π°ΠΊΠΈΠΌ ΠΎΠ±Ρ€Π°Π·ΠΎΠΌ, гавСрсинус — это просто Π³Ρ€Π΅Ρ… 2 (ΞΈ / 2). Π’ΠΎΡ‡Π½ΠΎ Ρ‚Π°ΠΊ ΠΆΠ΅ Π³Π°Π²Π΅Ρ€ΠΊΠΎΠ·ΠΈΠ½ Ρ€Π°Π²Π΅Π½ cos 2 (ΞΈ / 2). Если Ρƒ вас Π΅ΡΡ‚ΡŒ вычислСниС с использованиСм ΠΊΠ²Π°Π΄Ρ€Π°Ρ‚Π° синуса ΠΈΠ»ΠΈ косинуса, Π²Ρ‹ ΠΌΠΎΠΆΠ΅Ρ‚Π΅ ΠΈΡΠΏΠΎΠ»ΡŒΠ·ΠΎΠ²Π°Ρ‚ΡŒ Ρ‚Π°Π±Π»ΠΈΡ†Ρƒ гавСрсинусов ΠΈΠ»ΠΈ гавСрсинусов, ΠΈ Π²Π°ΠΌ Π½Π΅ Π½ΡƒΠΆΠ½ΠΎ Π²ΠΎΠ·Π²ΠΎΠ΄ΠΈΡ‚ΡŒ Π² ΠΊΠ²Π°Π΄Ρ€Π°Ρ‚ ΠΈΠ»ΠΈ ΠΈΠ·Π²Π»Π΅ΠΊΠ°Ρ‚ΡŒ ΠΊΠ²Π°Π΄Ρ€Π°Ρ‚Π½Ρ‹Π΅ ΠΊΠΎΡ€Π½ΠΈ.

Π”ΠΈΠ°Π³Ρ€Π°ΠΌΠΌΠ°, ΠΏΠΎΠΊΠ°Π·Ρ‹Π²Π°ΡŽΡ‰Π°Ρ синус, косинус ΠΈ Π²Π΅Ρ€ΡΠΈΡŽ ΡƒΠ³Π»Π°.Π˜Π·ΠΎΠ±Ρ€Π°ΠΆΠ΅Π½ΠΈΠ΅: Qef ΠΈ Π‘Ρ‚ΠΈΠ²Π΅Π½ Π”ΠΆ. ДТонсон, Ρ‡Π΅Ρ€Π΅Π· Wikimedia Commons.

ВСрсина — это довольно очСвидная триггСрная функция для опрСдСлСния ΠΈ, каТСтся, использовалась Π΅Ρ‰Π΅ Π² 400 Π³. Π½.э. Π² Индии. Но гавСрсинус ΠΌΠΎΠ³ Π±Ρ‹Ρ‚ΡŒ Π±ΠΎΠ»Π΅Π΅ Π²Π°ΠΆΠ½Ρ‹ΠΌ Π² Π±ΠΎΠ»Π΅Π΅ Π½Π΅Π΄Π°Π²Π½Π΅ΠΉ истории, ΠΊΠΎΠ³Π΄Π° ΠΎΠ½ использовался Π² Π½Π°Π²ΠΈΠ³Π°Ρ†ΠΈΠΈ. Π€ΠΎΡ€ΠΌΡƒΠ»Π° гавСрсинуса — ΠΎΡ‡Π΅Π½ΡŒ Ρ‚ΠΎΡ‡Π½Ρ‹ΠΉ способ вычислСния расстояний ΠΌΠ΅ΠΆΠ΄Ρƒ двумя Ρ‚ΠΎΡ‡ΠΊΠ°ΠΌΠΈ Π½Π° повСрхности сфСры с использованиСм ΡˆΠΈΡ€ΠΎΡ‚Ρ‹ ΠΈ Π΄ΠΎΠ»Π³ΠΎΡ‚Ρ‹ этих Π΄Π²ΡƒΡ… Ρ‚ΠΎΡ‡Π΅ΠΊ. Π€ΠΎΡ€ΠΌΡƒΠ»Π° гавСрсинуса — это ΠΏΠ΅Ρ€Π΅Ρ„ΠΎΡ€ΠΌΡƒΠ»ΠΈΡ€ΠΎΠ²ΠΊΠ° сфСричСского Π·Π°ΠΊΠΎΠ½Π° косинусов, Π½ΠΎ Ρ„ΠΎΡ€ΠΌΡƒΠ»ΠΈΡ€ΠΎΠ²ΠΊΠ° Π² Ρ‚Π΅Ρ€ΠΌΠΈΠ½Π°Ρ… гавСрсинусов Π±ΠΎΠ»Π΅Π΅ ΠΏΠΎΠ»Π΅Π·Π½Π° для ΠΌΠ°Π»Ρ‹Ρ… ΡƒΠ³Π»ΠΎΠ² ΠΈ расстояний.(Π‘ Π΄Ρ€ΡƒΠ³ΠΎΠΉ стороны, Ρ„ΠΎΡ€ΠΌΡƒΠ»Π° гавСрсинуса Π½Π΅ ΠΎΡ‡Π΅Π½ΡŒ Ρ…ΠΎΡ€ΠΎΡˆΠΎ справляСтся с ΡƒΠ³Π»Π°ΠΌΠΈ, Π±Π»ΠΈΠ·ΠΊΠΈΠΌΠΈ ΠΊ 90 градусам, Π½ΠΎ сфСричСский Π·Π°ΠΊΠΎΠ½ косинусов справляСтся с этим Ρ…ΠΎΡ€ΠΎΡˆΠΎ.) Π€ΠΎΡ€ΠΌΡƒΠ»Π° гавСрсинуса ΠΌΠΎΠΆΠ΅Ρ‚ Π΄Π°Ρ‚ΡŒ Ρ‚ΠΎΡ‡Π½Ρ‹Π΅ Ρ€Π΅Π·ΡƒΠ»ΡŒΡ‚Π°Ρ‚Ρ‹, Π½Π΅ трСбуя дорогостоящих вычислСний. ΠΊΠ²Π°Π΄Ρ€Π°Ρ‚Ρ‹ ΠΈ ΠΊΠ²Π°Π΄Ρ€Π°Ρ‚Π½Ρ‹Π΅ ΠΊΠΎΡ€Π½ΠΈ. Π•Ρ‰Π΅ Π² 1984 Π³ΠΎΠ΄Ρƒ Π»ΡŽΠ±ΠΈΡ‚Π΅Π»ΡŒΡΠΊΠΈΠΉ астрономичСский ΠΆΡƒΡ€Π½Π°Π» Sky & Telescope восхвалял Ρ„ΠΎΡ€ΠΌΡƒΠ»Ρƒ гавСрсинуса, которая ΠΏΠΎΠ»Π΅Π·Π½Π° Π½Π΅ Ρ‚ΠΎΠ»ΡŒΠΊΠΎ для Π½Π°Π·Π΅ΠΌΠ½ΠΎΠΉ Π½Π°Π²ΠΈΠ³Π°Ρ†ΠΈΠΈ, Π½ΠΎ ΠΈ для астрономичСских расчСтов.Π§Ρ‚ΠΎΠ±Ρ‹ ΡƒΠ·Π½Π°Ρ‚ΡŒ большС ΠΎ Ρ„ΠΎΡ€ΠΌΡƒΠ»Π΅ гавСрсинуса ΠΈ вычислСнии расстояний Π½Π° сфСрС, ΠΎΠ·Π½Π°ΠΊΠΎΠΌΡŒΡ‚Π΅ΡΡŒ с этой Π°Ρ€Ρ…ΠΈΠ²Π½ΠΎΠΉ ΠΊΠΎΠΏΠΈΠ΅ΠΉ страницы Π±ΡŽΡ€ΠΎ пСрСписи ΠΈΠ»ΠΈ этой ΡΡ‚Π°Ρ‚ΡŒΠ΅ΠΉ «БпроситС Π΄ΠΎΠΊΡ‚ΠΎΡ€Π° ΠœΠ°Ρ‚Π΅ΠΌΠ°Β».

Π£ мСня Π½Π΅ Ρ‚Π°ΠΊ ΠΌΠ½ΠΎΠ³ΠΎ ΠΈΠ½Ρ„ΠΎΡ€ΠΌΠ°Ρ†ΠΈΠΈ ΠΎΠ± истории Π΄Ρ€ΡƒΠ³ΠΈΡ… Ρ‚Ρ€ΠΈΠ³Π³Π΅Ρ€Π½Ρ‹Ρ… Ρ„ΡƒΠ½ΠΊΡ†ΠΈΠΉ Π² спискС. ВсС ΠΎΠ½ΠΈ ΠΌΠΎΠ³ΡƒΡ‚ ΡΠ΄Π΅Π»Π°Ρ‚ΡŒ вычислСния Π±ΠΎΠ»Π΅Π΅ Ρ‚ΠΎΡ‡Π½Ρ‹ΠΌΠΈ Π²Π±Π»ΠΈΠ·ΠΈ ΠΎΠΏΡ€Π΅Π΄Π΅Π»Π΅Π½Π½Ρ‹Ρ… ΡƒΠ³Π»ΠΎΠ², Π½ΠΎ я Π½Π΅ знаю, ΠΊΠ°ΠΊΠΈΠ΅ ΠΈΠ· Π½ΠΈΡ… ΠΎΠ±Ρ‹Ρ‡Π½ΠΎ использовались, Π° ΠΊΠ°ΠΊΠΈΠ΅ Π½Π°Π·Ρ‹Π²Π°Π»ΠΈΡΡŒ * Π°Π½Π°Π»ΠΎΠ³ΠΈΡ‡Π½ΠΎ Π΄Ρ€ΡƒΠ³ΠΈΠΌ функциям, Π½ΠΎ Ρ€Π΅Π΄ΠΊΠΎ использовались Π½Π° самом Π΄Π΅Π»Π΅. МнС Π»ΡŽΠ±ΠΎΠΏΡ‹Ρ‚Π½ΠΎ ΠΎΠ± этом, Ссли ΠΊΡ‚ΠΎ-Π½ΠΈΠ±ΡƒΠ΄ΡŒ Π·Π½Π°Π΅Ρ‚ большС ΠΎ ΠΏΡ€Π΅Π΄ΠΌΠ΅Ρ‚Π΅.

Когда Π›ΡƒΠΊΠΎΠ²ΠΈΡ†Π° ΠΈΠΌΠΈΡ‚ΠΈΡ€ΡƒΠ΅Ρ‚ Ρ€Π΅Π°Π»ΡŒΠ½ΡƒΡŽ Тизнь, это ΠΎΠ±Ρ‹Ρ‡Π½ΠΎ Ρ‚Ρ€Π°Π³ΠΈΡ‡Π½ΠΎ. Но Π² случаС сСкрСтных Ρ‚Ρ€ΠΈΠ³Π³Π΅Ρ€Π½Ρ‹Ρ… Ρ„ΡƒΠ½ΠΊΡ†ΠΈΠΉ ΡΡƒΡ‚ΡŒ ΠΏΡ€Π°Π²Π΄Ρ‹ Π² Π›ΡƒΠΊΠ΅ мСня Π½Π΅ ΠΎΠ³ΠΎΡ€Ρ‡ΠΈΠ»Π°. Нам ΠΎΡ‡Π΅Π½ΡŒ ΠΏΠΎΠ²Π΅Π·Π»ΠΎ, Ρ‡Ρ‚ΠΎ Ρ‚Π΅ΠΏΠ΅Ρ€ΡŒ ΠΌΡ‹ ΠΌΠΎΠΆΠ΅ΠΌ Ρ‚Π°ΠΊ Π»Π΅Π³ΠΊΠΎ ΡƒΠΌΠ½ΠΎΠΆΠ°Ρ‚ΡŒ, Π²ΠΎΠ·Π²ΠΎΠ΄ΠΈΡ‚ΡŒ Π² ΠΊΠ²Π°Π΄Ρ€Π°Ρ‚ ΠΈ ΠΈΠ·Π²Π»Π΅ΠΊΠ°Ρ‚ΡŒ ΠΊΠ²Π°Π΄Ρ€Π°Ρ‚Π½Ρ‹Π΅ ΠΊΠΎΡ€Π½ΠΈ, Π° наши ΠΊΠ°Π»ΡŒΠΊΡƒΠ»ΡΡ‚ΠΎΡ€Ρ‹ ΠΌΠΎΠ³ΡƒΡ‚ Ρ…Ρ€Π°Π½ΠΈΡ‚ΡŒ Ρ‚ΠΎΡ‡Π½ΡƒΡŽ ΠΈΠ½Ρ„ΠΎΡ€ΠΌΠ°Ρ†ΠΈΡŽ ΠΎ синусах, косинусах ΠΈ тангСнсах ΡƒΠ³Π»ΠΎΠ², Π½ΠΎ ΠΏΡ€Π΅ΠΆΠ΄Π΅ Ρ‡Π΅ΠΌ ΠΌΡ‹ смогли это ΡΠ΄Π΅Π»Π°Ρ‚ΡŒ, ΠΌΡ‹ ΠΏΡ€ΠΈΠ΄ΡƒΠΌΠ°Π»ΠΈ Ρ€Π°Π±ΠΎΡ‚Ρƒ -ΠΎΠΊΡ€ΡƒΠ³ Π² Π²ΠΈΠ΄Π΅ смСшного количСства Ρ‚Ρ€ΠΈΠ³Π³Π΅Ρ€Π½Ρ‹Ρ… Ρ„ΡƒΠ½ΠΊΡ†ΠΈΠΉ. Π›Π΅Π³ΠΊΠΎ Π·Π°Π±Ρ‹Ρ‚ΡŒ, Ρ‡Ρ‚ΠΎ люди, ΠΊΠΎΡ‚ΠΎΡ€Ρ‹Π΅ ΠΈΡ… ΠΎΠΏΡ€Π΅Π΄Π΅Π»ΠΈΠ»ΠΈ, Π½Π΅ Π±Ρ‹Π»ΠΈ садистскими учитСлями ΠΌΠ°Ρ‚Π΅ΠΌΠ°Ρ‚ΠΈΠΊΠΈ, ΠΊΠΎΡ‚ΠΎΡ€Ρ‹Π΅ хотят, Ρ‡Ρ‚ΠΎΠ±Ρ‹ люди Π·Π°ΠΏΠΎΠΌΠΈΠ½Π°Π»ΠΈ странныС Ρ„ΡƒΠ½ΠΊΡ†ΠΈΠΈ Π±Π΅Π· всякой ΠΏΡ€ΠΈΡ‡ΠΈΠ½Ρ‹.Π­Ρ‚ΠΈ Ρ„ΡƒΠ½ΠΊΡ†ΠΈΠΈ фактичСски сдСлали вычислСния Π±ΠΎΠ»Π΅Π΅ быстрыми ΠΈ ΠΌΠ΅Π½Π΅Π΅ ΠΏΠΎΠ΄Π²Π΅Ρ€ΠΆΠ΅Π½Π½Ρ‹ΠΌΠΈ ошибкам. Π’Π΅ΠΏΠ΅Ρ€ΡŒ, ΠΊΠΎΠ³Π΄Π° ΠΊΠΎΠΌΠΏΡŒΡŽΡ‚Π΅Ρ€Ρ‹ стали Π½Π°ΡΡ‚ΠΎΠ»ΡŒΠΊΠΎ ΠΌΠΎΡ‰Π½Ρ‹ΠΌΠΈ, ΠΏΡ€ΠΈΠ²Ρ‹Ρ‡ΠΊΠ° ΠΊ дискСтам ΡƒΠ»Π΅Ρ‚ΡƒΡ‡ΠΈΠ»Π°ΡΡŒ. Но я Π΄ΡƒΠΌΠ°ΡŽ, ΠΌΡ‹ всС ΠΌΠΎΠΆΠ΅ΠΌ ΡΠΎΠ³Π»Π°ΡΠΈΡ‚ΡŒΡΡ с Ρ‚Π΅ΠΌ, Ρ‡Ρ‚ΠΎ ΠΎΠ½ Π΄ΠΎΠ»ΠΆΠ΅Π½ Π²Π΅Ρ€Π½ΡƒΡ‚ΡŒΡΡ, хотя Π±Ρ‹ ΠΈΠ·-Π·Π° «классной» ΡˆΡƒΡ‚ΠΊΠΈ, ΠΊΠΎΡ‚ΠΎΡ€ΡƒΡŽ я ΠΏΡ€ΠΈΠ΄ΡƒΠΌΠ°Π», ΠΊΠΎΠ³Π΄Π° засыпал ΠΏΡ€ΠΎΡˆΠ»ΠΎΠΉ Π½ΠΎΡ‡ΡŒΡŽ: Π₯авСрсин? Π― Π΄Π°ΠΆΠ΅ Π½Π΅ знаю!

* Π― Ρ…ΠΎΡ‚Π΅Π» Π±Ρ‹ ΡΠ΄Π΅Π»Π°Ρ‚ΡŒ здСсь нСбольшоС отступлСниС Π² ΠΌΠΈΡ€ матСматичСских прСфиксов, Π½ΠΎ это ΠΌΠΎΠΆΠ΅Ρ‚ Π±Ρ‹Ρ‚ΡŒ Π½Π΅ для всСх. Вас ΠΏΡ€Π΅Π΄ΡƒΠΏΡ€Π΅Π΄ΠΈΠ»ΠΈ.

Π’ Ρ‚Π°Π±Π»ΠΈΡ†Π΅ сСкрСтных Ρ‚Ρ€ΠΈΠ³Π³Π΅Ρ€Π½Ρ‹Ρ… Ρ„ΡƒΠ½ΠΊΡ†ΠΈΠΉ Β«Ρ…Π°Β» явно ΠΎΠ·Π½Π°Ρ‡Π°Π΅Ρ‚ ΠΏΠΎΠ»ΠΎΠ²ΠΈΠ½Ρƒ; НапримСр, Ρ†Π΅Π½Π½ΠΎΡΡ‚ΡŒ гавСрсина составляСт ΠΏΠΎΠ»ΠΎΠ²ΠΈΠ½Ρƒ стоимости вСрсина.Β«Π‘ΠΎΒ» ΠΎΠ·Π½Π°Ρ‡Π°Π΅Ρ‚ Π²Ρ‹ΠΏΠΎΠ»Π½Π΅Π½ΠΈΠ΅ Ρ‚ΠΎΠΉ ΠΆΠ΅ Ρ„ΡƒΠ½ΠΊΡ†ΠΈΠΈ, Π½ΠΎ с Π΄ΠΎΠΏΠΎΠ»Π½ΠΈΡ‚Π΅Π»ΡŒΠ½Ρ‹ΠΌ ΡƒΠ³Π»ΠΎΠΌ. (Π”ΠΎΠΏΠΎΠ»Π½ΠΈΡ‚Π΅Π»ΡŒΠ½Ρ‹Π΅ ΡƒΠ³Π»Ρ‹ Π² суммС ΡΠΎΡΡ‚Π°Π²Π»ΡΡŽΡ‚ 90 градусов. Π’ ΠΏΡ€ΡΠΌΠΎΡƒΠ³ΠΎΠ»ΡŒΠ½ΠΎΠΌ Ρ‚Ρ€Π΅ΡƒΠ³ΠΎΠ»ΡŒΠ½ΠΈΠΊΠ΅ Π΄Π²Π° нСпрямых ΡƒΠ³Π»Π° Π΄ΠΎΠΏΠΎΠ»Π½ΡΡŽΡ‚ Π΄Ρ€ΡƒΠ³ Π΄Ρ€ΡƒΠ³Π°.) НапримСр, косинус ΡƒΠ³Π»Π° Ρ‚Π°ΠΊΠΆΠ΅ являСтся синусом Π΄ΠΎΠΏΠΎΠ»Π½ΠΈΡ‚Π΅Π»ΡŒΠ½ΠΎΠ³ΠΎ ΡƒΠ³Π»Π°. Π’ΠΎΡ‡Π½ΠΎ Ρ‚Π°ΠΊ ΠΆΠ΅ ΠΊΡ€Ρ‹ΡˆΠΊΠ° — это вСрсия Π΄ΠΎΠΏΠΎΠ»Π½ΠΈΡ‚Π΅Π»ΡŒΠ½ΠΎΠ³ΠΎ ΡƒΠ³Π»Π°, ΠΊΠ°ΠΊ Π²Ρ‹ ΠΌΠΎΠΆΠ΅Ρ‚Π΅ Π²ΠΈΠ΄Π΅Ρ‚ΡŒ Π³ΠΎΠ»ΡƒΠ±Ρ‹ΠΌ Ρ†Π²Π΅Ρ‚ΠΎΠΌ Π½Π°Π΄ ΠΎΠ΄Π½ΠΈΠΌ ΠΈΠ· красных синусов Π½Π° Π΄ΠΈΠ°Π³Ρ€Π°ΠΌΠΌΠ΅ Π²Π²Π΅Ρ€Ρ…Ρƒ сообщСния.

Одна бонусная триггСрная функция, которая мСня Π½Π΅ΠΌΠ½ΠΎΠ³ΠΎ смущаСт, — это вСркосинус.Если Π±Ρ‹ «со» Π² этом ΠΎΠΏΡ€Π΅Π΄Π΅Π»Π΅Π½ΠΈΠΈ ΠΎΠ·Π½Π°Ρ‡Π°Π»ΠΎ Π΄ΠΎΠΏΠΎΠ»Π½ΠΈΡ‚Π΅Π»ΡŒΠ½Ρ‹ΠΉ ΡƒΠ³ΠΎΠ», Ρ‚ΠΎΠ³Π΄Π° вСркозинус Π±Ρ‹Π» Π±Ρ‹ Ρ‚Π°ΠΊΠΈΠΌ ΠΆΠ΅, ΠΊΠ°ΠΊ ΠΏΠΎΠΊΡ€Ρ‹Π²Π°ΡŽΡ‰ΠΈΠΉ, Π° это Π½Π΅ Ρ‚Π°ΠΊ. ВмСсто этого вСркосинус — это вСрсия Π΄ΠΎΠΏΠΎΠ»Π½ΠΈΡ‚Π΅Π»ΡŒΠ½ΠΎΠ³ΠΎ ΡƒΠ³Π»Π° (Π΄ΠΎΠΏΠΎΠ»Π½ΠΈΡ‚Π΅Π»ΡŒΠ½Ρ‹Π΅ ΡƒΠ³Π»Ρ‹ Π² суммС ΡΠΎΡΡ‚Π°Π²Π»ΡΡŽΡ‚ 180 градусов), Π° Π½Π΅ Π΄ΠΎΠΏΠΎΠ»Π½ΠΈΡ‚Π΅Π»ΡŒΠ½Ρ‹ΠΉ. Π’ Π΄ΠΎΠΏΠΎΠ»Π½Π΅Π½ΠΈΠ΅ ΠΊ опрСдСлСниям ΠΊΠ°ΠΊ 1-cos (ΞΈ) ΠΈ 1 + cos (ΞΈ), вСрсин ΠΈ Π²Π΅Ρ€ΠΊΠΎΠ·ΠΈΠ½ ΠΌΠΎΠ³ΡƒΡ‚ Π±Ρ‹Ρ‚ΡŒ ΠΎΠΏΡ€Π΅Π΄Π΅Π»Π΅Π½Ρ‹ ΠΊΠ°ΠΊ versin (ΞΈ) = 2sin 2 (ΞΈ / 2) ΠΈ vercos (ΞΈ) = 2cos . 2 (ΞΈ / 2). Π’ случаС вСрсины я ΡΡ‡ΠΈΡ‚Π°ΡŽ, Ρ‡Ρ‚ΠΎ ΠΎΠΏΡ€Π΅Π΄Π΅Π»Π΅Π½ΠΈΠ΅, Π²ΠΊΠ»ΡŽΡ‡Π°ΡŽΡ‰Π΅Π΅ cos (ΞΈ), ΡΡ‚Π°Ρ€ΡˆΠ΅, Ρ‡Π΅ΠΌ ΠΎΠΏΡ€Π΅Π΄Π΅Π»Π΅Π½ΠΈΠ΅, Π²ΠΊΠ»ΡŽΡ‡Π°ΡŽΡ‰Π΅Π΅ синус Π² ΠΊΠ²Π°Π΄Ρ€Π°Ρ‚Π΅.Π― ΠΏΡ€Π΅Π΄ΠΏΠΎΠ»Π°Π³Π°ΡŽ, Ρ‡Ρ‚ΠΎ вСркосинус Π±Ρ‹Π» Π±ΠΎΠ»Π΅Π΅ ΠΏΠΎΠ·Π΄Π½ΠΈΠΌ Ρ‚Π΅Ρ€ΠΌΠΈΠ½ΠΎΠΌ, Π°Π½Π°Π»ΠΎΠ³ΠΎΠΌ опрСдСлСния ΠΊΠ²Π°Π΄Ρ€Π°Ρ‚Π° синуса вСрсина с использованиСм вмСсто Π½Π΅Π³ΠΎ косинуса. Если Π²Ρ‹ Π»ΡŽΠ±ΠΈΡ‚Π΅Π»ΡŒ истории Ρ‚Ρ€ΠΈΠ³ΠΎΠ½ΠΎΠΌΠ΅Ρ‚Ρ€ΠΈΠΈ ΠΈ Ρƒ вас Π΅ΡΡ‚ΡŒ Π΄ΠΎΠΏΠΎΠ»Π½ΠΈΡ‚Π΅Π»ΡŒΠ½Π°Ρ информация, Π΄Π°ΠΉΡ‚Π΅ ΠΌΠ½Π΅ Π·Π½Π°Ρ‚ΡŒ! Π’ любом случаС Ρ‚Π°Π±Π»ΠΈΡ†Π° супСрсСкрСтных Ρ‚Ρ€ΠΈΠ³Π³Π΅Ρ€Π½Ρ‹Ρ… Ρ„ΡƒΠ½ΠΊΡ†ΠΈΠΉ бонусов — Π·Π°Π±Π°Π²Π½ΠΎΠ΅ ΡƒΠΏΡ€Π°ΠΆΠ½Π΅Π½ΠΈΠ΅ для выяснСния Ρ‚ΠΎΠ³ΠΎ, Ρ‡Ρ‚ΠΎ ΠΎΠ·Π½Π°Ρ‡Π°ΡŽΡ‚ прСфиксы.

ИспользованиС Π΅Π΄ΠΈΠ½ΠΈΡ‡Π½Ρ‹Ρ… окруТностСй для соотнСсСния ΠΏΡ€ΡΠΌΠΎΡƒΠ³ΠΎΠ»ΡŒΠ½Ρ‹Ρ… Ρ‚Ρ€Π΅ΡƒΠ³ΠΎΠ»ΡŒΠ½ΠΈΠΊΠΎΠ² с синусом ΠΈ косинусом — стСнограмма Π²ΠΈΠ΄Π΅ΠΎ ΠΈ ΡƒΡ€ΠΎΠΊΠ°

ΠŸΡ€ΡΠΌΠΎΡƒΠ³ΠΎΠ»ΡŒΠ½ΠΈΠΊΠΈ ΠΈ Ρ‚Ρ€Π΅ΡƒΠ³ΠΎΠ»ΡŒΠ½ΠΈΠΊ

ΠŸΡ€ΡΠΌΠΎΡƒΠ³ΠΎΠ»ΡŒΠ½ΠΈΠΊΠΈ — Ρ‚Ρ€Π΅ΡƒΠ³ΠΎΠ»ΡŒΠ½ΠΈΠΊΠΈ с ΠΎΠ΄Π½ΠΈΠΌ прямым ΡƒΠ³Π»ΠΎΠΌ.ВсС Π»ΡŽΠ±ΡΡ‚ ΠΏΡ€ΡΠΌΠΎΡƒΠ³ΠΎΠ»ΡŒΠ½Ρ‹Π΅ Ρ‚Ρ€Π΅ΡƒΠ³ΠΎΠ»ΡŒΠ½ΠΈΠΊΠΈ, ΠΏΡ€Π°Π²Π΄Π°? Π§Ρ‚ΠΎ Π² Π½ΠΈΡ… Ρ‚Π°ΠΊΠΎΠ³ΠΎ Ρ…ΠΎΡ€ΠΎΡˆΠ΅Π³ΠΎ? Помимо ΠΏΡ€ΠΎΡ‡Π΅Π³ΠΎ, ΠΎΠ½ΠΈ ΠΎΠ±Π»Π°Π΄Π°ΡŽΡ‚ ΡƒΠ½ΠΈΠΊΠ°Π»ΡŒΠ½Ρ‹ΠΌΠΈ тригономСтричСскими свойствами.

Если ΠΌΡ‹ посмотрим Π½Π° этот ΡƒΠ³ΠΎΠ», тэта, Π½ΠΈΠΆΠ΅, ΠΌΡ‹ ΠΌΠΎΠΆΠ΅ΠΌ ΠΎΠ±ΠΎΠ·Π½Π°Ρ‡ΠΈΡ‚ΡŒ стороны ΠΎΡ‚Π½ΠΎΡΠΈΡ‚Π΅Π»ΡŒΠ½ΠΎ тэты. Π‘Ρ‚ΠΎΡ€ΠΎΠ½Π°, противополоТная тэтС, это … ΠΏΠΎΠ΄ΠΎΠΆΠ΄ΠΈΡ‚Π΅ … противополоТная сторона. БосСдняя сторона — Π΄Π° — сосСдняя сторона. И Π΅Ρ‰Π΅ Π΅ΡΡ‚ΡŒ Π³ΠΈΠΏΠΎΡ‚Π΅Π½ΡƒΠ·Π°, самая длинная сторона.

Π‘Ρ‚ΠΎΡ€ΠΎΠ½Ρ‹ ΠΏΡ€ΡΠΌΠΎΡƒΠ³ΠΎΠ»ΡŒΠ½ΠΎΠ³ΠΎ Ρ‚Ρ€Π΅ΡƒΠ³ΠΎΠ»ΡŒΠ½ΠΈΠΊΠ° ΠΌΠΎΠΆΠ½ΠΎ ΠΎΠ±ΠΎΠ·Π½Π°Ρ‡ΠΈΡ‚ΡŒ Π² зависимости ΠΎΡ‚ ΡƒΠ³Π»Π° Ρ‚Π΅Ρ‚Π°.

Π’Ρ€ΠΈ основных тригономСтричСских Ρ„ΡƒΠ½ΠΊΡ†ΠΈΠΈ — это синус, косинус ΠΈ тангСнс.Бинус Ρ‚Π΅Ρ‚Ρ‹ Ρ€Π°Π²Π΅Π½ противополоТности Π³ΠΈΠΏΠΎΡ‚Π΅Π½ΡƒΠ·Ρ‹. ΠšΠΎΡΠΈΠ½ΡƒΡ Ρ‚Π΅Ρ‚Ρ‹ Ρ€Π°Π²Π΅Π½ смСТному ΠΏΠΎ Π³ΠΈΠΏΠΎΡ‚Π΅Π½ΡƒΠ·Π΅. НаконСц, ΠΊΠ°ΡΠ°Ρ‚Π΅Π»ΡŒΠ½Π°Ρ Ρ‚Π΅Ρ‚Π° Ρ€Π°Π²Π½Π° ΠΏΡ€ΠΎΡ‚ΠΈΠ²ΠΎΠΏΠΎΠ»ΠΎΠΆΠ½ΠΎΠΉ ΠΏΠΎ сосСднСму. ΠœΡ‹ сокращаСм это Π΄ΠΎ Ρ„Ρ€Π°Π·Ρ‹ SOH CAH TOA .

Но Π·Π½Π°Π΅Ρ‚Π΅ Π»ΠΈ Π²Ρ‹, Ρ‡Ρ‚ΠΎ ΠΏΡ€ΡΠΌΠΎΡƒΠ³ΠΎΠ»ΡŒΠ½Ρ‹Π΅ Ρ‚Ρ€Π΅ΡƒΠ³ΠΎΠ»ΡŒΠ½ΠΈΠΊΠΈ соСдинСны с окруТностями? Ну, Π½Π΅ просто ΠΊΡ€ΡƒΠ³. Π― имСю Π² Π²ΠΈΠ΄Ρƒ Π΅Π΄ΠΈΠ½ΠΈΡ‡Π½Ρ‹ΠΉ ΠΊΡ€ΡƒΠ³.

Единичная ΠΎΠΊΡ€ΡƒΠΆΠ½ΠΎΡΡ‚ΡŒ

Единичная ΠΎΠΊΡ€ΡƒΠΆΠ½ΠΎΡΡ‚ΡŒ — ΠΎΠ΄ΠΈΠ½ ΠΈΠ· Π²ΠΎΠ»ΡˆΠ΅Π±Π½Ρ‹Ρ… матСматичСских инструмСнтов, ΠΊΠΎΡ‚ΠΎΡ€Ρ‹Π΅ ΠΎΠ±Π»Π΅Π³Ρ‡Π°ΡŽΡ‚ Π²Π°ΡˆΡƒ Тизнь.Π§Ρ‚ΠΎ особСнно Ρ…ΠΎΡ€ΠΎΡˆΠΎ Π² Π΅Π΄ΠΈΠ½ΠΈΡ‡Π½ΠΎΠΌ ΠΊΡ€ΡƒΠ³Π΅, Ρ‚Π°ΠΊ это Π΅Π³ΠΎ простота. Π­Ρ‚ΠΎ просто ΠΊΡ€ΡƒΠ³ с радиусом 1.

Π§Π΅ΠΌ ΠΎΠ½ ΠΏΠΎΠ»Π΅Π·Π΅Π½? НарисуСм. НачнитС с Π±Π°Π·ΠΎΠ²ΠΎΠΉ оси x ΠΈ y . Π’Π΅ΠΏΠ΅Ρ€ΡŒ Π΄ΠΎΠ±Π°Π²ΡŒΡ‚Π΅ ΠΊΡ€ΡƒΠ³ с Ρ†Π΅Π½Ρ‚Ρ€ΠΎΠΌ Π² Π½Π°Ρ‡Π°Π»Π΅ ΠΊΠΎΠΎΡ€Π΄ΠΈΠ½Π°Ρ‚. ΠžΠΏΡΡ‚ΡŒ ΠΆΠ΅, радиус ΠΊΡ€ΡƒΠ³Π° Ρ€Π°Π²Π΅Π½ 1.

Π•Π΄ΠΈΠ½ΠΈΡ‡Π½Ρ‹ΠΉ ΠΊΡ€ΡƒΠ³

Π—Π΄Π΅ΡΡŒ Π½Π΅ΠΌΠ½ΠΎΠ³ΠΎ Π²ΠΎΠ»ΡˆΠ΅Π±ΡΡ‚Π²Π°. Но этот ΠΏΠΎΡ‚Ρ€ΡΡΠ°ΡŽΡ‰ΠΈΠΉ ΡƒΡ€Π°Π³Π°Π½ Ρ‚ΠΎΠ»ΡŒΠΊΠΎ начинаСтся.

Если наша линия радиуса находится здСсь (ΠΏΠΎΠΊΠ°Π·Π°Π½ΠΎ Π½ΠΈΠΆΠ΅), ΠΈ ΠΌΡ‹ ΠΏΡ€ΠΎΠ²Π΅Π΄Π΅ΠΌ здСсь Π΅Ρ‰Π΅ ΠΎΠ΄Π½Ρƒ линию Π΄ΠΎ оси x (Ρ‚Π°ΠΊΠΆΠ΅ ΠΏΠΎΠΊΠ°Π·Π°Π½ΠΎ Π½ΠΈΠΆΠ΅), ΠΌΡ‹ ΠΏΠΎΠ»ΡƒΡ‡ΠΈΠΌ Ρ‡Ρ‚ΠΎ? ΠŸΡ€ΡΠΌΠΎΡƒΠ³ΠΎΠ»ΡŒΠ½Ρ‹ΠΉ Ρ‚Ρ€Π΅ΡƒΠ³ΠΎΠ»ΡŒΠ½ΠΈΠΊ.

ΠŸΡ€Π°Π²Ρ‹ΠΉ Ρ‚Ρ€Π΅ΡƒΠ³ΠΎΠ»ΡŒΠ½ΠΈΠΊ Π² Π΅Π΄ΠΈΠ½ΠΈΡ‡Π½ΠΎΠΌ ΠΊΡ€ΡƒΠ³Π΅

ΠœΡ‹ Π±ΡƒΠ΄Π΅ΠΌ Π½Π°Π·Ρ‹Π²Π°Ρ‚ΡŒ ΡƒΠ³ΠΎΠ» Π²Ρ‹ΡˆΠ΅ Ρ‚Π΅Ρ‚Π°. А Π³ΠΈΠΏΠΎΡ‚Π΅Π½ΡƒΠ·Π° Ρ‚Ρ€Π΅ΡƒΠ³ΠΎΠ»ΡŒΠ½ΠΈΠΊΠ° Ρ€Π°Π²Π½Π° 1. Π’Π΅ΠΏΠ΅Ρ€ΡŒ вСрнСмся ΠΊ Π½Π°ΡˆΠ΅ΠΌΡƒ Ρ‚Ρ€ΠΈΠ³Π³Π΅Ρ€Ρƒ.

Бинус ΠΈ косинус

Π§Ρ‚ΠΎ Ρ‚Π°ΠΊΠΎΠ΅ синус Ρ‚Π΅Ρ‚Ρ‹ снова? ΠŸΠΎΠΌΠ½ΠΈΡ‚Π΅ SOH CAH TOA . Бинус ΠΏΡ€ΠΎΡ‚ΠΈΠ²ΠΎΠΏΠΎΠ»ΠΎΠΆΠ΅Π½ Π³ΠΈΠΏΠΎΡ‚Π΅Π½ΡƒΠ·Π΅. Π˜Ρ‚Π°ΠΊ, синус Ρ‚Π΅Ρ‚Ρ‹ — это сторона Π³ΠΈΠΏΠΎΡ‚Π΅Π½ΡƒΠ·Ρ‹. Но ΠΏΠΎΠ΄ΠΎΠΆΠ΄ΠΈΡ‚Π΅ — Ссли Π³ΠΈΠΏΠΎΡ‚Π΅Π½ΡƒΠ·Π° Ρ€Π°Π²Π½Π° 1, Ρ‚ΠΎ синус Ρ‚Π΅Ρ‚Ρ‹ Ρ€Π°Π²Π΅Π½ любой Π΄Π»ΠΈΠ½Π΅.Π˜Ρ‚Π°ΠΊ, ΠΌΡ‹ ΠΌΠΎΠΆΠ΅ΠΌ Π½Π°Π·Π²Π°Ρ‚ΡŒ эту Π½ΠΎΠ³Ρƒ Π½ΠΈΠΆΠ΅ синуса.

А Ρ‡Ρ‚ΠΎ насчСт косинуса? ΠšΠΎΡΠΈΠ½ΡƒΡ — это сторона, ΠΏΡ€ΠΈΠ»Π΅Π³Π°ΡŽΡ‰Π°Ρ ΠΊ Π³ΠΈΠΏΠΎΡ‚Π΅Π½ΡƒΠ·Π΅. Π˜Ρ‚Π°ΠΊ, это сторона Π½Π°Π΄ Π΅Π΄ΠΈΠ½ΠΈΡ†Π΅ΠΉ ΠΈΠ»ΠΈ Ρ‚ΠΎΠ»ΡŒΠΊΠΎ эта сторона:

Π­Ρ‚ΠΎ ΠΎΠ·Π½Π°Ρ‡Π°Π΅Ρ‚, Ρ‡Ρ‚ΠΎ Ρ‚ΠΎΡ‡ΠΊΠ° Π² ΠΆΠ΅Π»Ρ‚ΠΎΠΉ Ρ‚ΠΎΡ‡ΠΊΠ΅, Π³Π΄Π΅ радиус достигаСт края ΠΊΡ€ΡƒΠ³Π°, ΠΌΠΎΠΆΠ΅Ρ‚ Π±Ρ‹Ρ‚ΡŒ ΠΈΠ΄Π΅Π½Ρ‚ΠΈΡ„ΠΈΡ†ΠΈΡ€ΠΎΠ²Π°Π½Π° ΠΊΠ°ΠΊ (cos (theta), sin (theta)). Π˜Ρ‚Π°ΠΊ, наша Π±Π°Π±ΠΎΡ‡ΠΊΠ° ΠΏΡ€ΡΠΌΠΎΡƒΠ³ΠΎΠ»ΡŒΠ½ΠΎΠ³ΠΎ Ρ‚Ρ€Π΅ΡƒΠ³ΠΎΠ»ΡŒΠ½ΠΈΠΊΠ° Π²Π·ΠΌΠ°Ρ…Π½ΡƒΠ»Π° ΠΊΡ€Ρ‹Π»ΡŒΡΠΌΠΈ, образуя синус ΠΈ косинус.И этот Π²Π΅Ρ‚Π΅Ρ€ добрался Π΄ΠΎ нашСй Π΅Π΄ΠΈΠ½ΠΈΡ‡Π½ΠΎΠΉ окруТности, Π³Π΄Π΅ снова ΠΏΠΎΡΠ²Π»ΡΡŽΡ‚ΡΡ Ρ‚Π΅ ΠΆΠ΅ синус ΠΈ косинус. Наш ΡƒΡ€Π°Π³Π°Π½ ΠΏΠΎΡ‚Ρ€ΡΡΠ°ΡŽΡ‰ΠΈΡ… возмоТностСй сСйчас Π΄Π΅ΠΉΡΡ‚Π²ΠΈΡ‚Π΅Π»ΡŒΠ½ΠΎ ΡƒΡ…ΠΎΠ΄ΠΈΡ‚.

Π’Ρ€Π΅ΡƒΠ³ΠΎΠ»ΡŒΠ½ΠΈΠΊΠΈ Π΅Π΄ΠΈΠ½ΠΈΡ‡Π½ΠΎΠΉ окруТности

Π”Π°Π²Π°ΠΉΡ‚Π΅ посмотрим Π½Π° это Π²Ρ€Π°Ρ‰Π΅Π½ΠΈΠ΅ ΡƒΡ€Π°Π³Π°Π½Π° (смотритС Π²ΠΈΠ΄Π΅ΠΎ, Π½Π°Ρ‡ΠΈΠ½Π°ΡŽΡ‰Π΅Π΅ΡΡ Π² 03:32). НСзависимо ΠΎΡ‚ Ρ‚ΠΎΠ³ΠΎ, ΠΊΡƒΠ΄Π° ΠΌΡ‹ ΠΏΠ΅Ρ€Π΅ΠΌΠ΅Ρ‰Π°Π΅ΠΌ Π½Π°ΡˆΡƒ линию радиуса, ΠΎΠ½Π° всС Ρ€Π°Π²Π½ΠΎ ΠΏΠΎΠΏΠ°Π΄Π°Π΅Ρ‚ Π² ΠΊΡ€Π°ΠΉ нашСго Π΅Π΄ΠΈΠ½ΠΈΡ‡Π½ΠΎΠ³ΠΎ ΠΊΡ€ΡƒΠ³Π° Π² Ρ‚ΠΎΡ‡ΠΊΠ΅ (cos (theta), sin (theta)). Когда ΠΎΠ½ двиТСтся, значСния косинуса Ρ‚Π΅Ρ‚Π° ΠΈ синус Ρ‚Π΅Ρ‚Π° просто ΠΈΠ·ΠΌΠ΅Π½ΡΡŽΡ‚ΡΡ вмСстС с ΠΈΠ·ΠΌΠ΅Π½Π΅Π½ΠΈΠ΅ΠΌ Ρ‚Π΅Ρ‚Ρ‹.

Π—Π΄Π΅ΡΡŒ Ρ‚Π΅Ρ‚Π° Ρ€Π°Π²Π½Π° 0:

Π­Ρ‚ΠΎ Π΄Π΅ΠΉΡΡ‚Π²ΠΈΡ‚Π΅Π»ΡŒΠ½ΠΎ Π½Π΅ Ρ‚Ρ€Π΅ΡƒΠ³ΠΎΠ»ΡŒΠ½ΠΈΠΊ, Π½ΠΎ ΠΏΡ€ΠΈΠ½Ρ†ΠΈΠΏ Π½Π΅ΠΈΠ·ΠΌΠ΅Π½Π΅Π½. ΠšΠΎΡΠΈΠ½ΡƒΡ-Ρ‚Π΅Ρ‚Π° — это всС Π΅Ρ‰Π΅ расстояниС ΠΎΡ‚ Π½Π°Ρ‡Π°Π»Π° ΠΊΠΎΠΎΡ€Π΄ΠΈΠ½Π°Ρ‚ Π΄ΠΎ ΠΆΠ΅Π»Ρ‚ΠΎΠΉ Ρ‚ΠΎΡ‡ΠΊΠΈ, поэтому косинус-Ρ‚Π΅Ρ‚Π° Ρ€Π°Π²Π΅Π½ 1. А ΠΊΠ°ΠΊ насчСт синус-Ρ‚Π΅Ρ‚Π°? ΠŸΠΎΡΠΊΠΎΠ»ΡŒΠΊΡƒ это ΠΏΠΎΡ…ΠΎΠΆΠ΅ Π½Π° Ρ‚Ρ€Π΅ΡƒΠ³ΠΎΠ»ΡŒΠ½ΠΈΠΊ, Π³Π΄Π΅ сторона, противополоТная тэтС, Ρ€Π°Π²Π½Π° 0, Ρ‚ΠΎΠ³Π΄Π° синус тэта Ρ€Π°Π²Π΅Π½ 0.

Π§Ρ‚ΠΎ, Ссли ΠΌΡ‹ пСрСмСстим тэту Π½Π° 90 градусов? Π’Π΅ΠΏΠ΅Ρ€ΡŒ косинус Ρ‚Π΅Ρ‚Π° Ρ€Π°Π²Π΅Π½ 0, Π° синус Ρ‚Π΅Ρ‚Π° Ρ€Π°Π²Π΅Π½ 1.

Когда ΠΌΡ‹ ΠΏΠ΅Ρ€Π΅Ρ…ΠΎΠ΄ΠΈΠΌ Π²ΠΎ Π²Ρ‚ΠΎΡ€ΠΎΠΉ ΠΊΠ²Π°Π΄Ρ€Π°Π½Ρ‚, косинус Ρ‚Π΅Ρ‚Π° становится ΠΎΡ‚Ρ€ΠΈΡ†Π°Ρ‚Π΅Π»ΡŒΠ½Ρ‹ΠΌ.ΠŸΠΎΡ‡Π΅ΠΌΡƒ? Наши значСния x ΠΎΡ‚Ρ€ΠΈΡ†Π°Ρ‚Π΅Π»ΡŒΠ½Ρ‹. Бинус Ρ‚Π΅Ρ‚Π° остаСтся ΠΏΠΎΠ»ΠΎΠΆΠΈΡ‚Π΅Π»ΡŒΠ½Ρ‹ΠΌ, ΠΏΠΎΠΊΠ° ΠΌΡ‹ Π½Π΅ Π΄ΠΎΠΉΠ΄Π΅ΠΌ Π΄ΠΎ ΠΊΠ²Π°Π΄Ρ€Π°Π½Ρ‚ΠΎΠ² III ΠΈ IV. Но Π² ΠΊΠ²Π°Π΄Ρ€Π°Π½Ρ‚Π΅ IV косинус снова становится ΠΏΠΎΠ»ΠΎΠΆΠΈΡ‚Π΅Π»ΡŒΠ½Ρ‹ΠΌ. ВсС это основано Π½Π° Π½Π°ΡˆΠΈΡ… значСниях x ΠΈ y .

Π˜Ρ‚Π°ΠΊ, Π²Π΅Ρ‚Π΅Ρ€ ΠΈΠ· ΠΏΡ€ΡΠΌΠΎΡƒΠ³ΠΎΠ»ΡŒΠ½ΠΎΠ³ΠΎ Ρ‚Ρ€Π΅ΡƒΠ³ΠΎΠ»ΡŒΠ½ΠΈΠΊΠ° всСгда являСтся Ρ‡Π°ΡΡ‚ΡŒΡŽ ΡƒΡ€Π°Π³Π°Π½Π° с Π΅Π΄ΠΈΠ½ΠΈΡ‡Π½Ρ‹ΠΌ ΠΊΡ€ΡƒΠ³ΠΎΠΌ. Он ΠΌΠΎΠΆΠ΅Ρ‚ Π΄ΡƒΡ‚ΡŒ Π½Π° сСвСр, восток, юг ΠΈΠ»ΠΈ Π·Π°ΠΏΠ°Π΄, Π½ΠΎ всС взаимосвязано.

ΠšΡ€Π°Ρ‚ΠΊΠΎΠ΅ содСрТаниС ΡƒΡ€ΠΎΠΊΠ°

Π’ ΠΈΡ‚ΠΎΠ³Π΅ Π½Π°ΠΌ ΡƒΠ΄Π°Π»ΠΎΡΡŒ ΡΠ²ΡΠ·Π°Ρ‚ΡŒ Π±Π°Π±ΠΎΡ‡Π΅ΠΊ ΠΈ ΡƒΡ€Π°Π³Π°Π½Ρ‹ с принцСссой Π›Π΅ΠΉΠ΅ΠΉ ΠΈ Ρ‚Ρ€ΠΈΠ³ΠΎΠ½ΠΎΠΌΠ΅Ρ‚Ρ€ΠΈΠ΅ΠΉ.Π§Ρ‚ΠΎ Π΅Ρ‰Π΅ Π±ΠΎΠ»Π΅Π΅ Π²Π°ΠΆΠ½ΠΎ, ΠΌΡ‹ рассмотрСли связи ΠΌΠ΅ΠΆΠ΄Ρƒ ΠΏΡ€ΡΠΌΠΎΡƒΠ³ΠΎΠ»ΡŒΠ½Ρ‹ΠΌΠΈ Ρ‚Ρ€Π΅ΡƒΠ³ΠΎΠ»ΡŒΠ½ΠΈΠΊΠ°ΠΌΠΈ, Π΅Π΄ΠΈΠ½ΠΈΡ‡Π½ΠΎΠΉ ΠΎΠΊΡ€ΡƒΠΆΠ½ΠΎΡΡ‚ΡŒΡŽ ΠΈ синусом ΠΈ косинусом.

ΠŸΡ€ΡΠΌΡ‹Π΅ Ρ‚Ρ€Π΅ΡƒΠ³ΠΎΠ»ΡŒΠ½ΠΈΠΊΠΈ — это Ρ‚Ρ€Π΅ΡƒΠ³ΠΎΠ»ΡŒΠ½ΠΈΠΊΠΈ с ΠΎΠ΄Π½ΠΈΠΌ прямым ΡƒΠ³Π»ΠΎΠΌ. Π’ ΠΏΡ€ΡΠΌΠΎΡƒΠ³ΠΎΠ»ΡŒΠ½ΠΎΠΌ Ρ‚Ρ€Π΅ΡƒΠ³ΠΎΠ»ΡŒΠ½ΠΈΠΊΠ΅ синус ΡƒΠ³Π»Π° Ρ€Π°Π²Π΅Π½ сторонС, ΠΏΡ€ΠΎΡ‚ΠΈΠ²ΠΎΠΏΠΎΠ»ΠΎΠΆΠ½ΠΎΠΉ Π³ΠΈΠΏΠΎΡ‚Π΅Π½ΡƒΠ·Π΅. ΠšΠΎΡΠΈΠ½ΡƒΡ Ρ€Π°Π²Π΅Π½ сторонС, ΠΏΡ€ΠΈΠ»Π΅Π³Π°ΡŽΡ‰Π΅ΠΉ ΠΊ Π³ΠΈΠΏΠΎΡ‚Π΅Π½ΡƒΠ·Π΅.

На Π΅Π΄ΠΈΠ½ΠΈΡ‡Π½ΠΎΠΉ окруТности ΠΌΡ‹ ΠΌΠΎΠΆΠ΅ΠΌ ΡΠΎΠ·Π΄Π°Ρ‚ΡŒ ΠΏΡ€ΡΠΌΠΎΡƒΠ³ΠΎΠ»ΡŒΠ½Ρ‹ΠΉ Ρ‚Ρ€Π΅ΡƒΠ³ΠΎΠ»ΡŒΠ½ΠΈΠΊ, Π΄ΠΎΠ±Π°Π²ΠΈΠ² Ρ€Π°Π΄ΠΈΡƒΡΠ½ΡƒΡŽ линию ΠΈ соСдинив Π΅Π΅ с осью x . ΠŸΠΎΡΠΊΠΎΠ»ΡŒΠΊΡƒ радиус Π΅Π΄ΠΈΠ½ΠΈΡ‡Π½ΠΎΠ³ΠΎ ΠΊΡ€ΡƒΠ³Π° Ρ€Π°Π²Π΅Π½ Π΅Π΄ΠΈΠ½ΠΈΡ†Π΅, Π³ΠΈΠΏΠΎΡ‚Π΅Π½ΡƒΠ·Π° этих Ρ‚Ρ€Π΅ΡƒΠ³ΠΎΠ»ΡŒΠ½ΠΈΠΊΠΎΠ² Ρ‚Π°ΠΊΠΆΠ΅ Ρ€Π°Π²Π½Π° Π΅Π΄ΠΈΠ½ΠΈΡ†Π΅.Π­Ρ‚ΠΎ ΠΎΠ·Π½Π°Ρ‡Π°Π΅Ρ‚, Ρ‡Ρ‚ΠΎ синус ΡƒΠ³Π»Π°, ΠΎΠ±Ρ€Π°Π·ΠΎΠ²Π°Π½Π½ΠΎΠ³ΠΎ радиусом ΠΈ осью x , Ρ€Π°Π²Π΅Π½ любой Π΄Π»ΠΈΠ½Π΅ здСсь:

И косинус Ρ€Π°Π²Π΅Π½ любой Π΄Π»ΠΈΠ½Π΅ здСсь:

Π Π΅Π·ΡƒΠ»ΡŒΡ‚Π°Ρ‚Ρ‹ обучСния

ПослС этого ΡƒΡ€ΠΎΠΊΠ° Π²Ρ‹ Π΄ΠΎΠ»ΠΆΠ½Ρ‹ ΡƒΠΌΠ΅Ρ‚ΡŒ:

  • ΠžΠ±ΡŠΡΡΠ½ΡΡ‚ΡŒ ΠΏΠΎΠ»Π΅Π·Π½ΠΎΡΡ‚ΡŒ Π΅Π΄ΠΈΠ½ΠΈΡ‡Π½ΠΎΠ³ΠΎ ΠΊΡ€ΡƒΠ³Π°
  • ΠžΠΏΠΈΡˆΠΈΡ‚Π΅, ΠΊΠ°ΠΊ ΡΠΎΠ·Π΄Π°Ρ‚ΡŒ ΠΏΡ€ΡΠΌΠΎΡƒΠ³ΠΎΠ»ΡŒΠ½Ρ‹ΠΉ Ρ‚Ρ€Π΅ΡƒΠ³ΠΎΠ»ΡŒΠ½ΠΈΠΊ Π½Π° Π΅Π΄ΠΈΠ½ΠΈΡ‡Π½ΠΎΠΉ окруТности
  • ΠžΠΏΡ€Π΅Π΄Π΅Π»ΠΈΡ‚Π΅ синус, косинус ΠΈ тангСнс с ΠΏΠΎΠΌΠΎΡ‰ΡŒΡŽ Π΅Π΄ΠΈΠ½ΠΈΡ‡Π½ΠΎΠΉ окруТности ΠΈ ΠΏΡ€ΡΠΌΠΎΡƒΠ³ΠΎΠ»ΡŒΠ½ΠΎΠ³ΠΎ Ρ‚Ρ€Π΅ΡƒΠ³ΠΎΠ»ΡŒΠ½ΠΈΠΊΠ°
.

Π”ΠΎΠ±Π°Π²ΠΈΡ‚ΡŒ ΠΊΠΎΠΌΠΌΠ΅Π½Ρ‚Π°Ρ€ΠΈΠΉ

Π’Π°Ρˆ адрСс email Π½Π΅ Π±ΡƒΠ΄Π΅Ρ‚ ΠΎΠΏΡƒΠ±Π»ΠΈΠΊΠΎΠ²Π°Π½. ΠžΠ±ΡΠ·Π°Ρ‚Π΅Π»ΡŒΠ½Ρ‹Π΅ поля ΠΏΠΎΠΌΠ΅Ρ‡Π΅Π½Ρ‹ *